0% found this document useful (0 votes)
167 views184 pages

SSC Steno 2025 English Subject Mock Format e Book With Solution

The document is a compilation of SSC Steno 2025 exam materials, including topic-wise English questions and solutions from all 9 shifts. It provides resources for complete preparation, including mock tests, previous year papers, and free lectures. Additionally, it includes links to download the RBE application and access various educational content on YouTube and other platforms.

Uploaded by

swaminishant5556
Copyright
© © All Rights Reserved
We take content rights seriously. If you suspect this is your content, claim it here.
Available Formats
Download as PDF, TXT or read online on Scribd
0% found this document useful (0 votes)
167 views184 pages

SSC Steno 2025 English Subject Mock Format e Book With Solution

The document is a compilation of SSC Steno 2025 exam materials, including topic-wise English questions and solutions from all 9 shifts. It provides resources for complete preparation, including mock tests, previous year papers, and free lectures. Additionally, it includes links to download the RBE application and access various educational content on YouTube and other platforms.

Uploaded by

swaminishant5556
Copyright
© © All Rights Reserved
We take content rights seriously. If you suspect this is your content, claim it here.
Available Formats
Download as PDF, TXT or read online on Scribd
You are on page 1/ 184

SSC Steno 2025 COMPILATION

(Exams held in Aug 2025)


All 9 shifts
English Subject [Topic-Wise]
Mock Format e-book with solution
RBE- Revolution By Education
JOIN US (click on Image) ON:-
Free important job
details
& Free lectures
http://youtube.com/c/RBERevolutionByEducation

All previous year


paper compilation
(SSC & Railway Exam)

https://t.me/RBE_S

RBE APP

https://play.google.com/st
https://www.instagra ore/apps/details?id=com.r
m.com/shubh_rbe/ evolution.education By Shubham Sir
RBE COMBO TEST SERIES
https://rbelearning.com/test-series

Maths+English+GK+Reasoning
FULL LENTH MOCKS
SECTIONAL MOCKS
ALL SUBJECTS
TYPE - WISE

TOPPERS
की पसंद
SSC CGL 2025
Air Air Air Air Air
8 16 13 22
5

SSC CGL 2024 Toppers - RBE paid Test Series Users

RBE WEBSITE - rbelearning.com


DOWNLOAD RBE APPLICATION NOW
https//play.google.com/store/apps/details?id=com.revolution.education

For SSC Exams Complete Preparation


FREE PLAYLISTS ON YOUTUBE

MATHS REVISION COURSE LATEST GOVT. JOBS

COMPUTER GK IMP. VIDEOS

GK CRASH COURSE MONTHLY TOP 100 CA

SSC BEST QUESTIONS SCIENCE THEORY + PYQS


Index
S.No Topic Questions Question Per shift
1 One Word Substitution 45 5.00
2 Spellings 42 4.67
3 Synonyms 43 4.78
4 Antonyms 48 5.33
5 Homonym 26 2.89
6 Idioms 48 5.33
7 Sentence Improvement 77 8.56
8 Error Detection 70 7.78
9 Fill in the Blanks 87 9.67
10 Close Test 55 6.11
11 Active & Passive Voice 85 9.44
12 Narration 92 10.22
13 Jumbled Sentences 99 11.00
14 Reading Comprehension 80 8.89
15 Miscellaneous 2 0.22
Download RBE application for SSC Exams Complete Preparation

SSC Steno 2025 T-1 One Word Substitution All 45 Questions with Detailed Solution and Answer key

Q1. Choose the correct one-word substitution for:


"The practice of deliberately expressing less
than what is meant." Q8. Choose the correct one-word substitution for:
(a) Hyperbole (b) Euphemism
(c) Litotes (d) Irony “A facility equipped with instruments for
studying stars, planets, and other celestial
bodies”
Q2. Choose the correct one-word substitution for: (a) Archive (b) Greenhouse
"An ideal society marked by justice, equality, (c) Workshop (d) Observatory
and happiness."
(a) Technocracy (b) Utopia
(c) Oligarchy (d) Dystopia Q9. Select the one-word term for:
“Someone who is consistently excluded or
ignored by a community due to tack of trust or
Q3. Which of the following words means popula rity .”
"prohibited by law or treaty from being imported or (a) Dissenter (b) Parian
exported"? (c) Loyalist (d) Noble
(a) Illegal (b) Banned
(c) Restricted (d) Contraband
Q10. What is the place for ammunition and
weapons called ?
Q4. Choose the correct one-word substitution for: (a) Library (b) Gallery
"A person who hates other people or (c) Arsenal (d) Laboratory
mankind"
(a) Misogynist (b) Optimist
(c) Egoist (d) Misanthrope Q11. Choose the correct one-word
substitution for:

Q5. Choose the correct one-word substitution A place where aircraft are kept
for: (a) Depot (b) Barracks
"A humorous poem of five lines" (c) Hangar (d) Dockyard
(a) Sonnet (b) Elegy
(c) Haiku (d) Limerick
Q12. Choose the one which can be substituted for
the given sentence.
Q6. Choose the correct one-word substitution A medicine that induces sleep
for: (a) Hypnotic (b) Narcotic
A person who is indifferent to pleasure or pain. (c) Analgesic (d) Sedative
(a) Masochist (b) Pessimist
(c) Epicure (d) Stoic
Q13. Choose the one which can be substituted for
the given sentence.
Q7. Choose the correct one-word substitution for: A place where wild animals are kept
“Property inherited from one’s father” (a) Abattoir (b) Zoo
(a) Testimony (b) Autocracy (c) Aquarium (d) Aviary
(c) Matrimony (d) Patrimony
Telegram (Previous year papers PDFs [SSC,Railway,DSSSB,UP SI]): https://t.me/RBE_S
YouTube (Free lectures and job updates): https://www.youtube.com/c/RBERevolutionByEducation
Download RBE application for SSC Exams Complete Preparation
Q14. Choose the one which can be substituted for Q22. Choose the correct one-word substitution
the given sentence. for:
One who studies the evolution of mankind "A person who studies the origin and
(a) Anthropologist (b) Sociologist development of human societies and cultures"
(c) Archaeologist (d) Philologist (a) Anthropologist (b) Archaeologist
(c) Historian (d) Sociologist

Q15. What is the one word for “A person who is


talkative”? Q23. What is the meaning of the word
(a) Introvert (b) Garrulous "Superannuation"?
(c) Taciturn (d) Reticent (a) A type of insurance for future health care
(b) A system of retirement benefits for employees
(c) The act of receiving a bonus
Q16. Choose the one which can be substituted for (d) The process of becoming too old to work
the given sentence.
A person’s peculiar habit or mannerism
(a) Eccentricity (b) Idiosyncrasy Q24. Choose the one which can be substituted for
(c) Hypocrisy (d) Illusion the given sentence:
A word formed from the initial letters of a name
Q17. Which of the following refers to the branch of (a) Homonym (b) Acronym
science concerned with the study of sound, its (c) Antonym (d) Pseudonym
production, transmission, and effects?
(a) Thermodynamics (b) Optics
(c) Acoustics (d) Geology Q25. Choose the correct one-word substitution for
the following description.
"A person who talks excessively"
Q18. Choose the correct one-word substitution (a) Garrulous (b) Silent
for: (c) Reticent (d) Introvert
"A speech delivered without preparation"
(a) Impromptu (b) Rebuttal
(c) Monologue (d) Soliloquy Q26. Choose the one which can be substituted for
the given sentence.
A political disorder and absence of government
Q19. Choose the one-word substitution for the (a) Monarchy (b) Plutocracy
following sentence. (c) Oligarchy (d) Anarchy
Someone who has been forced to flee his or her
country because of persecution, war or violence.
(a) Refugee (b) Sanctuary Q27. Choose the one which can be substituted for
(c) Asylum (d) Shelter the given sentence.
A person who writes books
(a) Editor (b) Publisher
Q20. Choose the correct one-word substitution (c) Orator (d) Author
for: " the act of placing a dead person in a
grave."
(a) Rebellion (b) Coronation Q28. Choose the one which can be substituted for
(c) Interment (d) Funeral the given sentence.
The murder of a king
(a) Patricide (b) Homicide
Q21. Which of the following words refers to a (c) Regicide (d) Fratricide
group of people who have graduated from a
particular school, college, or university?
(a) Student (b) Visitor Q29. What is the one-word substitute for “A poem
(c) Alumni (d) Teacher of mourning”?
(a) Ballad (b) Elegy
Telegram (Previous year papers PDFs [SSC,Railway,DSSSB,UP SI]): https://t.me/RBE_S
YouTube (Free lectures and job updates): https://www.youtube.com/c/RBERevolutionByEducation
Download RBE application for SSC Exams Complete Preparation
(c) Sonnet (d) Ode Q37. Choose the one which can be substituted for
the given sentence.
An imaginary ideal place of perfection
Q30. Choose the one which can be substituted for (a) Utopia (b) Monarchy
the given sentence. (c) Dystopia (d) Fable
A partner in crime
(a) Patriot (b) Accomplice
(c) Ally (d) Rival Q38. Choose the one which can be substituted for
the given sentence.
A person without a settled home
Q31. Choose the one which can be substituted for (a) Noble (b) Tenant
the given sentence. (c) Native (d) Vagabond
An official pardon for offenders
(a) Clemency (b) Absolution Q39. Choose the one which can be substituted for
(c) Amnesty (d) Reprieve the given sentence.
A short stay at a place
(a) Peregrination (b) Sojourn
Q32. Choose the one which can be substituted for
the given sentence. (c) Exodus (d) Odyssey
A short amusing story
(a) Anecdote (b) Chronicle Q40. Choose the one which can be substituted for
(c) Essay (d) Epistle the given sentence.
A person who has long experience in a field
Q33. Choose the one which can be substituted for (a) Novice (b) Veteran
(c) Trainee (d) Amateur
the given sentence.
The act of killing one’s brother
(a) Homicide (b) Suicide Q41. Choose the one which can be substituted for
(c) Patricide (d) Fratricide the given sentence.
A person who is extremely selective or difficult to
please
Q34. Choose the one which can be substituted for
(a) Docile (b) Gullible
the given sentence.
(c) Fastidious (d) Hostile
A man whose wife is dead
(a) Widower (b) Bachelor
(c) Divorcee (d) Heir Q42. Choose the one which can be substituted for
the given sentence.
A place where dead bodies are kept temporarily
Q35. Choose the correct one-word substitution
(a) Dispensary (b) Crematorium
for:
A symbol that serves as an emblem of a group of (c) Morgue (d) Graveyard
people
(a) Totem (b) Token Q43. What is the one-word substitution for the
(c) Trophy (d) Banner misappropriation of money?
(a) Embezzlement (b) Bribery
Q36. Choose the best one-word substitute for: (c) Extortion (d) Fraud
An inscription written on stone in the memory of a
deceased person. Q44. Choose the one which can be substituted for
(a) Memoir (b) Epitaph the given sentence.
(c) Epigraph (d) Autograph One who abandons one’s religious faith or
principles
(a) Infidel (b) Pagan
(c) Apostate (d) Heretic
Telegram (Previous year papers PDFs [SSC,Railway,DSSSB,UP SI]): https://t.me/RBE_S
YouTube (Free lectures and job updates): https://www.youtube.com/c/RBERevolutionByEducation
Download RBE application for SSC Exams Complete Preparation
Q45. Choose the one which can be substituted for (a) Patricide (b) Matricide
the given sentence. (c) Fratricide (d) Homicide
Killing of one’s mother

Answer Key:

Q1 c Q2 b Q3 d Q4 d Q5 d
Q6 d Q7 d Q8 d Q9 b Q10 c
Q11 c Q12 a Q13 b Q14 a Q15 b
Q16 b Q17 c Q18 a Q19 a Q20 c
Q21 c Q22 a Q23 b Q24 b Q25 a
Q26 d Q27 d Q28 c Q29 b Q30 b
Q31 c Q32 a Q33 d Q34 a Q35 a
Q36 b Q37 a Q38 d Q39 b Q40 b
Q41 c Q42 c Q43 a Q44 c Q45 b

Solution 1: Explanation "Optimist" means a person who has a positive


Litotes is a figure of speech that uses deliberate outlook.
understatement, often expressing something by "Egoist" means a self-centered person.
negating its opposite, effectively saying less than
is meant.
Hyperbole is exaggeration. Solution 5: Explanation
Euphemism is a mild or indirect expression used A limerick is a short, humorous poem consisting
to replace a harsh one. of five lines with a specific rhythm and rhyme
Irony involves saying something contrary to what scheme.
is meant, but not necessarily less than what is It is well-known as a form of comic or witty poetry
meant. often used for humorous effect.
Other terms like "epigram" or "satire" may refer to
humorous writing but do not specifically denote a
Solution 2: Explanation five-line h umorous poem.
"Utopia" refers to an ideal society characterized
by justice, equality, and happiness.
Technocracy is government by technical experts. Solution 6: Explanation
Oligarchy is rule by a small group of people. A "Stoic" is someone who endures pain or
Dystopia is an imagined society characterized by hardship without showing their feelings or
misery and oppression, opposite of utopia complaining.
"Masochist" refers to someone who enjoys pain.
"Pessimist" is someone who expects the worst.
Solution 3: Explanation "Epicure" is a person who enjoys fine food and
"Contraband" refers specifically to goods that are drink, seeking pleasure.
illegal to import or export according to law or
treaties.
"Illegal" and "Banned" are more general terms, Solution 7: Explanation
and "Restricted" means limited or controlled but "Patrimony" refers to an inheritance from a father
not necessarily prohibited. or ancestor.
"Testimony" is a formal statement or declaration.
"Autocracy" is a system of government by one
Solution 4: Explanation person with absolute power.
"Misanthrope" means a person who dislikes or "Matrimony" means marriage.
hates mankind.
"Misogynist" means hatred specifically towards
women.
Telegram (Previous year papers PDFs [SSC,Railway,DSSSB,UP SI]): https://t.me/RBE_S
YouTube (Free lectures and job updates): https://www.youtube.com/c/RBERevolutionByEducation
Download RBE application for SSC Exams Complete Preparation
Solution 8: Explanation: Solution 13: ✅ Correct Answer :
An observatory is a place equipped with Zoo
telescopes and other instruments to observe 📌 Point-wise Explanation :
celestial events and bodies. Meaning of the Sentence : "A place where wild
"Archive" refers to a place where records are animals are kept" refers to a location designed for
stored. housing and displaying animals, often for public
"Greenhouse" is for growing plants. viewing and conservation.
"Workshop" is a place for making or repairing Correct Substitution – Zoo : ✔ A zoo is a facility
things.
where wild animals are kept in enclosures, cared
Therefore, the correct choice is Observatory.
for, and often exhibited to the public.

Solution 9: Explanation:
Solution 14: ✅ Correct Answer: Anthropologist
"Parian" refers to an outcast or someone who is
🔍 Explanation:
socially rejected.
"Dissenter" is someone who disagrees or Definition : An anthropologist is someone who
opposes a majority opinion. studies the origin, development, and behavior of
"Loyalist" is someone loyal to a cause or leader. human beings , including the evolution of
"Noble" refers to someone of high moral mankind , cultures, and societies.
character or aristocratic status. Sociologist: ❌ Studies social behavior,
Therefore, the correct choice is Parian. institutions, and relationships — not human
evolution.
Archaeologist: ❌ Studies ancient artifacts and
Solution 10: Explanation: ruins — focuses more on material remains than
An arsenal is a storage place or facility for biological evolution.
weapons and ammunition. Philologist: ❌ Studies language and its
Library is for books. historical development — unrelated to human
Gallery is typically an art space or a place for evolution.
viewing.
Laboratory is for scientific experiments.
Therefore, the correct choice is Arsenal. Solution 15: ✅ Correct Answer: Garrulous
Correct Word :
Garrulous = excessively talkative, especially on
Solution 11: Explanation:
trivial matters
A hangar is a large building where aircraft are
Example: The garrulous old man kept the
stored and maintained.
children entertained with endless stories.
Depot is a storage place for goods or military
❌ Other Options Explained:
equipment.
Barracks are living quarters for soldiers. 1 ⃣ Introvert
Dockyard is a place where ships are built, Type : Noun (personality trait)
repaired, or docked. Meaning : A person who tends to be quiet,
Therefore, the correct choice is Hangar. reserved, and inward-focused.
Traits : Prefers solitude, avoids large social
gatherings, speaks less.
Solution 12: ✅ Correct Answer: Hypnotic Why Incorrect : Opposite of talkative — introverts
The sentence describes “a medicine that are typically quiet and reflective.
induces sleep.” 2 ⃣ Taciturn
A hypnotic is a specific type of drug used to Type : Adjective
induce sleep or treat insomnia . Meaning : Someone who is habitually silent or
It directly targets the brain’s sleep mechanisms, uncommunicative.
making it the most precise substitution. Traits : Speaks very little, avoids conversation
even when prompted.
Why Incorrect : Strong antonym of talkative —
Telegram (Previous year papers PDFs [SSC,Railway,DSSSB,UP SI]): https://t.me/RBE_S
YouTube (Free lectures and job updates): https://www.youtube.com/c/RBERevolutionByEducation
Download RBE application for SSC Exams Complete Preparation
taciturn individuals rarely speak. of burying a dead body in a grave or tomb.
3 ⃣ Reticent It’s a technical term used in legal, religious, and
Type : Adjective ceremonial contexts.
Meaning : Reluctant to speak or express Root : From Latin interrare — in (in) + terra
thoughts and feelings. (earth) → literally “to put into the earth.”
Traits : Hesitant, reserved, often avoids sharing
personal information.
Solution 21: The correct answer is Alumni .
Why Incorrect : Implies unwillingness to talk —
"Alumni" is the plural form of "alumnus" or
not suitable for describing a talkative person.
"alumna," and it specifically refers to a group of
individuals who were formerly enrolled at and
Solution 16: ✅ Correct Answer: Idiosyncrasy have graduated from a particular educational
🔍 Definition Match: institution. The other options, such as " Student "
(currently enrolled), "Visitor" (temporary guest), or
“A person’s peculiar habit or mannerism” This
"Teacher" (instructor), do not fit this definition.
refers to a unique, often quirky behavior or trait
specific to an individual.
Other options either broaden the scope ( Solution 22: The correct answer is "
Eccentricity ) or diverge entirely ( Hypocrisy , Anthropologist." An anthropologist is a person
Illusion ). who studies human societies, their origins,
development, and cultural aspects.
Archaeologists study past human activity through
Solution 17: ✅ Correct Answer: Acoustics
material remains, historians focus on historical
🔍 Definition Focus:
events, and sociologists’ study social behavior
The branch of science concerned with the study and structures, but only anthropologists
of sound , including its production , transmission , specifically study the origin and development of
and effect. societies and cultures.

Solution 18: ✅ Correct Answer: Impromptu Solution 23: The correct answer is " A system of
🔍 Target Definition: retirement benefits for employees."
“A speech delivered without preparation” → Superannuation refers to a pension scheme or
Refers to something spontaneous, unrehearsed, retirement fund system where employees and
or extemporaneous. employers make regular contributions during the
Rebuttal working years to provide financial support during
A counter-argument or response to criticism. retirement. It's commonly used in countries like
Monologue Australia and New Zealand. The other options
A long speech by one person (often in drama or don't accurately define this specific financial term.
performance.
Soliloquy means a speech in drama where a
character speaks thoughts aloud, typically while Solution 24: The correct answer is " Acronym."
alone, to reveal inner feelings to the audience. An acronym is a word formed from the initial
letters of a name or phrase, such as "NASA"
(National Aeronautics and Space Administration)
Solution 19: ✅ Correct Answer: Refugee or "FBI" (Federal Bureau of Investigation). The
Why “Refugee” is Correct : other options have different meanings: Homonym
A refugee is someone who has been displaced (words that sound alike but have different
from their homeland due to threats to life or meanings), Antonym (words with opposite
freedom . meanings), and Pseudonym (a fictitious name
used by an author).

Solution 20: ✅ Correct Answer: Interment


🔍 Word Analysis: Interment Solution 25: The correct answer is " Garrulous."
Meaning : Interment refers specifically to the act A garrulous person is someone who talks
Telegram (Previous year papers PDFs [SSC,Railway,DSSSB,UP SI]): https://t.me/RBE_S
YouTube (Free lectures and job updates): https://www.youtube.com/c/RBERevolutionByEducation
Download RBE application for SSC Exams Complete Preparation
excessively, often in a rambling or chatty manner. crimes. "Clemency" is mercy or leniency,
The other options are incorrect: "Silent" means "absolution" is forgiveness in a religious sense,
quiet or not speaking, "Reticent" means reserved and "reprieve" means a delay or cancellation of
or reluctant to speak, and "Introvert" refers to punishment. Only "amnesty" fits the definition of
someone who prefers solitude but doesn't an official pardon for offenders.
necessarily relate to excessive talking.

Solution 32: "Anecdote" is a brief, interesting, or


Solution 26: The term "anarchy" refers to a amusing story about a real incident or person.
situation characterized by political disorder due to The other options do not fit the definition:
the absence of government or authority. Unlike "chronicle" is a historical account, "essay" is a
monarchy (rule by one), plutocracy (rule by the written composition on a subject, and "epistle" is
wealthy), or oligarchy (rule by a few), "anarchy" a letter.
specifically means a lack of governing control and
resulting chaos or disorder.
Solution 33: The term "fratricide" specifically
refers to the act of killing one's brother.
Solution 27: A person who writes books is called "Homicide" means the killing of any person,
an "author." Unlike an editor (who refines "suicide" means killing oneself, and "patricide"
content), a publisher (who prints/distributes), or means killing one's father. Only "fratricide" fits the
an orator (who speaks publicly), an author is the definition in the question.
actual creator and writer of books.

Solution 34: The term "widower" specifically


Solution 28: "Regicide" specifically means the act refers to a man whose wife has died. "Bachelor"
of killing a king. "Homicide" means the killing of a is a man who has never married, "divorcee" is
human, "patricide" is the killing of one's father, someone who is divorced, and "heir" is someone
and "fratricide" is the killing of one's brother. entitled to inherit property or a title. Only
Therefore, "regicide" is the only word that "widower" matches the definition given in the
accurately fits the given definition. question.

Solution 29: An "elegy" is a poem written Solution 35: A "totem" is a symbol or object that
specifically to mourn and lament someone’s represents a group of people or tribe, serving as
death or express sorrow for loss. The other their emblem or spiritual symbol. The other
options ("ballad," "sonnet," and "ode") refer to options do not specifically refer to an emblem for
different types of poems with distinct purposes a group of people in this context.
and themes, not specifically associated with
mourning.
Solution 36: ✅ Correct Answer: Epitaph
🧠 Explanation:
Solution 30: The term "accomplice" refers to a The best one-word substitute for “an inscription
person who helps another commit a crime, written on stone in the memory of a deceased
making it the correct substitute for "a partner in person” is epitaph .
crime." "Patriot" refers to someone who loves An epitaph is a short text honoring a deceased
their country, "ally" generally means supporter or person, typically engraved on a tombstone or
friend, and "rival" means competitor or opponent; memorial plaque.
none of these fit the specific context of It often includes a tribute, dates, or a meaningful
partnership in criminal activity. quote.
Memoir → A written account of someone's life,
usually by the person themselves.
Solution 31: "Amnesty" refers specifically to an
Epigraph → A quote or phrase at the beginning
official pardon given by a government to people
of a book or chapter, not related to memorials.
who have committed offenses, especially political
Telegram (Previous year papers PDFs [SSC,Railway,DSSSB,UP SI]): https://t.me/RBE_S
YouTube (Free lectures and job updates): https://www.youtube.com/c/RBERevolutionByEducation
Download RBE application for SSC Exams Complete Preparation
Autograph → A person’s signature, not a Solution 41: ✅ Correct Answer: Fastidious
commemorative inscription. 🧠 Explanation:
Given phrase: A person who is extremely
selective or difficult to please 🔍 The most
Solution 37: ✅ Correct Answer: Utopia
accurate one-word substitution is Fastidious ,
🧠 Explanation:
which describes someone who:
The phrase “an imaginary ideal place of
Pays excessive attention to detail
perfection” refers to a concept where everything
Is hard to satisfy
is perfect—socially, politically, and morally. This
Often has high standards or is very particular
aligns exactly with the term:
✅ Example: ➡ She’s so fastidious about her
➡ Utopia — an imagined place or state of things
workspace that even a misplaced pen bothers
in which everything is perfect.
her.

Solution 38: ✅ Correct Answer: Vagabond


Solution 42: ✅ Correct Answer: Option 3 –
🧠 Explanation:
Morgue
Given Definition: ➡ A person without a settled 🔍 Explanation:
home Definition: A morgue is a place where dead
Correct One-Word Substitution: bodies are kept temporarily , especially before
➡ Vagabond — someone who wanders from identification, autopsy, or burial.
place to place without a permanent residence.

Solution 43: ✅ Correct Answer: Option 1 –


Solution 39: ✅ Correct Answer: Sojourn Embezzlement
🧠 Explanation: 🔍 Explanation:
Given phrase: A short stay at a place 🔍 The Misappropriation of money refers to the
word that best substitutes this meaning is wrongful or dishonest use of funds , especially
“Sojourn” , which refers to a temporary stay when someone entrusted with money uses it for
somewhere. personal gain.
✅ Example: ➡ During his sojourn in Paris, he The precise one-word substitution for this is
visited many art galleries. Embezzlement .

Solution 40: ✅ Correct Answer: Veteran Solution 44: ✅ Correct Answer: Option 3 –
🧠 Explanation: Apostate
Given phrase: A person who has long 🔍 Explanation:
experience in a field 🔍 The most suitable one- Definition: An apostate is someone who
word substitution is Veteran , which refers to renounces or abandons their religious faith,
someone who is highly experienced and beliefs, or principles , especially after previously
seasoned , especially through long-term adhering to them.
involvement.
✅ Example: ➡ He’s a veteran in the field of
Solution 45: ✅ Correct Answer: Option 2 –
journalism.
Matricide

Telegram (Previous year papers PDFs [SSC,Railway,DSSSB,UP SI]): https://t.me/RBE_S


YouTube (Free lectures and job updates): https://www.youtube.com/c/RBERevolutionByEducation
Download RBE application for SSC Exams Complete Preparation

SSC Steno 2025 T-1 Spellings All 42 Questions with Detailed Solution and Answer key

Q1. Choose the correctly spelt word: (c) odessy (d) oddysey
(a) Surveillance (b) Survilance
(c) Survailances (d) Servillance
Q9. Find the misspelt word.
(a) intelligent (b) Magnanimos
Q2. Which complex term is misspelt below? (c) influential (d) Magnanimous
(a) Epitaph (b) Ino c c u ous
(c) Circumlocution (d) Pseudonym
Q10. Choose the word that is not spelled
correctly.
Q3. Select the correct spelling of a word meaning (a) Maintenance (b) Colleague
'fond of company; sociable'. (c) Exagerate (d) Leisure
(a) Gregareous (b) Grigariou s
(c) Gregerious (d) Gregarious
Q11. Identify the misspelt word:
(a) Co noisseur (b) Absence
Q4. Spot the spelling error (c) Acquire (d) Acknowledge
(a) Quin tessential (b) Archaetype
(c) Resurgence (d) Misanthrope
Q12. Choose the correctly spelt word:
(a) Occational (b) Occasionale
Q5. Choose the correctly spelt word: (c) Occasional (d) Occassional
(a) Mishevious (b) Mischievious
(c) Misch ievous (d) Mischeivous
Q13. Choose the option that completes the
sentence using the correct spelling.
Q6. Which sentence contains a word that is After 35 years of service, Mr. Das finally retired
spelled incorrectly? and began receiving his ______ benefits every
(a) I t's important to maintain hygenic practices to month.
prevent illness. (a) superanualation (b) superanuation
(b) The scientist explained the hypothesis behind (c) superanuation (d) superannuation
the experiment clearly.
(c) The therapist used hypnosis to help the
patient recall past memories. Q14. Select the misspelt word from the list.
(d) The new hybrid model of the car is both fuel - (a) Camouflage (b) Reservoir
efficient and stylish. (c) Manoeuvre (d) Liason

Q7. Identity the word that Is not correctly spelled. Q15. Find the misspelt word.
(a) Discrepancy (b) Aberration (a) lightning (b) Perspicacious
(c) insurmountable (d) Surveliance (c) Perspicasiuos (d) loneliness

Q8. Select the correct spelling of a word meaning Q16. Choose the misspelt word in this list.
"a long adventurous journey. (a) Iridescant (b) Coercion
(a) odessay (b) odyssey (c) Bureaucracy (d) Aesthetics
Telegram (Previous year papers PDFs [SSC,Railway,DSSSB,UP SI]): https://t.me/RBE_S
YouTube (Free lectures and job updates): https://www.youtube.com/c/RBERevolutionByEducation
Download RBE application for SSC Exams Complete Preparation
(a) Entrepreneur (b) Entropreneur
(c) Enterprenuer (d) Entreprenure
Q17. Choose the correctly spelt word:
(a) Reconeile (b) Reconsile
(c) Reconcil (d) Reconcile Q26. Which sentence contains a word that is
spelled incorrectly?
(a) She carefully placed the vase on the shelf .
Q18. Which spelling is correct for a leadership (b) The soldier jumped out of the plane using a
role? parachute .
(a) Luitenant (b) Lieutenant (c) The students gathered together in the
(c) Leuitenant (d) Lieutanant assembly hall.
(d) The actors had a final rehearsel before the
big show.
Q19. Pick the spelling that is correct for an
educational qualification.
(a) Dipluma (b) Diploma Q27. Spot the misspelt word.
(c) Diplumma (d) Dipploma (a) Acheived (b) Accessible
(c) Accommodate (d) Committee
Q20. Choose the correctly spelt word:
(a) Questionaire (b) Questionnare Q28. Find the misspelt word.
(c) Questionnaire (d) Questionnair (a) Belligerent (b) Beligerant
(c) excellent (d) existence
Q21. Find the word with a spelling mistake.
(a) Preposterous (b) Paraphernalia Q29. Select the correct spelling of a word related
(c) Ephemeral (d) Alterior to bees.
(a) Apiary (b) Apairy
(c) Apeary (d) Apiery
Q22. Find the misspelt word.
(a) Prepostrous (b) legitimate
(c) Preposterous (d) kneel Q30. Which of these words is spelled wrongly?
(a) Idiosyncracy (b) Resplendent
(c) Vernacular (d) Dichotomy
Q23. Which sentence contains a word that is
spelled incorrectly?
(a) She is studying zoology at the university to Q31. Choose the correctly spelt word:
become a biologist. (a) Suppercede (b) Superc e ede
(b) The scientist wrote a paper on marine biology (c) Supirsede (d) Supersede
and its ecosystems.
(c) He is passionate about anthropology and
human cultures. Q32. Find the misspelt word.
(d) The weather forecast mentioned a change in (a) Idiosyncrasy (b) incredible
meterological patterns. (c) Idiosyncracy (d) guidance

Q24. Choose the option that completes the Q33. Choose the correct spelling for a word
sentence using the correct spelling. meaning 'not important'.
The newly fallen snow _______ transformed the (a) Triveel (b) Trivial
landscape. (c) Triviel (d) Triveal
(a) magically (b) magicaly
(c) majicelly (d) magickelly
Q34. Identify the incorrect spelling.
(a) Insatiable (b) Malfeasance
Q25. Choose the correctly spelt word: (c) Perserverance (d) Magnanimous
Telegram (Previous year papers PDFs [SSC,Railway,DSSSB,UP SI]): https://t.me/RBE_S
YouTube (Free lectures and job updates): https://www.youtube.com/c/RBERevolutionByEducation
Download RBE application for SSC Exams Complete Preparation
Q40. Which sentence contains a word that is
spelled incorrectly?
Q35. Choose the correctly spelt word: (a) The audience sat still, completely hypnotized
(a) Magnanimous (b) Magnanemous by the performance.
(c) Magnanamamous (d) Magnanimus (b) He explained the experiment in a way that
was both clear and engaging.
(c) She was absolutely intrigued by the
Q36. Find the misspelt word.
magician's clever tricks.
(a) Garrulous (b) guarantee
(d) The children were fasinated by the glowing
(c) Garulous (d) grateful
fireflies in the dark.

Q37. Choose the correctly spelt word:


Q41. Which sentence contains a word that is
(a) Imbellish (b) Embillish
spelled incorrectly?
(c) Embellish (d) Embelish
(a) The view from the mountaintop was
tramendous .
Q38. Which word is spelled wrongly? (b) She paid the tuition fee before the semester
(a) Permanent (b) Harass began.
(c) Calender (d) Government (c) They enjoyed a peaceful evening by the lake.
(d) His performance in the play was just
mediocre.
Q39. Choose the option that completes the
sentence using the correct spelling.
It is my _______ that municipal employees Q42. Find the misspelt word.
handle their jobs with great professionalism. (a) Ubiquitous (b) interrupt
(a) belief (b) beleaf (c) interference (d) Ubiquitus
(c) biliief (d) beleif

Answer Key:

Q1 a Q2 b Q3 d Q4 b Q5 c
Q6 a Q7 d Q8 b Q9 b Q10 c
Q11 a Q12 c Q13 d Q14 d Q15 c
Q16 a Q17 d Q18 b Q19 b Q20 c
Q21 d Q22 a Q23 d Q24 a Q25 a
Q26 d Q27 a Q28 b Q29 a Q30 a
Q31 d Q32 c Q33 b Q34 c Q35 a
Q36 c Q37 c Q38 c Q39 a Q40 d
Q41 a Q42 d

Solution 1: Explanation The misspelling with the extra 'c' is incorrect.


"Surveillance" means the careful watching or The other words "Epitaph," "Circumlocution," and
monitoring of a person, place, or activity, "Pseudonym" are spelled correctly.
especially by authorities like the police.
The other options such as "Survilance,"
"Survillance," and "Servillance" are incorrect Solution 3: Explanation
spellings. "Gregarious" means sociable or fond of the
company of others.
The other spellings—Gregareous, Grigariou s,
Solution 2: Explanation and Gregerious—are incorrect.
"Innocuous" means harmless or not likely to
cause harm or offense.
Telegram (Previous year papers PDFs [SSC,Railway,DSSSB,UP SI]): https://t.me/RBE_S
YouTube (Free lectures and job updates): https://www.youtube.com/c/RBERevolutionByEducation
Download RBE application for SSC Exams Complete Preparation
Solution 4: Explanation: Solution 11: ✅ Misspelt Word : Conoisseur
"Quintessential," "Resurgence," and 📌 Correct Spelling : Connoisseur
"Misanthrope" are spelled correctly. Memory Tip:
The incorrect option "Archaetype" is a misspelling Connoisseur = someone with refined taste or
of "Archetype," which means an original model or expertise
typical example. Think: “A connoisseur needs two Ns to know!”
(Double "n" + double "s" = refined spelling for
refined taste)
Solution 5: Explanation
"Mischievous" is the standard and accepted
spelling of the word. Solution 12: ✅ Correct Answer: Occasional
Other options like " Mishevious ," " Mischievious
🔍 Explanation:
," and " Mischeivous " are common misspellings.
Correct Spelling : Occasional is the correct
"Mischievous" has three syllables: mis- chie -
spelling, meaning something that happens from
vous , with the stress on the first syllable.
time to time or irregularly.
Therefore, the right choice is Mischievous.

Solution 13: ✅ Correct Answer:


Solution 6: Explanation:
"Hygienic" means conducive to maintaining health superannuation
and preventing disease. 🔍 Explanation:
" Hygenic " is a common misspelling and is not a Meaning of the Sentence : Mr. Das retired and
standard word. started receiving monthly benefits — this clearly
refers to retirement-related financial support ,
commonly known as superannuation .
Solution 7: The correct spelling is Surveillance.
The other words "Discrepancy," "Aberration," and
"Insurmountable" are correctly spelled. Solution 14: ✅ Correct Answer: Liason
🔍 Explanation:
Correct Spelling :
Solution 8: The correct spelling of the word The correct word is Liaison — meaning
meaning "a long adventurous journey" is: communication or cooperation between people or
odyssey organizations .
The other options are misspellings. Common misspelling: Liason (missing the
Therefore, the correct choice is odyssey. second “ i ”)

Solution 9: ✅ Correct Answer: Magnanimos Solution 15: ✅ Correct Answer: Perspicasiuos


“ Magnanimos ” is a misspelling of the correct 🔍 Explanation:
word “Magnanimous” , which means generous,
Correct Spelling :
noble-hearted, or forgiving , especially toward a
The correct word is Perspicacious — meaning
rival or someone less powerful.
having keen insight or understanding .
The correct spelling includes “magnanimous”
with “- mous ” at the end, not “-mos.”
All other options— intelligent , influential , and Solution 16: ✅ Correct Answer: Iridescant
magnanimous —are correctly spelled. The correct spelling is iridescent , derived from
“iris” (rainbow).
The suffix “-escent” indicates a quality of glowing
Solution 10: ✅ Incorrectly Spelled Word :
or shimmering.
Exagerate
“Iridescant” is a common misspelling due to
📌 Correct Spelling : Exaggerate
phonetic confusion.

Telegram (Previous year papers PDFs [SSC,Railway,DSSSB,UP SI]): https://t.me/RBE_S


YouTube (Free lectures and job updates): https://www.youtube.com/c/RBERevolutionByEducation
Download RBE application for SSC Exams Complete Preparation
Solution 17: ✅ Correct Answer: Reconcile (conforming to the law), "Preposterous" (the
🔍 Step-by-Step Explanation: correct spelling), and "kneel" (to go down on
Meaning of “Reconcile” : one's knee).
To restore friendly relations between people or
groups.
Solution 23: The spelling " meterological " is
Also used to mean settling differences or making
incorrect; the correct spelling is "meteorological."
two ideas consistent.
Spelling Logic :
Root: “re-” (again) + “concile” (from Latin Solution 24: The correct spelling is "magically."
concilium , meaning council or meeting). The other options are misspelled attempts at the
Correct spelling: Reconcile → with “cile” at the word. "Magically transformed" is the
end. grammatically correct and properly spelled phrase
for the sentence.

Solution 18: ✅ Correct Answer: Lieutenant


🔍 Word Meaning: Solution 25: The correct spelling is
Lieutenant : A leadership rank in the military or "Entrepreneur." All other options are misspellings.
police, often just below captain. "Entrepreneur" refers to a person who starts and
manages a business, taking on financial risks to
do so. Proper spelling is crucial for clarity and
Solution 19: The correct answer is " Diploma ." professional communication.
This is the standard and correct spelling for an
educational certificate or qualification awarded by
an educational institution. The other options are Solution 26: The incorrectly spelled word is "
incorrect spellings with various letter errors - " rehearsel ." The correct spelling is "rehearsal." All
Dipluma ," " Diplumma ," and " Dipploma " all other words in the options ("shelf," "parachute,"
contain spelling mistakes. "together") are correctly spelled. Identifying
spelling errors is vital for clear written
communication.
Solution 20: The correct answer is
"Questionnaire ." This is the proper spelling of a
set of questions used for gathering information or Solution 27: " Acheived " is incorrectly spelled;
conducting surveys. The word contains a double the correct spelling is "achieved." The other
"n" and ends with "- aire ." The other options options ("accessible," "accommodate,"
contain various spelling errors: missing letters, "committee") are correctly spelled and commonly
incorrect letter combinations, or wrong endings. used in written English. Proper spelling is
important for clarity and professional
communication.
Solution 21: The correct answer is " Alterior ."
This word is misspelled - the correct spelling is
"Ulterior" (as in "ulterior motive"). The other words Solution 28: " Beligerant " is a misspelling; the
are spelled correctly: "Preposterous" (extremely correct spelling is "belligerent," meaning hostile or
unreasonable), "Paraphernalia" (miscellaneous aggressive. The other words ("excellent,"
equipment), and "Ephemeral" (lasting for a short "existence," "belligerent") are spelled correctly
time). and are standard in English usage. Proper
spelling is essential for clear communication.

Solution 22: The correct answer is "


Prepostrous ." This word is misspelled - it should Solution 29: The correct spelling is "apiary." An
be "Preposterous" (meaning completely contrary apiary is a place where bees are kept, typically
to reason or common sense; absurd). The word is consisting of a collection of hives. The other
missing the letter "e" after "p" and before "r". The options feature incorrect spellings and do not
other words are spelled correctly: "legitimate"
Telegram (Previous year papers PDFs [SSC,Railway,DSSSB,UP SI]): https://t.me/RBE_S
YouTube (Free lectures and job updates): https://www.youtube.com/c/RBERevolutionByEducation
Download RBE application for SSC Exams Complete Preparation
correspond to any recognized term related to Misspelt Word: Garulous 🔍 The correct spelling
bees. is Garrulous , which means excessively talkative,
especially on trivial matters .

Solution 30: The word " Idiosyncracy " is


incorrectly spelled. The correct spelling is Solution 37: Correct Answer: ✅ Embellish
"Idiosyncrasy," meaning a distinctive or peculiar 🔍 Explanation:
feature or characteristic of a person or thing. The
The word “Embellish” means to decorate or
other options—"Resplendent," "Vernacular," and
enhance something by adding details or features .
"Dichotomy"—are all spelled correctly.
It is correctly spelt with:
Prefix: em -
Solution 31: The correct spelling is "supersede," Root: bell (from Latin bellus , meaning beautiful)
which means to replace or take the place of Suffix: - ish
something previously in use. The other options
are misspellings and are not recognized in
Solution 38: Correct Answer: ✅ Option 3 –
English usage.
Calender
🔍 Explanation:
Solution 32: The misspelt word is " Idiosyncracy The correct spelling is Calendar , not Calender .
." The correct spelling is "Idiosyncrasy," which A calendar refers to a system for organizing
means a peculiar or distinctive trait. The other days, months, and years.
words ("Idiosyncrasy," "incredible," and Calender (with an "e") is a rare technical term
"guidance") are correctly spelled. used in printing or textile industries, meaning a
machine that smooths paper or fabric — but that’s
not the intended meaning here.
Solution 33: ✅ Correct Answer: Trivial
📘 Explanation:
Word Meaning: ➡ Trivial means not important , Solution 39: ✅ Correct Answer: Option 1 –
insignificant , or minor . belief
The correct word here is belief , which means a
conviction or opinion held by someone.
Solution 34: ✅ Correct Answer:
Perserverance
Solution 40: ✅ Correct Answer: Option 4 – The
🔍 Explanation:
children were fasinated by the glowing
The word “ Perserverance ” is incorrectly
fireflies in the dark.
spelled. The correct spelling is “Perseverance”
🔍 Explanation:
— which means continued effort despite
difficulties . The word “ fasinated ” is spelled incorrectly .
The correct spelling is “fascinated” — with a “c”
after the “s”.
Solution 35: ✅ Correct Answer: Magnanimous
🧠 Explanation:
Solution 41: ✅ Correct Answer: Option 1 –
Magnanimous is the correct spelling. 🔹 It
“The view from the mountaintop was
means generous, noble, and forgiving , especially
tramendous .”
toward someone less powerful or who has made
🔍 Explanation:
a mistake.
The word “ tramendous ” is a misspelling of
✅ Example: ➡ Despite the insult, she remained
the correct word “tremendous.”
magnanimous and chose to forgive.

Solution 42: ✅ Correct Answer: Option 4 –


Solution 36: ✅ Correct Answer: Garulous
Ubiquitus
🧠 Explanation:
✔ Ubiquitous ❌ Ubiquitus
Telegram (Previous year papers PDFs [SSC,Railway,DSSSB,UP SI]): https://t.me/RBE_S
YouTube (Free lectures and job updates): https://www.youtube.com/c/RBERevolutionByEducation
Download RBE application for SSC Exams Complete Preparation

SSC Steno 2025 T-1 Synonyms All 43 Questions with Detailed Solution and Answer key

Q1. Select the correct Synonym of Q10. Select the synonym of Cacophony .
“EXASPERATE” . (a) Order (b) Discord
(a) Distract (b) Annoy (c) Harmony (d) Silence
(c) Calm (d) Inspire

Q11. Select the synonym of Loathe.


Q2. Select the correct synonym of 'ENERVATE'. (a) Love (b) Like
(a) Motivate (b) Strengthen (c) Accept (d) Hate
(c) Exhaust (d) R evive

Q12. Select the synonym of Abstruse.


Q3. Find the synonym of 'Encumbrance'. (a) Clear (b) Obvious
(a) Disinterest (b) Ignore (c) Simple (d) Difficult
(c) Offend (d) Obstacle

Q13. Select the synonym of Deteriorate.


Q4. Select the synonym of Diligent (a) Worsen (b) Grow
(a) Indolent (b) Hardworking (c) Revive (d) Improve
(c) Careless (d) Idle

Q14. Select the correct synonym for 'Abate'


Q5. Synonym of RECKLESS (a) Honest (b) Moderate
(a) Prudent (b) Careful (c) Evil (d) Increase
(c) Rash (d) Cautious

Q15. Select the synonym of Defer.


Q6. Select the correct Synonym of (a) Reject (b) Decline
"OBLITERATE' . (c) Proceed (d) Delay
(a) Alter (b) C reate
(c) Hide (d) Destroy
Q16. Select the synonym of Summon.
(a) Drive (b) Call
Q7. Select the synonym of Obsequious : (c) Send (d) Disperse
(a) Submissive (b) Defiant
(c) Aggressive (d) Proud
Q17. Select the synonym of Conundrum.
(a) Triumph (b) Harmony
Q8. Select the synonym of Pinnacle . (c) Problem (d) Celebration
(a) Decline (b) Base
(c) Bottom (d) Peak
Q18. What is the meaning of the word
"spendthrift"?
Q9. Synonym of Vigilant : (a) A person who spends money recklessly
(a) Alert (b) Inattentive (b) A person who saves money
(c) Careless (d) Sleepy (c) A person who borrows money
(d) A person who invests money wisely
Telegram (Previous year papers PDFs [SSC,Railway,DSSSB,UP SI]): https://t.me/RBE_S
YouTube (Free lectures and job updates): https://www.youtube.com/c/RBERevolutionByEducation
Download RBE application for SSC Exams Complete Preparation

Q19. Select the synonym of Alacrity. Q30. Select the synonym of Exquisite.
(a) Eagerness (b) Delay (a) Crude (b) Harsh
(c) Hesitation (d) Sluggishness (c) Ordinary (d) Delicate

Q20. Choose the correct synonym of 'Juvenile' : Q31. Select the synonym of Taciturn .
(a) Grown-up (b) Childish (a) Talkative (b) Noisy
(c) Ancient (d) Elder (c) Reserved (d) Bold

Q21. Select the synonym of Reticent. Q32. Select the synonym of Candid.
(a) Arrogant (b) Outspoken (a) Dubious (b) Frank
(c) Reserved (d) Talkative (c) Secretive (d) Insincere

Q22. Select the synonym of F oster. Q33. Pick the synonym of 'Frivolous'.
(a) Promote (b) Neglect (a) Significant (b) Powerful
(c) Oppose (d) Reject (c) Trivial (d) Essential

Q23. Select the synonym of R elinquish . Q34. Select the synonym of Vicinity.
(a) Continue (b) Abandon (a) Proximity (b) Absence
(c) Retain (d) Maintain (c) Departure (d) Distance

Q24. Select the synonym of Deft. Q35. Select the synonym of Amiable.
(a) Awkward (b) Inept (a) Rude (b) Friendly
(c) Clumsy (d) Skillful (c) Hostile (d) Cold

Q25. Choose the synonym of the word 'Bawdy'. Q36. Select the synonym of Insolent.
(a) Support (b) Coarse (a) Polite (b) Gentle
(c) Establish (d) Delay (c) Courteous (d) Disrespectful

Q26. Select the synonym of ‘Perfunctory’ Q37. Select the synonym of Esoteric.
(a) Painstaking (b) Diligent (a) Clear (b) Mysterious
(c) Careless (d) Detailed (c) Popular (d) Common

Q38. Which word is closest in meaning to


Q27. Select the synonym of Gratify. 'Castigate'?
(a) Insult (b) Displease (a) Dull (b) Vague
(c) Satisfy (d) Annoy (c) Dark (d) Condemn

Q28. Select the synonym of 'Carnal'. Q39. Select the synonym of Recalcitrant.
(a) Earthly (b) Demand (a) Supportive (b) Loyal
(c) Jealousy (d) Greed (c) Disobedient (d) Submissive

Q29. Select the synonym of Exaggerate . Q40. Which word is closest in meaning to ‘Allay”?
(a) Suppress (b) Minimize (a) Pacify (b) Clash
(c) Understate (d) Amplify (c) Noise (d) Dispute
Telegram (Previous year papers PDFs [SSC,Railway,DSSSB,UP SI]): https://t.me/RBE_S
YouTube (Free lectures and job updates): https://www.youtube.com/c/RBERevolutionByEducation
Download RBE application for SSC Exams Complete Preparation
(a) persistent (b) occasional
(c) Temporary (d) Fresh
Q41. Select the synonym of Feeble.
(a) Strong (b) Robust
(c) Sturdy (d) Weak Q43. Select the synonym of Ubiquitous.
(a) Rare (b) Isolated
(c) Occasional (d) Omnipresent
Q42. Select the synonym of Chronic.

Answer Key:

Q1 b Q2 c Q3 d Q4 b Q5 c
Q6 d Q7 a Q8 d Q9 a Q10 b
Q11 d Q12 d Q13 a Q14 b Q15 d
Q16 b Q17 c Q18 a Q19 a Q20 b
Q21 c Q22 a Q23 b Q24 d Q25 b
Q26 c Q27 c Q28 a Q29 d Q30 d
Q31 c Q32 b Q33 c Q34 a Q35 b
Q36 d Q37 b Q38 d Q39 c Q40 a
Q41 d Q42 a Q43 d

Solution 1: Explanation Solution 4: Explanation


"Exasperate" means to irritate or annoy someone "Diligent" means showing care and
intensely. conscientiousness in one's work or duties.
Other options: "Hardworking" similarly describes someone who
Distract: to divert attention. puts a lot of effort and dedication into work.
Calm: to soothe or make peaceful (opposite Other options like "Indolent," "Careless," and
meaning). "Idle" are antonyms or unrelated in meaning to
Inspire: to motivate or encourage (different "diligent".
meaning)

Solution 5: Explanation
Solution 2: Explanation "Reckless" means acting without care or concern
"Enervate" means to weaken, tire out, or drain of for consequences.
energy. "Rash" means acting without thinking or
Synonyms include exhaust, weaken, tire, considering the consequences, which is very
debilitate, and sap energy. close in meaning to "reckless."
The other options "Motivate," "Strengthen," and Other options like "Prudent," "Careful," and
"Revive" have opposite or different meanings, "Cautious" are antonyms, meaning the opposite
relating to increasing energy or encouraging of reckless.
rather than weakening.

Solution 6: Explanation:
Solution 3: Explanation "Obliterate" means to remove completely, to
"Encumbrance" means a burden or something destroy or erase all traces of something.
that hinders or obstructs progress. Among the options, "Destroy" best matches this
"Obstacle" is a close synonym as it means a meaning.
barrier or something that blocks or impedes Other options like "Alter," "Create," and "Hide" do
progress. not convey the meaning of complete destruction
The other options like "Disinterest," "Ignore," and or removal.
"Offend" do not align with the meaning of
encumbrance.
Solution 7: Explanation:
"Obsequious" means showing excessive
Telegram (Previous year papers PDFs [SSC,Railway,DSSSB,UP SI]): https://t.me/RBE_S
YouTube (Free lectures and job updates): https://www.youtube.com/c/RBERevolutionByEducation
Download RBE application for SSC Exams Complete Preparation
willingness to serve or please others; overly Solution 13: ✅ Correct Answer : Worsen
obedient or attentive. Deteriorate → To become worse or decline in
"Submissive" closely matches this meaning. condition.
Other options like "Defiant," "Aggressive," and Other options are antonyms of Deteriorate.
"Proud" have opposite or unrelated meanings.
Therefore, the correct choice is Submissive.
Solution 14: ✅ Correct Answer: Moderate
🔍 Explanation:
Solution 8: Explanation:
Meaning of 'Abate' : Abate means to reduce in
"Pinnacle" means the highest point or peak.
intensity, amount, or severity .
Among the options provided, "Peak" is the most
Correct Synonym – Moderate :
suitable synonym.
Moderate means to lessen or reduce the force
Other options like "Decline," "Base," and "Bottom"
or intensity of something , which matches the
are antonyms or unrelated.
meaning of abate .
Therefore, the correct choice is Peak.

Solution 15: ✅ Correct Answer: Delay


Solution 9: Explanation:
🔸 Meaning:
"Vigilant" means being watchful and alert to
potential danger or problems. Defer means to postpone or delay something to
"Alert" similarly means being attentive and ready a later time .
to notice things. It can also mean to yield respectfully to
The other options "Inattentive," "Careless," and someone’s opinion or authority , but in this
"Sleepy" are opposites or unrelated. context, we’re focusing on the time-related
Therefore, the correct choice is Alert. meaning .

Solution 10: Explanation: Solution 16: ✅ Correct Answer: Call


"Cacophony" means a harsh, discordant mixture 🔍 Word Analysis: Summon
of sounds or noise. Meaning : To formally or urgently ask someone
"Discord" similarly means lack of harmony or to come or be present.
agreement, often used in the context of sounds or Common contexts:
situations. Summon a witness
The other options "Order," "Harmony," and Summon courage
"Silence" are opposites or unrelated meanings. Summon the doctor
Therefore, the correct choice is Discord.

Solution 17: ✅ Correct Answer: Problem


Solution 11: ✅ Correct Answer: Hate 🔍 Word Analysis: Conundrum
The word "Loathe" means to feel intense dislike Meaning : A confusing or difficult problem or
or disgust for something or someone. Its closest question, often puzzling in nature.
synonym is "Hate" , which also expresses strong Common usage:
aversion. The committee faced a conundrum over budget
allocation.
It’s a moral conundrum with no easy answer.
Solution 12: ✅ Correct Answer: Difficult
Abstruse means something that is hard to
understand , complex , or obscure . Solution 18: ✅ Correct Answer: A person who
Example: The professor’s theory was too spends money recklessly
abstruse for most students. 🔍 Word Analysis: Spendthrift
The closest synonym is Difficult , which conveys Meaning : A spendthrift is someone who wastes
the idea of challenging to comprehend . money carelessly , often without thinking about
consequences or budgeting.
Telegram (Previous year papers PDFs [SSC,Railway,DSSSB,UP SI]): https://t.me/RBE_S
YouTube (Free lectures and job updates): https://www.youtube.com/c/RBERevolutionByEducation
Download RBE application for SSC Exams Complete Preparation
It’s not a compliment—it implies financial keeping or holding onto something, which are
irresponsibility. opposite in meaning to "relinquish."
Etymology Note : Interestingly, spendthrift
combines spend + thrift , but ironically means the
opposite of being thrifty. Solution 24: The correct answer is " Skillful."
"Deft" means showing great skill, especially
with the hands; quick and neat in movement
Solution 19: ✅ Correct Answer: Eagerness or action . "Skillful" has the same meaning,
🔍 Word Analysis: Alacrity referring to having or showing skill and expertise.
Meaning : Alacrity refers to cheerful readiness , The other options—"awkward," "inept," and
promptness , or eager willingness to do "clumsy"—are actually antonyms of "deft,"
something. describing lack of skill or grace.
It conveys enthusiasm + speed in response or
action.
Solution 25: The correct answer is "Coarse ."
Tone : Positive and energetic—often used to
"Bawdy" means indecent, vulgar, or crude,
describe someone who responds quickly and
especially in a humorous or sexual context.
enthusiastically.
"Coarse" is a synonym as it also means rough,
crude, vulgar, or lacking in refinement. The other
Solution 20: ✅ Correct Answer: Childish options—"support," "establish," and "delay"—
have completely different meanings and are not
🔍 Word Analysis: Juvenile
related to the concept of being vulgar or indecent.
Meaning : Juvenile refers to someone young or
immature , often used for children or adolescents.
It can also describe behavior that is typical of a Solution 26: The synonym of ‘Perfunctory’ is:
child—hence, childish . Careless
Tone : Neutral to slightly negative when referring Explanation:
to behavior (e.g., juvenile attitude ), but neutral Perfunctory means done without real interest,
when referring to age (e.g., juvenile court ). care, or enthusiasm; hasty and superficial.
Synonyms include careless, cursory, casual.
The other options, painstaking, diligent, and
Solution 21: The correct answer is " Reserved."
detailed, imply thoroughness and care, which are
The word "reticent" describes a person who is
opposites of perfunctory.
quiet or reluctant to speak, and "reserved" has a
Hence, the correct answer is: Careless .
similar meaning, indicating someone who is
restrained or does not share their thoughts or
feelings easily. The other options are antonyms or Solution 27: The synonym of "gratify" is "satisfy."
unrelated in meaning.

Solution 28: The synonym of "carnal" is "earthly,"


Solution 22: The correct answer is " Promote ." which relates to physical or sensual desires.
"Foster" means to encourage the development or
growth of something, which is closely related in
meaning to "promote." The other options— Solution 29: "Exaggerate" means to represent
"neglect," "oppose," and "reject"—are antonyms something as greater than it actually is; "
or unrelated in meaning. amplify" is a synonym because it means to
increase or intensify. "Suppress," "minimize,"
and "understate" are antonyms and do not
Solution 23: The correct answer is " Abandon." express a meaning similar to "exaggerate."
"Relinquish" means to give up, surrender, or
let go of something voluntarily. "Abandon" has
a similar meaning of giving up or deserting Solution 30: The word "exquisite" means
something completely. The other options— extremely beautiful, refined, or delicate. Among
"continue," "retain," and "maintain"—all suggest the options, "delicate" is the closest synonym,
Telegram (Previous year papers PDFs [SSC,Railway,DSSSB,UP SI]): https://t.me/RBE_S
YouTube (Free lectures and job updates): https://www.youtube.com/c/RBERevolutionByEducation
Download RBE application for SSC Exams Complete Preparation
reflecting something fine or beautifully made. Obscure or hard to grasp
"Crude," "harsh," and "ordinary" are antonyms or Often secretive or mysterious in nature
unrelated to the meaning of "exquisite." So, “Mysterious” is the closest synonym among
the given options.

Solution 31: "Taciturn" refers to a person who is


habitually silent or uncommunicative. The word Solution 38: ✅ Correct Answer: Condemn
"reserved" is the closest synonym, describing 🔍 Meaning of “Castigate”:
someone who does not easily show emotions or Castigate means to criticize or reprimand
talk much. The other options ("talkative," "noisy," someone severely , often with the intent to
and "bold") are unrelated or opposite in meaning. correct or punish.

Solution 32: "Candid" means open, honest, and Solution 39: ✅ Correct Answer: Disobedient
straightforward in speech or expression. The
🧠 Word Meaning:
synonym is "Frank," which carries the same
Recalcitrant (adjective): ➡ Stubbornly resisting
sense of openness and honesty. The other
options—"dubious," "secretive," and "insincere"— authority or control; unwilling to obey rules or
represent the opposite or unrelated qualities. orders.

Solution 33: The word "frivolous" refers to Solution 40: ✅ Correct Answer: Option 1 –
something not serious, unimportant, or lacking in Pacify
value. "Trivial" means insignificant or minor, 🔍 Meaning of “Allay”:
making it the most accurate synonym. The other To reduce , calm , or relieve something
options—"significant," "powerful," and unpleasant like fear, pain, or doubt
"essential"—connote importance or influence, Often used in emotional or psychological
which does not match the meaning of "frivolous." contexts: to allay concerns , to allay anxiety

Solution 34: The word "vicinity" means the area Solution 41: ✅ Correct Answer: Option 4 –
near or surrounding a place. "Proximity" is the Weak
correct synonym as it refers to nearness in 🔍 Meaning of “Feeble”:
relation to something. The other options Lacking physical strength or energy
("absence," "departure," and "distance") either Weak in force, intensity, or effectiveness
indicate being far away or unrelated concepts. Often used to describe a person, argument, or
effort that is fragile or ineffective

Solution 35: ✅ Correct Answer: Friendly


📘 Explanation: Solution 42: ✅ Correct Answer: Option 1 –
Word: Amiable Meaning: Pleasant, kind, and persistent
easy to get along with. 🔍 Explanation:
The word “chronic” refers to something that is
long-lasting , recurring , or continuing over
Solution 36: ✅ Correct Answer: Disrespectful
time , especially in the context of diseases,
📘 Explanation: problems, or conditions.
Word: Insolent Meaning: Boldly rude or Synonym: persistent – something that
disrespectful in speech or behavior. continues firmly or obstinately over a prolonged
period.

Solution 37: ✅ Correct Answer: Mysterious


🧠 Explanation: Solution 43: ✅ Correct Answer: Option 4 –
The word “Esoteric” refers to something that is: Omnipresent
Understood by only a small, specialized group 🔍 Explanation:
Telegram (Previous year papers PDFs [SSC,Railway,DSSSB,UP SI]): https://t.me/RBE_S
YouTube (Free lectures and job updates): https://www.youtube.com/c/RBERevolutionByEducation
Download RBE application for SSC Exams Complete Preparation
Word: Ubiquitous Meaning: Existing or being Omnipresent – Present everywhere
everywhere at the same time; widespread. simultaneously; all-pervasive.
✅ Synonym:

For SSC Exams Complete Preparation (Download RBE Application)


(Learn from those who have cleared the exam themselves)
https://play.google.com/store/apps/details?id=com.revolution.education

Telegram (Previous year papers PDFs [SSC,Railway,DSSSB,UP SI]): https://t.me/RBE_S


YouTube (Free lectures and job updates): https://www.youtube.com/c/RBERevolutionByEducation
Download RBE application for SSC Exams Complete Preparation

SSC Steno 2025 T-1 Antonym All 48 Questions with Detailed Solution and Answer key

Q1. Choose the antonym of ‘vindicate’. (c) Merciless (d) Revive


(a) Justify (b) Exonerate
(c) Defend (d) Accuse Q12. Choose the antonym of Commend .
(a) Approve (b) Applaud
Q2. Choose the antonym of “Slander”. (c) Criti cize (d) Praise
(a) Praise (b) Blame
(c) Mock (d) Abuse Q13. Choose the antonym of ‘meticulous’:
(a) Negligent (b) Thorough
Q3. Choose the antonym of ‘veracity: (c) Careful (d) Precise
(a) Honesty (b) Accuracy
(c) Falsehood (d) Truthfulness Q14. Find the antonym of ‘Impute’.
(a) Cheerful (b) Energetic
Q4. Select the antonym of ‘Oracular’. (c) Support (d) Lively
(a) Anxious (b) Impress
(c) Lucid (d) Idealistic Q15. Choose the antonym of ‘imitate’:
(a) Echo (b) Copy
Q5. Choose the antonym of ‘Pompous’ (c) Innovate (d) Replicate
(a) Delight (b) Submissive
(c) Urgency (d) Energy Q16. Choose the antonym of “enhance”:
(a) Enlarge (b) Diminish
Q6. Choose the antonym of “peevish” (c) Augment (d) Increase
(a) Tedious (b) Suave
(c) Dul l (d) Flat Q17. Choose the antonym of ‘lucid’:
(a) Clear (b) Coherent
Q7. Choose the antonym of ‘ magnanimous ’: (c) Vague (d) Transparent
(a) Forgiving (b) Generous
(c) Petty (d) Noble Q18. Choose the antonym of ‘inflate’:
(a) Expand (b) Increase
Q8. Identify the antonym of Quell . (c) Swell (d) Reduce
(a) Agitate (b) calm
(c) suppress (d) Silence Q19. Choose the antonym of ‘dominate’:
(a) Govern (b) Submit
Q9. Choose the antonym of ‘tangible’: (c) Rule (d) Control
(a) Definite (b) Intangible
(c) Real (d) Invisible Q20. Choose the antonym of 'Wan'.
(a) Long (b) Tiring
Q10. Choose the antonym of “affluent”: (c) Healthy (d) Wearisome
(a) Wealthy (b) Prosperous
(c) Destitute (d) Rich Q21. Choose the antonym of ‘benevolent’:
(a) Malevolent (b) Kind
Q11. Find the antonym of Wilt . (c) Generous (d) Humane
(a) Strict (b) Cruel
Q22. Choose the antonym of ‘hostile’:
Telegram (Previous year papers PDFs [SSC,Railway,DSSSB,UP SI]): https://t.me/RBE_S
YouTube (Free lectures and job updates): https://www.youtube.com/c/RBERevolutionByEducation
Download RBE application for SSC Exams Complete Preparation
(a) Rude (b) Bitter (a) Shallow (b) Deep
(c) Aggressive (d) Friendly (c) Surface (d) External

Q23. Choose the antonym of ‘resilient ’: Q36. Choose the antonym of ‘approve’:
(a) Tough (b) Buoyant (a) Endorse (b) Certify
(c) Flexible (d) Fragile (c) Sanction (d) Reject

Q24. Choose the antonym of ‘ mature’ : Q37. Choose the antonym of ‘ascend’:
(a) Experienced (b) Immature (a) Climb (b) Rise
(c) Grown (d) Developed (c) Descend (d) Soar

Q25. Choose the antonym of ‘authorize’: Q38. Select the antonym of 'Arraign'.
(a) Permit (b) Forbid (a) Gluttonous (b) Justify
(c) Empower (d) Allow (c) Selfish (d) Hungry

Q26. Choose the antonym of 'affirm': Q39. Pick the antonym of 'Knave'.
(a) Support (b) Acknowledge (a) Fuzzy (b) Paragon
(c) Accept (d) Refuse (c) Shady (d) Dark

Q27. Choose the antonym of 'prudent': Q40. Choose the antonym of ‘durable’:
(a) Sensible (b) Judicious (a) Sturdy (b) Strong
(c) Cautious (d) Thoughtless (c) Firm (d) Fragile

Q28. Select the antonym of 'Vilify'. Q41. Choose the antonym of 'candid':
(a) Defame (b) Malign (a) Honest (b) Deceitful
(c) Commend (d) Slur (c) Blunt (d) Frank

Q29. Choose the antonym of 'assert': Q42. Choose the antonym of ‘diligent’:
(a) Maintain (b) Deny (a) Lazy (b) Hardworking
(c) Declare (d) Proclaim (c) Meticulous (d) Efficient

Q30. Choose the antonym of ‘eminent’: Q43. Choose the antonym of ‘audacity’:
(a) Celebrated (b) Prominent (a) Modesty (b) Boldness
(c) Distinguished (d) Obscure (c) Bravery (d) Cowardice

Q31. Choose the antonym of ‘commendable’: Q44. Choose the antonym of ‘withdraw’:
(a) Blameworthy (b) Notable (a) Recede (b) Deduct
(c) Admirable (d) Laudable (c) Remove (d) Advance

Q32. Choose the antonym of 'Lax'. Q45. Choose the antonym of ‘prominent’:
(a) Stack (b) Careless (a) Eminent (b) Renowned
(c) Loose (d) Reliable (c) Conspicuous (d) Obscure

Q33. Choose the antonym of ‘liberation’. Q46. Choose the antonym of ‘deliberate’:
(a) Captivity (b) Freedom (a) Calculated (b) Rash
(c) Independence (d) Emancipation (c) Cautious (d) Intentional

Q34. Choose the antonym of ‘scrupulous’. Q47. Choose the antonym of ‘eloquent’:
(a) Principled (b) Ethical (a) Articulate (b) Fluent
(c) Dishonest (d) Careless (c) Persuasive (d) Inexpressive

Q35. Choose the antonym of ‘superficial’. Q48. Pick the antonym of ‘Yoke’.
Telegram (Previous year papers PDFs [SSC,Railway,DSSSB,UP SI]): https://t.me/RBE_S
YouTube (Free lectures and job updates): https://www.youtube.com/c/RBERevolutionByEducation
Download RBE application for SSC Exams Complete Preparation
(a) Liberate (b) Immortal
(c) Harness (d) Vicious

Answer Key:

Q1 d Q2 a Q3 c Q4 c Q5 b
Q6 b Q7 c Q8 a Q9 b Q10 c
Q11 d Q12 c Q13 a Q14 c Q15 c
Q16 b Q17 c Q18 d Q19 b Q20 c
Q21 a Q22 d Q23 d Q24 b Q25 b
Q26 d Q27 d Q28 c Q29 b Q30 d
Q31 a Q32 d Q33 a Q34 c Q35 b
Q36 d Q37 c Q38 b Q39 b Q40 d
Q41 b Q42 a Q43 a Q44 d Q45 d
Q46 b Q47 d Q48 a

Solution 1: Explanation Solution 5: Explanation:


"Vindicate" means to clear someone of blame or "Pompous" means having an exaggerated sense
suspicion. of self-importance, arrogant, or pretentious.
The opposite is to "accuse," meaning to charge Its antonyms relate to humility, modesty, and
someone with a fault or crime. being unassuming.
Other options like "Justify," "Exonerate," and Among the options, "Submissive" best fits as it
"Defend" are synonyms, not antonyms. means yielding or showing humility, opposite to
being pompous.
Solution 2: Explanation Other options like "Delight," "Urgency," and
"Slander" means making false and damaging "Energy" are unrelated in meaning to "pompous".
spoken statements about someone.
The opposite is to speak well of or commend Solution 6: Explanation
someone, hence "Praise" is the correct antonym. "Peevish" means easily irritated or annoyed.
Other options like "Blame," "Mock," and "Abuse" "Suave" means charming, confident, and elegant,
are similar in negative meaning and not which is opposite to being irritable or grumpy.
opposites. Other options like "Tedious," "Dull," and "Flat" do
not convey the opposite meaning of "peevish".
Solution 3: Falsehood
Explanation Solution 7: Explanation:
"Veracity" means truthfulness or accuracy. "Magnanimous" means generous, noble, and
The opposite is "falsehood," which means untruth forgiving.
or falseness. "Petty" means small-minded, spiteful, or
Other options like honesty, accuracy, and ungenerous, which is the opposite of
truthfulness are synonyms or related words, not magnanimous.
antonyms. Other options like "Forgiving," "Generous," and
"Noble" are synonyms or similar in meaning to
Solution 4: Explanation magnanimous.
"Oracular" describes something mysterious, Therefore, the correct choice is Petty.
cryptic, or prophetic.
The antonym of "oracular" is something clear, Solution 8: Explanation:
easily understood, or lucid. "Quell" means to suppress, subdue, or calm
Other options like "Anxious," "Impress," and something, often by force.
"Idealistic" do not convey the opposite meaning of "Agitate" means to stir up, provoke, or incite
"oracular." unrest or excitement, which is the opposite of
"quell."
Other options like "calm," "suppress," and
Telegram (Previous year papers PDFs [SSC,Railway,DSSSB,UP SI]): https://t.me/RBE_S
YouTube (Free lectures and job updates): https://www.youtube.com/c/RBERevolutionByEducation
Download RBE application for SSC Exams Complete Preparation
"silence" are similar in meaning to "quell," not something negative.
opposites. Example: He imputed the failure to poor planning.
The antonym would be a word that suggests
Solution 9: Explanation: defending, supporting, or absolving someone
"Tangible" means something that can be touched instead of blaming.
or physically felt. “Support” fits best as it implies standing by
"Intangible" means something that cannot be someone , helping , or defending them—
touched or physically perceived, like ideas or opposite of blaming.
feelings.
Other options like "definite," "real," and "invisible" Solution 15: ✅ Correct Answer: Innovate
do not accurately represent the opposite of Imitate means to copy or mimic someone or
tangible. something.
Therefore, the correct choice is Intangible. Example: She imitated the teacher’s voice
perfectly.
Solution 10: Explanation: An antonym is a word that expresses the
"Affluent" means wealthy or prosperous. opposite meaning .
"Destitute" means extremely poor, lacking the Innovate means to create something new , to
basic necessities of life, which is the opposite of introduce original ideas —the opposite of
affluent. copying.
Other options like "wealthy," "prosperous," and Therefore, Innovate is the most appropriate
"rich" are synonyms of affluent, not antonyms. antonym.
Therefore, the correct choice is Destitute.
Solution 16: ✅ Correct Answer: Diminish
Solution 11: Explanation: 🔍 Explanation:
"Wilt" means to droop, fade, or lose strength or Meaning of “Enhance” :
vigor . Enhance means to improve, intensify, or
"Revive" means to bring back to life, restore, or increase in value, quality, or attractiveness .
regain strength, making it the opposite of wilt. Common synonyms: boost, augment, elevate,
Other options like "Strict," "Cruel," and "Merciless" magnify
are unrelated in meaning. Antonym Logic :
Therefore, the correct choice is Revive. The opposite of enhancing something is
reducing, weakening, or lowering its impact or
Solution 12: Explanation: quality .
"Commend" means to praise or approve. Hence, diminish = to reduce or make less →
The opposite is to "criticize," which means to perfect antonym
express disapproval or find faults.
Other options like "Approve," "Applaud," and Solution 17: ✅ Correct Answer: Vague
"Praise" are synonyms, not antonyms.
🔸 Meaning:
Therefore, the correct choice is Criticize.
Lucid means clear, easily understood, or
mentally sound .
Solution 13: ✅ Correct Answer: Negligent
Often used to describe speech, writing, or
Meticulous means showing great attention to
thought that is logical and well-articulated.
detail; being very careful and precise.
Vague = lack of clarity , ambiguity, or
Its opposite would be someone who does not
confusion.
pay attention , is careless , or irresponsible .
They represent opposite ends of the clarity
Negligent perfectly captures this opposite
spectrum.
meaning—it refers to someone who fails to take
proper care.
Solution 18: ✅ Correct Answer: Reduce
🔍 Word Analysis: Inflate
Solution 14: ✅ Correct Answer: Support
Meaning : To increase in size, volume, or
“Impute” means to attribute or assign
value—often artificially or excessively. Common
blame/responsibility to someone, often for
Telegram (Previous year papers PDFs [SSC,Railway,DSSSB,UP SI]): https://t.me/RBE_S
YouTube (Free lectures and job updates): https://www.youtube.com/c/RBERevolutionByEducation
Download RBE application for SSC Exams Complete Preparation
contexts: are actually similar or related in meaning to
Inflate a balloon "resilient."
Inflated prices
Inflated ego Solution 24: The correct answer is " Immature."
So, the opposite would be to decrease, deflate, "Mature" means fully developed or having
or reduce . reached an advanced stage; its direct opposite is
"immature," which means not fully developed or
Solution 19: ✅ Correct Answer: Submit lacking maturity. The other options are similar in
🔍 Word Analysis: Dominate meaning to "mature," not antonyms.
Meaning : To exercise control or authority over;
to overpower or lead. Solution 25: The correct answer is "Forbid."
Common contexts: "Authorize" means to give official permission or
One team dominated the match. approval, while "forbid" means to refuse to allow
She dominates every discussion. something or to prohibit. "Permit," "empower,"
So, the opposite would be to yield , give in , or and "allow" are similar to "authorize" and
submit —i.e., to accept someone else's control. therefore not antonyms.

Solution 20: ✅ Correct Answer: Healthy Solution 26: The correct answer is " Refuse."
"Affirm" means to state or confirm something
🔍 Word Analysis: Wan
positively, to assert as true, or to validate. Its
Meaning : Wan describes someone who looks
antonym is "refuse," which means to decline,
pale, sickly, or weak , often due to illness, fatigue,
reject, or deny something. The other options—
or emotional distress.
"support," "acknowledge," and "accept"—are
Example: She gave a wan smile after the
similar in meaning to "affirm" rather than opposite.
exhausting day.
So, the opposite would be someone who looks
Solution 27: The correct answer is "
vibrant, strong, or healthy .
Thoughtless." " Prudent" means showing care,
wisdom, and good judgment in making
Solution 21: ✅ Correct Answer: Malevolent
decisions, especially regarding the future. Its
🔍 Word Analysis: Benevolent antonym is "thoughtless," which means
Meaning : Kind-hearted, well-meaning, and acting without consideration or care . The
charitable in nature. other options—"sensible," "judicious," and
Example: A benevolent leader always puts "cautious"—are all synonyms of "prudent" rather
others first. than antonyms.
So, the antonym must reflect ill intent or harmful
nature —which is exactly what malevolent means. Solution 28: The correct answer is " Commend."
" Vilify" means to speak or write about
Solution 22: ✅ Correct Answer: Friendly someone in an abusively disparaging manner
🔍 Word Analysis: Hostile or to defame someone. "Commend" is the
Meaning : Unfriendly, antagonistic, or aggressive antonym as it means to praise, approve, or
in attitude or behavior. speak favorably about someone . The other
Example: The crowd grew hostile after the options—"defame," "malign," and "slur"—are all
announcement. synonyms of "vilify" as they also involve speaking
So, the antonym must reflect warmth, kindness, negatively about someone.
or welcoming behavior— friendly fits perfectly.
Solution 29: The correct antonym of 'assert' is '
Solution 23: The correct answer is " Fragile." deny '.
While "resilient" refers to the ability to recover
quickly from difficulties or to withstand stress, Solution 30: The antonym of "eminent," which
"fragile" means easily broken, damaged, or means famous, respected, or well-known, is
unable to withstand adverse conditions, making it "obscure." "Obscure" means not well known,
the most appropriate antonym. The other options hidden, or unknown, which is the direct opposite
Telegram (Previous year papers PDFs [SSC,Railway,DSSSB,UP SI]): https://t.me/RBE_S
YouTube (Free lectures and job updates): https://www.youtube.com/c/RBERevolutionByEducation
Download RBE application for SSC Exams Complete Preparation
of "eminent." The other options all mean means to move downward. The other options—
distinguished or celebrated, which are similar in "climb," "rise," and "soar"—are all synonyms or
meaning rather than opposite. related to upward movement, not the opposite.

Solution 31: "Commendable" means deserving Solution 38: "Arraign" means to formally accuse
praise or admiration. The antonym "blameworthy" or charge someone in a court of law, often
means deserving blame or criticism, which is implying criticism or blame. The antonym,
opposite in meaning. "Notable," "admirable," and "justify," means to show or prove that someone is
"laudable" are synonyms or similar in meaning to right or reasonable, essentially defending rather
"commendable." than accusing. The other options are unrelated to
the meaning of "arraign."
Solution 32: "Lax" means not sufficiently strict,
severe, or careful. The word "reliable" is the Solution 39: ✅ Correct Answer: Paragon
opposite, indicating someone or something that is 📘 Explanation:
strict, dependable, and responsible. Other options Word: Knave Meaning: A dishonest ,
like "stack," "careless," and "loose" either have untrustworthy , or deceitful man.
unrelated meanings or reinforce the idea of being A paragon is someone who is a model of virtue ,
lax, not contradict it. honesty , or excellence .
It represents the moral opposite of a knave.
Solution 33: The word "liberation" means
freedom or release from confinement. Its antonym Solution 40: ✅ Correct Answer: Fragile
is "captivity," which means the state of being
🧠 Explanation:
imprisoned, confined, or enslaved. The other
Word: Durable ➡ Meaning: Something that lasts
options are synonyms or closely related to the
meaning of "liberation," and do not serve as long, is resistant to wear, and is strong or sturdy.
antonyms. Antonym: ➡ Fragile means easily broken,
delicate, or not long-lasting — the opposite of
Solution 34: "Scrupulous" means having moral durable.
integrity or being very careful and honest. The
antonym is "dishonest," which signifies a lack of Solution 41: ✅ Correct Answer: Deceitful
integrity, morality, or truthfulness. The other 🔍 Explanation:
options—"principled," "ethical," and "careless"— Word: Candid Meaning: Open, honest, and
do not express direct opposition in meaning to straightforward in speech or expression.
"scrupulous." 🔁 Antonym: A word that expresses the opposite
meaning .
Solution 35: The word "superficial" means ✅ Deceitful means dishonest, misleading, or
existing or occurring at or on the surface, lacking insincere — the direct opposite of candid .
depth or substance. Its antonym is "deep," which
describes something profound, thorough, or Solution 42: ✅ Correct Answer: Lazy
extending far below the surface. The other
🔍 Explanation:
options either have similar meanings to
"superficial" or do not express the opposite idea. Word: Diligent Meaning: Showing care, effort,
and persistence in one’s work or duties.
Solution 36: The word "approve" means to 🔁 Antonym: A word that expresses the opposite
accept, allow, or agree to something. Its antonym meaning — someone who does not work hard
is "reject," which means to refuse, deny, or not or carefully .
accept. The other options are all synonyms of ✅ Lazy means unwilling to work or use effort ,
"approve" and do not express the opposite which is the direct opposite of diligent .
meaning.
Solution 43: ✅ Correct Answer: Modesty
Solution 37: The word "ascend" means to move 🔍 Explanation:
upward or rise. Its antonym is "descend," which Word: Audacity Meaning: Boldness or daring,
Telegram (Previous year papers PDFs [SSC,Railway,DSSSB,UP SI]): https://t.me/RBE_S
YouTube (Free lectures and job updates): https://www.youtube.com/c/RBERevolutionByEducation
Download RBE application for SSC Exams Complete Preparation
often with a disregard for conventional rules or Solution 46: ✅ Correct Answer: Option 2 –
respect. Rash
🔁 Antonym: A word that expresses the opposite 🔍 Explanation:
meaning — someone who is humble, Deliberate means:
respectful, or reserved rather than daring or Done consciously and intentionally
brazen. Carefully thought out or planned
✅ Modesty means humility, lack of boldness or Opposite of impulsive or careless
arrogance , which directly contrasts with audacity . So, the antonym must reflect lack of careful
thought or planning , which is exactly what
Solution 44: ✅ Correct Answer: Advance rash means.
🔍 Explanation:
Word: Withdraw Meaning: To pull back, remove, Solution 47: ✅ Correct Answer: Option 4 –
or retreat from a position, place, or situation. Inexpressive
🔁 Antonym: A word that expresses the opposite 🔍 Explanation:
action — moving forward or entering instead of Word: Eloquent Meaning: Fluent, expressive,
pulling back. and persuasive in speaking or writing.
✅ Advance means to move forward or proceed , An antonym is a word that conveys the opposite
which is the direct opposite of withdraw . meaning .

Solution 45: ✅ Correct Answer: Option 4 – Solution 48: ✅ Correct Answer: Option 1 –
Obscure Liberate
🔍 Explanation: 🔍 Explanation:
The word prominent means: Word: Yoke Meaning:
Easily noticeable Literally: A wooden crosspiece fastened over the
Well-known or important necks of two animals, used for pulling a plow or
Standing out visibly or socially cart.
So, its antonym must convey the opposite — Figuratively: A symbol of bondage , oppression ,
something not well-known , hidden , or unclear . or control .

Telegram (Previous year papers PDFs [SSC,Railway,DSSSB,UP SI]): https://t.me/RBE_S


YouTube (Free lectures and job updates): https://www.youtube.com/c/RBERevolutionByEducation
Download RBE application for SSC Exams Complete Preparation

SSC Steno 2025 T-1 Homonym All 26 Questions with Detailed Solution and Answer key

Q1. Choose the sentence that uses a homonym of


the underlined word: The lead pipes were replaced due to safety
The judge will sentence the accused tomorrow. concerns.
(a) He received a ten-year sentence for the crime (a) The general will lead the troops into battle.
(b) She wrote a beautiful sentence in her diary. (b) They tested the Soil tor traces of lead.
(c) The courtroom fell silent during the sentence. (c) Exposure to lead can cause health issues in
(d) The lawyer defended the sentence fiercely. children .
(d) The house still had le ad i n Its plumbing
Q2. Choose the sentence that uses a system.
homonym of the underlined word:
He left the watch on the nightstand before Q7. Select the most appropriate homonym.
sleeping. The company refused to back the project.
(a) She bought a new gold watch for her (a) Finish (b) Leave
husband. (c) Support (d) Pack
(b) He set his digital watch to the right time.
(c) The guard was assigned to night watch duty. Q8. Select the most appropriate homonym.
(d) The watch was ticking softly beside the lamp. He was charged with theft.
(a) Energized (b) Ordered
Q3. Choose the sentence that uses a (c) Filled (d) Accused
homonym of the underlined word:
Please close the window before leaving. Q9. Select the most appropriate homonym.
(a) I always close the curtains at night. The diver reached the sea bed.
(b) Make sure to close the back door too. (a) Rest (b) Mattress
(c) Their bond grew close over the years. (c) Chair (d) Bottom
(d) He forgot to close the file before shutting the
laptop. Q10. Choose the sentence that uses the
homonym of the underlined word:
Q4. Choose the sentence that uses a The wind howled through the trees.
homonym of the underlined word: (a) He had to wind the old clock manually.
He had to tear the envelope to open it. (b) The wind was cold and sharp at night.
(a) He found a small tear in the corner. (c) The wind knocked over the garbage can.
(b) The comedian caused the audience to tear (d) A sudden gust of wind blew her hat away.
up with laughter.
(c) Please don't tear the paper again. Q11. Choose the sentence that uses a homonym
(d) She used tape to fix the tear on the bag. of the underlined word:
He bought a ticket for the evening show .
Q5. Select the most appropriate homonym. (a) That was the best stage show I’ve seen.
(b) The farmers decided to show their livestock.
The old fort was taken by storm . (c) We’re attending the school show tomorrow.
(a) Tornado (b) Rain (d) The magic show was amazing
(c) Assault (d) Breeze
Q12. Select the most appropriate homonym.
Q6. Choose the sentence that uses a The new CEO will chair the meeting.
homonym of the underlined word. (a) Cancel (b) Furniture
Telegram (Previous year papers PDFs [SSC,Railway,DSSSB,UP SI]): https://t.me/RBE_S
YouTube (Free lectures and job updates): https://www.youtube.com/c/RBERevolutionByEducation
Download RBE application for SSC Exams Complete Preparation
(c) Preside over (d) Observe Q20. Select the most appropriate homonym.
He gave her a cold shoulder at the party.
Q13. Select the most appropriate homonym. (a) Apology (b) Complaint
His name has been cleared of all charges. (c) Embrace (d) Ignore
(a) Exonerated (b) Shifted
(c) Made clean (d) Removed Q21. Select the most appropriate homonym.
The guards will watch over the premises.
Q14. Select the most appropriate homonym. (a) Protect (b) Timepiece
The opposition's motion was rejected. (c) Warn (d) Observe
(a) Proposal (b) Movement
(c) Commotion (d) Emotion Q22. Select the most appropriate homonym.
They had a heated row last night.
Q15. Select the most appropriate homonym. (a) Boat (b) Series
The old man could barely hear. (c) Argument (d) Line
(a) Listen (b) Hair
(c) See (d) Smell Q23. Select the most appropriate homonym.
She went to the court to file a case.
Q16. Select the most appropriate homonym. (a) Ground (b) Law place
He will bat for his friend's innocence. (c) Yard (d) Field
(a) Support (b) Criticize
(c) Play cricket (d) Hit Q24. Select the most appropriate homonym.
He was given a light punishment.
Q17. Select the most appropriate homonym. (a) Heavy (b) Serious
The king sat on a throne. (c) Dark (d) Mild
(a) Chair (b) Rule
(c) Crown (d) Sword Q25. Select the most appropriate homonym.
The current situation is tense.
Q18. Select the most appropriate homonym. (a) Present (b) Modern
The judge will pass the verdict. (c) Running (d) Electric flow
(a) Announce (b) Fail
(c) Shift (d) Cross Q26. Select the most appropriate homonym.
The police seized the counterfeit notes.
Q19. Select the most appropriate homonym. (a) Letters (b) Currency
This case calls for immediate action. (c) Messages (d) Music
(a) Example (b) Legal matter
(c) Scenario (d) Suitcase

Answer Key:

Q1 b Q2 c Q3 c Q4 b Q5 c
Q6 a Q7 c Q8 d Q9 d Q10 a
Q11 b Q12 c Q13 a Q14 a Q15 a
Q16 a Q17 a Q18 a Q19 b Q20 d
Q21 a Q22 c Q23 b Q24 d Q25 a
Q26 b

Solution 1: Explanation punishment or judgment.


The underlined word "sentence" in "The judge will The homonym "sentence" in the chosen option
sentence the accused tomorrow" means a legal means a group of words that form a complete
Telegram (Previous year papers PDFs [SSC,Railway,DSSSB,UP SI]): https://t.me/RBE_S
YouTube (Free lectures and job updates): https://www.youtube.com/c/RBERevolutionByEducation
Download RBE application for SSC Exams Complete Preparation
thought in writing. Solution 6: Explanation:
Both words are spelled the same but have In the original sentence, "lead" (pronounced "led")
different meanings, making them homonyms. refers to the metal used in pipes.
In option 1, "lead" is a verb meaning "to guide" or
Solution 2: Explanation: "to direct."
"Watch" (noun) in the original sentence refers to a
timepiece. Solution 7: ✅ Correct Answer: Support
The homonym "watch" (noun) in the selected In the sentence “The company refused to back
sentence means a period of duty or vigil, the project,” the word “back” is used as a verb .
especially during the night. Here, “back” means to support , endorse , or
Both words are spelled and pronounced the same fund something.
but have different meanings, making them So, the correct homonym (same spelling, different
homonyms. meaning) of “back” in this context is “support.”

Solution 3: Explanation: Solution 8: ✅ Correct Answer: Accused


"Close" in the original sentence is a verb Here, “charged” means to be formally accused
meaning "to shut." of a crime .
"Close" in the selected sentence is an The homonym “charged” can also mean:
adjective meaning "near or intimate," Energized (e.g., emotionally charged)
pronounced differently (/kloʊs/ vs. /kloʊz/). Filled (e.g., battery charged)
Both words share the same spelling but Ordered (e.g., charged a fee)
different meanings and pronunciations,
making them homonyms. Solution 9: ✅ Correct Answer : Bottom
The other options use "close" as a verb, similar in
📌 Point-wise Explanation :
meaning to the original sentence.
Meaning of "bed" in context : In the sentence
"The diver reached the sea bed," the word "bed"
Solution 4: Explanation:
refers to the bottom surface of the sea — not a
The underlined word "tear" in the sentence "He
piece of furniture.
had to tear the envelope to open it." is
Homonym Identification : The word "bed" has
a homograph—a word spelled the same as
multiple meanings (homonyms), such as:
another but having different meanings and
A place to sleep (e.g., mattress)
pronunciations:
A flower bed (garden area)
tear (verb) — pronounced like "tare" (/ ter /)
A riverbed or seabed (bottom surface)
meaning to rip or pull apart.
Example: "He had to tear the envelope to open
Solution 10: ✅ Correct Answer: He had to wind
it."
tear (noun) — pronounced like "tier" (/ tɪər /) the old clock manually.
meaning a drop of liquid from the eye when crying It changes both meaning and pronunciation :
or when someone is emotional. Original: wind = moving air
Example: "The comedian caused the audience to Option A: wind = to twist/turn
tear up with laughter."
Solution 11: ✅ Correct Answer: The farmers
Solution 5: Explanation: decided to show their livestock.
Here "taken by storm" means to capture or In the original sentence, show is a noun meaning
assault the fort, not a weather phenomenon. a performance.
The word "storm" can mean both a violent In Option B, show is a verb meaning to display or
weather event or a sudden attack. present livestock.
Other options like "Tornado," "Rain," and This shift in part of speech and meaning makes it
"Breeze" relate only to weather, not the context of a true homonym .
capturing a fort.
Therefore, the correct choice is Assault. Solution 12: ✅ Correct Answer: Preside over
🔍 Step-by-Step Explanation:
Telegram (Previous year papers PDFs [SSC,Railway,DSSSB,UP SI]): https://t.me/RBE_S
YouTube (Free lectures and job updates): https://www.youtube.com/c/RBERevolutionByEducation
Download RBE application for SSC Exams Complete Preparation
Target Sentence : authority or importance. In royal contexts, the
Meaning of “Chair” in Context : "throne" refers to the ceremonial or official chair
Here, “chair” is used as a verb , meaning to lead reserved for a monarch. The other options—
or preside over a meeting or discussion. "rule," "crown," and "sword"—are related to
It does not refer to furniture. kingship but do not share the same meaning as
Why “Preside over” is Correct : "chair" in this context.
It’s a direct synonym of the verb “chair” in this
context. Solution 18: In the context of a court, "pass" as a
Both imply taking charge or leading a formal homonym means "announce" (as in, to formally
gathering. deliver a judgment or verdict). The other options
("fail," "shift," "cross") represent alternate
Solution 13: ✅ Correct Answer: Exonerated meanings of "pass" but do not fit the judicial
🔍 Step-by-Step Analysis: usage in this sentence.
Sentence Context :
Implies legal or moral absolution —he is no Solution 19: In the context of "This case calls for
longer considered guilty. immediate action," "case" refers to a legal matter
Target Word Type : or situation requiring attention, not an example,
We’re looking for a homonym (same or similar scenario, or a suitcase. Therefore, "legal matter"
meaning in context), not just a literal synonym. is the most appropriate homonym for the
Meaning of “Exonerated” : sentence.
To officially declare someone not guilty or free
from blame . Solution 20: The phrase "cold shoulder" means to
Perfect fit for “cleared of all charges.” deliberately ignore someone or treat them with
indifference. "Ignore" is the correct interpretation,
Solution 14: The correct answer is " Proposal ." while the other options do not capture this
In this context, "motion" refers to a formal idiomatic meaning.
proposal or suggestion put forward in a legislative
or meeting setting. "Proposal" is the most Solution 21: ✅ Correct Answer: Protect
appropriate word that could replace "motion" in 🧠 Explanation:
this sentence while maintaining the same Sentence: The guards will watch over the
meaning. The other options don't fit the premises.
political/formal context of the sentence. 🔍 The phrase “watch over” is a phrasal verb
meaning to guard, protect, or keep safe — not
Solution 15: The homonym for " hear " is "listen," just to observe passively.
as both relate to the act of perceiving sound. So, the most appropriate homonym (same
Although "hair" is a homophone (sounds similar spelling, different meaning) of “watch” in this
but different meaning), "listen" is the word that context is:
connects to " hear " in context. "See" and "smell" ➡ Protect
relate to other senses and do not match as
homonyms for "hear."
Solution 22: ✅ Correct Answer: Argument
🔍 Explanation:
Solution 16: In this context, "bat" is used
Sentence: They had a heated row last night.
metaphorically to mean " support" or "defend "
🔸 The word “row” here is a homonym — a
someone's cause—in this sentence, his friend's
innocence. While "bat" commonly refers to hitting word that has multiple meanings and same
or playing cricket, here it is a homonym spelling but different contexts .
representing advocacy or siding with someone. In this sentence, “row” means a quarrel or
The other options do not fit the figurative use in argument , not a line or a boat.
the sentence. ✅ So, the most appropriate meaning is:
Argument
Solution 17: "Throne" is a homonym for "chair,"
as both represent a seat, specifically one of
Telegram (Previous year papers PDFs [SSC,Railway,DSSSB,UP SI]): https://t.me/RBE_S
YouTube (Free lectures and job updates): https://www.youtube.com/c/RBERevolutionByEducation
Download RBE application for SSC Exams Complete Preparation
Solution 23: In this context, "court" refers to a 🔍 Explanation:
place where legal cases are heard and justice is In the sentence “The current situation is tense,”
administered. The homonym "law place" the word “current” is used to mean present or
directly represents this meaning of "court, " existing now .
distinguishing it from other meanings like
"ground" (sports court) or "yard/field" (open area). Solution 26: ✅ Correct Answer: Option 2 –
Currency
Solution 24: ✅ Correct Answer: Option 4 – 🔍 Explanation:
Mild Sentence:
👉 Mild , which means gentle , not harsh , or not The police seized the counterfeit notes.
severe — matching the intended meaning of The word “notes” is a homonym , meaning it has
“light” here. multiple meanings depending on context. In this
case, it refers to currency — specifically fake
Solution 25: ✅ Correct Answer: Option 1 – currency bills .
Present

For SSC Exams Complete Preparation (Download RBE Application)


(Learn from those who have cleared the exam themselves)
https://play.google.com/store/apps/details?id=com.revolution.education

Telegram (Previous year papers PDFs [SSC,Railway,DSSSB,UP SI]): https://t.me/RBE_S


YouTube (Free lectures and job updates): https://www.youtube.com/c/RBERevolutionByEducation
Download RBE application for SSC Exams Complete Preparation

SSC Steno 2025 T-1 Idioms All 48 Questions with Detailed Solution and Answer key

Q1. Choose the underlined part of the (c) Create unnecessary complications
sentence with an appropriate idiom: (d) Tie unrelated issues together

As the president of a major company, Harshith is Q6. What does the idiom ‘Go the extra mile”
used to being the focus of attention. mean?
(a) The elephant in the room (a) Work harder than expected
(b) A dime a dozen (b) Work halt-heartedly
(c) Under the weather (c) Stay idle
(d) In the limelight (d) Leave midway

Q2. Choose the correct phrase for the word Q7. What does the idiom ‘Not one’s cup of tea”
highlighted in bold. mean?
Let’s try to think outside the box and come up (a) Something disliked
with a new solution to this problem. (b) Something enjoyable
(a) To reject all solutions without analysis. (c) very easy
(b) To consider unconventional and creative (d) impossible task
solutions.
(c) To follow traditional methods strictly. Q8. Choose the correct meaning of idiom “ To oil
(d) To solve problems using only existing rules. the hinges of silence ”.
(a) To speak diplomatically in tense moments
Q3. Which of the following sentences correctly (b) To gradually break uncomfortable silence
uses the idiom "a slip of the tongue"? (c) To bribe someone to remain quiet
(a) Ravi misspoke during the interview; it was just (d) To prevent secrets from being exposed
a slip of the tongue.
(b) Sneha carefully planned her speech to ensure Q9. Choose the correct meaning of idiom " Bite
a slip of the tongue. the bullet ”.
(c) Sunita lost her balance while walking and had (a) Endure something painful
a slip of the tongue. (b) Make 3 bold plan
(d) The chef burned his hand and let out a slip of (c) Speak without thinking
the tongue. (d) Tell a secret

Q4. Choose the correct meaning of the idiom: Q10. What does the idiom ‘Take to task’ mean?’
'Cast pearls before swine’ (a) Scold (b) congratulate
(a) Sell things at very high prices (c) Reward (d) Praise
(b) Share secrets with close friends
(c) Hide something precious from everyone Q11. What does the idiom ‘Give someone a cold
(d) Give valuable things to undeserving shoulder’ mean?
(a) Praise someone (b) Invite for dinner
Q5. Choose the correct meaning of idiom "Cut (c) Ignore someone (d) Treat warmly
the Gordian knot" .
(a) Solve the problem badly Q12. Choose the correct meaning of idiom: 'Show
(b) Disagree with authority a clean pair of heels'
(a) Imitate one (b) Spread quickly

Telegram (Previous year papers PDFs [SSC,Railway,DSSSB,UP SI]): https://t.me/RBE_S


YouTube (Free lectures and job updates): https://www.youtube.com/c/RBERevolutionByEducation
Download RBE application for SSC Exams Complete Preparation
(c) Run away (d) Be precise and careful (c) Poor Weather
(d) Give money
Q13. What does the idiom ‘In black and white’
mean? Q20. What does the idiom ‘Face the music’
(a) In secret (b) In verbal form mean?
(c) Officially in writing (d) Unclear terms (a) Confront difficulties (b) Hide feelings
(c) Avoid trouble (d) Run away
Q14. What does the idiom ‘To have the last laugh’
mean? Q21. What does the idiom ‘Spill the beans’ mean?
(a) Succeed eventually (a) Waste food (b) Hide a fact
(b) Be mocked (c) Lose patience (d) Reveal a secret
(c) Fail hopelessly
(d) Stay indifferent Q22. What does the idiom ‘ Turn a deaf ear’
mean?
Q15. Choose the correct meaning of the idiom: (a) Reply angrily (b) Ignore advice
'Tooth and nail' (c) Listen carefully (d) Argue
(a) In a confused state
(b) Secretly and silently Q23. What does the idiom ‘ Pull someone’s leg’
(c) Without any tools mean?
(d) With full strength and determination (a) Joke with them (b) Praise them
(c) Criticize them (d) Annoy them
Q16. What does the idiom ‘To turn over a new
leaf’ mean? Q24. What is the meaning of the proverb “ Don’t
(a) Stay unchanged count your chickens before they hatch”?
(b) Start afresh (a) Chickens should be counted immediately after
(c) Turn pages hatching.
(d) Repeat mistakes (b) Always count your livestock carefully.
(c) Don't assume success before it happens.
Q17. What does the idiom ‘Steal someone’s (d) Hatching eggs is an unpredictable process.
thunder’ mean?
(a) Ignore them (b) Take credit for their work Q25. What does the idiom 'Cry over spilt milk'
(c) Help them (d) Mock them mean?
(a) Be indifferent (b) Ignore problems
Q18. Which of the following sentences correctly (c) Regret a past loss (d) Work harder
uses the idiom "A picture is worth a thousand
words"? Q26. The phrase 'A white elephant' refers to:
(a) I prefer reading detailed descriptions rather (a) Lucky charm (b) Rare animal
than looking at images, even though a picture is (c) Valuable item (d) Costly burden
worth a thousand words.
(b) Since she couldn't take a picture, she decided Q27. Choose the correct meaning of the idiom:
to write a long paragraph, as a picture is worth a 'Cut no ice'
thousand words. (a) Have no effect (b) Be sharp
(c) The beautiful photograph captured the tragedy (c) Create trouble (d) Stop something
so well that no words were needed—truly, a
picture is worth a thousand words. Q28. What does the idiom ‘to grasp at straws’
(d) He described the entire scene in detail mean?
because a picture is worth a thousand words. (a) To show courage
(b) To make careful decisions
Q19. Choose the correct meaning of idiom: 'Face (c) To win easily
the music' (d) To try desperate measures
(a) Bear consequences
(b) Earn Money
Telegram (Previous year papers PDFs [SSC,Railway,DSSSB,UP SI]): https://t.me/RBE_S
YouTube (Free lectures and job updates): https://www.youtube.com/c/RBERevolutionByEducation
Download RBE application for SSC Exams Complete Preparation
Q29. Choose the correct meaning of Idiom "Flog (c) Fight again (d) Make peace
a dead horse".
(a) Beat an opponent unfairly Q39. The idiom 'Token strike' implies:
(b) Repeat a settled argument (a) Heavy protest (b) Warning strike
(c) Chase an impossible goal (c) Silent protest (d) No protest
(d) Waste time on a lost cause
Q40. What does the idiom ‘Speak your mind’
Q30. What does the idiom ‘To leave no stone mean?
unturned’ mean? (a) Gossip (b) Express frankly
(a) Relax completely (b) Ignore issues (c) Stay silent (d) Lie
(c) Search thoroughly (d) Avoid effort
Q41. What does the idiom ‘Get to the point’
Q31. What does the idiom ‘Barking up the wrong mean?
tree’ mean? (a) Speak indirectly (b) Interrupt
(a) Accusing wrongly (c) Avoid the matter (d) Speak directly
(b) Succeeding quickly
(c) Apologizing Q42. What does the idiom ‘Jack up the rates’
(d) Praising someone mean?
(a) Increase prices (b) Sell everything
Q32. What does the idiom ‘On cloud nine’ mean? (c) Drop prices (d) Stay unchanged
(a) Very angry (b) Lazy
(c) Confused (d) Extremely happy Q43. What does the idiom ‘To go for a song’
mean?
Q33. What does the idiom ‘Cry wolf’ mean? (a) Be unsold (b) Be expensive
(a) Raise a false alarm (b) Help others (c) Be sold cheaply (d) Be sung loudly
(c) Be truthful (d) Lie unnecessarily
Q44. What does the idiom ‘Clutching at straws’
Q34. The meaning of the idiom 'Make a clean mean?
breast of' is to: (a) Laughing loudly
(a) Whisper (b) Argue (b) Winning easily
(c) Accuse (d) Confess (c) Boasting unnecessarily
(d) Hoping without reason
Q35. What does the idiom ‘throw in the towel’
mean? Q45. What does the idiom ‘Sit on the fence’
(a) Boast (b) Start afresh mean?
(c) Work harder (d) Give up (a) Take a risk (b) Remain silent
(c) Stay neutral (d) Join both sides
Q36. What does the idiom ‘Jump the gun’ mean?
(a) Succeed quickly (b) Wait patiently Q46. Choose the correct meaning of the idiom:
(c) Fire a gun (d) Act prematurely The gift of the gab
(a) Ability to cook well
Q37. What does the idiom ‘To beat a dead horse’ (b) Power to make quick decisions
mean? (c) Habit of exaggerating stories
(a) Waste time on a hopeless issue (d) Talent for speaking fluently and persuasively
(b) Be overconfident
(c) Encourage others Q47. Choose the correct meaning of idiom:
(d) Win a competition All at sea
(a) Far away (b) Confused
Q38. What does the idiom ‘Bury the hatchet’ (c) Excited (d) Drowning
mean?
(a) Hide weapons (b) Ignore a problem Q48. What does the idiom ‘To bell the cat’ mean?
(a) Feed a pet (b) Take a risky initiative
Telegram (Previous year papers PDFs [SSC,Railway,DSSSB,UP SI]): https://t.me/RBE_S
YouTube (Free lectures and job updates): https://www.youtube.com/c/RBERevolutionByEducation
Download RBE application for SSC Exams Complete Preparation
(c) Make an easy move (d) Escape a problem

Answer Key:

Q1 d Q2 b Q3 a Q4 d Q5 a
Q6 a Q7 a Q8 b Q9 a Q10 a
Q11 c Q12 c Q13 c Q14 a Q15 d
Q16 b Q17 b Q18 c Q19 a Q20 a
Q21 d Q22 b Q23 a Q24 c Q25 c
Q26 d Q27 a Q28 d Q29 d Q30 c
Q31 a Q32 d Q33 a Q34 d Q35 d
Q36 d Q37 a Q38 d Q39 b Q40 b
Q41 d Q42 a Q43 c Q44 d Q45 c
Q46 d Q47 b Q48 b

Solution 1: Explanation or "Tie unrelated issues together."


"In the limelight" means being the center of So, the best answer based on the idiomatic sense
attention or in the spotlight. is closest to:
The other idioms have different meanings: Solve the problem badly (but ideally should be
"The elephant in the room" refers to an obvious "solve the problem decisively or boldly").
problem that is ignored.
"A dime a dozen" means something very Solution 6: Explanation:
common. This idiom refers to putting in more effort than
"Under the weather" means feeling ill. what is required or expected.
It implies dedication, commitment, and willingness
Solution 2: Explanation to do more to achieve a goal.
"Think outside the box" means to think differently, Other options like "Work half-heartedly," "Stay
innovatively, or creatively, especially beyond idle," and "Leave midway" have opposite or
traditional or obvious ideas. unrelated meanings.
The other options are contrary to the meaning of
the phrase. Solution 7: Explanation:
The idiom is used to indicate something that a
Solution 3: Explanation person does not like or is not interested in.
"A slip of the tongue" means a small mistake It is the opposite of something enjoyable or
made while speaking, such as saying something preferred.
incorrectly or unintentionally. Other options like "something enjoyable," "very
The other sentences misuse the idiom in contexts easy," or "impossible task" do not convey the
unrelated to speech errors. correct meaning.
Therefore, the correct choice is Something
Solution 4: Explanation disliked
This idiom means offering something valuable or
precious to people who do not appreciate or Solution 8: Explanation:
understand its worth. This idiom suggests making conversation easier
It metaphorically refers to giving pearls (valuable) or breaking an awkward silence gently, similar to
to swine (pigs), who cannot appreciate them. how oiling hinges helps them move smoothly.
It is about easing tension and encouraging
Solution 5: Explanation: communication, not about bribery, diplomacy, or
"Solve the problem badly" is closest but not secrecy.
accurate. Therefore, the correct choice is to gradually
The idiom does not mean "Disagree with break uncomfortable silence.
authority," "Create unnecessary complications,"
Telegram (Previous year papers PDFs [SSC,Railway,DSSSB,UP SI]): https://t.me/RBE_S
YouTube (Free lectures and job updates): https://www.youtube.com/c/RBERevolutionByEducation
Download RBE application for SSC Exams Complete Preparation
Solution 9: Explanation: succeeds , proving others wrong.
The idiom means to face a difficult or unpleasant
situation bravely and with endurance. Solution 15: ✅ Correct Answer: With full
It does not mean making a bold plan, speaking strength and determination
without thinking, or telling a secret. 🔍 Explanation:
Therefore, the correct choice is Endure Meaning of the Idiom : The phrase "tooth and
something painful. nail" means to fight or work with intense effort, full
strength, and fierce determination . 👉 It often
Solution 10: Explanation:
implies a struggle or battle where someone gives
To "take someone to task" means to criticize, their all.
scold, reprimand, or blame someone for
something they have done wrong.
Solution 16: ✅ Correct Answer: Start afresh
It does not mean to congratulate, reward, or
🔍 Explanation:
praise.
Therefore, the correct choice is Scold. Meaning of the Idiom :
To turn over a new leaf means to begin again
with a fresh attitude, behavior, or approach ,
Solution 11: ✅ Correct Answer: Ignore
especially after a period of mistakes or
someone
wrongdoing.
The idiom “Give someone a cold shoulder”
It originates from the idea of turning to a blank
means to deliberately ignore or show
page (leaf = page in old books) to begin anew.
indifference toward someone, often as a sign of
disapproval or rejection. It’s a subtle way of
Solution 17: ✅ Correct Answer: Take credit for
expressing emotional distance.
Example: their work
After their argument, she gave him the cold 🔍 Idiom: Steal someone’s thunder
shoulder at the party. 🔸 Meaning:
To use someone else's idea or achievement and
Solution 12: ✅ Correct Answer: Run away present it as your own, especially to gain praise
The idiom “Show a clean pair of heels” means or attention before they can.
to run away swiftly , often to escape danger,
responsibility, or confrontation. It paints a visual Solution 18: ✅ Correct Answer: The beautiful
image of someone running so fast that all you see photograph captured the tragedy so well that no
is the back of their heels. words were needed—truly, a picture is worth a
Example: thousand words.
When the thief saw the police, he showed a clean Idiom Meaning:
pair of heels. “A picture is worth a thousand words” → A visual
image can convey complex ideas or emotions
Solution 13: ✅ Correct Answer: Officially in more effectively than verbal description.
writing
Idiom : In black and white Solution 19: ✅ Correct Answer: Bear
Meaning : Something that is clearly written or consequences
printed , especially in an official or documented 🔍 Idiom Meaning:
form. “Face the music” → To accept and deal with the
Usage : “I need the agreement in black and unpleasant consequences of one’s actions,
white before I sign anything.” especially punishment or criticism.

Solution 14: ✅ Correct Answer : Solution 20: ✅ Correct Answer: Confront


Succeed eventually difficulties
📌 Point-wise Explanation : 🔍 Idiom Meaning: Face the music
Meaning of the Idiom : The phrase "to have the Definition : To accept the consequences of one’s
last laugh" means that someone who was actions, especially unpleasant ones.
doubted, mocked, or underestimated ultimately Often used when someone must deal with
Telegram (Previous year papers PDFs [SSC,Railway,DSSSB,UP SI]): https://t.me/RBE_S
YouTube (Free lectures and job updates): https://www.youtube.com/c/RBERevolutionByEducation
Download RBE application for SSC Exams Complete Preparation
criticism, punishment, or reality after a mistake or they were costly to maintain and couldn't be used
wrongdoing. for labor, making them burdensome possessions
rather than valuable assets.
Solution 21: ✅ Correct Answer: Reveal a secret
🔍 Idiom Meaning: Spill the beans Solution 27: The correct meaning of the idiom
Definition : To accidentally or deliberately "Cut no ice" is:
disclose confidential information or a secret. Have no effect
Often used when someone reveals something Explanation:
prematurely or unintentionally. The idiom "cut no ice" means that something
does not have any influence or effect on a
Solution 22: The correct answer is " Ignore situation.
advice." The idiom ‘turn a deaf ear’ means to The other options like "Be sharp," "Create
refuse to listen, especially to warnings or advice, trouble," and "Stop something" do not match the
as if one did not hear them at all. It implies meaning of this idiom.
consciously disregarding suggestions or criticism. Hence, the correct answer is: Have no effect .
The other options do not accurately capture the
meaning of this idiom. Solution 28: The idiom "to grasp at straws"
means to attempt anything in desperation,
Solution 23: The correct answer is "Joke with especially when all hope seems lost and there are
them." The idiom ‘pull someone’s leg’ means to no realistic solutions available. It does not mean
tease or joke with someone in a playful way, often making careful decisions or showing courage, but
by telling them something that is not true as a rather resorting to weak or unlikely options when
harmless prank. It does not mean to praise, facing failure or disaster.
criticize, or annoy someone.
Solution 29: The idiom "flog a dead horse" refers
Solution 24: The correct answer is "Don't to using time and energy on a situation that
assume success before it happens." This cannot be improved or resolved. It means
proverb is a metaphorical warning against insisting on trying or arguing about something that
being overconfident about future outcomes is already decided or fruitless, so the person is
that are not yet certain . It advises people not to merely "wasting time on a lost cause." The other
make plans based on expected results that may options do not correctly capture this meaning.
not materialize. The literal meaning about
counting chickens is used figuratively to represent Solution 30: The idiom "leave no stone unturned"
any situation where someone assumes a positive means to try every possible way or effort to
outcome before it actually occurs. achieve a goal or find something, i.e., to search
thoroughly. It implies persistent and exhaustive
Solution 25: The correct answer is " Regret a investigation or endeavor. The other options do
past loss." The idiom "cry over spilt milk" means not match the meaning of the idiom.
to be upset or regretful about something that has
already happened and cannot be changed or Solution 31: The idiom "barking up the wrong
undone. It suggests that worrying about past tree" means to make a mistaken accusation or to
mistakes or losses is pointless since they cannot pursue a false lead. It refers to acting based on a
be reversed. The expression advises against misunderstanding or error, especially by blaming
dwelling on things that are already done. the wrong person or focusing on the wrong issue.
The other options do not match the idiom’s
Solution 26: The correct answer is " Costly meaning.
burden ." The phrase "a white elephant" is an
idiom that refers to something that is expensive to Solution 32: The idiom "on cloud nine" is used
maintain but of little practical value or use. The to describe a state of extreme happiness or
term originates from the historical practice in joy . The other options do not match the idiomatic
Southeast Asia where white elephants were meaning. When someone is "on cloud nine," they
considered sacred and given as royal gifts, but are feeling elated or blissful, not angry, lazy, or
Telegram (Previous year papers PDFs [SSC,Railway,DSSSB,UP SI]): https://t.me/RBE_S
YouTube (Free lectures and job updates): https://www.youtube.com/c/RBERevolutionByEducation
Download RBE application for SSC Exams Complete Preparation
confused. 🧠 Explanation:
Idiom: Speak your mind Meaning: To say exactly
Solution 33: The idiom "cry wolf" means to give a what you think or feel, especially without
false warning or raise a false alarm when there is hesitation or fear of judgment.
no real danger, often resulting in people ignoring ✅ So, if someone speaks their mind , they are
future warnings when a real threat appears. This being honest, direct, and open about their
meaning comes from the story "The Boy Who thoughts.
Cried Wolf," where repeated false alarms led to
mistrust. Other options do not reflect the correct Solution 41: ✅ Correct Answer: Speak directly
meaning of the idiom.
🧠 Explanation:
Idiom: Get to the point Meaning: To speak
Solution 34: The idiom "make a clean breast of"
clearly and concisely, focusing on the main idea
means to confess or disclose something honestly
without unnecessary details or digressions.
and openly, especially secrets or wrongdoings. It
✅ So, when someone says “Get to the point,”
does not mean whispering, arguing, or accusing.
Only "confess" correctly matches the meaning of they’re asking the speaker to be direct and
the idiom. relevant .

Solution 35: The idiom "throw in the towel" comes Solution 42: ✅ Correct Answer: Increase
from boxing and means to quit or admit defeat. prices
Among the options, "give up" directly matches 🧠 Explanation:
this meaning, while the others do not relate to the Idiom: Jack up the rates 🔍 Meaning: To
sense of surrender or resignation. suddenly or sharply increase prices or rates ,
often in response to demand, inflation, or
Solution 36: The idiom "jump the gun" means to strategic profit-making.
do something too soon or act before the right ✅ Example: ➡ The hotel jacked up the rates
time, often resulting in mistakes or missed during the holiday season.
opportunities. "Act prematurely" is the accurate
meaning; the other options do not capture the Solution 43: ✅ Correct Answer: Option 3 – Be
essence of the idiom. sold cheaply
🔍 Explanation:
Solution 37: The idiom "to beat a dead horse"
The idiom “to go for a song” means that
means to waste time or effort on something that is
something is sold at a very low price , often
already settled or hopeless. The other options do
much lower than its actual value.
not capture this meaning.
It’s often used when someone gets a great
bargain or when something is undervalued.
Solution 38: ✅ Correct Answer: Make peace
This idiom means to end a conflict , forgive , or Solution 44: ✅ Correct Answer: Option 4 –
reconcile with someone after a disagreement.
Hoping without reason
🔍 Explanation:
Solution 39: ✅ Correct Answer: Warning strike
The idiom “Clutching at straws” refers to a
🧠 Explanation:
situation where someone is desperately trying
The idiom “token strike” refers to a brief, to find any small or unlikely hope , especially
symbolic protest —usually organized to warn when facing failure or danger.
authorities or express dissatisfaction without It implies irrational or baseless hope , often out
causing major disruption. of desperation.
It’s not meant to be a full-fledged or indefinite
strike, but rather a strategic gesture to signal Solution 45: ✅ Correct Answer: Option 3 –
potential escalation if demands aren’t met.
Stay neutral
🔍 Meaning of the Idiom: Sit on the fence
Solution 40: ✅ Correct Answer: Express
To “sit on the fence” means to avoid making a
frankly
Telegram (Previous year papers PDFs [SSC,Railway,DSSSB,UP SI]): https://t.me/RBE_S
YouTube (Free lectures and job updates): https://www.youtube.com/c/RBERevolutionByEducation
Download RBE application for SSC Exams Complete Preparation
decision or choosing a side , especially in a , or bewildered , especially in a situation where
debate or conflict. It reflects indecision or one doesn’t know what to do.
neutrality , often to avoid offending either party
or committing to a stance. Solution 48: ✅ Correct Answer: Option 2 –
Take a risky initiative
Solution 46: 🔍 Explanation: 🔍 Meaning of the Idiom: To bell the cat
Idiom: The gift of the gab Meaning: This idiom refers to volunteering to do a
The natural ability to speak confidently, dangerous or difficult task , especially one that
fluently, and convincingly—especially in a others agree is necessary but are unwilling to
way that charms or persuades others. attempt themselves.
It originates from a fable where mice propose
Solution 47: ✅ Correct Answer: Option 2 – putting a bell around a cat’s neck to be warned of
Confused its approach—but no one dares to do it.
🔍 Idiom: All at sea
Meaning: To be completely confused , uncertain

For SSC Exams Complete Preparation (Download RBE Application)


(Learn from those who have cleared the exam themselves)
https://play.google.com/store/apps/details?id=com.revolution.education

Telegram (Previous year papers PDFs [SSC,Railway,DSSSB,UP SI]): https://t.me/RBE_S


YouTube (Free lectures and job updates): https://www.youtube.com/c/RBERevolutionByEducation
Download RBE application for SSC Exams Complete Preparation

SSC Steno 2025 T-1 Sentence Improvement All 77 Questions with Detailed Solution and Answer key

Q1. Select the option that correctly replaces the (d) My uncle goes to that shop most frequently.
underlined phrase in the following sentence:
Q6. Choose the most suitable option to
The doctor gave the patient some medications replace the underlined part of the sentence. If
after he felt some pain on his left knee. the sentence is already correct, select "No
(a) has felt (b) did feel substitution required’.
(c) had felt (d) was felt The team has been preparing for the match since
two months without a break.
Q2. Select the most appropriate option to improve (a) during two months (b) for two months
the bold part of the sentence. (c) No substitution (d) from two months
The plane landed in the airport.
(a) towards airport (b) into the airport Q7. Select the sentence with correct use of
(c) at the airport (d) to the airport article ‘the.’
(a) Kalidas is the Shakespeare of India.
Q3. Choose the most suitable option to replace (b) Dr. Johnson met the Shakespeare in England,
the underlined part of the sentence. If the (c) Dr. Johnson had instructed the Shakespeare
sentence is already correct, select 'No to write The Tempest.
improvement'. (d) Dr. Johnson saw the Shakespeare acting in
By the time we will reach , the concert will have King Lear.
started.
(a) we reach (b) we reached Q8. Select the most appropriate option to
(c) we have reached (d) No improvement improve the bold part of the sentence.
If I was you, I would accept the offer.
Q4. Choose the most suitable option to (a) have been you (b) were you
replace the underlined part of the sentence. If (c) am you (d) would be you
the sentence is already correct, select "No
substitution required’. Q9. Select the most appropriate option to improve
The old lady was unable to climb stairs because the bold part of the sentence.
her legs is aching constantly since morning.
(a) is been aching constantly He said me a lie .
(b) have been aching constantly (a) told me lie (b) said to me lie
(c) Had been aches constantly (c) told me a lie (d) said lie
(d) no substitution required
Q10. Select the most appropriate option to
Q5. Choose the sentence which uses the improve the bold part of the sentence.
adjective/adverb in its superlative degree
correctly. The man is blind of one eye .
(a) My uncle goes to that shop mostly frequently. (a) from one eye (b) with one eye
(b) My uncle goes to that shop the more (c) b y one eye (d) in One eye
frequently.
(c) My uncle goes to that shop very most Q11. Choose the most suitable option to replace
frequently. the underlined part of the sentence, If the
sentence is already correct, select "No
Telegram (Previous year papers PDFs [SSC,Railway,DSSSB,UP SI]): https://t.me/RBE_S
YouTube (Free lectures and job updates): https://www.youtube.com/c/RBERevolutionByEducation
Download RBE application for SSC Exams Complete Preparation
substitution required’. (c) for helping me (d) to helping me

Each of the candidates have submitted their Q17. Choose the most suitable option to replace
documents for verification. the underlined part of the sentence, If the
(a) h ad submitted their document sentence is already correct, select "No
(b) is submitted their documents substitution required’.
(c) no substitution required
(d) has submitted their documents If I was you , I would consult a specialist.
(a) were you (b) am you
Q12. Select the most appropriate option to (c) No error (d) Would be you
improve the bold part of the sentence.
Q18. Select the most appropriate option to
I am not used with this kind of work . improve the bold part of the sentence.
(a) at this kind of work
(b) to this kind of work He has much works to finish.
(c) f or this kind of work (a) plenty works (b) A lot of work
(d) About this kind of work (c) many Works (d) lot of tasks

Q13. Select the most appropriate option to Q19. Select the most appropriate option to
improve the bold part of the sentence. improve the bold part of the sentence.
The higher you go, the cooler it get.
The teacher explained him the topic. (a) coolest it becomes (b) more cooler it gets
(a) explained the topic to him (c) cooler it getting (d) cooler it gets
(b) told him the topic
(c) taught him the topic Q20. Select the most appropriate option to
(d) explained to him the topic improve the bold part of the sentence.
The manager discussed on the issue in detail.
Q14. Select the most appropriate option to (a) discussed the issue
improve the bold part of the sentence. (b) talked over the issue
(c) explained the issue
She said that she will go to market later. (d) discussed about the issue
(a) would go (b) Can go
(c) shall go (d) could go Q21. Select the most appropriate option to
improve the bold part of the sentence.
Q15. Choose the most suitable option to She prevented me to go there.
replace the underlined part of the sentence. If (a) from going (b) for going
the sentence is already correct, select "No (c) going to (d) against going
substitution required’.
Q22. Select the most appropriate option to
The research paper was full of errors because the improve the bold part of the sentence.
data is not verified by any expert. He is a honest man.
(a) has not been verified (a) one honest (b) an honest
(b) Was Not verified (c) the honest (d) honest
(c) no substitution required
(d) is not being verted yet Q23. Select the most appropriate option to
improve the bold part of the sentence.
Q16. Select the most appropriate option to He came here with a view to meet his old friend.
improve the bold part of the sentence. (a) in view to meet (b) with a view to meeting
(c) to meet with (d) for meeting
He denied to help me .
(a) helping me (b) help to me

Telegram (Previous year papers PDFs [SSC,Railway,DSSSB,UP SI]): https://t.me/RBE_S


YouTube (Free lectures and job updates): https://www.youtube.com/c/RBERevolutionByEducation
Download RBE application for SSC Exams Complete Preparation
Q24. Select the most appropriate option to The students was asked to submit their
improve the bold part of the sentence. assignments by Friday.
Scarcely had I left when it was starting to rain. (a) Will asked to submit their assignments
(a) did start (b) had started (b) Were asked to submit their assignments
(c) started (d) was begun (c) re asked to submitted their assignments
(d) No substitution required
Q25. Select the most appropriate option to
improve the bold part of the sentence. Q33. Select the most appropriate option to
Each of the students have a book. improve the bold part of the sentence.
(a) is having (b) gets I came to know this from a friend.
(c) has (d) had (a) to know that from
(b) to know this through
Q26. Select the most appropriate option to (c) to know of it from
improve the bold part of the sentence. (d) to know it by
Seldom we have seen such a performance.
(a) had we seen (b) have we seen Q34. Which among the following sentences uses
(c) we had seen (d) did we saw the article (if required) before the word 'honour'
correctly?
Q27. Select the most appropriate option to (a) It is an honour to be here.
improve the bold part of the sentence. (b) It is a honour to be here.
They returned back to their home. (c) It is honour to be here.
(a) came back to home (b) went back home (d) It is the honour to be here.
(c) returned to house (d) returned home
Q35. Choose the most suitable option to replace
Q28. Select the most appropriate option to the underlined part of the sentence. If the
improve the bold part of the sentence. sentence is already correct, select 'No
She is my cousin sister. substitution required'.
(a) cousin (b) own cousin She aren't believe in wasting time during working
(c) cousin girl (d) cousin sibling hours.
(a) No substitution required
Q29. Select the most appropriate option to (b) doesn’t believe
improve the bold part of the sentence. (c) wasn’t believes
She is married with a doctor. (d) isn’t believe
(a) in a doctor (b) to a doctor
(c) by a doctor (d) from a doctor Q36. Select the most appropriate option to
improve the bold part of the sentence.
Q30. Select the most appropriate option to N o sooner I reached the platform than the train
improve the bold part of the sentence. left.
This is the boy who he stole the purse. (a) I had no sooner reached
(a) who stole (b) who had stole (b) No sooner had I reached
(c) whom stole (d) which stole (c) I no sooner reached
(d) No sooner did I reach
Q31. Select the most appropriate option to
improve the bold part of the sentence. Q37. Select the most appropriate option to
The doctor advised that he takes rest. improve the bold part of the sentence.
(a) should takes rest (b) take the rest I prefer tea than coffee.
(c) take rest (d) must takes rest (a) instead coffee (b) rather coffee
(c) over coffee (d) to coffee
Q32. Choose the most suitable option to replace
the underlined part of the sentence. If the Q38. Select the most appropriate option to
sentence is already correct, select substitution improve the bold part of the sentence.
required.
Telegram (Previous year papers PDFs [SSC,Railway,DSSSB,UP SI]): https://t.me/RBE_S
YouTube (Free lectures and job updates): https://www.youtube.com/c/RBERevolutionByEducation
Download RBE application for SSC Exams Complete Preparation
H ardly he had reached the station when the (a) scores (b) had been scoring
train left. (c) have scored (d) had scored
(a) He had hardly reached
(b) No sooner he reached Q46. Select the most appropriate option to
(c) He hardly had reached improve the bold part of the sentence.
(d) Hardly had he reached This session is an improvement rather the
previous one.
Q39. Select the most appropriate option to (a) very better (b) more good
improve the bold part of the sentence. (c) best (d) over
She prefers to stay at home than go out.
(a) to staying home than going out Q47. Select the most appropriate option to
(b) staying at home to going out improve the bold part of the sentence.
(c) to stay at home to going out It is high time you leave the room.
(d) staying home rather going out (a) had left the room (b) left the room
(c) have left the room (d) leaves the room
Q40. Select the most appropriate option to
improve the bold part of the sentence. Q48. Select the most appropriate option to
He worked very hardly to pass the exam. improve the bold part of the sentence.
(a) hard (b) harder He insisted to go home .
(c) regularly (d) sincerely (a) going on (b) on going
(c) that he goes (d) upon going
Q41. Select the most appropriate option to
improve the bold part of the sentence. Q49. Select the most appropriate option to
The meeting will begin by 3 PM. improve the bold part of the sentence.
(a) around (b) from I am used to eat late at night.
(c) at (d) on (a) for eating (b) to eating
(c) in eating (d) eating
Q42. Choose the correct option to replace the
underlined word/phrase in the following sentence. Q50. Select the most appropriate option to
The car is moving very f astly on the road. improve the bold part of the sentence.
(a) fastliness (b) quicker The teacher asked that who was absent.
(c) quickliness (d) fast (a) who (b) which student
(c) whose (d) that whom
Q43. Which among the following sentences uses
adjective with correct degree of comparison? Q51. Select the most appropriate option to
(a) New Delhi is more hotter than Chandigarh. improve the bold part of the sentence.
(b) New Delhi is hotter to Chandigarh. Hardly I had entered the room when the phone
(c) New Delhi is more hotter to Chandigarh. rang.
(d) New Delhi is hotter than Chandigarh. (a) Hardly had I entered
(b) I hardly had entered
Q44. Select the most appropriate option to (c) Hardly was I entering
improve the bold part of the sentence. (d) I had hardly entered
She is confident t o win the match .
(a) in win the match Q52. Select the most appropriate option to
(b) in winning improve the bold part of the sentence.
(c) of winning the match The price of petrol has increased yesterday .
(d) to winning the match (a) increased yesterday
(b) risen yesterday
Q45. Select the most appropriate option to (c) rose yesterday
improve the bold part of the sentence. (d) has been increased yesterday
He is one of those players who has scored the
most goals.
Telegram (Previous year papers PDFs [SSC,Railway,DSSSB,UP SI]): https://t.me/RBE_S
YouTube (Free lectures and job updates): https://www.youtube.com/c/RBERevolutionByEducation
Download RBE application for SSC Exams Complete Preparation
Q53. Which of the following sentences is Q60. Which of the following sentences is
grammatically correct? grammatically correct?
(a) When we reached the station, the train has (a) He went to college in a train.
already left. (b) He went to college with a train.
(b) When we reached the station, the train have (c) He went to college by a train.
already left. (d) He went to college on a train.
(c) When we reached the station, the train had
already left. Q61. Select the most appropriate option to
(d) When we reached the station, the train improve the bold part of the sentence.
already left. She succeeded by virtue of her talent.
(a) cause of (b) because
Q54. Select the most appropriate option to (c) due to (d) owing of
improve the bold part of the sentence.
He is living here since 2020. Q62. Select the most appropriate option to
(a) He has been living here since 2020. improve the bold part of the sentence.
(b) He is staying here since 2020. He behaves as if he knows everything.
(c) He is here since 2020. (a) has known everything
(d) He lives here since 2020. (b) known everything
(c) have known all
Q55. Select the most appropriate option to (d) knew everything
improve the bold part of the sentence.
The wall is decorated by posters. Q63. Select the most appropriate option to
(a) painted in posters improve the bold part of the sentence.
(b) full of posters You must refrain to smoke.
(c) decorated with posters (a) quit from smoke (b) stop of smoking
(d) covered in posters (c) refrain from smoking (d) stop to smoke

Q56. Select the most appropriate option to Q64. Select the most appropriate option to
improve the bold part of the sentence. improve the bold part of the sentence.
She said me the truth. The criminal was accused for theft.
(a) said to me (b) says me (a) of theft (b) with theft
(c) told me (d) spoke to me (c) about theft (d) On theft

Q57. Select the most appropriate option to Q65. Select the most appropriate option to
improve the bold part of the sentence. improve the bold part of the sentence.
We waited for him since two hours. We enjoyed during the party last night.
(a) by two hours (b) for two hours (a) had enjoyment during party
(c) until two hours (d) from two hours (b) enjoyed in the party
(c) enjoyed ourselves at the party
Q58. Select the most appropriate option to (d) have enjoyed in party
improve the bold part of the sentence.
I wish I am taller. Q66. Select the most appropriate option to
(a) would be taller (b) could be taller improve the bold part of the sentence.
(c) were taller (d) was taller She did her work very good.
(a) really great (b) very well
Q59. Select the most appropriate option to (c) too well (d) quite good
improve the bold part of the sentence.
Had I known about the delay, I will wait. Q67. Select the most appropriate option to
(a) will have waited (b) would waited improve the bold part of the sentence.
(c) would have waited (d) waited I have visited Delhi last year.
(a) was visiting (b) did visit

Telegram (Previous year papers PDFs [SSC,Railway,DSSSB,UP SI]): https://t.me/RBE_S


YouTube (Free lectures and job updates): https://www.youtube.com/c/RBERevolutionByEducation
Download RBE application for SSC Exams Complete Preparation
(c) visited (d) had visited (a) several data
(b) much information
Q68. Select the most appropriate option to (c) many pieces of information
improve the bold part of the sentence. (d) lots of knowledge
She was not enough tall to play basketball.
(a) not tall enough (b) very short Q74. Select the most appropriate option to
(c) too short (d) not tall improve the bold part of the sentence.
He is enough rich to buy a car.
Q69. Choose the correct option that replaces the (a) rich enough (b) rich too much
underline word/phrase in the following sentence: (c) sufficiently rich (d) too rich
Sujata put in the best dress for the party.
(a) put across (b) put on Q75. Select the most appropriate option to
(c) put off (d) put out improve the bold part of the sentence.
The fiction and non-fictional book, that I
Q70. Select the most appropriate option to borrowed, was interesting.
improve the bold part of the sentence. (a) which I borrowed (b) what I borrowed
She has did her homework already. (c) whom I borrowed (d) whose I
(a) doing her homework borrowed
(b) done her homework
(c) made homework Q76. Select the most appropriate option to
(d) made her homework improve the bold part of the sentence.
He doesn’t go to shopping as often as he used
Q71. Select the most appropriate option to to.
improve the bold part of the sentence. None of (a) go shopping
the option seems appropriate. (b) seldom goes at shop
(a) None of the options (b) No optionals (c) is rarely going to shop
(c) Any of the optionals (d) No one of the (d) rarely goes to purchase

Q72. Select the most appropriate option to Q77. Select the most appropriate option to
improve the bold part of the sentence. improve the bold part of the sentence.
He did not knew the answer. According to the teacher, the Earth is revolving.
(a) knows (b) know (a) the earth is round.
(c) knowing (d) knews (b) spoke that the Earth is round
(c) told that Earth was round
Q73. Select the most appropriate option to (d) said that the Earth is round
improve the bold part of the sentence.
She gave me many informations .

Answer Key:

Q1 c Q2 c Q3 a Q4 b Q5 d
Q6 b Q7 a Q8 b Q9 c Q10 d
Q11 d Q12 b Q13 a Q14 a Q15 b
Q16 a Q17 a Q18 b Q19 d Q20 a
Q21 a Q22 b Q23 b Q24 c Q25 c
Q26 b Q27 d Q28 a Q29 b Q30 a
Q31 c Q32 b Q33 b Q34 a Q35 b
Q36 b Q37 d Q38 d Q39 b Q40 a
Q41 c Q42 d Q43 d Q44 c Q45 c
Q46 d Q47 b Q48 b Q49 b Q50 a
Telegram (Previous year papers PDFs [SSC,Railway,DSSSB,UP SI]): https://t.me/RBE_S
YouTube (Free lectures and job updates): https://www.youtube.com/c/RBERevolutionByEducation
Download RBE application for SSC Exams Complete Preparation
Q51 a Q52 a Q53 c Q54 a Q55 c
Q56 c Q57 b Q58 c Q59 c Q60 d
Q61 c Q62 d Q63 c Q64 a Q65 c
Q66 b Q67 c Q68 a Q69 b Q70 b
Q71 a Q72 b Q73 c Q74 a Q75 a
Q76 a Q77 a

Solution 1: Explanation frequently," "very most frequently") are


The past perfect tense "had felt" is used to grammatically incorrect or improper usage of
indicate that the patient's feeling of pain superlative forms
happened before the doctor gave the medication.
The other options ("has felt," "did feel," "was felt") Solution 6: Explanation
are not appropriate for this sequence of past "Since" is used with a specific point in time (e.g.,
events. since January, since 2010).
"For" is used with a duration or period of time
Solution 2: Explanation (e.g., for two months, for three days).
The correct preposition to use with "landed" in Therefore, "for two months" is the correct phrase
this context is "at," as planes land at an airport. to express the duration of preparation.
The other options ("towards airport," "into the
airport," "to the airport") are incorrect or awkward Solution 7: Explanation
in this context. "The" is used here correctly because it refers to a
specific and well-known person (Shakespeare) to
Solution 3: Explanation express that Kalidas is being compared to this
In sentences with "By the time," the clause particular individual.
following it should be in the simple present tense, In the other options, "the" is incorrectly used
even if the main clause is in the future perfect before "Shakespeare" as if Shakespeare is a
tense. common noun rather than a proper noun/name.
The correct structure is: "By the time + present Proper nouns like names generally do not take an
simple, ... will have + past participle." article unless used in a comparative or group
Example: "By the time we reach, the concert will sense (as in the correct sentence).
have started."
The other options ("we reached," "we have Solution 8: Explanation
reached," "No improvement") are incorrect in this In conditional sentences expressing hypothetical
future perfect context. situations, the subjunctive mood uses "were"
instead of "was" after "If I."
Solution 4: Explanation "If I were you" is the grammatically correct and
The present perfect continuous tense "have been standard form.
aching" is appropriate here because the action
(aching) started in the past and continues up to Solution 9: Explanation
the present. The verb "say" does not take an indirect object
The subject "legs" is plural, so the auxiliary verb directly; it usually pairs with "to" (e.g., "said to
should be "have" (not "is"). me").
"Is been aching" and "Had been aches" are The verb "tell" correctly takes an indirect object
incorrect forms grammatically. without a preposition and is used when giving
information or stating something to someone.
Solution 5: Explanation Adding the article "a" before "lie" is necessary as
The word "frequently" is an adverb ending in -ly, "lie" is a countable noun here.
so the superlative form is made by adding "most"
before it ("most frequently"). Solution 10: Explanation:
Other options ("mostly frequently," "the more The correct preposition to use with "blind" when
Telegram (Previous year papers PDFs [SSC,Railway,DSSSB,UP SI]): https://t.me/RBE_S
YouTube (Free lectures and job updates): https://www.youtube.com/c/RBERevolutionByEducation
Download RBE application for SSC Exams Complete Preparation
referring to loss of sight in a specific eye is "in." "Has not been verified" is present perfect, which
"Blind in one eye" is a standard expression in doesn't fit the past context as well as simple past.
English. "Is not being verified yet" changes the meaning
Other options like "of one eye," "for one eye," or and tense improperly.
"with one eye" are grammatically incorrect in this Therefore, Was not verified is the best choice.
context.
Solution 16: Explanation:
Solution 11: Explanation: After the verb "deny," the gerund ( verb+ing ) form
The subject "Each" is singular, so it requires a is used.
singular verb. The structure "denied helping me" is
Therefore, "have" should be changed to "has" to grammatically correct, while "denied to help me"
agree with the singular subject. is incorrect.
The correct sentence is: "Each of the Therefore, the correct choice is helping me.
candidates has submitted their documents for
verification." Solution 17: Explanation:
In hypothetical or unreal conditional sentences,
Solution 12: Explanation: the subjunctive mood is used.
The correct phrase is "used to" when referring to The correct form is "If I were" instead of "If I was"
being accustomed to something. when talking about a situation contrary to fact or
Other options like "used with," "used for," or "used hypothetical.
about" are incorrect in this context. This is a standard grammatical rule for unreal
Therefore, the right choice is to this kind of work. conditions.

Solution 13: Explanation: Solution 18: Explanation:


The verb "explain" usually takes a direct object "Work" is an uncountable noun, so it does not
and a prepositional phrase with "to" to indicate take "much works" or "many works."
the person being explained to. "A lot of" is used with both countable and
"Explained him the topic" is incorrect because uncountable nouns and fits perfectly here with
"explain" is not used with an indirect object "work."
without "to." "Works" generally refers to multiple creations or
Other options like "told him the topic" or "taught pieces, which is not the meaning here.
him the topic" change the meaning slightly or are
less precise. Solution 19: ✅ Correct Answer: cooler it gets
"Explained to him the topic" is grammatically The sentence follows a comparative correlative
possible but less natural than "explained the topic structure : “The higher you go, the cooler it
to him." gets.”
The word “get” must agree with the subject “it,”
Solution 14: Explanation: so the correct form is “gets.
In reported speech, "will" changes to "would" to
reflect the shift in tense. Solution 20: ✅ Correct Answer : discussed the
Other options like "can go," "shall go," or "could issue
go" do not correctly follow the rules of tense 📌 Point-wise Explanation :
backshift in reported speech.
Verb Usage – "Discuss" : The verb "discuss" is
Therefore, the correct choice is would go.
transitive , which means it directly takes an object
without any preposition like "on" or "about".
Solution 15: Explanation:
Incorrect Phrase – "discussed on" : ❌ "discussed
The subject "data" is singular or treated as
on the issue" is grammatically incorrect because
singular here, so the verb should be "was."
"discuss" does not require "on".
The past tense "was not verified" matches the
Correct Phrase – "discussed the issue" : ✅ This
past tense "was full of errors."
"Is not verified" is present tense and inconsistent is the proper construction: ✔ "The manager
with past tense context. discussed the issue in detail."

Telegram (Previous year papers PDFs [SSC,Railway,DSSSB,UP SI]): https://t.me/RBE_S


YouTube (Free lectures and job updates): https://www.youtube.com/c/RBERevolutionByEducation
Download RBE application for SSC Exams Complete Preparation
Solution 21: ✅ Correct Answer : from going Solution 26: ✅ Correct Answer: have we seen
"Prevented me to go " ❌ is incorrect because “Seldom” is a negative adverb of frequency.
"prevent" is never followed by "to + verb" When placed at the beginning of a sentence, it
Correct structure: ✔ prevent someone from triggers inversion (like in questions).
doing something Correct structure: Seldom + auxiliary verb +
subject + main verb
Solution 22: ✅ Correct Answer : an honest
Solution 27: ✅ Correct Answer: returned home
Why "an honest" is correct : ✔ "honest" begins
The phrase “returned back” is redundant .
with a vowel sound → an honest
“Return” already implies coming back, so “back”
is unnecessary.
Solution 23: ✅ Correct Answer : with a view to
meeting
Solution 28: ✅ Correct Answer: cousin
🔍 Explanation:
The phrase “cousin sister” is non-standard in
Fixed Phrase Rule : The expression "with a view
English.
to" is a standard idiom in English. It means "with
In standard usage, “cousin” alone is gender-
the intention of doing something." 👉 Example: neutral and sufficient.
She enrolled in the course with a view to Adding “sister,” “girl,” or “sibling” is redundant
improving her skills. or incorrect .
Gerund Usage : After "to" in this idiom, we do not
use the base verb (like "meet"). Instead, we use Solution 29: ✅ Correct Answer: to a doctor
the gerund form (verb + ing ). So, it should be
🔍 Step-by-Step Analysis:
"meeting" , not "meet" .
Original Sentence :
❌ Incorrect usage of “with” in this context.
Solution 24: ✅ Correct Answer: started
Correct Preposition :
🔍 Explanation:
The standard expression is: married to [person]
Original Sentence : Scarcely had I left when it
“With” may be used informally in some dialects,
was starting to rain.
but “to” is grammatically correct and universally
This mixes past perfect ("had I left") with past
accepted.
continuous ("was starting") — which is
grammatically awkward.
Solution 30: ✅ Correct Answer: who stole
Rule of “Scarcely…when” :
“Scarcely” is used with past perfect , followed by 🔍 Step-by-Step Analysis:
simple past . Original Sentence :
Structure: Scarcely + had + subject + past ❌ Grammatically incorrect: “who he stole”
participle , when + simple past creates a redundancy.
Corrected Sentence : Scarcely had I left when it “Who” already acts as the subject of the relative
started to rain. ✅ clause, so “he” is unnecessary.
Correct Structure :
Solution 25: ✅ Correct Answer: has who + verb
→ who stole the purse
🔍 Explanation:
Subject Analysis :
Solution 31: ✅ Correct Answer: take rest
Each of the students is a singular subject , even
though “students” is plural. 🔍 Step-by-Step Analysis:
“Each” refers to individual members , so the verb Original Sentence :
must be singular . ❌ Incorrect: “takes” violates the subjunctive
Correct Verb Form : mood rule used after verbs like advise, suggest,
“Each of the students has a book.” ✅ recommend, demand, insist .
“has” = singular present tense verb matching Correct Structure :
“each” After “advised that,” we use the base form of the
verb (bare infinitive), regardless of subject.
So: “he take rest” is grammatically correct.
Telegram (Previous year papers PDFs [SSC,Railway,DSSSB,UP SI]): https://t.me/RBE_S
YouTube (Free lectures and job updates): https://www.youtube.com/c/RBERevolutionByEducation
Download RBE application for SSC Exams Complete Preparation
Common Usage : pronunciation begins with a vowel sound.
In Indian English, “take rest” is idiomatic and
widely accepted. Solution 35: ✅ Correct Answer
In other varieties, “get some rest” or “rest” may be doesn’t believe
preferred, but “take rest” is valid and contextually 🔍 Point-wise Explanation
appropriate here. Subject–Auxiliary Agreement
She → third person singular
Solution 32: ✅ Correct Answer aren’t → ❌ incorrect ; “aren’t” is plural or second
Were asked to submit their assignments person (you aren’t, they aren’t)
🔍 Detailed Explanation (Point-wise) Correct auxiliary for negative present simple:
Subject Identification doesn’t
The students → Plural subject Tense and Verb Form
Therefore, the verb must agree in plural form Present simple negative structure: Subject +
Verb Form Check doesn’t + base verb
Was asked → Singular past tense passive So: She doesn’t believe ✅
❌ Incorrect for plural subject
Correct Passive Construction Solution 36: The correct answer is " No sooner
Passive voice structure: Subject + was/were + had I reached." The expression "No
past participle sooner...than..." requires an inverted structure
For plural subject ( students ), use were asked with the auxiliary "had" coming before the subject
Corrected Sentence for correct grammar (past perfect tense). The
✅ The students were asked to submit their other options are either incorrect in structure or
assignments by Friday. do not use the idiomatic form appropriately.

Solution 37: The correct answer is " to coffee ."


Solution 33: ✅ Correct Answer The verb "prefer" is correctly followed by "to"
to know this through when comparing two things ("prefer A to B").
🔍 Point-wise Explanation Therefore, the correct and idiomatic expression is
Original Phrase : "I prefer tea to coffee." The other options do not fit
I came to know this from a friend standard English usage in this context.
Grammatically acceptable, but “through” is more
idiomatic and formal in this context. Solution 38: The correct answer is "Hardly had
Meaning & Usage : he reached." The correct structure with "hardly" at
“Through a friend” implies the medium or channel the beginning of a sentence is an inverted form:
by which the information was received. "Hardly had he reached the station when the train
It sounds more polished and natural in formal or left." This follows the standard English inversion
written English. rule for sentences beginning with negative
Improved Sentence : adverbs.
✅ I came to know this through a friend.
Solution 39: The correct answer is " staying at
Solution 34: ✅ Correct Answer home to going out ." The structure with "prefer"
is "prefer [doing A] to [doing B]." Therefore, "She
It is an honour to be here.
prefers staying at home to going out" is the
🔍 Point-wise Explanation
grammatically correct and idiomatic form. The
Article Usage Rule
other options use incorrect words or prepositions.
Use “an” before words that begin with a vowel
sound , not just a vowel letter.
Solution 40: The correct answer is " hard." The
“Honour” begins with a silent ‘h’ , so the word
word "hardly" means "scarcely" or "barely," which
starts with an ‘o’ sound → vowel sound.
doesn't fit the context. The sentence should read
Correct Article
"He worked very hard to pass the exam," where
✅ “an honour” is correct because the
"hard" is an adverb meaning "with great effort or
energy." The other options don't fit grammatically
Telegram (Previous year papers PDFs [SSC,Railway,DSSSB,UP SI]): https://t.me/RBE_S
YouTube (Free lectures and job updates): https://www.youtube.com/c/RBERevolutionByEducation
Download RBE application for SSC Exams Complete Preparation
or contextually in this sentence structure. context.

Solution 41: The correct answer is " at ." When Solution 47: The correct usage after "It is high
specifying the exact time something begins, we time" is the simple past tense. Therefore, "It is
use "at." "The meeting will begin at 3 PM" high time you left the room" is grammatically
indicates the meeting starts exactly at 3 PM. "By correct. The other options ("had left," "have left,"
3 PM" would mean no later than 3 PM, which "leaves") do not fit the idiomatic usage of this
suggests it could start earlier. The other options phrase.
don't fit: "around" is too vague, "from" suggests
duration, and "on" is incorrect for specific times. Solution 48: The correct form after "insist" is
"insist on going," not "to go." The verb "insist" is
Solution 42: The correct answer is " fast ." The properly followed by "on" + gerund, making "He
word " fastly " is incorrect because "fast" functions insisted on going home" the grammatically
as both an adjective and an adverb. When accurate construction. The other options do not fit
describing how something moves, we use "fast" standard English usage with the verb "insist."
as an adverb, not " fastly ." The correct sentence
should read: "The car is moving very fast on the Solution 49: The correct phrase is "used to
road." The other options are either non- standard eating," which uses "to" as a preposition followed
words or don't fit the grammatical structure. by a gerund. "I am used to eating late at night" is
grammatically correct. The structure "used to"
Solution 43: The correct answer is " New Delhi is should always be followed by a noun or gerund,
hotter than Chandigarh ." This sentence uses not an infinitive ("to eat"). The other options do
the correct comparative form "hotter" (not " more not conform to the proper idiomatic usage.
hotter " which is a double comparative error) and
the correct preposition "than" for comparisons. Solution 50: The correct phrase is "The teacher
The other options contain either double asked who was absent." The word "that" is
comparative errors (" more hotter ") or incorrect unnecessary and ungrammatical in this context.
prepositions ("to" instead of "than"). "Who" is the appropriate interrogative pronoun to
use when asking about the identity of someone
Solution 44: The correct answer is " of winning who was absent. Other options either do not fit
the match." The adjective "confident" is the sentence context or are incorrect forms.
followed by the preposition "of" when
expressing certainty about an outcome or Solution 51: The correct construction is "Hardly
ability. "She is confident of winning the match" is had I entered," which is an example of inversion
the grammatically correct construction. "Confident used after negative adverbials like "hardly,"
to win" is not standard English usage, and the "scarcely," or "no sooner." The verb comes before
other options either have grammatical errors or the subject for emphasis and correct grammar.
are less appropriate than the correct answer. Other options do not correctly follow this rule or
change the intended meaning.
Solution 45: The correct answer is "have scored"
because the relative clause "who have scored" Solution 52: The use of "has increased" is
refers to the plural antecedent "players." Thus, incorrect with a definite past time marker such as
the verb must be in the plural form to ensure "yesterday." For actions completed at a known
subject-verb agreement. time in the past, the simple past tense should be
used. Therefore, the correct usage is "The price
Solution 46: The phrase "rather the previous one" of petrol increased yesterday." "Risen yesterday"
is incorrect. The correct comparative expression and "rose yesterday" are either grammatically
is "over the previous one," as in "This session is incorrect or do not fit with "price" as the noun.
an improvement over the previous one." The "Has been increased yesterday" is incorrect
other options ("very better," " more good ," "best") because present perfect passive does not
are grammatically incorrect or do not fit the combine with a specific past time.

Telegram (Previous year papers PDFs [SSC,Railway,DSSSB,UP SI]): https://t.me/RBE_S


YouTube (Free lectures and job updates): https://www.youtube.com/c/RBERevolutionByEducation
Download RBE application for SSC Exams Complete Preparation
Solution 53: The past perfect tense ("had already other options are either grammatically incorrect or
left") is used to show that one past action (the do not fit the context of speculative past actions.
train leaving) was completed before another
(reaching the station). Other options either use Solution 60: The correct preposition to use for
incorrect tenses, have a subject-verb traveling in a vehicle such as a train is "on." "He
disagreement, or do not properly sequence past went to college on a train" is grammatically
actions. correct. Other options use incorrect or awkward
prepositions for this context.
Solution 54: The correct tense with "since" for a
period extending from the past until now is the Solution 61: ✅ Correct Answer: due to
present perfect continuous. "He has been living 🎯 Explanation:
here since 2020" accurately expresses an action The original phrase “by virtue of her talent” is
that started in the past and is still ongoing. The grammatically correct and formal. However, if
other options use incorrect tense forms and do we’re looking to improve or simplify it with a
not convey the ongoing nature as required. more commonly used alternative, “due to” is the
most appropriate choice.
Solution 55: The correct usage is "decorated with
posters," as "with" indicates the material or object Solution 62: ✅ Correct Answer: knew
used for decoration. "By" is used for agents
everything
performing an action and is incorrect in this
🧠 Explanation:
context. The other options do not fit standard
The sentence “He behaves as if he knows
English expressions for describing walls with
everything” uses the phrase “as if” , which
posters.
typically requires a subjunctive mood when
expressing something unreal, imaginary, or
Solution 56: The verb "said" cannot be used
contrary to fact .
directly with an object; instead, "told" is the
So, even though he doesn’t actually know
correct verb to use when stating information given
everything , the correct form is:
to a person. The correct expression is "She told
➡ “He behaves as if he knew everything.”
me the truth," which is grammatically accurate.
Other options either use incorrect verb forms or This use of past tense (“knew”) reflects a
prepositions. hypothetical or doubtful situation , not a real
one.
Solution 57: The correct phrase is "for two
hours," not " since two hours." "Since" is used Solution 63: ✅ Correct Answer: refrain from
with a specific starting point in time, while "for" is smoking
used with a period or duration. The sentence 🧠 Explanation:
should be "We waited for him for two hours." The verb “refrain” is always followed by the
Other options are not grammatically correct for preposition “from” and a gerund (– ing form) of
this context. the verb.
So, the correct construction is:
Solution 58: The correct form is "I wish I were ➡ “You must refrain from smoking.”
taller." In English grammar, "were" is used for
unreal or hypothetical wishes contrary to present Solution 64: ✅ Correct Answer: of theft
reality, regardless of the subject ("I" takes "were"). 🧠 Explanation:
The other options either change the meaning or The verb “accused” is always followed by the
don't follow the subjunctive mood structure. preposition “of” when referring to the crime or
wrongdoing someone is charged with.
Solution 59: The correct expression for the ➡ Correct sentence: The criminal was accused
context of an unreal condition in the past is
of theft.
"would have waited." The form "Had I known
about the delay, I would have waited" correctly
Solution 65: ✅ Correct Answer: enjoyed
conveys the hypothetical nature of the action. The
ourselves at the party
Telegram (Previous year papers PDFs [SSC,Railway,DSSSB,UP SI]): https://t.me/RBE_S
YouTube (Free lectures and job updates): https://www.youtube.com/c/RBERevolutionByEducation
Download RBE application for SSC Exams Complete Preparation
🧠 Explanation: Solution 70: ✅ Correct Answer: done her
Original Sentence: ➡ We enjoyed during the homework
party last night. Why “done her homework” Is Correct:
🔴 Error: The verb “enjoy” is not used alone The auxiliary verb “has” requires the past
when referring to personal experiences. ✅ participle form of the main verb.
Correct usage: enjoy oneself → “We enjoyed ✅ do → did → done ➤ “ done ” is the correct past
ourselves…” participle to follow “ has ” .

Solution 66: ✅ Correct Answer: very well Solution 71: Correct Answer: ✅ Option 1 –
🧠 Explanation: None of the options
🔍 Explanation:
Original Sentence: ➡ She did her work very
good. The original sentence is: “None of the option
seems appropriate.”
🔴 Error: “Good” is an adjective, but here we
This contains a grammatical error :
need an adverb to describe how she did her
“Option” should be plural → “options”
work.
The verb “seems” is correct because “None” is
✅ Correct adverb: “well” ➡ She did her work
treated as singular in formal usage.
very well.
Solution 72: ✅ Correct Answer: Option 2 –
Solution 67: ✅ Correct Answer: visited
know
🧠 Explanation: 🔍 Explanation:
Original Sentence: I have visited Delhi last year. Original Sentence: He did not knew the answer.
🔸 This sentence is incorrect because “have This is grammatically incorrect because:
visited” (present perfect) should not be used with The auxiliary verb “did” already indicates past
a specific time reference like last year . tense.
✅ Correct Usage: So, the main verb must be in its base form , not
When a specific past time is mentioned (e.g., past tense.
last year, yesterday, in 2020 ), we use simple
past tense . Solution 73: ✅ Correct Answer: Option 3 –
🔁 Corrected Sentence: ➡ I visited Delhi last many pieces of information
year. 🔍 Explanation:
The word “information” is uncountable , so it
Solution 68: ✅ Correct Answer: not tall cannot be used with “many” or made plural as
enough informations . To express quantity correctly, we
🧠 Why This Is Correct: use:
In English, the adjective comes before “enough” “ much information” – general quantity
when describing degree or sufficiency . “ many pieces of information” – specific
✅ Correct structure: [adjective] + enough → tall countable units
enough “ lots of knowledge” – related, but “knowledge”
❌ Incorrect: enough tall (violates standard word ≠ “information” exactly
order) “ several data” – incorrect; data is plural of
datum , but usage varies
Solution 69: ✅ Correct Answer: put on
Solution 74: ✅ Correct Answer: Option 1 – rich
🧠 Why “put on” Is Correct:
enough
“Put on” means to wear or dress oneself in
🔍 Explanation:
something .
It’s the standard phrasal verb used when referring Original Sentence: He is enough rich to buy a
to clothing or accessories . car. This is grammatically incorrect because:
The word “enough” should come after the
adjective ( rich ) — not before.

Telegram (Previous year papers PDFs [SSC,Railway,DSSSB,UP SI]): https://t.me/RBE_S


YouTube (Free lectures and job updates): https://www.youtube.com/c/RBERevolutionByEducation
Download RBE application for SSC Exams Complete Preparation
Correct structure: adjective + enough 🔍 Explanation:
Original Sentence:
Solution 75: ✅ Correct Answer: Option 1 – He doesn’t go to shopping as often as he used to.
which I borrowed The phrase “go to shopping” is grammatically
🔍 Explanation: incorrect. The correct expression is:
Original Sentence: The fiction and non-fictional 👉 “go shopping” (without “to”)
book, that I borrowed , was interesting.
The phrase “that I borrowed” is grammatically Solution 77: 🔍 Explanation:
acceptable in informal usage, but when preceded Original Sentence:
by a comma , the correct relative pronoun is According to the teacher, the Earth is revolving.
“which” , not “that”. This sentence is grammatically correct but
Also, “which” is used for non-defining relative factually imprecise . The intended meaning
clauses , which this sentence contains due to the seems to be a general scientific statement —
comma. that the Earth is round , not that it is "revolving"
(which is also true, but less likely to be the
Solution 76: ✅ Correct Answer: Option 1 – go teacher’s quoted statement in this context).
shopping

For SSC Exams Complete Preparation (Download RBE Application)


(Learn from those who have cleared the exam themselves)
https://play.google.com/store/apps/details?id=com.revolution.education

Telegram (Previous year papers PDFs [SSC,Railway,DSSSB,UP SI]): https://t.me/RBE_S


YouTube (Free lectures and job updates): https://www.youtube.com/c/RBERevolutionByEducation
Download RBE application for SSC Exams Complete Preparation

SSC Steno 2025 T-1 Error Detection All 70 Questions with Detailed Solution and Answer key

Q1. Find the part of the sentence that contains Q6. Find the incorrect part of the sentence:
an error: He was senior than me in college.
(a) No error (b) He was
Each participant who registered online for the (c) senior than (d) me in the college
event (1)/ were given a welcome kit (2)/ along
with the program schedule and map (3)/ at the Q7. Identify the part containing a grammatical
helpdesk near the entrance. (4)/ or structural error:
(a) along with the program schedule and map (3) Neither the Principal nor the teachers was
(b) Each participant who registered online for the present.
event (1) (a) nor the teachers (b) No error
(c) were given a welcome kit (2) (c) Neither the principal (d) was present
(d) at the helpdesk near the entrance. (4)
Q8. Find the part of the sentence that contains
Q2. Identify the error in the sentence. an error:
Identify the error in the sentence. The teacher who had conducted the workshop
By the time they arrive, the concert had earlier, (1)/ along with his assistants, (2)/ have
ended. submitted the final report to the board (3)/ before
(a) ended (b) arrive the deadline last week. (4)
(c) time (d) concert (a) along with his assistants (2)
(b) before the deadline last week (4)
Q3. Find the part of the sentence that contains (c) have submitted the final report to the board (3)
an error: (d) The teacher who had conducted the workshop
One of the guest speakers who addressed the earlier {1)
seminar (1)/ and interacted with students (2)/
were praised widely for their clarity and Q9. Find the incorrect part of the sentence:
enthusiasm (3)/ during the closing ceremony. (4) Neither Ram nor his brothers is reliable.
(a) were praised widely for their clarity and (a) No error (b) Neither Ram
enthusiasm (3) (c) nor his brothers (d) is reliable
(b) and interacted with students (2)
(c) during the closing ceremony (4) Q10. I dentify the part containing a
(d) One of the guest speakers who addressed the grammatical or structural error:
seminar (1) The behavior of these students leave much to be
desired
Q4. Identify the error in the sentence. (a) of these students (b) leave much
Can you pick up me from the airport tomorrow? (c) to be desired (d) The behavior
(a) airport tomorrow (b) me from
(c) from the airport (d) pick up me Q11. Identify the error in the sentence.
He has saw the movie three times already.
Q5. Find the incorrect part of the sentence: (a) saw (b) already
She has been teaching in this school since five (c) movie (d) three
years.
(a) in the school (b) No error Q12. Find the incorrect part of the sentence:
(c) She has been teaching (d) Since 5 years She is more smarter than her sister.
(a) She is (b) than her sister
Telegram (Previous year papers PDFs [SSC,Railway,DSSSB,UP SI]): https://t.me/RBE_S
YouTube (Free lectures and job updates): https://www.youtube.com/c/RBERevolutionByEducation
Download RBE application for SSC Exams Complete Preparation
(c) no error (d) More Smarter (c) the pressure (d) cope up with

Q13. Identify the word i n this sentence that Q22. Choose the part of the sentence that
contains an error. contains an error:
Neither the teacher nor the students was present.
She arrived to the office early this morning. (a) nor the students (b) was present
(a) to (b) morning (c) Neither the teacher (d) No error
(c) office (d) arrived
Q23. Choose the part of the sentence that
Q14. Find the incorrect part of the sentence: contains an error:
She gave me a good advice.
One must keep his promises. (a) No error (b) good
(a) his promises (b) One (c) a good advice (d) She gave me
(c) no error (d) must keep
Q24. Find the incorrect part of the sentence:
Q15. Identify the part containing a I returned back from Mumbai last week.
grammatical or structural error: (a) last week (b) from Mumbai
(c) I returned back (d) No error
It is high time you start studying seriously.
(a) studying seriously (b) no error Q25. Identify the error in the sentence.
(c) It is high time (d) you start She danced graceful on stage during the annual
show.
Q16. Identify the word in this sentence that (a) stage (b) Show
contains an error. (c) Graceful (d) during
The keys are inside the table drawer.
(a) table (b) keys Q26. Find the incorrect part of the sentence:
(c) inside (d) drawer She is angry upon me.
(a) me (b) No error
Q17. Find the incorrect part of the sentence: (c) She is (d) angry upon
She is junior than me.
(a) No error (b) me Q27. Identify the part containing a grammatical or
(c) Junior than (d) She is structural error:
No sooner he had arrived than the meeting
Q18. Identify the error in the sentence. started.
This route is most safer than the highway. (a) No error (b) he had arrived
(a) highway (b) most (c) than the meeting started (d) No sooner
(c) route (d) safer
Q28. Find the incorrect part of the sentence:
Q19. Find the incorrect part of the sentence: The officer has issued an order to arrest him
The police has arrested the culprit. immediately.
(a) the culprit (b) No error (a) an order to arrest him immediately
(c) has arrested (d) The police (b) No error
(c) The officer
Q20. Identify the part containing a grammatical (d) Has issued
error.
Neither Ramesh nor his friends is willing to join. Q29. Identify the error in the sentence.
(a) No error (b) is willing to join Mount Everest is more highest peak in the world.
(c) Neither Ramesh (d) nor his friends (a) E verest (b) world
(c) more (d) highest
Q21. Find the incorrect part of the sentence:
He could not cope up with the pressure.
(a) He could not (b) No error
Telegram (Previous year papers PDFs [SSC,Railway,DSSSB,UP SI]): https://t.me/RBE_S
YouTube (Free lectures and job updates): https://www.youtube.com/c/RBERevolutionByEducation
Download RBE application for SSC Exams Complete Preparation
Q30. Identify the part containing a grammatical or Q39. Find the incorrect part of the sentence:
structural error: You should not laugh on others.
The teacher advised to listen attentively. (a) You should not (b) laugh on
(a) advised (b) The teacher (c) No error (d) others
(c) No error (d) to listen attentively
Q40. Find the incorrect part of the sentence:
Q31. Find the incorrect part of the sentence: This book comprises of five chapters.
She congratulated him for his success. (a) This book (b) No error
(a) him for (b) She congratulated (c) five chapters (d) comprises of
(c) No error (d) his success!
Q41. Choose the part of the sentence that
Q32. Identify the error in the sentence. contains an error:
He made a quick shower before heading to the You should avail of every opportunity.
office. (a) No error (b) avail of
(a) made (b) quick (c) every opportunity (d) You should
(c) before (d) shower
Q42. Identify the part containing a grammatical or
Q33. Identify the part containing a grammatical or structural error:
structural error: The father with his sons are arriving soon.
The manager together with his staff have gone for (a) No error (b) The father
lunch. (c) are arriving soon (d) with his sons
(a) No error (b) have gone for lunch
(c) The manager (d) together with his staff Q43. Find the incorrect part of the sentence:
The judge ordered the prisoner to hung till death.
Q34. Find the incorrect part of the sentence: (a) The judge ordered (b) the prisoner
She is one of the best singer in the city. (c) to hung till death (d) No error
(a) singer in the city (b) of the best
(c) No error (d) She is one Q44. Find the incorrect part of the sentence:
Neither of the two brothers are trustworthy.
Q35. Identify the error in the sentence. (a) are (b) Neither of the two brothers
She adopted a honest approach to resolving the (c) No error (d) trustworthy
issue.
(a) honest (b) issue Q45. Find the incorrect part of the sentence:
(c) a (d) resolving He divided the sweets between his five friends.
(a) between his five friends (b) the sweets
Q36. Identify the error in the sentence. (c) He divided (d) No error
He is a fastly growing leader in the organization.
(a) leader (b) growing Q46. Choose the part of the sentence that
(c) fastly (d) organization contains an error:
The child is afraid from the dark.
Q37. Identify the part containing a grammatical or (a) the dark (b) The child is
structural error: (c) No error (d) afraid from
One of the questions were very difficult.
(a) were (b) very difficult Q47. Identify the part containing a grammatical or
(c) No error (d) One of the questions structural error:
The jury was divided in their opinions.
Q38. Choose the part of the sentence that (a) The jury (b) in their opinions
contains an error: (c) No error (d) was divided
He succeeded to win the prize.
(a) the prize (b) He succeeded Q48. Find the incorrect part of the sentence:
(c) No error (d) to win The old man fell down from the stairs.
(a) No error (b) The old man
Telegram (Previous year papers PDFs [SSC,Railway,DSSSB,UP SI]): https://t.me/RBE_S
YouTube (Free lectures and job updates): https://www.youtube.com/c/RBERevolutionByEducation
Download RBE application for SSC Exams Complete Preparation
(c) fell down (d) from the stairs Q57. Choose the part of the sentence that
contains an error:
Q49. Choose the part of the sentence that She resembles with her mother.
contains an error: (a) mother (b) with her mother
My elder sister is married with a doctor. (c) No error (d) She resembles
(a) No error (b) a doctor
(c) My elder sister (d) is married with Q58. Find the incorrect part of the sentence:
She is better in mathematics than English.
Q50. Choose the part of the sentence that (a) better in mathematics (b) than English
contains an error: (c) No error (d) She is
She insisted to accompany me.
(a) She insisted (b) No error Q59. Choose the part of the sentence that
(c) me (d) to accompany contains an error:
She does not know to swim.
Q51. Find the incorrect part of the sentence: (a) She does not (b) No error
No less than fifty students were absent. (c) Swim (d) Know to swim
(a) fifty students (b) were absent
(c) No error (d) No less than Q60. Identify the part containing a grammatical or
structural error:
Q52. Identify the part containing a grammatical or The news are too good to be true.
structural error: (a) are too good (b) The news
Hardly had he left the room when the guests have (c) to be true (d) No error
arrived.
(a) left the room Q61. Identify the part containing a grammatical or
(b) when the guests have arrived structural error:
(c) Hardly had he He asked me that where I was going.
(d) No error (a) He asked me (b) I was going
(c) No error (d) that where
Q53. Choose the part of the sentence that
contains an error: Q62. Find the incorrect part of the sentence:
He is living in Chennai since five years. The sceneries of Ooty were wonderful.
(a) in Chennai (b) He is living (a) were wonderful (b) No error
(c) No error (d) since five years (c) The sceneries (d) of Ooty

Q54. Find the incorrect part of the sentence: Q63. Find the incorrect part of the sentence:
He has come yesterday. I have visited Agra last month.
(a) He has come (b) come (a) Agra (b) I have visited
(c) No error (d) yesterday (c) No error (d) last month

Q55. Find the incorrect part of the sentence: Q64. Find the incorrect part of the sentence:
The ship drowned in the sea. She married with a doctor.
(a) in the sea (b) The ship (a) No error (b) with a doctor
(c) No error (d) drowned (c) She married (d) with

Q56. Choose the part of the sentence that Q65. Choose the part of the sentence that
contains an error: contains an error:
I prefer tea than coffee. The committee have given its decision.
(a) I prefer (b) tea than (a) its decision (b) have given
(c) coffee (d) No error (c) No error (d) The committee

Q66. Find the incorrect part of the sentence:


She prevented me to enter the hall.
Telegram (Previous year papers PDFs [SSC,Railway,DSSSB,UP SI]): https://t.me/RBE_S
YouTube (Free lectures and job updates): https://www.youtube.com/c/RBERevolutionByEducation
Download RBE application for SSC Exams Complete Preparation
(a) She prevented (b) the hall (c) this year (d) have increased
(c) No error (d) me to enter
Q69. Find the incorrect part of the sentence:
Q67. Find the incorrect part of the sentence: He as well as his brothers were present.
The thief was hung for his crimes. (a) He as well as (b) his brothers
(a) No error (b) The thief was (c) No error (d) were present
(c) for his crimes (d) hung
Q70. Choose the part of the sentence that
Q68. Identify the part containing a grammatical or contains an error:
structural error: The team were playing well.
The number of applicants have increased this (a) No error (b) well
year. (c) were playing (d) The team
(a) No error (b) The number of applicants

Answer Key:

Q1 c Q2 b Q3 a Q4 d Q5 d
Q6 c Q7 d Q8 c Q9 d Q10 b
Q11 a Q12 d Q13 a Q14 a Q15 d
Q16 c Q17 c Q18 b Q19 c Q20 b
Q21 d Q22 b Q23 c Q24 c Q25 c
Q26 d Q27 b Q28 b Q29 c Q30 a
Q31 a Q32 a Q33 b Q34 a Q35 c
Q36 c Q37 a Q38 d Q39 b Q40 d
Q41 a Q42 c Q43 c Q44 a Q45 a
Q46 d Q47 b Q48 d Q49 d Q50 d
Q51 d Q52 b Q53 d Q54 a Q55 d
Q56 b Q57 b Q58 a Q59 d Q60 a
Q61 d Q62 c Q63 b Q64 b Q65 b
Q66 d Q67 d Q68 d Q69 d Q70 c

Solution 1: Explanation concert ended before they arrived (simple past).


Subject-Verb Agreement
The subject of the sentence is "Each participant Solution 3: Explanation
who registered online for the event", where "Each The subject of the sentence is "One of the guest
participant" is singular. speakers," which is singular.
The verb "were given" is plural, which does not The relative clause "who addressed the seminar
agree with the singular subject. and interacted with students" describes the "guest
The correct verb should be "was given" to match speaker" and does not change the number of the
the singular subject. subject.
Therefore, the verb should be singular to agree
Solution 2: Explanation with the singular subject "One."
The sentence mixes present tense ("arrive") with The correct verb is was praised (singular), not
past perfect tense ("had ended"), which creates a "were praised" (plural).
tense inconsistency.
Since the action of arriving is expected to occur Solution 4: Explanation
before the concert ended, "arrive" should be in "Pick up" is a phrasal verb where the pronoun
the past tense to match the sequence of events. object is placed between the two words, not after
The corrected sentence is: both.
"By the time they arrived, the concert had ended." Preposition "from the airport" is correctly used
Here, "had ended" (past perfect) shows the here.
Correct usage
Telegram (Previous year papers PDFs [SSC,Railway,DSSSB,UP SI]): https://t.me/RBE_S
YouTube (Free lectures and job updates): https://www.youtube.com/c/RBERevolutionByEducation
Download RBE application for SSC Exams Complete Preparation
The correct phrase is pick me up (the pronoun Hence, the error is in the verb phrase "is
"me" comes between "pick" and "up"). reliable" which should be "are reliable."
The sentence should be: "Can you pick me up
from the airport tomorrow?" Solution 10: Explanation:
The subject of the sentence is "The behavior ,"
Solution 5: Explanation which is singular.
"Since" should be followed by a specific point in The verb must agree in number with the subject.
time (e.g., since 2018, since January). Since " behavior " is singular, the verb should be
For a duration or period of time, "for" should be singular: "leaves" instead of "leave."
used instead of "since." Correct sentence: "The behavior of these
The correct sentence is: students leaves much to be desired."
She has been teaching in this school for five
years. Solution 11: Explanation:
Using "since" with a duration like "five years" is The sentence is in the present perfect tense
grammatically incorrect. ("has").
After "has," the past participle form of the verb
Solution 6: Explanation should be used, not the simple past.
The adjective "senior" should be followed by the The correct past participle of "see" is "seen."
preposition "to", not "than," when making Correct sentence: "He has seen the movie three
comparisons. times already."
The correct phrase is: "He was senior to me in
college." Solution 12: Explanation:
Hence, the error is in the phrase "senior than," "Smarter" is the comparative form of "smart."
which should be corrected to "senior to." Using both "more" and the "-er" suffix together is
a double comparative, which is incorrect.
Solution 7: Explanation The correct sentence should be: "She is smarter
In sentences with "Neither...nor," the verb agrees than her sister."
with the noun or pronoun nearest to it.
Here, "teachers" is plural and closest to the verb, Solution 13: Explanation:
so the verb should be plural. The correct preposition to use after "arrived" in
The corrected sentence should be: "Neither the this context is "at" or "in", not "to."
Principal nor the teachers were present." Correct sentence: "She arrived at the office early
Therefore, the error is in the verb "was" which this morning."
should be "were." Therefore, the incorrect word is to.

Solution 8: Explanation Solution 14: Explanation:


The subject "The teacher" is singular. The subject "One" is indefinite and singular, so
The phrase "along with his assistants" is a the pronoun referring to it should be gender-
parenthetical and does not change the number of neutral.
the subject. The correct pronoun to use is "one's" instead of
Therefore, the verb must be singular to agree with "his."
the singular subject. The correct sentence should be: "One must
"Have submitted" is plural; the correct verb form keep one's promises."
should be "has submitted." Therefore, the error lies in his promises.

Solution 9: Explanation: Solution 15: Explanation:


When using "neither... nor ," the verb should The correct usage after "it is high time" is to use
agree with the subject closer to it. the past simple tense to indicate that something
In this sentence, "brothers" is plural, so the verb should have been done already.
should be plural ("are") to agree with "brothers." Therefore, "start" should be "started."
Corrected sentence: "Neither Ram nor his Correct sentence: "It is high time
brothers are reliable."
Telegram (Previous year papers PDFs [SSC,Railway,DSSSB,UP SI]): https://t.me/RBE_S
YouTube (Free lectures and job updates): https://www.youtube.com/c/RBERevolutionByEducation
Download RBE application for SSC Exams Complete Preparation
you started studying seriously." friends are willing to join."

Solution 16: Explanation: Solution 21: ✅ Correct Answer: cope up with


The word "inside" is not typically used with the 🔍 Explanation:
phrase "table drawer." Standard Usage :
The correct preposition here should be in, as in The correct phrase is “cope with” , not “cope up
"The keys are in the table drawer." with” .
"In" is the appropriate preposition to indicate “Cope” is a verb that means to deal effectively
something is located within a container or with something difficult .
enclosed space like a drawer. It is not a phrasal verb with “up” — adding “up” is
Therefore, the error is in the word inside. a common error

Solution 17: Explanation: Solution 22: ✅ Correct Answer: was present


The adjective "junior" is a comparative adjective,
🔍 Explanation:
but it should be followed by "to" instead of "than."
Subject–Verb Agreement Rule (Neither…nor) :
The correct phrase is "She is junior to me."
When “neither…nor” connects two subjects, the
Therefore, the error is in Junior than.
verb agrees with the subject closer to it (i.e., the
second subject).
Solution 18: ✅ Correct Answer: most
In this sentence:
Sentence : This route is most safer than the Subject 1 : the teacher (singular)
highway. Subject 2 : the students (plural)
Error : The word “most” is incorrectly used Verb: was → incorrect , because it doesn’t agree
before the comparative adjective “safer.” with students
Rule : Correct Usage : Neither the teacher nor the
Use “more” with adjectives that don’t have a
students were present. ✅
comparative form (e.g., more beautiful ).
“were” matches the plural subject “students”

Solution 19: ✅ Correct Answer: has arrested


Solution 23: ✅ Correct Answer: a good advice
📌 Point-wise Explanation :
🔍 Explanation:
Subject Identification : The subject of the
Key Rule :
sentence is "The police" .
The noun “advice” is uncountable , so it cannot
Grammatical Nature of 'Police' : Although "police"
be used with the article “a” .
appears singular, it is treated as a plural collective
You can say:
noun in standard English usage.
✅ some good advice
Subject–Verb Agreement Rule : A plural subject
must take a plural verb . Hence, "has" (singular) ✅ a piece of good advice
is incorrect; it should be "have" (plural). ❌ a good advice

Solution 20: ✅ Correct Answer : Solution 24: ✅ Correct Answer: I returned back
is willing to join The verb “returned” already means “came
📌 Point-wise Explanation : back.”
Subject Structure : The sentence uses Adding “back” is unnecessary and
"Neither...nor..." to connect two subjects: 🔹 grammatically incorrect in formal usage.
Correct version: “I returned from Mumbai
Ramesh (singular) 🔹 his friends (plural)
last week.”
Verb Agreement Rule : When subjects are joined
by "neither...nor..." , the verb agrees with the
Solution 25: ✅ Correct Answer: Graceful
nearest subject (the one closest to the verb).
Nearest Subject : 🔹 his friends is plural , so the The verb “danced” requires an adverb to
describe how she danced.
verb must also be plural .
“Graceful” is an adjective ; the correct adverb
Correct Sentence : ✔ "Neither Ramesh nor his
form is “gracefully.”
✅ Corrected sentence: She danced gracefully
Telegram (Previous year papers PDFs [SSC,Railway,DSSSB,UP SI]): https://t.me/RBE_S
YouTube (Free lectures and job updates): https://www.youtube.com/c/RBERevolutionByEducation
Download RBE application for SSC Exams Complete Preparation
on stage during the annual show. Solution 30: ✅ Correct Answer
advised
Solution 26: ✅ Correct Answer: angry upon 🔍 Point-wise Explanation
The correct prepositions after “angry” are: Verb Pattern Rule
angry with someone → She is angry with me. The verb “advise” typically requires an object
angry at someone/something → She is angry at before an infinitive verb.
the delay. Correct structure: advise + someone + to do
something
Solution 27: ✅ Correct Answer: he had arrived Error in the Sentence
🔍 Step-by-Step Analysis: “advised to listen” → ❌ Incorrect
Original Sentence : Missing object after “advised” — we don’t know
Rule of Inversion with “No sooner” : who was advised.
When a sentence begins with “No sooner” , it Correct Usage Examples
requires inversion (auxiliary verb before subject). ✅ The teacher advised the students to listen
Correct structure: No sooner had + subject + attentively.
past participle + than... ✅ She advised me to take rest.

Solution 28: ✅ Correct Answer: No error Solution 31: ✅ Correct Answer


🔍 Step-by-Step Analysis: Option: him for
Full Sentence : 🔍 Point-wise Explanation
Tense Check : Verb–Preposition Collocation Rule
Present perfect: has issued → ✅ correct for The verb “congratulate” is followed by the
recent action. preposition “on” , not “for”
Subject–Verb Agreement : Correct usage: congratulate someone on
The officer → singular subject something
has issued → matches perfectly → ✅ Error in the Sentence
Phrase Check : “congratulated him for his success” → ❌
an order to arrest him immediately → idiomatic Incorrect
and grammatically correct. Should be : ✅ “ congratulated him on his
“to arrest” is an infinitive expressing purpose → success”
✅ Correct Usage Examples
“immediately” is a valid adverb modifying the ✅ She congratulated him on his promotion.
action → ✅ ✅ They congratulated her on winning the award.

Solution 29: ✅ Correct Answer Solution 32: ✅ Correct Answer


more Option: made
🔍 Point-wise Explanation 🔍 Point-wise Explanation
Error Type Verb Usage Error
Incorrect use of comparative and superlative The verb “made” is incorrect when referring to
forms together taking a shower.
Rule We use “take” or “have” with “shower” — not
Words like “more” are used with comparative “make”
adjectives (e.g., more beautiful , more intelligent ) Correct Collocation
“Highest” is a superlative adjective , which ✅ He took a quick shower before heading to the
already expresses the maximum degree office.
So, using “more” before “highest” is redundant
✅ He had a quick shower before heading to the
and grammatically incorrect
office.
Correct Sentence
✅ Mount Everest is the highest peak in the
Solution 33: The correct answer is " have gone
world.
for lunch." The error lies here because the
Telegram (Previous year papers PDFs [SSC,Railway,DSSSB,UP SI]): https://t.me/RBE_S
YouTube (Free lectures and job updates): https://www.youtube.com/c/RBERevolutionByEducation
Download RBE application for SSC Exams Complete Preparation
subject "The manager," which is singular, should "to consist of" and should not be followed by "of."
be followed by the singular verb "has." Phrases The correct sentence should be "This book
like "together with his staff" do not affect the comprises five chapters." People often confuse
verb’s number, so the sentence should be "The "comprise" with "consist of," but "comprise"
manager together with his staff has gone for stands alone without a preposition, while "consist"
lunch." requires "of."

Solution 34: The correct answer is "singer in the Solution 41: The usage in the sentence is correct:
city." The error is in the noun phrase: after "avail of every opportunity" is the standard
"one of the best," a plural noun should follow, structure when "avail" is used as a verb in British
so it should be "singers in the city " instead of English. There is no grammatical error in the
"singer in the city." The rest of the sentence is sentence, so "No error" is the right answer.
grammatically correct.
Solution 42: The subject "The father with his
Solution 35: The correct answer is "a." The error sons" is singular, so the verb should also be
is in the use of the article "a" before "honest." singular: "is arriving soon " instead of "are
Since "honest" begins with a vowel sound, it arriving soon." The error lies in subject-verb
should be "an honest approach " (not " a agreement—"with his sons" is a prepositional
honest approach"). The other parts are correct. phrase and does not make the subject plural.

Solution 36: The correct answer is " fastly ." The Solution 43: The error is in "to hung till death." It
word " fastly " is incorrect because "fast" is should be "to be hanged till death" because
both an adjective and an adverb . The phrase "hanged" is the correct past participle when
should be "a fast growing leader," not " fastly referring to execution by hanging. "Hung" is used
growing." The other words in the sentence are otherwise (e.g., pictures on a wall). The correct
used correctly. sentence is "The judge ordered the prisoner to be
hanged till death."
Solution 37: The correct answer is "were." The
error is in subject-verb agreement. " One of the Solution 44: The word "neither" is singular, so the
questions" has "One" as the main subject, verb should also be singular. The correct
which is singular, so it should take the sentence is "Neither of the two brothers is
singular verb "was" instead of the plural verb trustworthy." Therefore, "are" is the incorrect part
"were." The correct sentence should read: "One because it should be "is."
of the questions was very difficult."
Solution 45: The error is in the use of "between."
Solution 38: The correct answer is "to win." The When distributing among more than two
verb "succeed" should be followed by "in" + people or things, "among" should be used,
gerund (verb + - ing ), not "to" + infinitive. The not "between ." The correct sentence is "He
correct sentence should be "He succeeded in divided the sweets among his five friends."
winning the prize." This is the standard
grammatical construction with the verb "succeed." Solution 46: The error is "afraid from". The
correct preposition after "afraid" is "of," not
Solution 39: The correct answer is "laugh on ." "from." The sentence should be "The child is
The error is in the preposition used with the verb afraid of the dark." The use of "from" is incorrect
"laugh." T he correct preposition is "at," not in this context.
"on." T he sentence should read "You should not
laugh at others." We use "laugh at" when referring Solution 47: The error lies in "in their opinions."
to mocking or making fun of someone, while "Jury" is a collective noun treated as singular in
"laugh on" is not a standard English construction. British English, so it should be "in its opinion."
Using "their" suggests the jury is plural, which is
Solution 40: The correct answer is " comprises not correct in this context. All other parts are
of ." The verb "comprise" means "to include" or
Telegram (Previous year papers PDFs [SSC,Railway,DSSSB,UP SI]): https://t.me/RBE_S
YouTube (Free lectures and job updates): https://www.youtube.com/c/RBERevolutionByEducation
Download RBE application for SSC Exams Complete Preparation
grammatically correct. Solution 55: The error is in using "drowned." The
word "drown" is used for living beings, typically
Solution 48: The phrase "fell down from the referring to people or animals dying in water. The
stairs" is incorrect. The correct usage is "fell down correct verb for ships is "sank" or "was sunk," so
the stairs," not "fell down from the stairs." The the sentence should be "The ship sank in the
word "from" is unnecessary and ungrammatical sea."
here. The rest of the sentence is correct.
Solution 56: This sentence contains a
Solution 49: The error is in "is married with." The comparative error in the use of the word “than.”
correct usage is "is married to a doctor ," not 🔍 Correct Usage:
"with." The preposition "to" should be used when The verb “prefer” is followed by “to” , not
specifying the person someone is married to. The “than.”
other parts of the sentence are grammatically ✅ Correct sentence: “I prefer tea to coffee.”
correct.
Solution 57: ✅ Correct Answer: with her
Solution 50: The error is in "to accompany." The
mother
verb "insist" is always followed by "on" and the
📘 Explanation:
gerund form, not "to" with an infinitive. The
Incorrect Sentence: ➡ She resembles with her
correct structure is: "She insisted on
accompanying me." Therefore, the incorrect part mother.
of the sentence is "to accompany." Correct Sentence: ➡ She resembles her mother.

Solution 51: The error is in "No less than." The Solution 58: ✅ Correct Answer: better in
correct expression should be "No fewer than fifty mathematics
students were absent." The word "fewer" is used 🔍 Explanation:
for countable nouns such as "students," while The error lies in the phrase “better in
"less" is used for uncountable nouns. The other mathematics” . The correct preposition after
parts of the sentence are grammatically correct. “better” when comparing skills or subjects is “at”
, not “in”.
Solution 52: The error is in the phrase "when the
guests have arrived." In this sentence, after Solution 59: ✅ Correct Answer: Know to swim
"Hardly had he left the room," the correct form 🧠 Explanation:
should be "when the guests arrived" (simple past) The sentence “She does not know to swim” is
or "when the guests had arrived" (past perfect), grammatically incorrect because the verb “know”
not "have arrived" (present perfect), since the is not followed by “to swim” in standard English
past perfect is needed for such constructions. usage.
Instead, we say:
Solution 53: The error is in " since five years." ➡ “She does not know how to swim.”
The correct expression should use "for five years"
The correct structure is: know how to + base
instead of "since," because "since" is used before
verb
a point in time, while "for" is used for a duration.
The sentence should read: "He has been living in
Solution 60: ✅ Correct Answer: are too good
Chennai for five years."
🧠 Explanation:
Solution 54: The error is "He has come." The Sentence: The news are too good to be true.
present perfect tense ("has come") should not be 🔍 Let's break it down:
used with a definite past time marker like “News” is an uncountable noun and is treated
"yesterday." The correct form is "He came as singular , even though it ends in “-s”.
yesterday." The other parts of the sentence are Therefore, it should take a singular verb — “is” ,
correct. not “are” .

Telegram (Previous year papers PDFs [SSC,Railway,DSSSB,UP SI]): https://t.me/RBE_S


YouTube (Free lectures and job updates): https://www.youtube.com/c/RBERevolutionByEducation
Download RBE application for SSC Exams Complete Preparation
Solution 61: ✅ Correct Answer: that where 🔍 Explanation:
✅ Correct Structure for Reported Questions: Incorrect Sentence: She prevented me to enter
When reporting a wh -question , we do not use the hall.
“that” . Instead, we directly use the question This is grammatically incorrect because:
word (e.g., where, why, what ) after the reporting The verb “prevent” is not followed by an
verb. infinitive (to + verb) .
🔁 Correct sentence: ➡ He asked me where I Instead, it should be followed by a gerund (verb
was going. + ing ) or the structure prevent + object + from
+ verb- ing .
Solution 62: ✅ Correct Answer: The sceneries
Solution 67: ✅ Correct Answer: Option 4 –
🧠 Explanation:
hung
Sentence: The sceneries of Ooty were wonderful.
🔍 Explanation:
🔸 The word “sceneries” is incorrect because
Sentence: The thief was hung for his crimes.
“scenery” is an uncountable noun in English. It
The word “hung” is incorrect in this context.
does not take a plural form like sceneries .
When referring to execution by hanging , the
✅ Correct usage: ➡ The scenery of Ooty was
correct past participle is “hanged” , not “hung”.
wonderful.
Hung is used for objects (e.g., The painting was
hung on the wall. )
Solution 63: ✅ Correct Answer: I have visited Hanged is used for people in the context of
🧠 Why “I have visited” is Incorrect: capital punishment.
The phrase “last month” refers to a specific
time in the past . Solution 68: ✅ Correct Answer: Option 4 –
With specific past time expressions (e.g., have increased
yesterday, last week, last month, in 2020 ), we 🔍 Explanation:
use Simple Past Tense , not Present Perfect .
Sentence: The number of applicants have
increased this year.
Solution 64: ✅ Correct Answer: with The subject here is “The number” , which is
Why “with” Is Incorrect: singular .
The verb “marry” is transitive , meaning it Therefore, the verb should also be singular →
directly takes an object without a preposition. “has increased” , not “have increased” .
✅ Correct usage: married someone
❌ Incorrect: married with someone Solution 69: ✅ Correct Answer: Option 4 –
“were present”
Solution 65: ✅ Correct Answer: Option 2 – 🧠 Rule:
have given When using “as well as” , the subject before it
🔍 Explanation: determines the verb. 👉 The phrase “He as well
Sentence: The committee have given its as his brothers” is not plural — the main
decision. subject is “He” (singular).
Error: The verb “have” is incorrect here. So, the verb should be “was” , not “were.”
Why? The subject “committee” is a collective
noun and is treated as singular when acting as a Solution 70: ✅ Correct Answer: Option 3 –
unit. were playing
Therefore, the correct verb is “has” , not “have”. Here, “team” is acting as a single unit → so the
verb should be singular : 👉 The team was
Solution 66: ✅ Correct Answer: Option 4 – me playing well.
to enter

Telegram (Previous year papers PDFs [SSC,Railway,DSSSB,UP SI]): https://t.me/RBE_S


YouTube (Free lectures and job updates): https://www.youtube.com/c/RBERevolutionByEducation
Download RBE application for SSC Exams Complete Preparation

SSC Steno 2025 T-1 Fill in the Blanks All 87 Questions with Detailed Solution and Answer key

Q1. Choose the correct option to complete the Q8. Fill in the blank with the appropriate tense.
sentence. Jobin has been working all day. He _____ tired.
(a) can be (b) must be
The engineer visited the construction _____ to (c) will be (d) shall be
check the progress of the building.
(a) sit y (b) site Q9. Select the most appropriate option to fill in
(c) cite (d) sight the blank.
She was absorbed ______ reading.
Q2. Select the most appropriate option to fill in (a) to (b) in
the blank . (c) by (d) at
Listen carefully ________ what I say.
(a) in (b) to Q10. F il l in the blank with appropriate
(c) by (d) on prepositions.
The soldier was suspected _____being a spy and
Q3. Fill in the blank with an appropriate degree was put behind bars.
of comparison. (a) of (b) by
Hari is ________ than Sohan in solving (c) up (d) after
mathematical problems.
(a) talentest (b) most talented Q11. Fill in the blank with appropriate parts of
(c) talenter (d) more talented speech.
We spent an ______ day at school yesterday
Q4. Select the most appropriate option to fill in (a) enjoyed (b) enjoyable
the blank. (c) enjoyment (d) enjoying
The boy was indifferent _______ to his studies.
(a) in (b) on Q12. Choose the correct option to complete
(c) at (d) to the sentence.

Q5. The violinist played a beautiful _______ She tied the belt tightly around he r_____.
during the concert. (a) weist (b) waste
(a) piece (b) pace (c) waist (d) west
(c) peace (d) peas
Q13. Select the most appropriate option to fill
Q6. in the blank.
Fill in the blank with an appropriate article.
There is ________ hour left for you to get ready. He is afrai d ______ snakes.
(a) an (b) the (a) of (b) for
(c) a (d) many (c) to (d) with

Q7. Select the most appropriate option to fill in Q14. Select the most appropriate option to fill
the blank. in the blank.
The manager was satisfied _______ the work. He is good ______ mathematics .
(a) to (b) by (a) at (b) by
(c) for (d) with (c) to (d) in

Telegram (Previous year papers PDFs [SSC,Railway,DSSSB,UP SI]): https://t.me/RBE_S


YouTube (Free lectures and job updates): https://www.youtube.com/c/RBERevolutionByEducation
Download RBE application for SSC Exams Complete Preparation
Q15. Select the most appropriate option to fill (c) from (d) with
in the blank.
Q23. Select the most appropriate option to fill in
The soldier laid down his life ____ the nation, the blank.
(a) for (b) by She is married ___ a businessman.
(c) with (d) to (a) for (b) with
(c) at (d) to
Q16. F ill in the blank with an appropriate adverb.
The food is ____ good. Q24. Select the most appropriate option to fill in
(a) annually (b) cautiously the blank.
(c) extremely (d) boldly Please switch ___ the lights.
(a) up (b) over
Q17. Choose the correct option to complete the (c) off (d) down
sentence.
Q25. Choose the correct option to complete the
Do you think it is going to _____ this afternoon? sentence.
(a) rain (b) rein The volunteers will _____ the neighbourhood to
(c) raign (d) reign gather support for the new policy.
(a) canvas (b) cannabis
Q18. Select the most appropriate option to fill (c) canopies (d) canvass
in the blank.
Q26. Select the most appropriate option to fill in
The soldiers died _____ their country. the blank.
(a) at (b) for The officials connived ___ the illegal activities.
(c) by (d) in (a) over (b) on
(c) for (d) at
Q19. Select the most appropriate option to fill
in the blank. Q27. Select the most appropriate option to fill in
the blank.
The beggar was blind _____o ne eye. She adhered strictly ___ the code of conduct.
(a) on (b) at (a) by (b) to
(c) in (d) by (c) for (d) at

Q20. Select the most appropriate option to fill Q28. Select the most appropriate option to fill in
in the blank. the blank.
I saw a man ___ was wearing a black coat.
The police looked _____ the matter. (a) which (b) whose
(a) for (b) by (c) who (d) whom
(c) at (d) into
Q29. Select the most appropriate option to fill in
Q21. Select the most appropriate option to fill in the blank.
the blank. We prefer tea ___ coffee.
My house is ___ the bank and the post office. (a) for (b) on
(a) across (b) between (c) with (d) to
(c) among (d) beside
Q30. Select the most appropriate option to fill in
Q22. Select the most appropriate option to fill in the blank.
the blank. She was annoyed ___ the noise.
The witness was reluctant to part ___ vital (a) at (b) with
information. (c) to (d) by
(a) by (b) of

Telegram (Previous year papers PDFs [SSC,Railway,DSSSB,UP SI]): https://t.me/RBE_S


YouTube (Free lectures and job updates): https://www.youtube.com/c/RBERevolutionByEducation
Download RBE application for SSC Exams Complete Preparation
Q31. Select the most appropriate option to fill in Q39. Choose the correct option to complete the
the blank. sentence.
The students were keen ___ participating. The new policy has ______ the use of plastic
(a) on (b) With bags in the store.
(c) for (d) in (a) banned (b) bend
(c) bond (d) band
Q32. Select the most appropriate option to fill in
the blank. Q40. Choose the correct option to complete the
She has great affection ___ her brother. sentence.
(a) of (b) for She couldn't help but _______ up during an
(c) to (d) with emotional scene in the movie.
(a) tier (b) tyre
Q33. Select the most appropriate option to fill in (c) tire (d) tear
the blank.
The book consists ___ ten chapters. Q41. Select the most appropriate option to fill in
(a) Of (b) In the blank.
(c) By (d) from The thief broke ____ the house.
(a) in (b) into
Q34. Fill in the blank with the appropriate tense. (c) by (d) at
The student ________ his homework yet.
(a) was not completed (b) did not complete Q42. Select the most appropriate option to fill in
(c) have not completed (d) has not completed the blank.
You must cope ____ difficulties.
Q35. Choose the correct option to complete the (a) with (b) for
sentence. (c) by (d) in
She read the story aloud to the class in a clam
and steady __________. Q43. Select the most appropriate option to fill in
(a) Miner (b) manor the blank.
(c) manner (d) meaner The train arrived late ____ the station.
(a) to (b) in
Q36. Fill in the blank with appropriate (c) on (d) at
prepositions.
Mahesh threw the ball ________ his friend, but Q44. Fill in the blank with appropriate
he could not catch it. prepositions.
(a) to (b) for When you have completed reading the book,
(c) against (d) at keep it ________ the shelf.
(a) in (b) across
Q37. Select the most appropriate option to fill in (c) on (d) against
the blank.
The minister was apprised ___ the current Q45. Choose the correct option to complete the
situation. sentence.
(a) about (b) at She walked gracefully down the _____ during the
(c) to (d) of wedding ceremony .
(a) isle (b) aisle
Q38. Select the most appropriate option to fill in (c) oil (d) ail
the blank.
The teacher advised him to abstain ___ bad Q46. Select the most appropriate option to fill in
company. the blank.
(a) of (b) from She is capable ____ handling the situation.
(c) with (d) for (a) to (b) by
(c) of (d) in

Telegram (Previous year papers PDFs [SSC,Railway,DSSSB,UP SI]): https://t.me/RBE_S


YouTube (Free lectures and job updates): https://www.youtube.com/c/RBERevolutionByEducation
Download RBE application for SSC Exams Complete Preparation
Q47. Select the most appropriate option to fill in Q55. Select the most appropriate option to fill in
the blank. the blank.
They congratulated him _____ winning the match. He speaks English ___ than I do.
(a) on (b) for (a) good (b) more good
(c) at (d) of (c) better (d) best

Q48. Select the most appropriate option to fill in Q56. Select the most appropriate option to fill in
the blank. the blank.
The audience was mesmerized _____ the His actions are incompatible ___ his words.
speaker's eloquence. (a) to (b) for
(a) with (b) for (c) by (d) with
(c) to (d) by
Q57. Select the most appropriate option to fill in
Q49. Fill in the blank with the most appropriate the blank.
option: She was credited ___ inventing the device.
The director's remarks did not ___ well with the (a) to (b) for
board members, leading to immediate backlash. (c) by (d) with
(a) go over (b) go off
(c) go in (d) go out Q58. Select the most appropriate option to fill in
the blank.
Q50. Select the most appropriate option to fill in I am not interested ___ politics.
the blank. (a) at (b) in
The boy was punished ___ his disobedience. (c) on (d) of
(a) at (b) to
(c) for (d) in Q59. Select the most appropriate option to fill in
the blank.
Q51. Select the most appropriate option to fill in He was accused ___ theft.
the blank. (a) by (b) of
The ___ of the movie was unexpected. (c) for (d) with
(a) end (b) ends
(c) ended (d) ending Q60. Select the most appropriate option to fill in
the blank.
Q52. Select the most appropriate option to fill in He is not eligible ___ this post.
the blank. (a) to (b) for
The patient was devoid ___ any hope of recovery. (c) by (d) with
(a) at (b) with
(c) in (d) of Q61. Select the most appropriate option to fill in
the blank.
Q53. Select the most appropriate option to fill in There is ___ apple on the table.
the blank. (a) an (b) one
He congratulated me ___ my success. (c) the (d) a
(a) of (b) of
(c) at (d) on Q62. Choose the correct option to complete the
sentence.
Q54. Select the most appropriate option to fill in She gave me a nice _______ on my new haircut.
the blank. (a) complement (b) complimente
They arrived ___ the airport on time. (c) compleement (d) compliment
(a) in (b) over
(c) at (d) on Q63. Select the most appropriate option to fill in
the blank.
The editor dispensed ___ unnecessary details.
(a) to (b) with
Telegram (Previous year papers PDFs [SSC,Railway,DSSSB,UP SI]): https://t.me/RBE_S
YouTube (Free lectures and job updates): https://www.youtube.com/c/RBERevolutionByEducation
Download RBE application for SSC Exams Complete Preparation
(c) of (d) by Q72. Select the most appropriate option to fill in
the blank.
Q64. Select the most appropriate option to fill in The sun rises ___ the east.
the blank. (a) on (b) by
I have no objection ___ your proposal. (c) at (d) in
(a) with (b) of
(c) for (d) to Q73. Select the most appropriate option to fill in
the blank.
Q65. Choose the correct option to complete the He insisted ___ paying the bill.
sentence. (a) by (b) at
The carpenter replaced the broken ____ in the (c) for (d) on
window.
(a) plain (b) pain Q74. Select the most appropriate option to fill in
(c) peyn (d) pane the blank.
She was brought ___ by her grandmother.
Q66. Select the most appropriate option to fill in (a) in (b) out
the blank. (c) off (d) up
He was deprived ___ his rights.
(a) with (b) by Q75. Select the most appropriate option to fill in
(c) of (d) from the blank.
The child has been missing ___ yesterday.
Q67. Select the most appropriate option to fill in (a) since (b) on
the blank. (c) for (d) from
Ramesh is senior ____ me in service.
(a) with (b) of Q76. Choose the correct option to complete the
(c) from (d) to sentence.
The hospital staff ensured proper ________
Q68. Choose the correct option to complete the conditions in every ward.
sentence. (a) sanitery (b) sanitarium
The car remained ______ at the red light. (c) sanitary (d) sanity
(a) stationary (b) stationry
(c) stationery (d) stationerie Q77. Select the most appropriate option to fill in
the blank.
Q69. Choose the correct option to complete the The workers agitated ____ better working
sentence. conditions.
Due to the accident, she had a deep _______ on (a) at (b) by
her arm. (c) to (d) for
(a) wound (b) wounded
(c) wand (d) wind Q78. Choose the correct option to complete the
sentence.
Q70. Select the most appropriate option to fill in She contracted a cold and had problems with
the blank. stuffy ________.
The teacher was angry ___ the students. (a) nose (b) news
(a) to (b) by (c) knows (d) noose
(c) with (d) on
Q79. Select the most appropriate option to fill in
Q71. Select the most appropriate option to fill in the blank.
the blank. They prefer tea __ coffee.
She ___ watching that movie already. (a) than (b) over
(a) have (b) having (c) to (d) from
(c) had (d) h as been

Telegram (Previous year papers PDFs [SSC,Railway,DSSSB,UP SI]): https://t.me/RBE_S


YouTube (Free lectures and job updates): https://www.youtube.com/c/RBERevolutionByEducation
Download RBE application for SSC Exams Complete Preparation
Q80. Select the most appropriate option to fill in (c) on (d) for
the blank.
She ___ to the market every morning. Q84. Select the most appropriate option to fill in
(a) gone (b) go the blank.
(c) going (d) goes She hasn’t called me ___ Monday.
(a) since (b) for
Q81. Select the most appropriate option to fill in (c) at (d) on
the blank.
On that pleasant autumn day, I was accompanied Q85. Select the most appropriate option to fill in
______ my daughter, Zenani. the blank.
(a) for (b) by He presides ___ all important meetings.
(c) in (d) of (a) with (b) over
(c) at (d) on
Q82. Select the most appropriate option to fill in
the blank. Q86. Select the most appropriate option to fill in
The manager was dissatisfied ___ the poor the blank.
attendance. He is interested ___ learning foreign languages.
(a) with (b) to (a) for (b) in
(c) by (d) of (c) at (d) to

Q83. Select the most appropriate option to fill in Q87. Select the most appropriate option to fill in
the blank. the blank.
She was intent ___ pursuing higher studies She writes essays ___.
abroad. (a) neat (b) neatly
(a) by (b) to (c) neatest (d) neater

Answer Key:

Q1 a Q2 b Q3 d Q4 d Q5 a
Q6 a Q7 d Q8 b Q9 b Q10 a
Q11 b Q12 c Q13 a Q14 a Q15 a
Q16 c Q17 a Q18 b Q19 c Q20 d
Q21 b Q22 d Q23 d Q24 c Q25 d
Q26 d Q27 b Q28 c Q29 d Q30 d
Q31 a Q32 b Q33 a Q34 d Q35 c
Q36 a Q37 d Q38 b Q39 a Q40 d
Q41 b Q42 a Q43 d Q44 c Q45 b
Q46 c Q47 a Q48 d Q49 a Q50 c
Q51 d Q52 d Q53 d Q54 c Q55 c
Q56 d Q57 d Q58 b Q59 b Q60 b
Q61 a Q62 d Q63 b Q64 d Q65 d
Q66 c Q67 d Q68 a Q69 a Q70 c
Q71 d Q72 d Q73 d Q74 d Q75 a
Q76 c Q77 d Q78 a Q79 c Q80 d
Q81 b Q82 a Q83 c Q84 a Q85 b
Q86 b Q87 b

Solution 1: Explanation quote or refer), and "sight" (something seen) do


"Site" means a location or place, which fits not fit the context.
perfectly with "construction site."
The other options "sity" (a misspelling), "cite" (to
Telegram (Previous year papers PDFs [SSC,Railway,DSSSB,UP SI]): https://t.me/RBE_S
YouTube (Free lectures and job updates): https://www.youtube.com/c/RBERevolutionByEducation
Download RBE application for SSC Exams Complete Preparation
Solution 2: Explanation "Can be" expresses possibility but is less certain.
The verb "listen" is commonly followed by the "Will be" and "shall be" indicate future tense,
preposition "to" when indicating the object of which does not fit the context here.
listening.
Other options like "in," "by," or "on" are not correct Solution 9: Explanation
in this context. The correct preposition used with "absorbed" in
this context is "in."
Solution 3: Explanation "Absorbed in" means deeply engaged or
When comparing two people or things, use engrossed in something.
the comparative degree, which often uses "more" Other prepositions like "to," "by," and "at" are not
with adjectives of two or more syllables. correct in this context.
"Talented" is a three-syllable adjective, so "more
talented" is the correct comparative form. Solution 10: Explanation
Other options like "talentest" and "talenter" are The correct preposition used with the word
incorrect forms. "suspected" in this context is "of."
"Most talented" is a superlative used when "Suspected of" means believed to be guilty or
comparing three or more things, not just two. involved in something.
Other options like "by," "up," and "after" do not fit
Solution 4: Explanation correctly in this context.
The word "indifferent" is always followed by the
preposition "to" when showing lack of interest or Solution 11: Explanation
concern. "Enjoyable" is an adjective describing the noun
"day."
Solution 5: Explanation The other options are incorrect parts of speech
"Piece" refers to a musical composition or work. for this context:
"Pace" means speed or rate of movement. "enjoyed" is a past tense verb
"Peace" means tranquility or absence of conflict. "enjoyment" is a noun
"Peas" are a type of vegetable. "enjoying" is a present participle/gerund
Therefore, "piece" fits the context of music.
Solution 12: Explanation
Solution 6: Explanation "Waist" refers to the part of the body between the
The word "hour" begins with a silent 'h' and ribs and hips.
sounds like it starts with a vowel sound. "Weist," "waste," and "west" are incorrect in this
For words beginning with a vowel sound, the context:
article "an" is used. "Weist" is a misspelling.
"A" is used before words starting with a "Waste" means to use or expend carelessly.
consonant sound. "West" refers to a cardinal direction.
"The" is a definite article and "many" is not an
article. Solution 13: Explanation
The correct preposition used with "afraid" when
Solution 7: Explanation expressing fear of something is "of."
The adjective "satisfied" is commonly followed by "Afraid for" indicates concern for someone's
the preposition "with" when expressing safety, not fear of something.
contentment about something. "Afraid to" is used before a verb (e.g., afraid to
Other prepositions like "to," "by," or "for" are not go).
correct in this context. "Afraid with" is incorrect in this context.

Solution 8: Explanation Solution 14: Explanation


"Must be" is used to express a logical conclusion The correct preposition used with "good" when
or assumption based on evidence. referring to a skill or subject is "at."
Since Jobin has been working all day, it is "By," "to," and "in" are incorrect in this context.
reasonable to assume he is tired.
Telegram (Previous year papers PDFs [SSC,Railway,DSSSB,UP SI]): https://t.me/RBE_S
YouTube (Free lectures and job updates): https://www.youtube.com/c/RBERevolutionByEducation
Download RBE application for SSC Exams Complete Preparation
Solution 15: Explanation phrase “part with” .
The correct preposition to show sacrifice or
purpose in this context is "for." Solution 23: ✅ Correct Answer :
"By," "with," and "to" do not fit the meaning here. to
📌 Point-wise Explanation :
Solution 16: Explanation Correct Preposition with "married" : The verb
"Extremely" is used to intensify an adjective, such "married" is followed by the preposition "to" when
as "good." indicating the person someone is married to.
Other options like "annually" (time), "cautiously" Correct Usage : ✔ "She is married to a
(manner), and "boldly" (manner) do not fit well as businessman." This shows the relationship
intensifiers for "good."
between the subject and the person she married.

Solution 17: Explanation:


Solution 24: ✅ Correct Answer : off
"Rain" means precipitation from the sky.
Phrasal Verb – "switch off" :
"Rein" refers to the straps used to control a horse.
"Switch off" means to turn off an electrical device
" Raign " is not a correct word.
or light.
"Reign" means to rule as a monarch.
It’s the standard and idiomatic usage in English.
Therefore, the correct choice is rain.

Solution 25: ✅ Correct Answer : canvass


Solution 18: Explanation:
"Died for" means they sacrificed their lives in 📌 Completed Sentence :
service or defense of their country. The volunteers will canvass the neighbourhood
Other prepositions do not fit the meaning to gather support for the new policy.
correctly. Usage Tip:
Canvass (verb) = to actively seek votes,
Solution 19: Explanation: opinions, or support Example: They canvassed
"Blind in one eye" is the correct phrase indicating the area before the election.
loss of sight in a particular eye. Canvas (noun) = fabric used for tents or painting
Other prepositions do not fit this expression. Example: She painted a landscape on canvas.
Therefore, the correct choice is in.
Solution 26: ✅ Correct Answer : at
Solution 20: Explanation: Usage Tip:
"Look into" means to investigate or examine a Connive at something = to secretly allow or
matter. overlook wrongdoing.
Other prepositions like "for," "by," and "at" do not Example: The manager connived at corruption in
fit correctly in this context. the department.
Therefore, the correct choice is into. Connive with someone = to secretly cooperate in
wrongdoing.
Solution 21: ✅ Correct Answer: between Example: The officials connived with the
Here, we are talking about two specific smugglers.
locations —the bank and the post office. The
correct preposition to use when referring to two Solution 27: ✅ Correct Answer: to
distinct points is “between.” 🔍 Explanation:
Correct Phrase : The verb “adhere” is always
Solution 22: ✅ Correct Answer: with followed by the preposition “to” when referring to
The phrase “part with” is a common phrasal verb rules, principles, or standards.
that means to give up or relinquish something , Usage :
often reluctantly. She adhered strictly to the code of conduct.
In this sentence, the witness is hesitant to give This means she followed the rules with full
away or share vital information. discipline and commitment.
So, the correct preposition is “with” , forming the

Telegram (Previous year papers PDFs [SSC,Railway,DSSSB,UP SI]): https://t.me/RBE_S


YouTube (Free lectures and job updates): https://www.youtube.com/c/RBERevolutionByEducation
Download RBE application for SSC Exams Complete Preparation
Solution 28: ✅ Correct Answer: who Correct Structure :
🔍 Explanation: Subject : The book
Sentence Structure : I saw a man ___ was Verb : consists
wearing a black coat. We need a relative pronoun Preposition : of
that refers to “a man” and acts as the subject of Object : ten chapters
the clause “was wearing a black coat.”
Solution 34: ✅ Correct Answer: has not
Solution 29: ✅ Correct Answer: to completed
🔍 Explanation: 🔍 Step-by-Step Analysis:
Verb–Preposition Collocation : The verb “prefer” Sentence Structure :
is always followed by the preposition “to” when The word “yet” signals use of present perfect
comparing two things. tense , especially in negative or interrogative
Correct Usage : forms.
We prefer tea to coffee. Subject :
This structure shows a clear preference of one The student → third person singular
item over another. Correct Tense Form :
Present perfect negative: has + not + past
Solution 30: ✅ Correct Answer: by participle → has not completed
🔍 Sentence:
Solution 35: ✅ Correct Answer: manner
“She was annoyed ___ the noise.”
🔍 Sentence Context:
We’re dealing with a passive construction
expressing emotional reaction to a cause. She read the story aloud to the class in a calm
🔍 Why “by” Works Best: and steady __________.
The blank needs a noun that describes how she
“Annoyed by” is the standard collocation when
read—i.e., her style or way of doing something .
referring to the source or cause of irritation.
The correct word is “manner” , which means
“Noise” is an external factor, making “by” the most
method, style, or approach .
grammatically and idiomatically accurate choice.

Solution 36: ✅ Correct Answer: to


Solution 31: ✅ Correct Answer: on
🔍 Sentence:
The adjective “keen” is commonly followed by
the preposition “on” when expressing interest or Mahesh threw the ball to his friend, but he could
enthusiasm about an activity. not catch it.
The correct collocation is: keen on + verb-ing "To" is used when the action involves intended
. direction or transfer toward someone.
Implies Mahesh wanted his friend to receive the
Solution 32: ✅ Correct Answer: for ball.
🔍 Step-by-Step Explanation:
Solution 37: ✅ Correct Answer
Target Phrase :
of
Correct Collocation :
The noun “affection” is commonly followed by the 🔍 Point-wise Explanation
preposition “for” when expressing emotional Verb–Preposition Collocation
attachment or fondness. The verb “apprise” is always followed by the
Standard usage: affection for someone preposition “of” when referring to the information
Why “for” is Correct : being conveyed.
It indicates emotional connection or fondness Standard structure: apprise + someone + of +
directed toward someone . something
Example: He showed deep affection for his pet. Meaning
Apprise means to inform or notify
Solution 33: ✅ Correct Answer: Of So: The minister was apprised of the current
situation = The minister was informed about the
🔍 Step-by-Step Analysis:
Verb Phrase :
Telegram (Previous year papers PDFs [SSC,Railway,DSSSB,UP SI]): https://t.me/RBE_S
YouTube (Free lectures and job updates): https://www.youtube.com/c/RBERevolutionByEducation
Download RBE application for SSC Exams Complete Preparation
current situation. on the surface of the shelf. The other options do
not fit the physical relationship described in the
Solution 38: ✅ Correct Answer sentence.
from
🔍 Point-wise Explanation Solution 45: The correct answer is " aisle." An
Verb–Preposition Collocation "aisle" is the walkway or passage between
The verb “abstain” is always followed by the rows of seats in a church or ceremony venue,
preposition “from” when referring to avoiding which is exactly what a bride walks down
something. during a wedding ceremony . "Isle" means a
Standard usage: abstain from + noun/gerund small island, "oil" is a liquid substance, and "ail"
Meaning means to be sick or cause problems—none of
Abstain from bad company = Avoid associating these fit the wedding context.
with bad company
Solution 46: The correct answer is "of." The
Solution 39: The correct answer is "banned." standard prepositional phrase is "capable of"
The sentence refers to prohibiting the use of when expressing someone's ability to do
plastic bags, and "banned" is the correct verb something. "She is capable of handling the
meaning to officially or legally prohibit something. situation" is the grammatically correct
The other options do not logically fit the context of construction. The other prepositions ("to," "by,"
the sentence. "in") do not form correct idiomatic expressions
with "capable" in this context.
Solution 40: The correct answer is " tear." The
phrase "tear up" means to start getting tears in Solution 47: The correct answer is " on." The
one’s eyes, usually because of strong standard prepositional phrase is "congratulate
emotion. None of the other options ("tier," " tyre someone ON something" when referring to an
," "tire") relate to becoming emotional or crying. achievement or success. "They congratulated him
on winning the match" follows the correct
Solution 41: The correct answer is "into ." The grammatical pattern. The other prepositions ("for,"
phrasal verb "break into" is the correct and "at," "of") do not form the correct idiomatic
idiomatic expression used to describe expression with "congratulated" in this context.
someone entering a place, especially illegally or
by force. The other options ("in," "by," and "at") do Solution 48: The correct answer is "by." When
not fit the context of the sentence. someone is mesmerized, they are captivated
or enchanted BY something or someone . The
Solution 42: The correct answer is "with." The preposition "by" indicates the agent or cause of
verb "cope" is always followed by the the mesmerizing effect. "The audience was
preposition "with" when referring to dealing mesmerized by the speaker's eloquence"
with problems or difficulties . The correct correctly shows that the speaker's eloquence was
phrase is "cope with difficulties" and the other the cause of the audience being mesmerized.
options do not form a standard English The other prepositions do not form correct
collocation. grammatical constructions with "mesmerized."

Solution 43: T he correct answer is "at ." The Solution 49: The correct answer is " go over."
standard prepositional phrase is "arrive at the The phrasal verb "go over well" means to be
station" when referring to railway or bus stations. received favorably or accepted positively . In
"To," "in," and "on" are incorrect in this context, as this context, "did not go over well" indicates that
"at" correctly indicates the specific location where the director's remarks were poorly received by the
the train arrived. board members, which explains the immediate
backlash. The other phrasal verbs don't fit this
Solution 44: The correct answer is " on." The context of reception or acceptance.
standard prepositional phrase is " on the shelf "
when referring to placing an object so that it rests
Telegram (Previous year papers PDFs [SSC,Railway,DSSSB,UP SI]): https://t.me/RBE_S
YouTube (Free lectures and job updates): https://www.youtube.com/c/RBERevolutionByEducation
Download RBE application for SSC Exams Complete Preparation
Solution 50: The most appropriate option to fill in Solution 55: The correct comparative form to use
the blank is for . here is "better." "Better" is the comparative of
Correct Answer "good," used for making comparisons. The other
The boy was punished for his disobedience. options are either incorrect comparative forms or
Explanation do not fit the sentence grammatically.
The preposition " for " is used to indicate the
reason or cause of an action, which fits the Solution 56: The correct preposition is "with." The
context of punishment due to disobedience. standard phrase is "incompatible with," which
Options like "at," "to," and "in" do not correctly means that two things do not go together or do
convey the causal relationship required in this not match. Other prepositions are not idiomatic or
sentence. grammatically correct in this context.

Solution 51: ✅ Correct Answer: ending Solution 57: The correct preposition in this
🧠 Explanation: context is "with." The phrase "credited with" is
Let’s evaluate each option grammatically and standard in English when acknowledging
contextually: someone's accomplishment or achievement. The
End – ❌ grammatically awkward here. “ The end other options do not fit idiomatically or
of the movie ” is possible, but “ was unexpected ” grammatically in the sentence.
fits better with a noun form that implies an event
or outcome. Solution 58: The correct preposition is "in." The
Ends – ❌ incorrect tense and subject-verb phrase "interested in" is commonly used to show
one's interest regarding a subject. Thus, "I am not
agreement. “ Ends ” is a verb, not suitable after “
interested in politics" is grammatically accurate,
The ___ of the movie. ”
while the other options are incorrect in this
Ended – ❌ past tense verb; doesn ’ t fit after “
context.
The ___ of the movie. ”
Ending – ✔ correct. “The ending of the movie Solution 59: The correct preposition is "of." The
was unexpected” is natural and grammatically standard phrase is "accused of" when specifying
sound. the crime or reason for accusation. Thus, "He was
accused of theft" is the proper usage; the other
Solution 52: The correct preposition to use with prepositions are incorrect in this context.
"devoid" is "of." The standard phrase is "devoid
of," meaning completely lacking something. Solution 60: The correct preposition is "for." The
Therefore, "The patient was devoid of any hope of phrase "eligible for" is the standard form in
recovery" is grammatically and idiomatically English when referring to suitability or
accurate. The other options do not collocate qualification relating to a position or opportunity.
correctly with "devoid." Therefore, "He is not eligible for this post" is
grammatically accurate, while the other options
Solution 53: The correct preposition in this are incorrect in this context.
context is "on." The standard collocation is
"congratulate (someone) on (something)." Thus, Solution 61: The correct option is "an" because
the grammatically correct sentence is "He "apple" begins with a vowel sound. In English
congratulated me on my success." The other grammar, we use " an " before words starting
options do not fit standard English usage for with vowel sounds to make the sentence smooth
congratulations. and accurate: "There is an apple on the table."
The other options do not fit the grammatical rule
Solution 54: The correct preposition for locations for indefinite articles before vowel sounds.
such as "airport" is "at." The phrase "arrived at
the airport" follows standard English usage for Solution 62: The correct word is "compliment,"
referring to points of arrival at places. The other which means a polite expression of praise or
options do not fit the required context for this admiration. The other options are either
sentence. misspellings or "complement," which refers to
Telegram (Previous year papers PDFs [SSC,Railway,DSSSB,UP SI]): https://t.me/RBE_S
YouTube (Free lectures and job updates): https://www.youtube.com/c/RBERevolutionByEducation
Download RBE application for SSC Exams Complete Preparation
something that completes or goes well with Sentence: ➡ The teacher was angry ___ the
something else, and does not fit the meaning in students.
this context. Therefore, "She gave me a nice Correct Preposition: ➡ with – used to show the
compliment on my new haircut" is the target of anger or frustration.
grammatically and semantically accurate answer.
Solution 71: ✅ Correct Answer: has
Solution 63: The correct phrase is "dispensed
🧠 Explanation:
with unnecessary details." "Dispense with" means
Sentence: She ___ watching that movie already.
to do without or get rid of something. The other
Let’s analyze grammatically:
options ("to," "of," "by") do not create a
Subject: She → third person singular
grammatically correct or idiomatic expression in
Verb form needed: Present perfect tense → has
this context.
+ past participle / been + - ing form
Solution 64: The correct preposition in this
Solution 72: ✅ Correct Answer: in
context is "to." The phrase is "no objection to"
followed by a noun or gerund, so the correct 🧠 Explanation:
sentence is "I have no objection to your Sentence: The sun rises ___ the east.
proposal." Other prepositions do not form an 🔍 The correct preposition to indicate location or
idiomatic or grammatically correct expression direction in this context is “in” .
here. ✅ Final sentence: ➡ The sun rises in the east.

Solution 65: The correct word is "pane," which Solution 73: ✅ Correct Answer: on
means a single sheet of glass in a window or 🧠 Explanation:
door. The other options—"plain," "pain," and " Sentence: He insisted ___ paying the bill.
peyn "—do not fit the context or are incorrect
🔍 The verb “insist” is always followed by the
spellings.
preposition “on” when used with a gerund (verb +
ing ).
Solution 66: The correct preposition is "of." The
✅ Final sentence: ➡ He insisted on paying the
standard phrase is "deprived of" to indicate losing
bill.
or being denied something, such as rights,
opportunities, or possessions. Other prepositions
do not fit idiomatically or grammatically in this Solution 74: ✅ Correct Answer: up
context. 🧠 Explanation:
Sentence: She was brought ___ by her
Solution 67: The correct preposition is "to." The grandmother.
standard usage is "senior to" (not "senior than"), 🔍 The correct phrasal verb is “brought up” ,
especially when comparing rank, position, or which means raised or reared , especially in the
length of service. Other prepositions do not fit context of childhood and upbringing.
standard English grammar for this phrase. ✅ Final sentence: ➡ She was brought up by her
grandmother.
Solution 68: The correct word is "stationary,"
meaning not moving or standing still. "Stationery" Solution 75: ✅ Correct Answer: since
refers to writing materials, and the other spellings 🧠 Explanation:
are incorrect. Thus, "The car remained stationary
Sentence: The child has been missing ___
at the red light" is the correct usage.
yesterday.
🔍 The correct preposition here is “since” ,
Solution 69: ✅ Correct Answer: wound
which is used to indicate the starting point of a
wound – a noun meaning an injury to the body,
time period — especially with specific points in
especially one where the skin is cut or broken.
time like yesterday , Monday , 2010 , etc.
✅ Final sentence: ➡ The child has been missing
Solution 70: ✅ Correct Answer: with
since yesterday.
📘 Explanation:
Telegram (Previous year papers PDFs [SSC,Railway,DSSSB,UP SI]): https://t.me/RBE_S
YouTube (Free lectures and job updates): https://www.youtube.com/c/RBERevolutionByEducation
Download RBE application for SSC Exams Complete Preparation
Solution 76: ✅ Correct Answer: sanitary Structure: accompanied by [person]
🧠 Explanation:
Sentence: The hospital staff ensured proper Solution 82: ✅ Correct Answer: Option 1 –
________ conditions in every ward. with
🔍 The blank requires an adjective that describes 🔍 Explanation:
cleanliness and hygiene , especially in a Sentence: The manager was dissatisfied ___ the
medical setting. poor attendance.
✅ Sanitary means clean and hygienic , which The correct preposition to use with “dissatisfied”
perfectly fits the context of hospital conditions. is “with” .
Structure: dissatisfied + with + noun/gerund
Solution 77: ✅ Correct Answer: for
Solution 83: ✅ Correct Answer: Option 3 – on
🧠 Explanation:
🔍 Explanation:
Sentence: The workers agitated ____ better
working conditions. Sentence: She was intent ___ pursuing higher
🔍 The correct preposition is “for” , which is used studies abroad.
The adjective “intent” is always followed by the
to indicate the cause or purpose of the agitation.
preposition “on” when expressing determination
✅ Final sentence: ➡ The workers agitated for
or focus toward an action.
better working conditions.
The correct structure is: intent on + verb- ing
(gerund)
Solution 78: ✅ Correct Answer: nose
🧠 Sentence Completion: Solution 84: ✅ Correct Answer: Option 1 –
Correct Sentence: ➡ She contracted a cold and since
had problems with stuffy nose . 🔍 Explanation:
🔍 Why “nose” Is Correct: Sentence: She hasn’t called me ___ Monday.
A stuffy nose is a common symptom of a cold, The verb phrase “hasn’t called” is in the present
caused by nasal congestion. perfect tense , which is typically followed by:
The word fits both grammatically and “since” → to indicate the starting point of a time
contextually . period.
“for” → to indicate the duration of time.
Solution 79: ✅ Correct Answer: to
🔍 Why “to” Is Correct: Solution 85: ✅ Correct Answer: Option 2 –
The verb “prefer” is followed by “to” when over
comparing two things. 📘 Sentence:
✅ Structure: prefer A to B He presides over all important meetings.
➤ Example: I prefer books to movies. 🔍 Explanation:
The verb “preside” is typically followed by the
Solution 80: ✅ Correct Answer: Option 4 – preposition “over” when referring to someone
goes who leads, governs, or chairs a formal event or
🔍 Explanation: meeting.
Sentence: She ___ to the market every morning.
The phrase “every morning” indicates a Solution 86: ✅ Correct Answer: Option 2 – in
habitual action . 🔍 Explanation:
For habitual actions with third person singular Sentence:
subjects (he, she, it), we use the simple present He is interested ___ learning foreign languages.
tense with -s or -es added to the verb. The correct preposition to follow “interested” is
“in” when referring to an activity or subject.
Solution 81: ✅ Correct Answer: Option 2 – by
The correct preposition after “accompanied” is Solution 87: ✅ Correct Answer: Option 2 –
“by” when referring to the person who is with neatly
you. 🔍 Explanation:
Telegram (Previous year papers PDFs [SSC,Railway,DSSSB,UP SI]): https://t.me/RBE_S
YouTube (Free lectures and job updates): https://www.youtube.com/c/RBERevolutionByEducation
Download RBE application for SSC Exams Complete Preparation
Sentence: She writes essays ___. adverb , not an adjective or a
We need a word that describes how she writes — comparative/superlative form.
that is, the manner of writing. This calls for an

For SSC Exams Complete Preparation (Download RBE Application)


(Learn from those who have cleared the exam themselves)
https://play.google.com/store/apps/details?id=com.revolution.education

Telegram (Previous year papers PDFs [SSC,Railway,DSSSB,UP SI]): https://t.me/RBE_S


YouTube (Free lectures and job updates): https://www.youtube.com/c/RBERevolutionByEducation
Download RBE application for SSC Exams Complete Preparation

SSC Steno 2025 T-1 Cloze Test All 55 Questions with Detailed Solution and Answer key

Q1-5. In the following passage, some words Q6-10. In the following passage, some words
have been deleted. Read the passage carefully have been deleted. Read the passage carefully
and select the most appropriate option to fill and select the most appropriate option to fill
in each blank: each blank:
Modern ethical debates tend to concern the Researchers in quantum computing face the twin
conflict between deontological and difficulties of (1) _____ qubit coherence and (2)
consequentialist approaches, each providing a (1) _____ error rates. While superconducting qubits
_____ methodology. Deontology requires respect have shown promise, maintaining coherence
for moral (2) _____ irrespective of the result, beyond microseconds remains (3) _____ due to
while consequentialism judges on the basis of the environmental decoherence. Furthermore, scaling
net (3) _____. Objections against pure up the number of qubits requires (4) _____
consequentialism suggest it will allow (4) _____ control electronics that themselves introduce
actions if these bring about maximum overall noise. To overcome these hurdles, teams are
welfare. In reply, some recommend (5) _____ exploring (5) _____ techniques such as surface
hybrid theories that combine obligations with codes and topological qubits.
outcome sensitivity.
What should come in the place of blank 1?
What should come in the place of blank (a) to durable (b) ephemeral
number 1? (c) Permanent (d) Perennial
(a) quarrelsome (b) rigorous
(c) cursory (d) vacuous What should come in the place of blank 2 ?
(a) negligible (b) prohibitive
What should come in the place of blank (c) cheap (d) low
number 2 ?
(a) mood (b) actions What should come in the place of blank 3 ?
(c) conjectues (d) fallacies (a) tractable (b) elusive
(c) innate (d) redundant

What should come in the place of blank What should come in the place of blank 4 ?
number 3 ? (a) austere (b) sophisticated
(a) goods (b) outcomes (c) rudimentary (d) obsolete
(c) benefits (d) profit
What should come in the place of blank 5 ?
What should come in the place of blank (a) fallacios (b) Q uantum -
number 4 ? error-correcting
(a) Laudable (b) immoral (c) presumptive (d) monochromatic
(c) Beneficial (d) Delightful
Q11-15. I n the following question, a sentence
with a blank is given, followed by five words
What should come in the place of blank labeled (1), (2), (3), (4), and (5).
number 5 ?
(a) good (b) Well-balanced Choose the word that fills the blank
(c) over simple (d) Irrelevant appropriately in the context of the passage.

The truth of the self-conscious individual lies not


Telegram (Previous year papers PDFs [SSC,Railway,DSSSB,UP SI]): https://t.me/RBE_S
YouTube (Free lectures and job updates): https://www.youtube.com/c/RBERevolutionByEducation
Download RBE application for SSC Exams Complete Preparation
in its immediate certainty, but in the dialectical (a) undermines (b) were undermine
movement of its becoming. (c) undermined (d) are undermining
Consciousness, in its pursuit of knowledge,
initially assumes that its object is something Use the appropriate for of the verb to
external and independent. Yet, through the complete the sentence. ( 4 )
labor of experience, it discovers that this object is (a) Yield (b) Justification
not truly separate but _____ (1) by consciousness (c) Detachment (d) Retraction
itself.
Use the appropriate for of the verb to
The apparent duality of subject and object is thus complete the sentence. ( 5 )
sublated in the dialectic, revealing that knowledge (a) Possessive pronoun
is not the passive reception of an external truth (b) Demonstrative pronoun
but an active process in which consciousness (c) Reflexive pronoun
shapes and ______ (2) its own reality. (d) Interrogative pronouns

This movement unfolds through a series of Q16-20. In the following passage, some words
contradictions, in which each stage of have been omitted. Read the passage carefully
consciousness, believing itself to have attained and choose the most appropriate option from the
certainty, encounters its own inadequacy. The given alternatives to fill in each blank. Mark your
master-slave dialectic exemplifies this: self- answer for each question.
consciousness seeks recognition from another, For many young professionals, choosing a career
yet in asserting its dominance over the other, it is no longer just about earning a salary—it’s
paradoxically ______ ( 3) its own freedom. about finding meaning in what they do. Some are
drawn to high-paying jobs, while others feel more
The master, in reducing the slave to a mere (1) ______ working in fields that align with their
instrument, deprives himself of a fully realized personal values. This growing divide between
recognition. Meanwhile, the slave, through passion and paycheck has led to tough decisions
labor and self-transformation, attains a deeper for many. While financial stability is important,
awareness of autonomy, demonstrating that ignoring one’s true interests can lead to long-term
freedom is not given but achieved (2) ______. Experts suggest that a balanced
through negation and _______ ( 4 ). spirit (Geist) approach is possible. One can pursue a stable
emerges as the reconciliation of these conflicts— job and still nurture their passions through part-
not a static substance but a dynamic process of time work, volunteering, or creative outlets.
self-realization. Spirit unfolds historically, However, such a path requires careful (3)
manifesting itself _____ ( 5 ) in institutions, ______. Making impulsive career shifts without
culture, and philosophy, gradually moving toward planning can lead to regret. It’s important to
absolute knowing. This historical progression is evaluate not just income, but also growth,
not linear but dialectical, driven by the negation of purpose, and job (4) ______. After all, the goal is
old forms and the emergence of new ones. not just to survive—but to build a life that is both
secure and (5) ______.
Use the appropriate for of the verb to
complete the sentence. (1) (1)
(a) Mediating (b) Mediated (a) challenged (b) comfortable
(c) Mediates (d) has mediated (c) proud (d) fulfilled

Use the appropriate for of the verb to (2)


complete the sentence. ( 2 ) (a) success (b) burnout
(a) Transforms (b) Transports (c) freedom (d) boredom
(c) Interrupts (d) Disabuses
(3)
Use the appropriate for of the verb to (a) luck (b) hesitation
complete the sentence. ( 3 )
Telegram (Previous year papers PDFs [SSC,Railway,DSSSB,UP SI]): https://t.me/RBE_S
YouTube (Free lectures and job updates): https://www.youtube.com/c/RBERevolutionByEducation
Download RBE application for SSC Exams Complete Preparation
(c) planning (d) risk given alternatives to fill in each blank. Mark your
answer for each question.
(4) As artificial intelligence becomes more integrated
(a) comfort (b) loyalty into daily life, debates around its ethical use are
(c) stability (d) attitude growing louder. Machines now make decisions
that were once the (1) ______ of human
(5) judgment—from recommending medical
(a) wealthy (b) creative treatments to scanning resumes. While AI offers
(c) rewarding (d) routine speed and efficiency, it lacks moral (2) ______,
raising concerns about bias, accountability, and
Q21-25. In the following passage, some words transparency. Critics argue that without strict
have been deleted. Read the passage carefully ethical frameworks, AI could reinforce social
and select the most appropriate option to fill in inequalities or even make harmful choices with no
each blank: clear line of (3) ______. Developers face the
Progress in digital technology has (1) _____ challenge of embedding values into code—yet
transformed contemporary life. From artificial whose values should apply? In multicultural
intelligence to smartphones, technologies are (2) societies, defining what is “right” is rarely (4)
_____ transforming industries and societies. But ______. Some suggest collaborative oversight
with speed comes (3) _____ as numerous involving ethicists, technologists, and
workers are left uncertain about their future jobs. policymakers. Others warn that regulation must
To adapt to this disruption, people need to be (4) evolve faster than innovation to prevent misuse.
_____, constantly renewing their (5) _____ to In the race to build smarter systems, the real
stay competitive. question remains: can we create machines that
are not just intelligent, but also (5) ______?
Select the most appropriate option to fill in blank
number 1. (1)
(a) marginally (b) sporadically (a) domain (b) version
(c) occasionally (d) drastically (c) burden (d) process

Select the most appropriate option to fill in blank (2)


number 2. (a) clarity (b) depth
(a) hesitantly (b) rarely (c) instinct (d) reasoning
(c) accidentally (d) invariably
(3)
Select the most appropriate option to fill in blank (a) argument (b) command
number 3. (c) accountability (d) conduct
(a) celebration (b) incentive
(c) harmony (d) disruption (4)
(a) visible (b) consistent
Select the most appropriate option to fill in blank (c) obvious (d) universal
number 4.
(a) proactive (b) stubborn (5)
(c) reactive (d) adaptable (a) controlled (b) obedient
(c) ethical (d) productive
Select the most appropriate option to fill in blank
number 5. Q31-35. In the following passage, some words
(a) strategies (b) diversions have been omitted. Read the passage carefully
(c) obstacles (d) competencies and choose the most appropriate option from the
given alternatives to fill in each blank. Mark your
Q26-30. In the following passage, some words answer for each question.
have been omitted. Read the passage carefully In modern societies, the concept of freedom is
and choose the most appropriate option from the often celebrated as an essential right. However,
Telegram (Previous year papers PDFs [SSC,Railway,DSSSB,UP SI]): https://t.me/RBE_S
YouTube (Free lectures and job updates): https://www.youtube.com/c/RBERevolutionByEducation
Download RBE application for SSC Exams Complete Preparation
true freedom is not the absence of rules but the programs that prepare students for space-related
presence of thoughtful (1) ______. Without it, fields. These programs not only focus on science,
freedom risks turning into chaos, where individual but also build problem-solving and leadership
actions undermine collective welfare. A society skills. Working in space will bring its own
that prizes only personal liberty without shared challenges—long durations, isolation, and
responsibility may soon face moral and civic (2) technical failures will require ( 2) _____
______. While people often equate freedom with preparation. Yet, the chance to be part of
doing as they please, meaningful liberty requires something bigger than oneself is what draws
a commitment to boundaries that preserve dignity many young minds. With space tourism rising and
and (3) ______ for all. This balance is difficult to private companies joining the race, the job market
maintain, especially in democracies where voices is expected to ( 3) _______ . However, to
are many and interests often (4) ______. Yet, it is succeed in this field, one must stay curious,
precisely through dialogue, compromise, and adaptable, and ( 4) _____ . In the future, reaching
mutual respect that freedom finds its most for the stars might not be a dream, but a ( 5) _
enduring expression. In the end, freedom _____.
divorced from responsibility is not progress, but a
subtle (5) ______ of what it truly means to be What will come in (1)
free. (a) ambition (b) option
(c) mystery (d) illusion
(1)
(a) control (b) governance What will come in ( 2 )
(c) restraint (d) leadership (a) casual (b) emotional
(c) intense (d) frequent
(2)
(a) revival (b) collapse What will come in ( 3 )
(c) education (d) loyalty (a) collapse (b) expand
(c) decline (d) compete
(3)
(a) justice (b) convenience What will come in ( 4 )
(c) access (d) equality (a) reliable (b) silent
(c) proud (d) stubborn
(4)
(a) vary (b) divide What will come in ( 5 )
(c) contradict (d) clash (a) mistake (b) memory
(c) reality (d) direction
( 5)
(a) rejection (b) misunderstanding Q41-46. In the following passage, some words
(c) distortion (d) creation have been omitted. Read the passage carefully
and choose the most appropriate option from the
Q36-40. In the following passage, some words given alternatives to fill in each blank. Mark your
have been omitted. Read the passage carefully answer for each question.
and choose the most appropriate option from the Ravi and Arjun had been rivals on the football
given alternatives to fill in each blank. Mark your field since their first year in school. Both were
answer for each question. talented players, and their competition often
As technology continues to advance, space brought out the best—and sometimes the worst—
exploration is no longer limited to astronauts and in them. When the school team was selected for
scientists. In the coming decades, space could the district championship, both were chosen to
become a place for engineers, architects, doctors, play. At first, their rivalry made teamwork difficult.
and even artists. The idea of working beyond Passes were missed, tempers (1) ______ and the
Earth, once considered science fiction, is slowly coach had to step in. He reminded them that
becoming a realistic ( 1) ______ . Many countries personal pride should never come before the
have already started investing in training team. Slowly, Ravi and Arjun began to (2) ______
Telegram (Previous year papers PDFs [SSC,Railway,DSSSB,UP SI]): https://t.me/RBE_S
YouTube (Free lectures and job updates): https://www.youtube.com/c/RBERevolutionByEducation
Download RBE application for SSC Exams Complete Preparation
one another’s strengths instead of focusing on choosing self-growth over temporary approval
their differences. As trust grew, their coordination brings long-term (4) ______. True friends will
improved, and the team’s performance soared. In never ask you to (5) ______ your dreams just to
the semi-final match, it was a pass from Ravi that fit in.
helped Arjun score the winning goal. The crowd
cheered not just for the win, but for the (1)
sportsmanship. What began as a clash ended in (a) skills (b) media
a strong (3) ______ that taught them the value of (c) Image (d) Circle
unity, mutual (4) ______ and shared (5) ______.
(2)
What will come in (1) ____? (a) unnecessary (b) mild
(a) broke (b) cooled (c) academic (d) emotional
(c) flared (d) paused
(3)
What will come in (2) _______? (a) no (b) later
(a) ignore (b) appreciate (c) sorry (d) yes
(c) compete (d) prove
(4)
What will come in ( 3) _ ____? (a) struggle (b) regret
(a) partnership (b) challenge (c) respect (d) benefit
(c) lesson (d) match
(5)
What will come in (4) ____? (a) share (b) delay
(a) effort (b) celebration (c) sacrifice (d) chase
(c) respect (d) rivalry
Q51-55. In the following passage, some words
What will come in (5) _____? have been omitted. Read the passage carefully
(a) pressure (b) fear and choose the most appropriate option from the
(c) pride (d) conflict given alternatives to fill in each blank. Mark your
answer for each question.
Q46-50. In the following passage, some words Social media has become an important part of
have been omitted. Read the passage carefully modern life, especially among young people. It
and choose the most appropriate option from the allows them to connect, share ideas, and express
given alternatives to fill in each blank. Mark your themselves freely. However, this freedom also
answer for each question. brings challenges. Many teenagers feel pressure
For high school students, the final years before to look perfect or appear popular online. They
graduation are often filled with pressure from all often compare their lives to others, which can
directions. While teachers and parents focus on affect their self-confidence. Experts suggest that
marks and entrance exams, many students also it’s important to build a healthy ( 1) _ _____
face pressure from friends to fit in or appear toward social media. Instead of trying to copy
“cool.” This creates a constant struggle between others, young users should focus on their real-life
staying focused on studies and maintaining social goals and (2) _____ . Parents and schools also
(1) ______. The fear of missing out on group have a role to play by teaching students how to
activities or being judged can distract even the use digital tools (3) _____ . Setting screen-time
most serious learners. Over time, this pressure limits and encouraging outdoor activities can help
builds up and may lead to (2) ______ stress. restore a sense of balance. In the end,
Some students feel torn between expectations technology should be a tool for growth, not a
and personal goals, unsure of what truly matters. source of (4) ____ . By using social media wisely,
Experts say the key is to set clear priorities and young people can turn it into a space of learning,
learn to say (3) ______ when needed. Building (5) _____ and real connection.
confidence and developing time-management
skills can help students find balance. In the end, What will come in (1)
Telegram (Previous year papers PDFs [SSC,Railway,DSSSB,UP SI]): https://t.me/RBE_S
YouTube (Free lectures and job updates): https://www.youtube.com/c/RBERevolutionByEducation
Download RBE application for SSC Exams Complete Preparation
(a) attitude (b) habit (c) effectively (d) blindly
(c) relationship (d) thought
What will come in (4)
What will come in (2) (a) fun (b) distraction
(a) wishes (b) schedules (c) stress (d) failure
(c) priorities (d) dreams
What will come in (5)
What will come in (3) (a) fame (b) growth
(a) widely (b) safely (c) fun (d) creativity

Answer Key:

Q1 b Q2 b Q3 b Q4 b Q5 b
Q6 b Q7 b Q8 b Q9 b Q10 b
Q11 b Q12 a Q13 a Q14 c Q15 c
Q16 d Q17 b Q18 c Q19 c Q20 c
Q21 d Q22 d Q23 d Q24 d Q25 d
Q26 a Q27 c Q28 c Q29 d Q30 c
Q31 c Q32 b Q33 d Q34 d Q35 c
Q36 b Q37 c Q38 b Q39 a Q40 c
Q41 c Q42 b Q43 a Q44 c Q45 c
Q46 c Q47 d Q48 a Q49 d Q50 c
Q51 a Q52 c Q53 b Q54 c Q55 d

Solution 1: Explanation Solution 4: Explanation


The passage discusses two ethical approaches, The passage mentions objections against
portraying each as a serious, well-defined consequentialism suggesting it could
system—a rigorous methodology fits this context. allow immoral actions if those actions bring about
"Quarrelsome" (argumentative), "cursory" maximum overall welfare.
(superficial), and "vacuous" (empty) do not This highlights a common critique that
convey the intended meaning of a thorough, consequentialism might justify unethical means
systematic method. for a perceived greater good.

Solution 2: Explanation Solution 5: Explanation


In the context of deontological ethics, respect for The passage refers to hybrid theories that
moral actions irrespective of results fits the combine obligations with outcome sensitivity,
passage, emphasizing the importance of the suggesting a well-balanced approach between
nature of actions themselves rather than deontological and consequentialist ethics.
consequences. "Good" is too vague, "over simple" (likely intended
The other options—mood, conjectures, fallacies— as "oversimple") is a negative descriptor, and
do not logically fit the theme of moral respect "irrelevant" does not fit the positive
within this passage. recommendation context.

Solution 3: Explanation Solution 6: Explanation


Consequentialism judges actions based on the "Ephemeral" means short-lived or transient, which
net outcomes they produce, which encompasses fits the challenge of maintaining qubit coherence
both positive and negative results. in quantum computing.
While "goods," "benefits," and "profit" focus more The other options—"to durable," "Permanent,"
narrowly on positive results, "outcomes" is a and "Perennial"—contradict the difficulty
broader term that fits the ethical evaluation described, as qubit coherence currently is not
context better. permanent or durable.

Telegram (Previous year papers PDFs [SSC,Railway,DSSSB,UP SI]): https://t.me/RBE_S


YouTube (Free lectures and job updates): https://www.youtube.com/c/RBERevolutionByEducation
Download RBE application for SSC Exams Complete Preparation
Solution 7: Explanation consciousness.
The passage discusses difficulties, so "prohibitive Other options like "mediating" (present participle),
error rates"—meaning very high or unacceptable "mediates" (present tense), or "has mediated"
error rates—fits the context of challenges in (present perfect tense) do not fit grammatically or
quantum computing. contextually.
"Negligible," "cheap," and "low" suggest minimal
or affordable error rates, which contradict the Solution 12: Explanation:
challenge described . The sentence is: "knowledge is not the passive
reception of an external truth but an active
Solution 8: Explanation process in which consciousness shapes and
"Elusive" means difficult to achieve or grasp, ______(2) its own reality."
which fits the challenge of maintaining qubit The verb needed must suggest that
coherence beyond microseconds due to consciousness actively changes or develops its
environmental decoherence. own reality.
The other options—"tractable" (manageable), "Transforms" fits this meaning perfectly as it
"innate" (natural), and "redundant" conveys active change and creation.
(unnecessary)—do not align with the difficulty Other options, "Transports," "Interrupts," and
described in the passage.The part of the "Disabuses," do not fit the context. "Transports"
sentence that contains an error is in section (2) — means carries from one place to another without
"were given a welcome kit." The subject "Each change, "Interrupts" means stops or breaks, and
participant" is singular, so the verb should be "Disabuses" means frees from a misconception.
singular, "was given," instead of plural "were
given." Solution 13: Explanation:
The sentence is: "self-consciousness seeks
Solution 9: Explanation recognition from another, yet in asserting its
Scaling up the number of qubits dominance over the other, it paradoxically ______
requires sophisticated control electronics, (3) its own freedom."
meaning advanced and complex technology, The verb needed must indicate that the action
which fits the challenge described. taken by self-consciousness weakens or subverts
The other options—"austere," "rudimentary," and its own freedom.
"obsolete"—imply simplicity or outdatedness, "Undermines" means to weaken or damage
which would not meet the needs of advanced something gradually, which fits perfectly here.
quantum computing systems. Other options like "were undermine,"
"undermined," and "are undermining" do not fit
Solution 10: Explanation grammatically or contextually as well as
The passage discusses overcoming hurdles in "undermines," which is simple present tense
quantum computing, so "quantum-error-correcting agreeing with "self-consciousness."
techniques" such as surface codes and
topological qubits fits perfectly. Solution 14: Explanation:
The other options—fallacious, presumptive, The sentence discusses freedom being achieved
monochromatic—do not relate to the technical through negation and _______ (4).
context of quantum computing error correction. The word "Detachment" fits as it conveys a sense
of separation or distancing needed in the
Solution 11: Explanation: dialectical process, which aligns with the
The passage states: "it discovers that this object philosophical tone.
is not truly separate but _____(1) by The other options "Yield," "Justification," and
consciousness itself." "Retraction" do not fit well in the context of
The object is something that consciousness negation and dialectical movement.
shapes or influences, so it is acted upon or "Detachment" suggests the mindful distancing
"mediated" by consciousness. inherent in the dialectical sublation and
"Mediated" is the past participle form used here to reconciliation process described as "spirit (Geist)
express that the object is influenced or shaped by
Telegram (Previous year papers PDFs [SSC,Railway,DSSSB,UP SI]): https://t.me/RBE_S
YouTube (Free lectures and job updates): https://www.youtube.com/c/RBERevolutionByEducation
Download RBE application for SSC Exams Complete Preparation
emerges." making before changing careers.

Solution 15: Explanation: Solution 19: ✅ Correct Answer: stability


The sentence is: "Spirit unfolds historically, The sentence says:
manifesting itself _____(5) in institutions, culture, “It’s important to evaluate not just income, but
and philosophy..." also growth, purpose, and job (4) ______.”
The word "itself" is a reflexive pronoun, used here Here, the list includes growth , purpose , and the
to indicate that the spirit is acting back on or missing word must logically align with these
manifesting itself. career-related qualities. The most fitting word is
The reflexive pronoun reflects the subject (spirit) stability , which refers to job security and
back onto itself, which is grammatically correct consistency —a key factor in long-term career
and contextually appropriate. satisfaction.
Possessive pronouns (like its), demonstrative
pronouns (like this/that), and interrogative Solution 20: ✅ Correct Answer: rewarding
pronouns (like which/what) do not fit correctly The final sentence says:
here. “After all, the goal is not just to survive—but to
build a life that is both secure and (5) ______.”
Solution 16: ✅ Correct Answer: fulfilled Here, the contrast is between mere survival and
The sentence says: a life that offers fulfillment and satisfaction .
“Some are drawn to high-paying jobs, while The word rewarding fits perfectly—it conveys the
others feel more (1) ______ working in fields that idea of a life that brings personal and emotional
align with their personal values.” value , not just financial security.
Here, the contrast is between money-driven
choices and value-driven satisfaction . The Solution 21: ✅ Correct Answer: drastically
most appropriate word is fulfilled , which conveys Contextual Fit:
a sense of emotional and personal satisfaction The phrase “Progress in digital technology has
—exactly what one feels when their work aligns ___ transformed contemporary life” demands a
with their values. word that reflects intensity and scale .
“Drastically” aligns with the profound and
Solution 17: ✅ Correct Answer: burnout sweeping changes brought by AI, smartphones,
The sentence says: etc.
“While financial stability is important, ignoring
one’s true interests can lead to long-term (2) Solution 22: ✅ Correct Answer: invariably
______.” Contextual Fit:
Here, the idea is that neglecting personal The sentence: “From artificial intelligence to
passions for the sake of money may result in smartphones, technologies are (2) _____
emotional exhaustion or dissatisfaction over transforming industries and societies.” demands a
time. The word burnout perfectly captures this— word that conveys consistency and certainty .
it's a state of chronic stress, fatigue, and loss of “Invariably” reinforces the idea that this
motivation due to misalignment between work transformation is ongoing and predictable ,
and personal values. aligning with the tone of the passage.

Solution 18: ✅ Correct Answer: planning Solution 23: ✅ Correct Answer: disruption
The sentence says: Contextual Fit:
“However, such a path requires careful (3) The sentence: “But with speed comes (3) _____
______. Making impulsive career shifts without as numerous workers are left uncertain about
planning can lead to regret.” their future jobs.” clearly implies a negative
The second sentence already warns against consequence of technological advancement.
impulsive decisions , so the blank must refer to “Disruption” captures the economic and social
the opposite— careful preparation or instability caused by automation and digital
forethought . The word planning fits perfectly, as transformation.
it emphasizes the need for strategic decision-
Telegram (Previous year papers PDFs [SSC,Railway,DSSSB,UP SI]): https://t.me/RBE_S
YouTube (Free lectures and job updates): https://www.youtube.com/c/RBERevolutionByEducation
Download RBE application for SSC Exams Complete Preparation
Solution 24: ✅ Correct Answer: adaptable The phrase “no clear line of accountability” is a
Contextual Fit: common and precise expression in ethics and
The sentence: “To adapt to this disruption, people governance.
need to be (4) _____, constantly renewing It highlights the absence of a responsible party
their…” clearly calls for a trait that reflects when AI makes decisions with serious
flexibility and readiness to evolve . consequences.
“Adaptable” directly supports the verb “adapt” and
aligns with the theme of continuous learning and Solution 29: ✅ Correct Answer: universal
change . 🔍 Sentence Context:
“In multicultural societies, defining what is ‘right’ is
Solution 25: ✅ Correct Answer: adaptable rarely (4) ______.”
Contextual Fit: This sentence highlights the challenge of ethical
The sentence: “…constantly renewing their (5) consensus across diverse cultures and belief
_____ to stay competitive.” clearly refers to skills system
or capabilities that help individuals thrive in a 🔍 Why “Universal” Is Ideal:
changing job market. In multicultural societies , moral values vary
“Competencies” is the most precise term for widely.
professional abilities that require regular updating. The word “universal” precisely captures the idea
that no single ethical standard applies to all .
Solution 26: ✅ Correct Answer: domain It aligns with the broader theme of the passage:
🔍 Explanation: the difficulty of embedding shared values into AI
Let’s analyze the sentence: systems.
“Machines now make decisions that were once
the (1) ______ of human judgment…” Solution 30: ✅ Correct Answer: ethical
🔸 Contextual Meaning: 🔍 Sentence Context:
The sentence refers to areas traditionally handled “In the race to build smarter systems, the real
by humans , such as medical recommendations question remains: can we create machines that
and resume screening. are not just intelligent, but also (5) ______?”
The word must reflect territory or scope of human This final sentence contrasts intelligence with a
decision-making. deeper moral quality—suggesting that ethical
behavior is the true goal.
Solution 27: ✅ Correct Answer: instinct 🔍 Why “Ethical” Is Ideal:
The blank needs a noun that reflects innate moral The passage revolves around AI’s moral
judgment or ethical intuition , which AI lacks. limitations and the need for value-driven design .
🔍 Why “Instinct” Fits Best: “Ethical” directly answers the question: can
Humans use moral instinct to make ethically machines be morally responsible , not just smart?
sensitive decisions. It ties back to earlier concerns: bias,
AI, being rule-based, lacks this natural ethical accountability, transparency, and cultural values.
compass , which is the core concern in the
passage. Solution 31: The correct answer is " restraint." In
the sentence, "true freedom is not the absence of
Solution 28: ✅ Correct Answer: accountability rules but the presence of thoughtful restraint," the
🔍 Sentence Context: word "restraint" best fits the context. It suggests
that freedom must be balanced with self-control
“Critics argue that without strict ethical
and responsible boundaries to prevent chaos and
frameworks, AI could reinforce social inequalities
protect collective welfare. The other options, while
or even make harmful choices with no clear line
related to management and direction, do not
of (3) ______.”
convey the necessary sense of self-imposed
This sentence discusses the lack of responsibility
limits inherent in preserving meaningful liberty.
or answerability when AI systems make harmful
decisions.
Solution 32: The correct answer is " collapse." In
🔍 Why “Accountability” Wins:
the sentence, "A society that prizes only personal
Telegram (Previous year papers PDFs [SSC,Railway,DSSSB,UP SI]): https://t.me/RBE_S
YouTube (Free lectures and job updates): https://www.youtube.com/c/RBERevolutionByEducation
Download RBE application for SSC Exams Complete Preparation
liberty without shared responsibility may soon accessible.
face moral and civic collapse," the word Illusion – implies something deceptive or unreal,
"collapse" best describes the potential downfall or which again contradicts the passage’s optimistic
breakdown of societal values and systems when tone.
responsibility is neglected. The other options do
not convey the same sense of failure or Solution 37: ✅ Correct Answer for (2): intense
breakdown in the context of societal 🧠 Explanation:
consequences. Let’s evaluate each option in context of the
sentence: "Working in space will bring its own
Solution 33: The correct answer is " equality." In challenges—long durations, isolation, and
the context of the sentence, "meaningful liberty technical failures will require (2) _____
requires a commitment to boundaries that preparation."
preserve dignity and (3) equality for all," the word Casual – ❌ inappropriate; space missions
"equality" best fits. It highlights that true freedom
demand seriousness, not informality.
is maintained only when everyone’s rights and
Emotional – ❌ while emotional resilience is
dignity are respected equally, underscoring the
important, the word doesn ’ t fit the context of “
importance of fairness in society. The other
technical failures ” and “ long durations. ”
options do not capture the idea of equal treatment
Intense – ✔ best fit. It conveys the rigorous,
for all members of society.
demanding nature of preparation required for
Solution 34: The correct answer is "clash." In the space challenges.
sentence, "democracies where voices are many Frequent – ❌ refers to how often something
and interests often (4) clash," the word "clash" happens, not the quality or depth of preparation.
most accurately describes situations where
differing interests or opinions come into conflict Solution 38: ✅ Correct Answer for (3): expand
with each other. This term emphasizes the 🧭 Explanation:
frequent disagreements present in diverse, Let’s revisit the sentence: "With space tourism
democratic societies. The other options do not rising and private companies joining the race, the
convey the same sense of direct opposition. job market is expected to (3) ______."
Now evaluate the options:
Solution 35: The correct answer is "distortion." Collapse – ❌ contradicts the optimistic tone and
In the sentence, "freedom divorced from the idea of growth in space-related careers.
responsibility is not progress, but a subtle (5) Expand – ✔ fits perfectly. It reflects the
distortion of what it truly means to be free," the anticipated growth in employment opportunities
word "distortion" communicates that without due to increasing interest and investment in
responsibility, the real meaning of freedom is space.
warped or misrepresented. The other options do Decline – ❌ illogical in this context, as the
not accurately express this idea of the meaning of
passage highlights rising interest and investment.
freedom being twisted or altered in a negative
Compete – ❌ grammatically awkward. The job
way.
market doesn ’ t “ compete ” ; it grows , shrinks ,
or changes .
Solution 36: ✅ Correct Answer for (1): option
🧠 Explanation:
Solution 39: ✅ Correct Answer for (4): reliable
Let’s evaluate each choice in context:
🧠 Explanation:
Ambition – refers to a strong desire, but doesn’t
fit grammatically with “becoming a realistic ___.” Let’s examine the sentence: "…to succeed in this
field, one must stay curious, adaptable, and (4)
Option – ✔ fits perfectly. The phrase “becoming
_____."
a realistic option” means it is now a viable
We’re looking for a quality that complements
possibility , which aligns with the passage’s
curious and adaptable —traits essential for
tone.
thriving in space-related careers.
Mystery – implies something unknown, which
Reliable – ✔ best fit. It implies dependability ,
contradicts the idea of space becoming
Telegram (Previous year papers PDFs [SSC,Railway,DSSSB,UP SI]): https://t.me/RBE_S
YouTube (Free lectures and job updates): https://www.youtube.com/c/RBERevolutionByEducation
Download RBE application for SSC Exams Complete Preparation
which is crucial in high-stakes environments like Solution 44: The best word for blank (4) is
space. "respect." The passage describes how Ravi and
Silent – ❌ irrelevant and out of place in this Arjun learned the value of unity and mutual
context. respect through their teamwork and
Proud – ❌ doesn ’ t align with the tone of sportsmanship. "Effort," "celebration," and
practical qualities needed for success. "rivalry" do not fit the intended moral lesson of
Stubborn – ❌ negative connotation; contradicts appreciating each other and working together.
adaptability.
Solution 45: The correct word for blank (5) is
"pride." The passage concludes with Ravi and
Solution 40: ✅ Correct Answer for (5): reality
Arjun sharing not just victory, but the pride in their
🧠 Explanation:
improved teamwork, sportsmanship, and mutual
Let’s look at the final sentence: "In the future, respect .
reaching for the stars might not be a dream, but a
(5) ______."
Solution 46: ✅ Correct Answer for (1): Image
We need a word that contrasts with “dream” and
“Maintaining social ______”
affirms the possibility or actualization of space
The most fitting word here is image , which refers
ambitions.
to how one is perceived socially —especially in
Mistake – ❌ illogical and negative; doesn ’ t fit
terms of popularity or trendiness.
the hopeful tone.
Memory – ❌ implies something from the past, Solution 47: ✅ Correct Answer for (2):
while the sentence is about the future. emotional
Reality – ✔ perfect fit. It completes the contrast: The passage discusses how peer pressure and
“not a dream, but a reality.” academic stress affect students emotionally.
Direction – ❌ vague and doesn ’ t complete the
contrast effectively. Solution 48: ✅ Correct Answer for (3): no
Saying “no” is a powerful skill in managing peer
Solution 41: The correct word to fill in the blank pressure and protecting one’s goals.
for tempers is "flared." In the context of rivalry and It reflects assertiveness and boundary-setting ,
missed passes, "flared" indicates that emotions which are essential for emotional well-being and
grew intense or heated, which is suitable here. time management.
"Broke," "cooled," and "paused" do not match the
context of escalating rivalry and missed teamwork Solution 49: ✅ Correct Answer for (4): benefit
opportunities. “In the end, choosing self-growth over temporary
approval brings long-term (4) ______.”
Solution 42: The correct choice is "appreciate," The sentence emphasizes the positive outcome
as the passage describes Ravi and Arjun moving of prioritizing personal development over peer
from rivalry to recognizing and valuing each pressure.
other's strengths. This marked a shift away from
focusing on their differences and towards
Solution 50: ✅ Correct Answer for (5):
collaboration and mutual respect, fitting the
sacrifice
context of improved teamwork in the story.
“True friends will never ask you to (5) ______
your dreams just to fit in.”
Solution 43: The correct word for blank (3) is
The sentence emphasizes that genuine friends
"partnership." The passage describes the
support your goals , not demand you give them
transformation of Ravi and Arjun’s relationship
up for social acceptance.
from rivalry to teamwork, emphasizing how their
conflict resulted in a strong partnership that
Solution 51: Answer for Blank (1): The most
benefited themselves and the entire team. Other
appropriate word to fill in the blank is:
options do not convey the idea of enduring
(1) attitude – This fits perfectly with the phrase “a
teamwork and unity emerging from their rivalry.
healthy attitude toward social media,” which
implies having a balanced and thoughtful
Telegram (Previous year papers PDFs [SSC,Railway,DSSSB,UP SI]): https://t.me/RBE_S
YouTube (Free lectures and job updates): https://www.youtube.com/c/RBERevolutionByEducation
Download RBE application for SSC Exams Complete Preparation
approach to its use. Solution 54: The most appropriate word here is:
stress – It aligns perfectly with the tone of the
Solution 52: The most appropriate word here is: passage, which discusses the emotional pressure
( 3) priorities – It pairs naturally with “goals” and and comparison anxiety that social media can
emphasizes personal progress and success. cause among young users.

Solution 53: Answer for Blank (3): Solution 55: The most appropriate word here is:
safely – This fits the context of responsible digital creativity – It complements “learning” and “real
behavior and addresses concerns like privacy connection,” suggesting that social media can be
and overuse. a platform for expressive and innovative
engagement .

For SSC/Railway/DSSB Exams Complete Preparation (Download RBE Application)


(Learn from those who have cleared the exam themselves)
https://play.google.com/store/apps/details?id=com.revolution.education

Telegram (Previous year papers PDFs [SSC,Railway,DSSSB,UP SI]): https://t.me/RBE_S


YouTube (Free lectures and job updates): https://www.youtube.com/c/RBERevolutionByEducation
Download RBE application for SSC Exams Complete Preparation

SSC Steno 2025 T-1 Active- Passive Voice All 85 Questions with Detailed Solution and Answer key

Q1. Convert the sentence provided below from Q4. Select the correct passive form of the
its passive voice structure to an active voice given sentence.
structure. "He will submit the report by Friday."
The poem was composed by the student during (a) The report will be submitted by nim by Friday.
the literature class. (b) The report will have been submitted by him.
(a) The poem is being composed by the student (c) The report submitted by him by Friday.
during the class. (d) The report is submitted by him by Friday.
(b) The student composed the poem during the
literature class. Q5. Select the correct active form of tne given
(c) The student is composing the poem during the sentence.
literature class. "The window had been broken by the boy."
(d) The student had composed the poem during (a) The boy had broken the window,
the class. (b) The boy is breaking the window.
(c) The boy broke the window.
Q2. Change the following from active to (d) The boy has broken the window.
passive:
By the time the team arrived, the technicians had Q6. Convert the sentence provided below from
already repaired the control panel and tested all its passive voice structure to an active voice
functions twice. structure:
(a) Everything was fixed before the team reached The emergency steps had been taken by the
the site. authorities before the storm hit.
(b) By the time the team arrived, the control panel (a) The emergency steps were taken before the
had already been repaired, and all functions had storm hit.
been tested twice by the technicians. (b) The authorities took emergency steps.
(c) The technicians repaired and tested (c) The storm was faced with emergency steps.
everything twice before the team arrived. (d) The authorities had taken the emergency
(d) All functions were being tested by the time the steps before the storm hit.
team arrived.
Q7. Select the correct passive form of the
Q3. Convert the sentence provided below from given sentence.
its passive voice structure to an active voice “She is painting the wall.”
structure: (a) The wall was being painted.
All the entries must have been verified by the (b) The wall has painted.
auditor before submission. (c) The wall is being painted by her.
(a) The auditor was verifying all entries before (d) The wall is painted by her,
submission,
(b) The auditor must have verified all the entries Q8. Convert the sentence provided below from
before submission. its passive voice structure to an active voice
(c) All entries were verified by the auditor. structure:
(d) All the entries are to be verified before su The book was read by the student in just two
bmission . days.
(a) The student has read the book in just two
days.

Telegram (Previous year papers PDFs [SSC,Railway,DSSSB,UP SI]): https://t.me/RBE_S


YouTube (Free lectures and job updates): https://www.youtube.com/c/RBERevolutionByEducation
Download RBE application for SSC Exams Complete Preparation
(b) The book ss read by the student in Just two (a) The meal will prepare by the che f by noon,
Gays. (b) The chef will be prepared the
(c) The student read the book in just two days. meal.
(d) The student was reading the book in just two (c) The meal would be prepared by noon.
days. (d) The meal will be prepared by the chef by
noon.
Q9. Select the correct active form of the given
sentence. Q13. Change the following from active to
"The book is being read by him." passive:
(a) He reads the book.
(b) He is reading the book. People believe that the company will launch the
(c) He has read the book. new product before the end of this fiscal year.
(d) He was reading the book. (a) The company believes to launch the product
before the end of the fiscal year.
Q10. Convert the sentence provided below (b) It has been believed that the new product will
from its passive voice structure to an active launch this year.
voice structure: (c) The new product i s believed to be launched
before the fiscal year ends.
The groceries are delivered to the house by the (d) It is believed that the company will launch the
shopkeeper every evening. new product before the end of this fiscal year.
(a) The shopkeeper had delivered the groceries
to the house. Q14. Convert the sentence provided below
(b) The groceries are being delivered 10 the from its passive voice structure to an active
house every evening. voice structure.
(c) The groceries were delivered to the house by
the shopkeeper, It i s believed that the minister will announce the
(d) The shopkeeper delivers the groceries to the new reforms next week
house every evening. (a) People believe that the minister will announce
the new reforms next week,
Q11. Convert the sentence provided below (b) The new reforms are announced by the
from its passive voice structure to an active minister,
voice structure: (c) It was believed that the reforms would be
announced .
The house is cleaned by her every morning (d) People think the reforms have already been
before breakfast. announced.
(a) She cleans the house every morning before
breakfast. Q15. Select the correct passive form of the
(b) She i s cleaning the house every morning given sentence.
before breakfast.
(c) She cleaned the house every morning before "The manager will approve the application.”
breakfast , (a) The application has been approved.
(d) She has cleaned the house every morning (b) The application will be approved by the
before breakfast. manager,
(c) The application was being approved.
Q12. (d) The application Is approved by the manager.

Select the correct passive form of the given Q16. Select the correct active form of the
sentence. given sentence.

"The chef will prepare the meal by noon." "The letters were delivered by the postman."
(a) The postman h ad delivered the letters.
(b) The postman delivers the letters,
Telegram (Previous year papers PDFs [SSC,Railway,DSSSB,UP SI]): https://t.me/RBE_S
YouTube (Free lectures and job updates): https://www.youtube.com/c/RBERevolutionByEducation
Download RBE application for SSC Exams Complete Preparation
(c) The postman delivered the letters. Q21. Select the correct passive form of the given
(d) The postman has delivered the letters. sentence.
"We may have misunderstood the instructions."
Q17. Select the correct active form of the given (a) The instructions are being misunderstood.
sentence. (b) The instructions may have misunderstood.
(c) The instructions may have been
"The hall is being decorated by the staff." misunderstood.
(a) The staff has decorated the hall. (d) The instructions may be misunderstood.
(b) The sta ff was decorating the hall.
(c) The staff decorates the hall. Q22. Select the correct passive form of the given
(d) The staff is decorating the h all. sentence.
"They serve food to the guests."
Q18. Transform the following sentence into (a) Food is being served by them.
active voice. (b) Food was served to the guests.
(c) Food has served by them.
The royal sword is said to have been transferred (d) Food is served to the guests by them.
to a new museum where it would have been
preserved property. Q23. Transform the following sentence into active
(a) Someone is said to have transferred the royal voice.
sword to a new museum where they would The tree was felled by the farmer.
preserve it properly. (a) The farmer fell the tree.
(b) Someone is said to transfer the royal sword (b) The farmer felled the tree.
to a new museum where they would have (c) The farmer has felled the tree.
preserved i t properly. (d) The farmer fell from the tree.
(c) Someone is said to have transferred the royal
sword to a new museum which they would have Q24. Select the correct passive form of the given
preserved it properly. sentence.
(d) Someone is said to have transferred the royal "They can be delivering the goods tomorrow."
sword to a new museum where they would have (a) The goods shall be delivering tomorrow.
preserved it properly . (b) The goods can be delivered tomorrow.
(c) The goods can deliver tomorrow.
Q19. Change the voice of the following sentence. (d) The goods can be being delivered tomorrow.
It is going to be constructed by them within a few
weeks. Q25. Choose the correct active voice form of the
(a) They are going to construct it within a few sentence:
weeks. "The editorial is written by Mamatha."
(b) They are going to construct it with a few (a) Mamatha writes the editorial.
weeks. (b) Mamatha is writes the editorial.
(c) They are going to constructed it within a few (c) Mamatha writing the editorial.
weeks. (d) Mamatha has writing the editorial.
(d) They are going to construct something within
a few weeks. Q26. Select the correct passive form of the given
sentence.
Q20. Select the correct active form. "The doctor prescribed the medicine."
"A decision will have been taken by the (a) The medicine is prescribed.
committee." (b) The medicine has been prescribed.
(a) The committee will have taken a decision. (c) The medicine was prescribed by the doctor.
(b) The committee took a decision. (d) The medicine had been prescribing.
(c) The committee has taken a decision.
(d) The committee will take a decision. Q27. Select the correct active form of the given
sentence.
"The hungry dog was gently fed by the owner."
Telegram (Previous year papers PDFs [SSC,Railway,DSSSB,UP SI]): https://t.me/RBE_S
YouTube (Free lectures and job updates): https://www.youtube.com/c/RBERevolutionByEducation
Download RBE application for SSC Exams Complete Preparation
(a) The owner fed the hungry dog. Q33. Select the correct passive form of the given
(b) The owner gently feeding the hungry dog. sentence.
(c) The owner gently fed the hungry dog. "He wrote the letter."
(d) The owner gently fed up the hungry dog. (a) The letter had been wrote.
(b) The letter is written by him.
Q28. Select the correct active form. (c) The letter wrote by him.
"The truth must be told by you." (d) The letter was written by him.
(a) You must have told the truth.
(b) You ought to tell the truth. Q34. Transform the following sentence into active
(c) You must tell the truth. voice.
(d) You must be telling the truth. This law which was passed in the parliament last
year should have been reviewed properly by the
Q29. Select the correct active form of the given members of the committee.
sentence. (a) The members of the committee should have
"The plants were watered by the gardener every reviewed the law properly, which the parliament
morning." passed last year.
(a) The gardener has watered the plants every (b) The members of the committee should have
morning. reviewed the law properly, which was passed in
(b) The gardener is watering the plants every the parliament last year.
morning. (c) The parliament should have reviewed the law
(c) The gardener watered the plants every properly, which the members of the committee
morning. passed last year.
(d) The gardener waters the plants every (d) The law should have reviewed the members
morning. of the committee properly which the parliament
passed last year.
Q30. Select the correct passive form of the given
sentence. Q35. Select the correct passive form of the given
"The editor has approved the final version." sentence.
(a) The final version is approved. "The mechanic may be checking the brakes now."
(b) The final version was approved by the editor. (a) The brakes may be checking now.
(c) The final version was being approved. (b) The brakes may be being checked now.
(d) The final version has been approved by the (c) The brakes may be check now.
editor. (d) The brakes may checked now.

Q31. Convert the sentence provided below from Q36. Select the correct active form of the given
its passive voice structure to an active voice sentence.
structure: "The walls are being painted by the workers."
The letter has been written by the manager (a) The workers paint the walls.
carefully. (b) The workers are painting the walls.
(a) The manager is writing the letter carefully. (c) The workers painted the walls.
(b) The manager was writing the letter carefully. (d) The workers were painting the walls.
(c) The manager will write the letter carefully.
(d) The manager has written the letter carefully. Q37. Convert the sentence provided below from
its passive voice structure to an active voice
Q32. Select the correct active form. structure:
"The article ought to have been revised by the The keys were found by him under the old
editor." wooden table.
(a) The editor must have revised the article. (a) He finds the keys under the old wooden table.
(b) The editor ought to have revised the article. (b) He is finding the keys under the old wooden
(c) The editor ought to revise the article. table.
(d) The editor ought to be revising the article. (c) He found the keys under the old wooden
table.
Telegram (Previous year papers PDFs [SSC,Railway,DSSSB,UP SI]): https://t.me/RBE_S
YouTube (Free lectures and job updates): https://www.youtube.com/c/RBERevolutionByEducation
Download RBE application for SSC Exams Complete Preparation
(d) He was finding the keys under the old wooden thoroughly.
table.
Q43. Select the correct passive form of the given
Q38. Select the correct active form of the given sentence.
sentence. "They should have been following the instructions
"The documents are being submitted by the carefully."
secretary." (a) The instructions should have been being
(a) The secretary has submitted the documents. followed carefully.
(b) The secretary is submitting the documents. (b) The instructions should have been followed
(c) The secretary submits the documents. carefully.
(d) The secretary was submitting the documents. (c) The instructions should followed carefully.

Q39. Convert the sentence provided below from (d) The instructions should be followed carefully.
its passive voice structure to an active voice
structure: Q44. Select the correct active form.
"The cake was baked by Mary for her brother's "He is known to me."
birthday party." (a) I know about him. (b) I knew him.
(a) Her brother had the cake baked by Mary. (c) I had known him. (d) I know him.
(b) Mary is baking the cake for her brother's
birthday party. Q45. Select the correct passive form of the given
(c) Mary baked the cake for her brother's birthday sentence.
party. "We are cleaning the room."
(d) The birthday cake was made by Mary for her (a) The room was cleaned.
brother. (b) The room is being cleaned by us.
(c) The room has been cleaning.
Q40. Select the correct passive form of the given (d) The room is cleaned.
sentence. "He is solving a complex problem."
(a) A complex problem had solved. Q46. Select the correct active form of the given
(b) A complex problem has solved. sentence.
(c) A complex problem is solved. "The bills are paid by him every month."
(d) A complex problem is being solved by him. (a) He was paying the bills every month.
(b) He is paying the bills every month.
Q41. Select the correct active form. (c) He paid the bills every month.
"The town was destroyed by the earthquake." (d) He pays the bills every month.
(a) The earthquake was destroying the town.
(b) The earthquake destroyed the town. Q47. Transform the following sentence into active
(c) The earthquake had destroyed the town. voice:
(d) The earthquake destroys the town. Were you instructed by them how to fill in the
form?
Q42. Convert the sentence provided below from (a) Was he informed how to fill in the form by
its passive voice structure to an active voice them?
structure. (b) Did you instruct them how to fill in the form?
The final draft of the report has been reviewed by (c) Did they instruct you how to fill in the form?
the supervisor thoroughly. (d) Were they informing you how to fill in the
(a) The supervisor was reviewing the final draft form?
thoroughly.
(b) The supervisor has reviewed the final draft of Q48. Select the correct passive form of the given
the report thoroughly. sentence.
(c) The supervisor had reviewed the final draft "The judge is hearing the final arguments."
thoroughly. (a) The final arguments was being heard.
(d) The final draft was reviewed by the supervisor (b) The final arguments has heard.

Telegram (Previous year papers PDFs [SSC,Railway,DSSSB,UP SI]): https://t.me/RBE_S


YouTube (Free lectures and job updates): https://www.youtube.com/c/RBERevolutionByEducation
Download RBE application for SSC Exams Complete Preparation
(c) The final arguments are being heard by the Q54. Select the correct active form.
judge. "Why was the match postponed by the
(d) The final arguments were heard. organizers?"
(a) Why are the organizers postponing the
Q49. Select the correct passive form of the given match?
sentence. (b) Why did the organizers postpone the match?
"I will complete the work tomorrow." (c) Why do the organizers postpone the match?
(a) The work will have completed by me. (d) Why have the organizers postponed the
(b) The work shall completed by me. match?
(c) The work is completed tomorrow by me.
(d) The work will be completed by me tomorrow. Q55. Select the correct active form of the given
sentence:
Q50. Transform the following sentence into active "The assignment work has been successfully
voice. finished ahead of time by Pragna."
All the structures built in that area had been (a) Pragna has successfully finished her
demolished due to encroachment. assignment ahead of time.
(a) They had demolished all the structures built in (b) Pragna has successfully finished her
that area due to encroachment by the people. assignment work ahead of time.
(b) They had demolished all the structures built in (c) Pragna have successfully finished her
that area due to encroachment. assignment work ahead of time.
(c) They had demolished all the structures build in (d) Pragna has finished her assignment work
that area due to encroachment. ahead of time.
(d) They had demolished all the structures that
were built in that area due to encroachment. Q56. Select the correct passive form of the given
sentence.
Q51. Select the correct active form. "The director should announce the results today."
"Was your phone being charged by someone?" (a) The results should be announce today.
(a) Is someone charging your phone? (b) The results should have been announced.
(b) Was someone charging your phone? (c) The results should be announced today.
(c) Has someone charged your phone? (d) The results should being announced.
(d) Did someone charge your phone?
Q57. Select the correct passive form of the given
Q52. Select the correct active form. sentence.
"Was your phone being charged by someone?" "They might be repairing the engine now."
(a) Is someone charging your phone? (a) The engine might be repaired now.
(b) Was someone charging your phone? (b) The engine may be repairing now.
(c) Has someone charged your phone? (c) The engine might repaired now.
(d) Did someone charge your phone? (d) The engine might be being repaired now.

Q53. Select the correct active form of the given Q58. Select the correct active form of the given
sentence. sentence.
"The dining hall was being thoroughly cleaned by "The cake had been decorated by the baker."
the workers." (a) The baker will decorate the cake.
(a) The workers were thoroughly cleaned the (b) The baker had decorated the cake.
dining hall. (c) The baker is decorating the cake.
(b) The workers had thoroughly cleaning the (d) The baker decorates the cake.
dining hall.
(c) The workers was thoroughly cleaning the Q59. Select the correct passive form of the given
dining hall. sentence.
(d) The workers were thoroughly cleaning the "She cleaned the house yesterday."
dining hall. (a) The house is cleaned by her yesterday.
(b) The house was cleaned by her yesterday.
Telegram (Previous year papers PDFs [SSC,Railway,DSSSB,UP SI]): https://t.me/RBE_S
YouTube (Free lectures and job updates): https://www.youtube.com/c/RBERevolutionByEducation
Download RBE application for SSC Exams Complete Preparation
(c) The house was being cleaned. structure.
(d) The house has cleaned. The documents are signed by the director before
submission.
Q60. Select the correct active form of the given (a) The director signs the documents before
sentence. submission. (b) The documents are
"The house was painted by them last month." being signed by the director before submission.
(a) They were painting the house last month. (c) The director has signed the documents before
(b) They painted the house last month. submission. (d) The director signed the
(c) They have painted the house last month. documents before submission.
(d) They paint the house last month.
Q67. Change the voice of the following sentence.
Q61. Select the correct passive form of the given The satellite has been launched successfully by
sentence. ISRO.
"They ought to have completed the task." (a) ISRO has launched the satellite successfully.
(a) The task ought be completed. (b) ISRO have launched the satellite successfully.
(b) The task ought to completed . (c) ISRO has launch the satellite successfully.
(c) The task ought to have been completed. (d) ISRO has launched the satellite successful.
(d) The task ought have completed.
Q68. Select the correct passive form of the given
Q62. Transform the following sentence into active sentence.
voice: "The principal might be watching the surveillance
Let the mobile be charged. footage."
(a) One must charge the mobile. (a) The surveillance footage might watched .
(b) Charge the mobile. (b) The surveillance footage is being watched.
(c) Will you charge the mobile? (c) The surveillance footage might be watched.
(d) Someone should charge the mobile. (d) The surveillance footage might be being
watched by the principal.
Q63. Select the correct passive form of the given
sentence. Q69. Select the correct active form.
"They have discussed the issue already." "This error could have been avoided by careful
(a) The issue discusses by them. planning."
(b) The issue has been discussed already by (a) This error could be avoided by careful
them. planning.
(c) The issue had been discussed. (b) Careful planning could have avoided this
(d) The issue was discussed by them. error.
(c) Someone could have avoided this error.
Q64. Select the correct active form. (d) This error could not have been made.
"Has the file been uploaded by you?"
(a) Are you uploading the file? Q70. Select the correct active form.
(b) Had you uploaded the file? "Was the culprit being chased by the police?"
(c) Did you upload the file? (a) Was the police chasing the culprit?
(d) Have you uploaded the file? (b) Is the police chasing the culprit?
(c) Had the police chased the culprit?
Q65. Select the correct active form. (d) Did the police chase the culprit?
"By whom is this rule obeyed?"
(a) Who is obeying this rule? Q71. Select the correct passive form of the given
(b) Who does obey this rule? sentence.
(c) Who obeys this rule? "She is cooking dinner."
(d) Who has obeyed this rule? (a) Dinner has been cooked by her.
(b) Dinner is being cooked by her.
Q66. Convert the sentence provided below from
its passive voice structure to an active voice
Telegram (Previous year papers PDFs [SSC,Railway,DSSSB,UP SI]): https://t.me/RBE_S
YouTube (Free lectures and job updates): https://www.youtube.com/c/RBERevolutionByEducation
Download RBE application for SSC Exams Complete Preparation
(c) Dinner was cooked by her. How beautifully has the picture been drawn by the
(d) Dinner is cooked by her. artist!
(a) How beautifully has the artist drew the picture!
Q72. Select the correct passive form of the given (b) How beautifully has the artist drawn the
sentence. picture!
"The committee had rejected the proposal." (c) How beautifully has the artist been drew the
(a) The proposal was rejected by the committee. picture!
(b) The proposal had been rejected by the (d) How beautifully has the artist been drawn by
committee. the picture!
(c) The proposal was rejecting.
(d) The proposal is rejected. Q78. Select the correct active form.
"The temple has been visited by thousands of
Q73. Choose the correct active voice form of the pilgrims."
sentence: (a) Thousands of pilgrims had visited the temple.
"All the books had been neatly arranged on the (b) Thousands of pilgrims have visited the
shelves by my grandfather." temple.
(a) My grandfather has neatly arranged all the (c) Thousands of pilgrims were visiting the
books on the shelves. temple.
(b) My grandfather have neatly arranged all the (d) Thousands of pilgrims are visiting the temple.
books on the shelves.
(c) My grandfather had arranged all the books on Q79. Select the correct active form of the given
the shelves. sentence:
(d) My grandfather had neatly arranged all the "The preliminary results have been officially
books on the shelves. announced by the matriculation board."
(a) The matriculation board has officially
Q74. Change the following sentence into active announced the final results.
voice: (b) The matriculation board has officially
The bill was passed in the parliament. announcing the preliminary results.
(a) They have passed the bill in the parliament. (c) The matriculation board has officially
(b) The parliament passed the bill. announced the preliminary results.
(c) The bill has passed in the parliament. (d) The matriculation board will has officially
(d) The members of the parliament passed the announced the preliminary results.
bill in the parliament.
Q80. Select the correct passive form of the given
Q75. Select the correct active form of the given sentence.
sentence. "He helps the poor."
"The homework is being checked by the teacher." (a) The poor had been helped.
(a) The teacher was checking the homework. (b) The poor are helped by him.
(b) The teacher is checking the homework. (c) The poor were helped by him.
(c) The teacher checks the homework. (d) The poor have helped.
(d) The teacher has checked the homework.
Q81. Select the correct passive form of the given
Q76. Transform the following sentence into active sentence.
voice: “He had lost the keys before the meeting.”
The glass was broken. (a) The keys were lost by him.
(a) He broken the glass. (b) The keys had lost before the meeting.
(b) He broke the glass. (c) The keys has been lost.
(c) Someone broke the glass. (d) The keys had been lost by him before the
(d) No one broke the glass. meeting.

Q77. Transform the following sentence into active Q82. Select the correct active form of the given
voice: sentence.
Telegram (Previous year papers PDFs [SSC,Railway,DSSSB,UP SI]): https://t.me/RBE_S
YouTube (Free lectures and job updates): https://www.youtube.com/c/RBERevolutionByEducation
Download RBE application for SSC Exams Complete Preparation
"The damaged road will be repaired by the (c) The truth may have told to them.
municipality." (d) The truth has been told.
(a) The municipality is begin repairing the
damaged road. Q84. Select the correct active form.
(b) The municipality will beginning repairing the "Was the novel written by her?"
damaged road. (a) Was she writing the novel?
(c) The municipality will begin repairing the (b) Has she written the novel?
damaged road. (c) Did she write the novel?
(d) The municipality will have begin repairing the (d) Is she writing the novel?
damaged road.
Q85. Select the correct passive form of the given
Q83. Select the correct passive form of the given sentence.
sentence. "Someone could have been using this password."
"She may have told them the truth." (a) This password could have been being used.
(a) The truth may told them. (b) This password can be being used.
(b) The truth may have been told to them. (c) This password could have been used.
(d) This password could be used.

Answer Key:

Q1 b Q2 b Q3 b Q4 a Q5 a
Q6 d Q7 c Q8 c Q9 b Q10 d
Q11 a Q12 d Q13 d Q14 a Q15 b
Q16 c Q17 d Q18 d Q19 a Q20 a
Q21 c Q22 d Q23 b Q24 d Q25 a
Q26 c Q27 c Q28 c Q29 d Q30 d
Q31 d Q32 b Q33 d Q34 a Q35 b
Q36 b Q37 c Q38 b Q39 c Q40 d
Q41 b Q42 b Q43 b Q44 d Q45 b
Q46 d Q47 c Q48 c Q49 d Q50 b
Q51 b Q52 b Q53 d Q54 b Q55 b
Q56 c Q57 d Q58 b Q59 b Q60 b
Q61 c Q62 b Q63 b Q64 d Q65 c
Q66 a Q67 a Q68 d Q69 b Q70 a
Q71 b Q72 b Q73 d Q74 b Q75 b
Q76 c Q77 b Q78 b Q79 c Q80 b
Q81 d Q82 c Q83 b Q84 c Q85 a

Solution 1: Explanation tense ("had repaired," "had tested").


In passive voice, the subject "The poem" receives In passive voice, the verb changes to "had been +
the action. past participle" to keep the tense consistent.
To convert to active voice, make the agent of the The object of the active sentence ("the control
action ("the student") the subject. panel" and "all functions") becomes the subject in
Use the simple past tense verb "composed" to the passive sentence.
match the original tense. The doer ("the technicians") is introduced by "by."
The other options have incorrect tense or This option perfectly preserves the tense,
continuous forms that do not align with the meaning, and structure of the original sentence in
original sentence. passive voice.

Solution 2: Explanation Solution 3: Explanation


The original active voice is in past perfect In passive voice, the subject receives the action
Telegram (Previous year papers PDFs [SSC,Railway,DSSSB,UP SI]): https://t.me/RBE_S
YouTube (Free lectures and job updates): https://www.youtube.com/c/RBERevolutionByEducation
Download RBE application for SSC Exams Complete Preparation
(here, "all the entries" are verified by "the In active voice, the subject "the student" performs
auditor"). the action.
To convert to active voice, "the auditor" (doer) The verb changes to simple past tense: "read"
becomes the subject, and "all the entries" (pronounced like "red").
(receiver) becomes the object. The object "the book" remains the object in active
The modal construction "must have been verified" voice.
turns into "must have verified" for the active
structure. Solution 9: Explanation
The passive sentence is in the present
Solution 4: Explanation continuous tense.
In simple future tense, the passive structure is: In active voice, the subject "he" performs the
object + will be + past participle + by + agent . action.
"The report" (object) becomes the subject in The verb changes to present continuous form: "is
passive voice. reading."
"will be submitted" maintains the future tense. The object "the book" remains the object of the
"by him" identifies the agent, and "by Friday" verb in active voice.
specifies the time.
Solution 10: Explanation:
Solution 5: Explanation The subject in the passive voice ("The groceries")
The passive sentence is in the past perfect becomes the object in the active voice.
passive tense ("had been broken"). The object in the passive voice ("the
In active voice, the subject ("the boy") becomes shopkeeper") becomes the subject in the active
the doer of the action. voice.
The verb changes to past perfect tense: "had The preposition "by" is removed.
broken." The verb is changed from passive form ("are
The object ("the window") becomes the object in delivered") to active present tense ("delivers") to
active voice. match "every evening."
Other options are incorrect because they use
Solution 6: Explanation wrong tenses or passive voice remains .
The passive sentence is in past perfect passive
tense ("had been taken"). Solution 11: Explanation:
In active voice, the subject ("the authorities") In passive voice, the subject ("the house")
performs the action. becomes the object in active voice.
The verb changes to past perfect tense: "had The agent ("her") in passive becomes the subject
taken." ("She") in active voice.
The object ("the emergency steps") becomes the The verb changes from passive form "is cleaned"
object in the active sentence. to active present tense "cleans," agreeing with
This preserves the tense and meaning accurately. "every morning" (habitual action).
Other options are incorrect because of tense or
Solution 7: Explanation structure issues, or they keep passive voice.
The sentence is in present continuous tense.
The passive voice in present continuous is Solution 12: Explanation:
formed as: In the passive voice, the object of the active
object + is/are being + past participle . sentence ("the meal") becomes the subject.
"The wall" (object) becomes the subject. The verb "will prepare" in future tense becomes
"is being painted" is the correct passive verb "will be prepared" (will + be + past participle).
form. The agent "the chef" follows after the preposition
"by her" specifies the doer of the action. "by."
This structure follows the rule for forming the
Solution 8: Explanation passive voice in the future tense.
The passive sentence is in simple past tense
(was read).
Telegram (Previous year papers PDFs [SSC,Railway,DSSSB,UP SI]): https://t.me/RBE_S
YouTube (Free lectures and job updates): https://www.youtube.com/c/RBERevolutionByEducation
Download RBE application for SSC Exams Complete Preparation
Solution 13: Explanation: The performer "Someone" replaces the "by"
The subject "People" is generalized and often phrase typical of passive voice.
omitted in passive constructions with "It is The verbs and modal forms are maintained in a
believed." suitable active voice construction.
Passive voice uses "It is believed" followed by the "Where they would have preserved it properly"
original statement in passive form. correctly refers to the action done by the museum
The product and company retain their roles; the in active voice sense.
product is the subject in passive voice.
Other options either use incorrect tense, Solution 19: ✅ Correct Answer: They are going
structure, or omit necessary elements. to construct it within a few weeks.
In passive voice, the focus is on the action and
Solution 14: Explanation: the receiver of the action. In active voice, the
In passive voice, the sentence starts with "It is focus shifts to the doer (subject). Here, “them”
believed" — the subject is vague or general. becomes “they” (subject), and “construct” remains
In active voice, the subject becomes specific or in its base form after “going to.”
general people ("People believe").
The verb changes accordingly from passive to Solution 20: ✅ Correct Answer :
active form. The committee will have taken a decision.
The remaining part of the sentence stays the 📌 Point-wise Explanation :
same.
Voice Transformation Rule : The sentence is in
future perfect passive voice : 🔹 "A decision will
Solution 15: Explanation:
have been taken by the committee."
In the future tense passive voice, the structure is:
Structure of Future Perfect Passive : 🔹 Passive:
subject + will be + past participle + (by + agent)
Here, "application" is the subject receiving the will have been + past participle 🔹 Active: subject
action, "will be approved" is the passive verb + will have + past participle
phrase, and "by the manager" indicates who Subject and Object Swap :
performs the action. Passive Subject: A decision
Passive Agent: by the committee
Solution 16: Explanation: Active Subject: The committee
The original sentence is in the past passive tense. Active Object: a decision
The corresponding active tense is simple past: Correct Active Form : ✔ "The committee will
"The postman delivered the letters." have taken a decision."
Other options either use different tenses or forms
(past perfect, present simple, present perfect). Solution 21: ✅ Correct Answer :
Therefore, the correct choice is : The postman The instructions may have been misunderstood.
delivered the letters. 🔄 Transformation Logic :
Modal + Perfect Tense :
Solution 17: Explanation: "may have misunderstood" → Passive: may have
The original sentence is in the present continuous been misunderstood
passive voice. Object becomes Subject :
The active form of present continuous passive is: "the instructions" (object in active) becomes
Subject (doer) + am/is/are + verb- ing + object. subject in passive.
"The staff is decorating the hall" clearly shows the Correct Passive Structure : ✔ Modal (may) +
doer performing the action in the present have been + past participle (misunderstood)
continuous tense.
Therefore, the correct choice is The staff is Solution 22: ✅ Correct Answer :
decorating the hall.
Food is served to the guests by them.
🔄 Transformation Logic :
Solution 18: Explanation:
Tense :
The passive subject "The royal sword" becomes
"serve" is simple present tense → Passive: is
the object in active voice ("the royal sword").
served
Telegram (Previous year papers PDFs [SSC,Railway,DSSSB,UP SI]): https://t.me/RBE_S
YouTube (Free lectures and job updates): https://www.youtube.com/c/RBERevolutionByEducation
Download RBE application for SSC Exams Complete Preparation
Structure : ✔ Passive = Subject (object of active) Active Voice Conversion :
+ is/are + past participle + (optional agent) ✔ Subject becomes: The owner
"Food is served to the guests by them" Verb: fed (simple past)
Why this is correct : Object: the hungry dog
It retains both direct object ("food") and indirect Adverb placement: gently — placed before the
object ("to the guests") verb for natural flow ✅ Final sentence: The owner
Includes the agent ("by them") for clarity gently fed the hungry dog.

Solution 23: ✅ Correct Answer : The farmer Solution 28: ✅ Correct Answer: You must tell
felled the tree. the truth.
Grammar Insight: 🔍 Explanation:
Passive Voice : The tree was felled by the Original Sentence (Passive Voice) : The truth
farmer. must be told by you.
Active Voice : The farmer felled the tree. → Modal verb: must
Subject (doer) + verb + object Passive structure: must be + past participle (told)
Agent: by you
Solution 24: Not avaialble Yet Active Voice Conversion :
Subject: You
Solution 25: ✅ Correct Answer: Mamatha writes Verb: must tell (modal + base verb)
the editorial. Object: the truth ✅ Final sentence: You must tell
🔍 Explanation: the truth.
Original Sentence (Passive Voice) : "The editorial
is written by Mamatha ." 👉 Passive structure: Solution 29: ✅ Correct Answer: The gardener
[Object] + is + past participle + by [Agent] waters the plants every morning.
Identifying the Tense : The verb "is written" is in 🔸 Tense: Simple past passive
simple present passive . So, the active voice must But the phrase “every morning” suggests a
also be in simple present tense . habitual action , which aligns better with simple
Active Voice Structure : 👉 Subject + base verb present tense in active voice.
(s/ es ) + object ✅ Mamatha writes the editorial. Voice Transformation Rule:
Passive: Object + was/were + past participle + by
+ subject
Solution 26: ✅ Correct Answer: The medicine
Active: Subject + verb (tense adjusted) + object
was prescribed by the doctor.
🔍 Explanation:
Solution 30: ✅ Correct Answer: The final version
Original Sentence (Active Voice) : The doctor
has been approved by the editor.
prescribed the medicine.
Passive Voice Rule:
Tense: Simple past
Active: Subject + has/have + past participle +
Subject: The doctor
object
Object: the medicine
Passive: Object + has/have + been + past
Passive Voice Structure :
participle + by + subject
Object + was/were + past participle + by + subject
So: The medicine was prescribed by the doctor.
Solution 31: ✅ Correct Answer: The manager
Solution 27: ✅ Correct Answer: The owner has written the letter carefully.
🔍 Step-by-Step Analysis:
gently fed the hungry dog.
🔍 Explanation: Passive Sentence Given :
Voice Type :
Original Sentence (Passive Voice) : The hungry
Passive voice in present perfect tense
dog was gently fed by the owner.
Structure: has/have been + past participle
Tense: Simple past passive
Active Voice Conversion :
Structure: was + past participle (fed)
Subject: The manager
Subject: The hungry dog
Verb: has written (present perfect)
Agent: by the owner
Telegram (Previous year papers PDFs [SSC,Railway,DSSSB,UP SI]): https://t.me/RBE_S
YouTube (Free lectures and job updates): https://www.youtube.com/c/RBERevolutionByEducation
Download RBE application for SSC Exams Complete Preparation
Object: the letter Verb: passed → Object: the law
Adverb: carefully (retained)
Solution 35: ✅ Correct Answer: The brakes
Solution 32: ✅ Correct Answer: The editor may be being checked now.
ought to have revised the article. 🔍 Step-by-Step Analysis:
🔍 Step-by-Step Analysis: Active Sentence Given :
Passive Sentence Given : Tense: Modal + be + present participle
Voice Type : (continuous)
Passive voice with modal verb “ought to have” Modal verb: may
Structure: ought to have been + past participle Verb: be checking
Active Voice Conversion : Object: the brakes
Subject: The editor Passive Voice Structure for Modal + Continuous :
Verb: ought to have revised Object + modal + be + being + past participle
Object: the article → The brakes may be being checked
Tense Consistency : Time Reference :
“ought to have” expresses duty or expectation in “now” remains unchanged—indicates present
the past time
Must retain the perfect infinitive form: have + past
participle Solution 36: ✅ Correct Answer
The workers are painting the walls.
Solution 33: ✅ Correct Answer: The letter was 🔍 Point-wise Explanation
written by him. Voice Identification
🔍 Step-by-Step Analysis: Original sentence is in passive voice : Subject
Active Sentence Given : (walls) + are being + past participle (painted) + by
Tense: Simple past agent (workers)
Subject: He Tense Recognition
Verb: wrote are being painted → Present Continuous Passive
Object: the letter So, active voice must be in Present Continuous
Passive Voice Structure : Tense
Object + was/were + past participle + by + subject Active Voice Structure
“The letter” becomes the subject Subject (agent) + is/are + verb-ing + object
“wrote” → was written The workers are painting the walls
“by him” retains the agent
Solution 37: ✅ Correct Answer
Solution 34: ✅ Correct Answer: He found the keys under the old wooden table.
The members of the committee should have 🔍 Point-wise Explanation
reviewed the law properly, which the parliament Voice Identification
passed last year. Original sentence is in Passive Voice
🔍 Step-by-Step Analysis: Structure: Object + was/were + past participle +
Original Passive Sentence : by + agent
Voice Type : Tense Recognition
Main clause: Passive (should have been were found → Simple Past Passive
reviewed) So, active voice must be in Simple Past Tense
Relative clause: Passive (was passed) Active Voice Structure
Active Voice Conversion : Subject (agent) + verb (past form) + object +
Main clause : The members of the committee additional details
should have reviewed the law properly → Transformation
Subject: members of the committee → Verb: Passive: The keys were found by him under the
should have reviewed → Object: the law old wooden table.
Relative clause : which the parliament passed Active: ✅ He found the keys under the old
last year → Passive “was passed” becomes
active “passed” → Subject: the parliament →
Telegram (Previous year papers PDFs [SSC,Railway,DSSSB,UP SI]): https://t.me/RBE_S
YouTube (Free lectures and job updates): https://www.youtube.com/c/RBERevolutionByEducation
Download RBE application for SSC Exams Complete Preparation
wooden table. aspect "following" becomes the simple past
participle "followed" in passive construction.
Solution 38: The correct answer is "The
secretary is submitting the documents." This Solution 44: The correct answer is "I know him ."
is the active voice equivalent of the passive To convert from passive to active voice, the agent
sentence "The documents are being submitted by "me" becomes the subject "I," and "he" becomes
the secretary," using the present continuous the object "him." The present tense is maintained.
tense. The other options either change the tense "I know about him" changes the meaning, while "I
or do not reflect the continuous action described knew him" and "I had known him" incorrectly
in the original sentence. change the tense from present to past.

Solution 39: The correct answer is "Mary baked Solution 45: The correct answer is " The room is
the cake for her brother's birthday party." This being cleaned by us." The original sentence is in
option changes the passive structure "The cake present continuous active voice ("are cleaning").
was baked by Mary" into the active structure with To convert to passive voice, we maintain the
"Mary" as the subject performing the action same tense: present continuous passive uses
(baked). The sentence also maintains the past "is/are being + past participle." The subject "We"
tense and the meaning of the original sentence. becomes "by us," and the object "the room"
Other options either change the tense or the becomes the subject. The other options use
meaning. incorrect tenses or are grammatically wrong.

Solution 40: The correct answer is "A complex Solution 46: The correct answer is "He pays the
problem is being solved by him." The passive bills every month." To convert from passive to
form of the present continuous tense ("is active voice: the agent "him" becomes the subject
solving") is made by using "is being" + past "He," the passive verb "are paid" becomes the
participle, so "is being solved" is correct . The active verb "pays" (maintaining present tense),
other options use incorrect tenses or structures. and "the bills" remains the object. The other
options incorrectly change the tense from present
Solution 41: The correct answer is " The to past continuous, present continuous, or past
earthquake destroyed the town." This option simple respectively.
correctly converts the passive voice sentence into
active voice, keeping the subject (the earthquake) Solution 47: The correct active voice is “Did they
and verb tense (simple past) consistent with the instruct you how to fill in the form?”
original meaning. The other options change the
tense or do not accurately reflect the action as Solution 48: The correct passive form is "The
described in the original sentence. final arguments are being heard by the judge."

Solution 42: The correct answer is "The Solution 49: The correct passive form uses "will
supervisor has reviewed the final draft of the be + past participle": "The work will be completed
report thoroughly." To convert from passive to by me tomorrow." The other options are
active voice, the subject (supervisor) should grammatically incorrect or use tense/structure
perform the action, and the tense should remain that does not match the required passive
the same (present perfect). The other options construction for future actions.
either use incorrect tenses or remain in passive
voice. Solution 50: The correct active voice form
maintains the tense, agent, and object
Solution 43: The correct answer is "The relationships: "They had demolished all the
instructions should have been followed structures built in that area due to encroachment."
carefully." When converting from active to This uses "they" as the subject and ensures the
passive voice, the object "instructions" becomes past perfect tense is preserved appropriately.
the subject, and the modal "should" with perfect Other options contain unnecessary phrases or
aspect "have been" is retained. The continuous
Telegram (Previous year papers PDFs [SSC,Railway,DSSSB,UP SI]): https://t.me/RBE_S
YouTube (Free lectures and job updates): https://www.youtube.com/c/RBERevolutionByEducation
Download RBE application for SSC Exams Complete Preparation
incorrect verb forms. and the time expression "today." The other
options contain errors in tense, form, or
Solution 51: The passive voice "Was your phone completeness.
being charged by someone?" is converted to the
active voice as "Was someone charging your Solution 57: The correct passive form is "The
phone?". This keeps the tense (past continuous) engine might be being repaired now." This
and meaning unchanged, with the agent maintains the structure of the modal verb "might,"
("someone") as the subject and the object ("your followed by "be being," which is used for forming
phone") following. passive voice with continuous aspect. The other
options either use incorrect tense, modal, or verb
Solution 52: The passive voice "Was your phone forms.
being charged by someone?" is transformed into
active voice as "Was someone charging your Solution 58: The passive sentence "The cake
phone?" This retains the past continuous tense had been decorated by the baker." should be
and follows the standard active structure with the converted into the active voice as "The baker had
agent ("someone") as subject and "your phone" decorated the cake." This maintains the past
as the object. Other options use incorrect tense or perfect tense structure and makes the subject
do not match the time frame given in the original (the baker) the doer of the action. The other
sentence. options do not match the tense and meaning of
the original sentence.
Solution 53: The correct active form keeps the
verb tense and meaning. "Was being thoroughly Solution 59: The sentence "She cleaned the
cleaned" (passive past continuous) becomes house yesterday." in passive voice is "The house
"were thoroughly cleaning" (active past was cleaned by her yesterday." The past simple
continuous). The subject ("the workers") performs tense ("cleaned") is appropriately changed to
the action on the object ("the dining hall"). Other "was cleaned" in passive. The other options are
options do not match the correct grammatical either incorrect in tense, voice, or structure.
form.
Solution 60: The passive sentence "The house
Solution 54: The original sentence uses the was painted by them last month." should be
passive voice in simple past tense ("was converted to the active form as "They painted the
postponed"). The corresponding active voice form house last month." This option uses simple past
is "Why did the organizers postpone the match?" tense, correctly matching the original time
which properly matches the tense and agent. The reference. Other options use incorrect tenses or
other options use tense forms or aspects that do do not agree with the time expression "last
not fit the original sentence. month."

Solution 55: The passive voice sentence is Solution 61: The correct passive construction is
correctly changed to active voice by making "The task ought to have been completed." In
"Pragna" the subject, followed by "has passive voice, the object "the task" becomes the
successfully finished her assignment work ahead subject, and "ought to have been completed"
of time." This version maintains the present correctly presents the modal verb structure with
perfect tense and includes all essential perfect aspect. The other options contain
components and adverbs from the original grammatical errors or omit necessary auxiliary
sentence. Other options either omit necessary words.
words, use incorrect tense, or do not match the
meaning precisely. Solution 62: In imperative sentences, the active
voice conveys a direct order. The passive "Let the
Solution 56: The passive form uses "should be mobile be charged" becomes "Charge the mobile"
announced" to correctly reflect the modal in the active voice, which is a clear command.
verb structure . The subject ("the results") The other options either add unnecessary words
comes first, followed by "should be announced"
Telegram (Previous year papers PDFs [SSC,Railway,DSSSB,UP SI]): https://t.me/RBE_S
YouTube (Free lectures and job updates): https://www.youtube.com/c/RBERevolutionByEducation
Download RBE application for SSC Exams Complete Preparation
or change the sentence form. Passive Voice Structure: ➡ [Object] + might be
being + past participle + by [subject]
Solution 63: The passive voice of "They have “ might be watching” → “might be being
discussed the issue already" is "The issue has watched”
been discussed already by them." This correctly Subject: The principal
keeps the present perfect tense and positions the Object: The surveillance footage
agent (by them) appropriately. The other options
either use incorrect tense or are grammatically Solution 69: ✅ Correct Answer: Careful
incorrect. planning could have avoided this error.
Subject (Agent): Careful planning
Solution 64: The correct active form of "Has the Verb Phrase: could have avoided
file been uploaded by you?" is "Have you Object: this error
uploaded the file?" Both sentences use present
perfect tense to ask about the completion of the Solution 70: ✅ Correct Answer: Was the police
action. The other options use incorrect tense or
chasing the culprit?
verb forms. Subject: the police
Verb: was chasing
Solution 65: The sentence "By whom is this rule Object: the culprit
obeyed?" in passive voice changes to "Who
➡ Active Voice: Was the police chasing the
obeys this rule?" in active voice. The subject
culprit?
"who" becomes the doer of the action, with the
simple present tense remaining unchanged for a
Solution 71: ✅ Correct Answer: Dinner is
general or habitual question. Other options use
different tenses, which do not match the meaning being cooked by her.
of the original sentence. 🔄 Active to Passive Transformation:
Active Voice: She is cooking dinner. Tense:
Solution 66: The active voice of "The documents Present Continuous Structure: Subject +
are signed by the director before submission" is is/are/am + verb- ing + object
"The director signs the documents before 🔁 Passive Voice Structure for Present
submission." This correctly shifts the agent (the Continuous: ➡ Object + is/are/am + being + past
director) to subject position and maintains present participle + by + subject
tense habitual action using the simple present ✅ So, the passive becomes: ➡ Dinner is being
form. The other options use different tenses or cooked by her.
remain in passive voice.
Solution 72: ✅ Correct Answer: The proposal
Solution 67: ✅ Step-by-Step Transformation: had been rejected by the committee.
Identify the Subject and Object: 🔄 Active to Passive Transformation:
Subject (doer): ISRO Active Voice: The committee had rejected the
Object (receiver): the satellite proposal. Tense: Past Perfect Structure: Subject
Verb phrase: has been launched + had + past participle + object
Change the Verb from Passive to Active: 🔁 Passive Voice Structure for Past Perfect: ➡
Passive: has been launched
Object + had been + past participle + by + subject
Active: has launched
✅ So, the passive becomes: ➡ The proposal
✅ Use has with singular subject “ISRO”
had been rejected by the committee.
Retain the Adverb:
Successfully remains unchanged
Solution 73: ✅ Correct Answer: My
❌ Don’t replace it with “successful” (which is an
grandfather had neatly arranged all the books
adjective)
on the shelves.
🔄 Passive to Active Voice Conversion:
Solution 68: ✅ Correct Answer: The
Passive Sentence: All the books had been neatly
surveillance footage might be being watched
arranged on the shelves by my grandfather.
by the principal.
Telegram (Previous year papers PDFs [SSC,Railway,DSSSB,UP SI]): https://t.me/RBE_S
YouTube (Free lectures and job updates): https://www.youtube.com/c/RBERevolutionByEducation
Download RBE application for SSC Exams Complete Preparation
🔍 Tense: Past Perfect Passive 🔁 Active Voice by thousands of pilgrims.
Structure: ➡ Subject + had + past participle + Active Voice: Thousands of pilgrims have visited
object + modifiers the temple.
✅ Active Voice: ➡ My grandfather had neatly
arranged all the books on the shelves. Solution 79: ✅ Correct Answer: Option 3 – The
matriculation board has officially announced
Solution 74: ✅ Correct Answer: The parliament the preliminary results.
🔍 Explanation:
passed the bill.
🧠 Why This Is Correct: Passive Voice (Given): “The preliminary results
have been officially announced by the
Subject (doer) is now The parliament .
matriculation board.”
Verb is in simple past : passed .
To convert this into Active Voice , follow this
Object remains the bill .
structure: Subject + has/have + past participle
The sentence is concise, grammatically correct,
+ object
and retains the original meaning.
Subject: The matriculation board
Verb: has announced
Solution 75: ✅ Correct Answer: The teacher is
Object: the preliminary results
checking the homework.
Adverb: officially
🧠 Why This Is Correct:
Tense Used: Present Continuous ➤ is being Solution 80: ✅ Correct Answer: Option 2 – The
checked → is checking poor are helped by him.
Subject (doer): The teacher 🔍 Explanation:
Object: The homework
Active Voice: He helps the poor.
The sentence retains both tense and meaning
Tense: Simple Present
accurately.
Structure: Subject + Verb + Object
To convert into Passive Voice in Simple Present:
Solution 76: 🔍 Explanation: Use: Object + is/are + past participle + by +
Passive Voice Sentence: “The glass was subject
broken.”
To convert it into Active Voice , we need: Solution 81: ✅ Correct Answer: Option 4 – The
Subject (doer of the action) → Unknown in this
keys had been lost by him before the meeting.
case, so we use “Someone”
🔍 Explanation:
Verb → “broke” (past tense of “break”)
Active Voice: He had lost the keys before the
Object → “the glass”
meeting.
✔ So, the correct active voice is: “Someone
Tense: Past Perfect
broke the glass.”
Structure: Subject + had + past participle +
object
Solution 77: ✅ Correct Answer: Option 2 – To convert into Passive Voice in Past Perfect:
How beautifully has the artist drawn the Use: Object + had been + past participle + by +
picture! subject
🔄 Voice Transformation: Passive → Active
Passive Voice: How beautifully has the picture Solution 82: ✅ Correct Answer: Option 3 – The
been drawn by the artist! municipality will begin repairing the damaged
Active Voice: How beautifully has the artist road.
drawn the picture! 🔍 Explanation:
Passive Voice: The damaged road will be
Solution 78: ✅ Correct Answer: Option 2 – repaired by the municipality.
Thousands of pilgrims have visited the This is in future simple passive : will be + past
temple. participle (repaired)
🔄 Passive to Active Voice Transformation Active Voice Structure (Future Simple):
Passive Sentence: The temple has been visited
Telegram (Previous year papers PDFs [SSC,Railway,DSSSB,UP SI]): https://t.me/RBE_S
YouTube (Free lectures and job updates): https://www.youtube.com/c/RBERevolutionByEducation
Download RBE application for SSC Exams Complete Preparation
Subject + will + base verb + object To convert this into Active Voice , follow these
steps .
Solution 83: ✅ Correct Answer: Option 2 – The
truth may have been told to them. Solution 85: ✅ Correct Answer: Option 1 –
🔍 Explanation: This password could have been being used.
Active Voice: She may have told them the truth. 🧠 Step-by-Step Transformation:
This is in modal perfect tense : may have + past Subject (doer): Someone
participle (told) Object (receiver): this password → becomes the
Passive Voice Structure (Modal Perfect): subject in passive voice.
Object + modal + have been + past participle Verb structure:
could have been using → modal + perfect
Solution 84: ✅ Correct Answer: Option 3 – Did continuous
she write the novel? Passive form of have been using → have been
🔍 Explanation: being used
Passive Voice: Passive Construction: 👉 This password could
Was the novel written by her? have been being used.

For SSC Exams Complete Preparation (Download RBE Application)


(Learn from those who have cleared the exam themselves)
https://play.google.com/store/apps/details?id=com.revolution.education

Telegram (Previous year papers PDFs [SSC,Railway,DSSSB,UP SI]): https://t.me/RBE_S


YouTube (Free lectures and job updates): https://www.youtube.com/c/RBERevolutionByEducation
Download RBE application for SSC Exams Complete Preparation

SSC Steno 2025 T-1 Narration All 92 Questions with Detailed Solution and Answer key

Q1. A sentence is provided in direct speech. (b) The teacher said, "The sun rose in the east."
From the four given options, choose the one (c) The teacher said, "The sun rising in the east."
that most accurately conveys the sentence in (d) The teacher said, "The sun rises in the east."
corresponding indirect speech.
Q5. Choose the correct indirect speech form
The teacher said, "Open your books." of the following sentence.
(a) The teacher ordered them opening the books. Roshni said to Rakesh, "Time and tide waits for
(b) The teacher asked them to open their books. none."
(c) The teacher said them open your books. (a) Roshni told Rakesh that time and tide waited
(d) The teacher asked to open books. for none.
(b) Roshni told Rakesh if time and tide waited for
Q2. A sentence is provided in direct speech. none.
From the four given options, choose the one (c) Roshni told Rakesh that time and tide waits for
that most accurately conveys the sentence in none.
its corresponding indirect speech. (d) Roshni told Rakesh whether time and tide
She said to her friend, "What a beautiful painting waited for none.
you have made!"
(a) She exclaimed to her friend that she ha s Q6. A sentence is provided in direct speech.
made a very beautiful painting. From the four given options, choose the one
(b) She exclaimed to her friend that she had that most accurately conveys the sentence in
made a very beautiful painting. corresponding indirect speech.
(c) She shouted that painting was very nice. They said, "We shall visit the museum tomorrow."
(d) She told her that the painting is beautiful. (a) They said that they would visit the museum
the next day.
Q3. A sentence is provided in direct speech. (b) They said they shall visit museum tomorrow.
From the four given options, choose the one (c) They said that we will visit the museum next
that most accurately conveys the sentence in day.
its corresponding indirect speech. (d) They said they would be visiting the museum.
The officer said, "Had I known about the problem
earlier, I would have helped." Q7. Choose the correct indirect speech form
(a) The officer said that he know if he had helped. of the following sentence.
(b) The officer said he had helped if he had Rehan said to Mallika, "The Somnath Temple is in
known it. Prabhas Patan in Gujarat."
(c) The officer told if he knew about the problem, (a) Rehan told Mallika that the Somnath Temple
he would help. was in Prabhas Patan in Gujarat.
(d) The officer said that if he had known about the (b) Rehan told Mallika that the Somnath Temple
problem earlier, he would have helped. had been in Prabhas Patan in Gujarat
(c) Rehan told Mallika that the Somnath Temple
Q4. A sentence is provided in indirect speech. is in Prabhas Patan in Gujarat..
From the four given options, choose the one (d) Rehan told Mallika that the Somnath Temple
that most accurately conveys the sentence in has been in Prabhas Patan in Gujarat.
its corresponding direct speech.
The teacher said that the sun rises in the east. Q8. Identify the word that doesn't fit the part of
(a) The teacher said, "The sun has risen in the speech needed.
east." She made a quickly decision without thinking.
Telegram (Previous year papers PDFs [SSC,Railway,DSSSB,UP SI]): https://t.me/RBE_S
YouTube (Free lectures and job updates): https://www.youtube.com/c/RBERevolutionByEducation
Download RBE application for SSC Exams Complete Preparation
(a) made (b) decision (c) Rashmi told Reshma that Mahima had
(c) thinking (d) Quickly danced on the floor when she had been called by
her mother.
Q9. A sentence is provided in indirect speech. (d) Rashmi told Reshma that Mahima was
From the four given options, choose the one dancing on the floor when she was called by her
that most accurately conveys the sentence in mother.
its corresponding direct speech .
My friend said that he could meet me after lunch. Q14. Choose the correct Indirect speech form
(a) My friend said, "I could meet you after lunch.” of the following sentence.
(b) My friend said, "I can meet you after lunch.”
(c) My friend said, May I meet you after lunch?” Rajesh says to Malini, "We did not like the dance
(d) My friend said, Shall I meet you after lunch?” last year."
(a) Rajesh tells Malini that they did not like the
Q10. Which figure of speech is used in "The old dance the previous year.
lady wept a river of Tears”. (b) Rajesh tells Malini that they had not liked the
(a) Hyperbole (b) Euphemism dance the previous year .
(c) Anticlimax (d) Alliteration (c) Rajesh told Malini that they did not like the
dance last year.
Q11. The following sentence is in indirect (d) Rajesh told Malini that they had not liked the
speech. Choose the option that conveys it in dance the previous year .
direct speech.
He said that he had been waiting there for more Q15. Choose the correct direct speech form
than two hours before the gates finally opened. of the following sentence .
(a) He said, I was waiting here for more than two
hours before the gates finally opened. They said that they had already completed their
(b) He said, "I have been waiting here since two homework.
hours before the gates opened.” (a) They said, “ We have already completed
(c) He said, "I had waiting there before the gates homework of ours,”
opened.” (b) They said, “ We have already completed our
(d) He said, I waited here two hours before gates homework."
opened.” (c) They said, "We have completed our
homework."
Q12. Choose the correct indirect speech form (d) They said, "We already completed their
of the following sentence. homework."

They said, "They had a very good habit." Q16. Choose the correct indirect speech form
(a) They told that they had a very good habit. of the following sentence.
(b) They told that we had a very good habit.
(c) They told that we had had a very good habit. The advocate said to his client, "Did you really
(d) They told that they had had a very good habit. see that there were black clouds in the sky and it
was about to rain when the car skidded hitting
Q13. Choose the correct Indirect speech form the lamp post?"
of the following sentence. (a) The advocate asked his client If he had really
seen that there were black clouds in the sky and
Rashmi said to Reshma, "Mahima was dancing that it was about to rain when the car
on the floor when she was called by her mother." had skidded hitting the lamp post.
(a) Rashmi told Reshma that Mahima had been (b) The advocate asked his client If he had really
dancing on the floor when she had been called by seen that there had been black clouds in the sky
her mother, and that it had been about to rain
(b) Rashmi told Reshma that Mahima had been when the car skid and hit the lamp post.
ganching on the floor when was called by her (c) The advocate asked his client whether he had
mother. really seen that there had been black clouds in
Telegram (Previous year papers PDFs [SSC,Railway,DSSSB,UP SI]): https://t.me/RBE_S
YouTube (Free lectures and job updates): https://www.youtube.com/c/RBERevolutionByEducation
Download RBE application for SSC Exams Complete Preparation
the sky and that it had been about to (b) He said, “ Did I take the medications?”
rain when the car had skidded hitting the lamp (c) He said, “ Whether you take the
post. medications?”
(d) The advocate asked his client whether he (d) He said, “ Did you took the medications?”
really saw that there were black clouds in the sky
and that it was 3000 to rain when the car Q20. Choose the correct Indirect speech form
skidded hitting the lamp post. of the following sentence.

Q17. Choose the correct indirect speech form Roshan says, " I have completed my homework."
of the following sentence. (a) Roshan said that he had completed his
Homework .
The laborers said to the contractor, "Did you ask (b) Roshan says that he has completed his
the manager when we are going to receive our homework .
remuneration that has been pending for the last (c) Roshan said that he has completed his
three months?" homework.
(a) The laborers asked the contractor if he had (d) Roshan says that he had completed his
asked the manager when they were going to homework .
receive their remuneration that had been pending
for the previous three months. Q21. Choose the correct direct speech form of the
(b) The laborers asked the contractor if he had following sentence.
asked the manager when they are going to The interviewer asked me if I could drive a car.
receive their remuneration that was pending for (a) The interviewer said, "Shall I drive a car?"
the previous three months. (b) The interviewer said, "You can drive a car."
(c) The laborers asked the contractor if had he (c) The interviewer said, "Can you drive a car?"
asked the manager when they are going to (d) The interviewer said, "Will you drive a car?"
receive their remuneration that was pending for
the last three months. Q22. Select the option that expresses the given
(d) The laborers asked the contractor if he asked sentence in Direct to Indirect Speech.
the manager when they were going to receive She said, "My parents are visiting next week."
their remuneration that had been pending for the (a) She said that her parents were visiting the
previous three months. following week.
(b) She said that her parents are visiting next
Q18. Choose the correct direct speech form of week.
the following sentence. (c) She said that my parents were visiting the
next week.
He told that when they had reached the station, (d) She said that her parents will visit next week.
the train had already left.
(a) He said, “ When we have reach the station, Q23. Select the option that expresses the given
the train had already left.” sentence in Direct Speech.
(b) He said, “ When we were reaching the She said that she could have passed the exam.
station, the train was already leaving." (a) She said, "I would have passed the exam."
(c) He said, “ When we reached the station, the (b) She said, "I could have passed the exam."
train had already left.” (c) She said, "I might have passed the exam."
(d) He said, “ When we reach the station, the (d) She said, "I should have passed the exam."
train has already left.”
Q24. Select the option that expresses the given
Q19. Choose the correct direct speech form of sentence in Direct to Indirect Speech.
the following sentence. Rita said, "I am happy today."
(a) Rita said that she was happy today.
He asked me whether I had taken the (b) Rita said that she was happy that day.
medications. (c) Rita said that she had been happy that day.
(a) He said, “ Did you take the medications?”
Telegram (Previous year papers PDFs [SSC,Railway,DSSSB,UP SI]): https://t.me/RBE_S
YouTube (Free lectures and job updates): https://www.youtube.com/c/RBERevolutionByEducation
Download RBE application for SSC Exams Complete Preparation
(d) Rita said that she is happy today. The reporter reported that five people had been
drowned in a boat accident.
Q25. Select the option that expresses the given (a) The reporter said, "Five people drowned in a
sentence in Direct to Indirect Speech. road accident."
He said, "I may go there." (b) The reporter said, "Five people were drowning
(a) He said that he might go there. in a boat accident."
(b) He said that he may go there. (c) The reporter said, "Five people are drowned in
(c) He said that he would go there. a boat accident."
(d) He said that he goes there. (d) The reporter said, "Five people were drowned
in a boat accident."
Q26. Select the option that expresses the given
sentence in Direct to Indirect Speech. Q31. Choose the correct direct speech form of the
She said, “How wonderful the view is!” following sentence.
(a) She exclaimed that the view was very They requested to close the door if he was feeling
wonderful. cold.
(b) She exclaimed that the view was wonderful. (a) He said, "Please close the door if feel cold."
(c) She said that the view is wonderful. (b) He said, "Please close the door if you is
(d) She said how wonderful the view is. feeling cold."
(c) He said, "Please closed the door if you are
Q27. Select the option that expresses the given feeling cold."
sentence in Direct Speech. (d) He said, "Please close the door if you are
She told me that I should have been more careful. feeling cold."
(a) She said, "You would have been more
careful." Q32. Choose the correct direct speech form of the
(b) She said, "You should have been more following sentence.
careful." Meenal told Renu that she had been a very good
(c) She said, "You must have been more careful." football player.
(d) She said, "You should be more careful." (a) Meenal said to Renu, "I am a very good
football player."
Q28. Select the option that expresses the given (b) Meenal said to Renu, "I was a very good
sentence in Direct to Indirect Speech. football player."
He said, "Let me do it myself." (c) Meenal said to Renu, "You have being a very
(a) He asked to do it himself. good football player."
(b) He suggested doing it himself. (d) Meenal said to Renu, "You had a very good
(c) He asked to be allowed to do it himself. football player."
(d) He requested that he do it himself.
Q33. Select the option that expresses the given
Q29. Choose the correct direct speech form of the sentence in Direct to Indirect Speech.
following sentence. She said, "I need some rest."
Robinson said that he would be planting trees (a) She said that she needed some rest.
along the bank of the river. (b) She said that she needs some rest.
(a) Robinson said, "I shall have to plant trees (c) She said that she had needed some rest.
along the bank of the river." (d) She said that she will need some rest.
(b) Robinson said, "I shall plant trees along the
bank of the river." Q34. Choose the correct direct speech form of the
(c) Robinson said, "I am planting trees along the following sentence.
bank of the river." Rima said that if she had wings, she would fly in
(d) Robinson said, "I shall be planting trees along the sky.
the bank of the river." (a) Rima said, "If I have wings, I will flew in the
sky."
Q30. Choose the correct direct speech form of the (b) Rima said, "If I had wings, I would fly in the
following sentence. sky."
Telegram (Previous year papers PDFs [SSC,Railway,DSSSB,UP SI]): https://t.me/RBE_S
YouTube (Free lectures and job updates): https://www.youtube.com/c/RBERevolutionByEducation
Download RBE application for SSC Exams Complete Preparation
(c) Rima said, "If I had wings, I will fly in the sky." (d) Rukmini asked Sanjana why had she not
(d) Rima said, "If I have wings, I would fly in the appeared in the UPSC Preliminary Examination
sky." the previous year.

Q35. Select the option that expresses the given Q40. Choose the correct indirect speech form of
sentence in Direct to Indirect Speech. the following sentence.
She said, "O that I were a bird!" He said to Renuka, "We must do it now and go
(a) She said that she wished she were a bird. there as soon as possible."
(b) She said that she was a bird. (a) He told Renuka that they must do it now and
(c) She said that she has been a bird. go here as soon as possible.
(d) She said that she wanted to be a bird. (b) He told Renuka that they must do it then and
go there as soon as possible.
Q36. Choose the correct direct speech form of the (c) He told Renuka that they had to do it then and
following sentence. go there as soon as possible.
He said that this had been in the box. (d) He told Renuka that they have to do it now
(a) He said, "This was in the box." and go there as soon as possible.
(b) He said, "This had in the box."
(c) He said that, "It is in the box." Q41. Select the option that expresses the given
(d) He said, "That have in the box." sentence in Direct to Indirect Speech:
Ravi said, "I will be joining the new office soon."
Q37. Choose the correct indirect speech form of (a) Ravi said that he would join the new office
the following sentence. soon.
They said, "Please bring the basket here." (b) Ravi said that he will be joining the new office
(a) They requested me to bring the basket there. soon.
(b) They requested to take the basket here. (c) Ravi said that he would be joining the new
(c) They requested me to take the basket there. office soon.
(d) They requested me to bring the basket here. (d) Ravi said that he joins the new office soon.

Q38. Select the option that expresses the given Q42. Select the option that expresses the given
sentence in Direct to Indirect Speech. sentence in Direct Speech.
The man said, "Be quiet and listen carefully." They told us that we needn’t bring anything.
(a) The man asked to be quiet and listen. (a) They said, "You didn’t need to bring anything."
(b) The man ordered them to be quiet and listen (b) They said, "You don’t need to bring anything."
carefully. (c) They said, "You needn’t bring anything."
(c) The man advised them to be quiet. (d) They said, "You must not bring anything."
(d) The man said to be quiet and to listen
carefully. Q43. Find the incorrect part of the sentence:
She delivered a speech in the prize distribution
Q39. Choose the correct indirect speech form of function.
the following sentence. (a) No error
Rukmini said to Sanjana, "Why didn't you appear (b) a speech
in the UPSC Preliminary Examination last year?" (c) in the prize distribution function
(a) Rukmini asked Sanjana why she did not (d) She delivered
appear in the UPSC Preliminary Examination last
year. Q44. Select the option that expresses the given
(b) Rukmini told Sanjana why she had not sentence in Direct to Indirect Speech:
appeared in the UPSC Preliminary Examination They said, "We enjoy watching cricket."
the last year. (a) They said that they enjoy watching cricket.
(c) Rukmini asked Sanjana why she had not (b) They said that they had enjoyed watching
appeared in the UPSC Preliminary Examination cricket.
the previous year. (c) They said that they have enjoyed watching
cricket.
Telegram (Previous year papers PDFs [SSC,Railway,DSSSB,UP SI]): https://t.me/RBE_S
YouTube (Free lectures and job updates): https://www.youtube.com/c/RBERevolutionByEducation
Download RBE application for SSC Exams Complete Preparation
(d) They said that they enjoyed watching cricket. (d) He told me to come there.

Q45. Choose the correct direct speech form of the Q50. Choose the correct indirect speech form of
following sentence. the following sentence.
The farmer asked his wife why she was upset He said, "Come here."
when he had just returned from the city. (a) He told to go there.
(a) The farmer said to his wife, "Why are you (b) He told me to came here.
upset when I have just returned from the city?" (c) He told me to go there.
(b) The farmer said to his wife, "Why are you so (d) He told me to come there.
upset when I just returned from the city?"
(c) The farmer said to his wife, "Why did you Q51. Choose the correct indirect speech form of
upset when I returned from the city?" the following sentence.
(d) The farmer said to his wife, "Why are you Madhubala said to Dilip, "I had a very sumptuous
upset when I was just returned from the city?" meal yesterday."
(a) Madhubala told Dilip that she had a very
Q46. Choose the correct direct speech form of the sumptuous meal the previous day.
following sentence. (b) Madhubala told Dilip that she had had a very
They requested the officer to permit them to go sumptuous meal the previous day.
on leave for three days. (c) Madhubala told Dilip that she had had a very
(a) They said to the officer, "Please permit us to sumptuous meal the following day.
go on leave for three days." (d) Madhubala told Dilip that she had had a very
(b) They said to the officer, "Request to permit us sumptuous meal the day after.
to go on leave for three days."
(c) They said to the officer, "Please permit us Q52. Select the option that expresses the given
going on leave by three days." sentence in Direct Speech.
(d) They said to the officer, "Please permit them She told us that she had not seen the movie.
to goes on leave for three days." (a) She said, “I had not seen the movie.”
(b) She said, "I haven’t seen the movie."
Q47. Select the option that expresses the given (c) She said, "She hadn’t seen the movie."
sentence in Direct to Indirect Speech (d) She said, "I didn’t see the movie."
She said, "I dare not oppose him."
(a) She said she has not dared to oppose him. Q53. Select the option that expresses the given
(b) She said she dared not oppose him. sentence in Direct to Indirect Speech.
(c) She said she had dared not oppose him. She said, "Can I ask a question?"
(d) She said she dare not oppose him. (a) She asked whether she can ask a question.
(b) She said whether she could ask a question.
Q48. Select the option that expresses the given (c) She asked whether she could ask a question.
sentence in Direct to Indirect Speech (d) She asked whether she might ask a question.
The captain said, "Stand at ease!"
(a) The captain advised them to stand at ease. Q54. Select the option that expresses the given
(b) The captain said that they should stand at sentence in Direct to Indirect Speech.
ease. He said, "I have completed the task."
(c) The captain ordered them to stand at ease. (a) He said that he had completed the task.
(d) The captain said to stand at ease. (b) He said that he has completed the task.
(c) He said that he was completing the task.
Q49. Choose the correct indirect speech form of (d) He said that he completed the task.
the following sentence.
He said, "Come here." Q55. In the following question, a sentence has
(a) He told to go there. been given in Indirect Speech. Choose the
(b) He told me to came here. correct Direct Speech form.
(c) He told me to go there.
The climatologist said that if global temperatures
Telegram (Previous year papers PDFs [SSC,Railway,DSSSB,UP SI]): https://t.me/RBE_S
YouTube (Free lectures and job updates): https://www.youtube.com/c/RBERevolutionByEducation
Download RBE application for SSC Exams Complete Preparation
kept rising, sea levels would threaten major Q60. Select the option that expresses the given
coastal cities. sentence in Direct Speech.
(a) The climatologist said, “If global temperatures She said that she was learning French.
had kept rising, sea levels would have threatened (a) She said, "I have learned French."
major coastal cities.” (b) She said, "I learn French."
(b) The climatologist said, “If global temperatures (c) She said, "I was learning French."
kept rising, sea levels would threaten major (d) She said, "I am learning French."
coastal cities.”
(c) The climatologist said, “If global temperatures Q61. In the following question, a sentence has
keep rising, sea levels threaten major coastal been given in Indirect Speech. Choose the
cities.” correct Direct Speech form.
(d) The climatologist said, “If global temperatures The analyst stated that inflation could have been
kept rising, sea levels will threaten major coastal controlled if monetary policies had been revised
cities.” earlier.
(a) The analyst stated, “Inflation had been
Q56. Select the option that expresses the given controlled if monetary policies had been revised
sentence in Direct to Indirect Speech. earlier.”
He said, "I don’t like spicy food." (b) The analyst stated, “Inflation could have been
(a) He said that he likes spicy food. controlled if monetary policies had been revised
(b) He said that he didn’t like spicy food. earlier.”
(c) He said that he hadn’t liked spicy food. (c) The analyst stated, “Inflation could be
(d) He said that he doesn’t like spicy food. controlled if monetary policies were revised
earlier.”
Q57. Select the option that expresses the given (d) The analyst stated, “Inflation can be controlled
sentence in Direct to Indirect Speech. if monetary policies are revised earlier.”
She said, "I had eaten breakfast."
(a) She said that she has eaten breakfast. Q62. Choose the correct direct speech form of the
(b) She said that she had eaten breakfast. following sentence.
(c) She said that she would have eaten breakfast. He asked if I had been aware of the policies of
(d) She said that she ate breakfast. the company.
(a) He said, "Were you awared of the policies of
Q58. Choose the correct direct speech form of the the company?"
following sentence. (b) He said, "Was you aware of the policies of the
Simran said that she had been writing a letter to company?"
the principal. (c) He said, "Were you aware of the policies of
(a) Simran said, "I was written a letter to the the company?"
principal." (d) He said, "Did you be aware of the policies of
(b) Simran said, "I have been writing a letter to the company?"
the principal."
(c) Simran said, "I was being writing a letter to the Q63. Select the option that expresses the given
principal." sentence in Direct Speech.
(d) Simran said, "I have writing a letter to the She said that she had never encountered such
principal." behavior.
(a) She said, "I have never encountered such
Q59. Select the option that expresses the given behavior."
sentence in Direct to Indirect Speech. (b) She said, "I never encounter such behavior."
She said to me, "Would you like some coffee?" (c) She said, “I had never encountered such
(a) She offered me some coffee. behavior.”
(b) She asked that I would like some coffee. (d) She said, "I was never encountering such
(c) She asked me would I like some coffee. behavior."
(d) She asked me if I would like some coffee.

Telegram (Previous year papers PDFs [SSC,Railway,DSSSB,UP SI]): https://t.me/RBE_S


YouTube (Free lectures and job updates): https://www.youtube.com/c/RBERevolutionByEducation
Download RBE application for SSC Exams Complete Preparation
Q64. Choose the correct indirect speech form of (d) Rina said to Suman, "You leave the city
the following sentence. immediately."
He said, "Will you finish your task on time?"
(a) He asked me if I should finish my task on Q69. Select the option that expresses the given
time? sentence in Direct to Indirect Speech.
(b) He asked me if I should finish my task on He said to me, "Where are you going?"
time. (a) He asked me where I had gone.
(c) He asked me if I should have finished my task (b) He asked me where I was going.
on time. (c) He asked me where I am going.
(d) He asked me if I should finish his task on (d) He asked me where I go.
time.
Q70. Choose the correct indirect speech form of
Q65. Select the option that expresses the given the following sentence.
sentence in Direct to Indirect Speech. Rohan said to Manasi, "I have no rose to give you
They said, "How foolish of him to say that!" now."
(a) They exclaimed how foolish of him to say that. (a) Rohan told Manasi that he had had no rose to
(b) They exclaimed that it had been very foolish give her then.
of him to say that. (b) Rohan told Manasi that he has no rose to give
(c) They exclaimed that he is very foolish to say her then.
that. (c) Rohan told Manasi that he had no rose to give
(d) They said it was very foolish of him to say her now.
that. (d) Rohan told Manasi that he had no rose to give
her then.
Q66. The figure of speech used in "I wandered
lonely as a cloud..."? Q71. Select the option that expresses the given
(a) Simile (b) Metaphor sentence in Direct to Indirect Speech.
(c) Hyperbole (d) Personification She said, "Has he left already?"
(a) She asked had he left already.
Q67. Choose the correct direct speech form of the (b) She said if he had left already.
following sentence. (c) She asked if he had left already.
He exclaimed that the flower was looking very (d) She asked whether had he left already.
beautiful, which was placed in the vase.
(a) He said, "Which beautiful the flower was Q72. Select the option that expresses the given
looking which is placed in the vase!" sentence in Direct to Indirect Speech.
(b) He said, "What beautiful the flower is looking He said, "It must be raining outside."
which was placed in the vase!" (a) He said it had to be raining outside.
(c) He said, "How beautiful the flower is looking, (b) He said it might be raining outside.
which is placed in the vase!" (c) He said it should be raining outside.
(d) He said, "The flower is looking very beautiful, (d) He said it must be raining outside.
which is placed in the vase!"
Q73. Select the option that expresses the given
Q68. Choose the correct direct speech form of the sentence in Direct to Indirect Speech
following sentence. She said, "How I wish I could meet him once!"
Rina told Suman to leave the city immediately. (a) She hoped she would meet him.
(a) Rina said to Suman, "Please leave the city (b) She asked that she meet him once.
immediately." (c) She said that she wished she could meet him
(b) Rina said to Suman, "Leave the city once.
immediately now." (d) She exclaimed that she could meet him once.
(c) Rina said to Suman, "Leave the city
immediately." Q74. Choose the correct indirect speech form of
the following sentence.

Telegram (Previous year papers PDFs [SSC,Railway,DSSSB,UP SI]): https://t.me/RBE_S


YouTube (Free lectures and job updates): https://www.youtube.com/c/RBERevolutionByEducation
Download RBE application for SSC Exams Complete Preparation
Mohan told Radha, "I have seen you many times (d) Ravi said to his friend, "I request you to turn
in the market since last year." the music down."
(a) Mohan told Radha that she had seen him
many times in the market since the previous year. Q79. Select the option that expresses the given
(b) Mohan told Radha that he had seen her many sentence in Direct to Indirect Speech.
times in the market since last year. She said, "Who knows the answer?"
(c) Mohan told Radha that he had seen her many (a) She asked who knows the answer.
times in the market since the previous year. (b) She asked who knew the answer.
(d) Mohan told Radha that he had seen her many (c) She said who is knowing the answer.
times in the market since the following year. (d) She told who knows the answer.

Q75. Choose the correct indirect speech form of Q80. Choose the correct direct speech form of the
the following sentence. following sentence.
He said, "Where are you going at this time of the Sobhit said that he could speak more fluently
day?" than Rahul.
(a) He asked me where I was going at this time of (a) Sobhit said, "I can spoke more fluently than
the day. Rahul."
(b) He asked me where was I going at this time of (b) Sobhit said, "I can speak most fluently than
the day. Rahul."
(c) He asked me where was I going at that time of (c) Sobhit said, "I can speak more fluently than
the day. Rahul."
(d) He asked me where I was going at that time (d) Sobhit said, "I can speak more fluentierr than
of the day. Rahul."

Q76. Select the option that expresses the given Q81. Select the option that expresses the given
sentence in Direct Speech. sentence in Direct to Indirect Speech.
They said that they were supposed to arrive He said, "I must reach early."
earlier. (a) He said that he will reach early.
(a) They said, "We should arrive earlier." (b) He said that he should reach early.
(b) They said, "We are supposed to arrive (c) He said that he had to reach early.
earlier." (d) He said that he must reach early.
(c) They said, "We were supposed to arrive
earlier." Q82. Select the option that expresses the given
(d) They said, "We have to arrive earlier." sentence in Direct to Indirect Speech.
The teacher said, "Let him try on his own."
Q77. Select the option that expresses the given (a) The teacher said that he should let him try on
sentence in Direct to Indirect Speech his own.
They said, "We are watching a movie." (b) The teacher suggested that he try on his own.
(a) They said that they watched a movie. (c) The teacher ordered him to try on his own.
(b) They said that they were watching a movie. (d) The teacher said that let him try on his own.
(c) They said that they are watching a movie.
(d) They said that they had watched a movie. Q83. Select the option that expresses the given
sentence in Direct Speech.
Q78. Choose the correct direct speech form of the She said that she liked chocolate.
following sentence. (a) She said, "I like chocolate."
Ravi requested his friend to turn the music down. (b) She said, "I had liked chocolate."
(a) Ravi said, "Will you please turn the music (c) She said, "She likes chocolate."
down?" (d) She said, "I liked chocolate."
(b) Ravi said to his friend, "Are you going to turn
the music down?" Q84. Select the option that expresses the given
(c) Ravi said to his friend, "Please turn the music sentence in Direct Speech.
down." She said that she was going to the market.
Telegram (Previous year papers PDFs [SSC,Railway,DSSSB,UP SI]): https://t.me/RBE_S
YouTube (Free lectures and job updates): https://www.youtube.com/c/RBERevolutionByEducation
Download RBE application for SSC Exams Complete Preparation
(a) She said, "I will go to the market." (b) The explorer remarked, “I rarely saw such a
(b) She said, "I have gone to the market." breathtaking view in all my travels.”
(c) She said, "I was going to the market." (c) The explorer remarked, “Rarely had I seen
(d) She said, "I am going to the market." such a breathtaking view in all my travels.”
(d) The explorer remarked, “Rarely I had seen
Q85. Select the option that expresses the given such a breathtaking view in all my travels.”
sentence in Direct to Indirect Speech.
She said, "I must go now." Q90. In the following question, a sentence has
(a) She said that she will go now. been given in Indirect Speech. Choose the
(b) She said that she had to go then. correct Direct Speech form.
(c) She said that she was going then. The astronomer noted that Jupiter’s rotation was
(d) She said that she must go now. the fastest among all planets in the solar system.
(a) The astronomer noted, “Jupiter’s rotation is
Q86. Select the option that expresses the given the fastest among all planets in the solar system.”
sentence in Direct to Indirect Speech. (b) The astronomer noted, “Jupiter’s rotation had
He said, "May God bless you." been the fastest among all planets in the solar
(a) He wished God bless me. system.”
(b) He said that God may bless me. (c) The astronomer noted, “Jupiter’s rotation has
(c) He told that God should bless me. been the fastest among all planets in the solar
(d) He prayed that God might bless me. system.”
(d) The astronomer noted, “Jupiter’s rotation was
Q87. Select the option that expresses the given the fastest among all planets in the solar system.”
sentence in Direct Speech.
They said that they were enjoying the trip. Q91. Select the option that expresses the given
(a) They said, "We were enjoying the trip." sentence in Direct to Indirect Speech.
(b) They said, "We have enjoyed the trip." They said, "Our teacher will explain the topic
(c) They said, "We are enjoying the trip." tomorrow."
(d) They said, "We enjoy the trip." (a) They said that the teacher should explain the
topic tomorrow.
Q88. Select the option that expresses the given (b) They said that the teacher explained the topic
sentence in Direct to Indirect Speech. the next day.
She said, “I was tired.” (c) They said that their teacher would explain the
(a) She said that she will be tired. topic the next day.
(b) She said that she was tired. (d) They said that our teacher will explain the
(c) She said that she had been tired. topic tomorrow.
(d) She said that she is tired.
Q92. Select the option that expresses the given
Q89. In the following question, a sentence has sentence in Direct Speech.
been given in Indirect Speech. Choose the I told him that I was busy.
correct Direct Speech form. (a) I told him, "You are busy."
The explorer remarked that rarely had he seen (b) I told him, "I was busy."
such a breathtaking view in all his travels. (c) I said, "I am busy."
(a) The explorer remarked, “I had rarely seen (d) I told him, “I am busy.”
such a breathtaking view in all my travels.”

Answer Key:

Q1 b Q2 b Q3 d Q4 d Q5 c
Q6 a Q7 c Q8 d Q9 b Q10 a
Q11 a Q12 d Q13 a Q14 a Q15 b
Q16 c Q17 a Q18 c Q19 a Q20 b
Telegram (Previous year papers PDFs [SSC,Railway,DSSSB,UP SI]): https://t.me/RBE_S
YouTube (Free lectures and job updates): https://www.youtube.com/c/RBERevolutionByEducation
Download RBE application for SSC Exams Complete Preparation
Q21 c Q22 a Q23 b Q24 b Q25 a
Q26 b Q27 b Q28 c Q29 d Q30 d
Q31 d Q32 b Q33 a Q34 b Q35 a
Q36 a Q37 c Q38 b Q39 c Q40 c
Q41 c Q42 c Q43 c Q44 d Q45 a
Q46 a Q47 b Q48 c Q49 c Q50 c
Q51 b Q52 a Q53 c Q54 a Q55 b
Q56 b Q57 b Q58 b Q59 d Q60 d
Q61 b Q62 c Q63 c Q64 b Q65 d
Q66 a Q67 c Q68 c Q69 b Q70 d
Q71 c Q72 b Q73 c Q74 c Q75 d
Q76 c Q77 b Q78 c Q79 b Q80 c
Q81 c Q82 b Q83 a Q84 d Q85 b
Q86 d Q87 c Q88 b Q89 c Q90 d
Q91 c Q92 d

Solution 1: Explanation the tense remains the same (present simple)


When reporting a command politely, "asked" + when changing from indirect to direct speech.
"to" + infinitive is appropriate. The other options change tense incorrectly or use
The other options are grammatically incorrect or improper forms that do not fit the original
awkward: meaning.
"The teacher ordered them opening the books"
(incorrect verb form) Solution 5: Explanation
"The teacher said them open your books" When converting into indirect speech, the present
(incorrect structure) tense "waits" changes to the past tense "waited"
"The teacher asked to open books" (missing due to backshifting in reported speech.
object "them" and possessive "their"). The sentence is a statement, so "that" is used to
introduce the indirect speech.
Solution 2: Explanation The other options are either grammatically
The original sentence is an exclamation, so incorrect or use inappropriate structures such as
"exclaimed" is a suitable reporting verb. "if" or "whether" which are used in reported
The present perfect tense "have made" changes questions, not statements.
to past perfect "had made" in indirect speech.
The pronouns are correctly changed from "you" to Solution 6: Explanation
"she" and possessive "your" to "her." "Shall" changes to "would" in indirect speech
The other options either have tense errors or when the reporting verb is in the past tense.
incorrect phrasing. "Tomorrow" changes to "the next day" in indirect
speech to reflect the change in time reference.
Solution 3: Explanation Pronouns change accordingly from "We" to
The original sentence is a third conditional "they."
sentence (unreal past condition) using "Had I This option follows all the standard rules of
known... I would have helped." converting direct speech to in d irect speech.
In indirect speech, the sentence structure stays
the same for third conditional sentences with Solution 7: Explanation
appropriate reporting verbs. Facts and universal truths do not change tense in
The other options are grammatically incorrect or indirect speech.
incomplete and do not retain the conditional Therefore, the present tense "is" remains
meaning accurately. unchanged when converting to reported speech
here.
Solution 4: Explanation
Since the sentence expresses a universal truth,
Telegram (Previous year papers PDFs [SSC,Railway,DSSSB,UP SI]): https://t.me/RBE_S
YouTube (Free lectures and job updates): https://www.youtube.com/c/RBERevolutionByEducation
Download RBE application for SSC Exams Complete Preparation
Solution 8: Explanation speech with a statement.
"Quickly" is an adverb, but in the sentence "She The other options either use incorrect pronouns
made a quickly decision without thinking," an (like "we") or incorrect tense forms.
adjective is needed to describe the noun
"decision." Solution 13: Explanation:
The correct adjective form is quick (e.g., "a quick The direct speech "Mahima was dancing" (past
decision"). continuous) changes to past perfect continuous
Hence, "quickly" is incorrect in this context "had been dancing" in indirect speech.
because it does not properly modify the noun "She was called" changes to past perfect passive
"decision." "had been called."
The pronouns "Rashmi" and "Reshma" remain
Solution 9: Explanation the same as the speakers are reporting to
In indirect speech, modal verbs like "can" often another third party.
change to "could" when reported by a past tense The verb "said to" changes to "told," which is the
reporting verb ("said"). correct reporting verb for indirect speech with an
When converting back to direct speech, the past object.
form "could" changes back to the present form
"can." Solution 14: Explanation:
The other options use incorrect modal verbs or The reporting verb "says" is in the present tense,
sentence structures for this context. so the tense in the reported clause does not
"Could" in direct speech would imply a past change.
conditional or less certainty; here, presence of "We" changes to "they" in indirect speech.
"could" in indirect speech implies original direct "Last year" changes to "the previous year."
speech used "can." Since the reporting verb is in present tense
("says"), there is no backshifting of tense
Solution 10: Explanation required.
Hyperbole is an exaggerated statement not
meant to be taken literally, used for emphasis or Solution 15: Explanation:
effect. The tense in direct speech is present perfect
"A river of tears" exaggerates the amount of "have completed" which becomes past perfect
weeping to emphasize the old lady's sadness. "had completed" in indirect speech.
Euphemism, anticlimax, and alliteration do not fit Possessive pronoun "our homework" is correct for
this expression. "their homework."
The phrase should be clear and natural in direct
Solution 11: Explanation: speech.
The indirect speech uses past perfect continuous Therefore, the correct choice is:
("had been waiting"), which typically changes to They said, “We have already completed our
past continuous ("was waiting") in direct speech. homework."
The reporting verb "said" is used with a comma
before the quotation. Solution 16: Explanation:
The pronouns change accordingly from third The question in direct speech changes to an
person indirect ("he") to first person direct ("I"). indirect question with "whether."
The time expressions remain similar to reflect the Tenses shift back: "were" → "had been," "was" →
same meaning. "had been," "skidded" → "had skidded."
Pronouns and word order are adjusted for indirect
Solution 12: Explanation: speech.
In reported speech, the past perfect tense ("had This version captures all the details and follows
had") indicates an action that was already correct tense and structure rules for indirect
completed before the time of speaking. speech.
The pronoun "they" is retained as the speaker
refers to the same group. Solution 17: Explanation:
The verb "told" is correct to introduce indirect The reporting verb "asked" is used for a question.
Telegram (Previous year papers PDFs [SSC,Railway,DSSSB,UP SI]): https://t.me/RBE_S
YouTube (Free lectures and job updates): https://www.youtube.com/c/RBERevolutionByEducation
Download RBE application for SSC Exams Complete Preparation
The question is converted to indirect speech next week." – This is a present continuous
using "if." sentence with a future time reference.
Tense changes: "are going" becomes "were Indirect Speech Rules Applied:
going," "has been pending" becomes "had been Tense Change: Present continuous ( are visiting
pending." ) becomes past continuous ( were visiting ).
Pronouns are adjusted accordingly. Pronoun Change: “My” becomes “her” to match
"Previous three months" is used instead of "last the speaker.
three months" for proper indirect speech time Time Expression Change: “next week” becomes
reference. “the following week” .

Solution 18: Explanation: Solution 23: ✅ Correct Answer: She said, "I
In indirect speech, the past perfect tense remains could have passed the exam."
unchanged in direct speech. To convert it to Direct Speech , we:
The pronoun changes from "they" to "we" in direct Retain the same modal verb: “could have”
speech when reporting oneself. Restore the original pronoun: “she” → “I”
The time sequence "had reached" and "had Keep the sentence structure as a declarative
already left" is kept consistent as "reached" statement , not a question.
(simple past) and "had already left" (past perfect)
in direct speech. Solution 24: ✅ Correct Answer: Rita said that
she was happy that day.
Solution 19: Explanation: Tense Change: Present tense “am” becomes
In indirect speech, "whether" or "if" is used for past tense “was” .
yes/no questions. Pronoun Change: “I” becomes “she” (to match
The tense in direct speech changes to simple the speaker).
past for "had taken" in indirect speech. So, "had Time Expression Change: “today” becomes
taken" becomes "did take" in direct speech. “that day” .
The pronouns and verbs adjust accordingly.
The sentence is a question, so direct speech Solution 25: ✅ Correct Answer: He said that he
uses an interrogative form with "Did you take...?"
might go there.
The direct sentence is: He said, "I may go
Solution 20: Explanation:
there."
When the reporting verb is in the present tense Reporting verb is “said” (past tense), so we apply
(says), the tense of the direct speech remains the backshift of tense in indirect speech.
same in indirect speech. “May” changes to “might” in indirect speech.
"I have completed" remains "he has completed." The pronoun “I” becomes “he” to match the
Pronouns and possessives are changed subject of the reporting clause.
appropriately.
Therefore, the correct choice is:
Solution 26: ✅ Correct Answer :
Roshan says that he has completed his
She exclaimed that the view was wonderful.
homework.
📌 Point-wise Explanation :
Sentence Type : The direct speech is an
Solution 21: ✅ Correct Answer: The
exclamatory sentence expressing admiration:
interviewer said, "Can you drive a car?"
“How wonderful the view is!”
To convert it to Direct Speech , we need to:
Reporting Verb : Since the speaker is expressing
Restore the original question format.
emotion (admiration), we use "exclaimed" instead
Use the present tense modal verb “can” instead
of "said".
of “could” .
Tense Change Rule : The reporting verb "said" is
Keep the question structure intact.
in the past tense, so the present tense "is"
changes to "was" in indirect speech.
Solution 22: ✅ Correct Answer: She said that
Structure of Indirect Speech : ✔ Exclamatory →
her parents were visiting the following week.
Statement ✔ Remove exclamation and use "that"
Direct Speech: She said, "My parents are visiting
Telegram (Previous year papers PDFs [SSC,Railway,DSSSB,UP SI]): https://t.me/RBE_S
YouTube (Free lectures and job updates): https://www.youtube.com/c/RBERevolutionByEducation
Download RBE application for SSC Exams Complete Preparation
clause Final form: She exclaimed that the view Tense Conversion Rule :
was wonderful. In direct speech, “had been drowned” becomes
“were drowned” (past passive).
Solution 27: ✅ Correct Answer : This maintains the passive voice and matches the
She said, "You should have been more careful." past narrative.
Pronoun Change : Subject and Voice :
"I" (in indirect) becomes "you" in direct speech, “Five people” remains the subject.
because she was speaking to "me". “Were drowned” keeps the passive construction
Tense Retention : intact.
"should have been" remains unchanged in direct
speech. Solution 31: ✅ Correct Answer: He said, "Please
Reporting Verb : close the door if you are feeling cold."
"told me" becomes "said" , since the direct 🔍 Explanation:
speech doesn't require an object. Original Sentence (Indirect Speech) : They
requested to close the door if he was feeling cold.
Solution 28: ✅ Correct Answer : He asked to be Verb of reporting: requested
allowed to do it himself. Reported action: to close the door if he was
Grammar Insight: feeling cold
“Let me…” in direct speech often transforms to Implied direct speech: a polite request using
“asked to be allowed…” in indirect speech. “Please”
It conveys a polite request for permission , not a Conversion to Direct Speech :
command or suggestion. Request → imperative with “please”
Pronoun shift: he → you
Solution 29: ✅ Correct Answer : Robinson said, Tense shift: was feeling → are feeling (present
"I shall be planting trees along the bank of the continuous for direct speech) ✅ Final form: He
river." said, "Please close the door if you are feeling
🔍 Explanation : cold."
Reported Speech (Indirect) : Robinson said that
he would be planting trees along the bank of the Solution 32: ✅ Correct Answer: Meenal said to
river. 👉 This is in future continuous tense (would Renu, "I was a very good football player."
be planting). Indirect : Meenal told Renu that she had been a
Tense Conversion Rule : very good football player.
Indirect → Direct Speech Direct : Meenal said to Renu, "I was a very good
“ would be planting ” in indirect becomes “shall football player."
be planting” in direct (for first person subject “I”). 🔸 Key Elements:
Pronoun Change : Reported verb : “told” → past tense
“He” in indirect becomes “I” in direct (Robinson is Reported clause : “she had been…” → past
the speaker). perfect
Correct Direct Speech : ✅ Robinson said, "I shall In direct speech , past perfect often becomes
be planting trees along the bank of the river." simple past when expressing a past state.

Solution 30: ✅ Correct Answer: The reporter Solution 33: ✅ Correct Answer: She said that
said, "Five people were drowned in a boat she needed some rest.
accident." ✅ Correct Answer:
🔍 Explanation (Step-by-Step): She said that she needed some rest.
Original Sentence (Indirect Speech) : The 🔍 Step-by-Step Explanation:
reporter reported that five people had been Reporting Verb :
drowned in a boat accident. The verb “said” is in the past tense , so we apply
Tense: Past perfect passive ("had been the standard rule of backshifting the tense in
drowned") reported speech.
Reporting verb: reported (past tense) Tense Shift :
Telegram (Previous year papers PDFs [SSC,Railway,DSSSB,UP SI]): https://t.me/RBE_S
YouTube (Free lectures and job updates): https://www.youtube.com/c/RBERevolutionByEducation
Download RBE application for SSC Exams Complete Preparation
The original sentence uses simple present : “I “this” remains unchanged in direct speech
need” No need to shift to “that” or “it”
In indirect speech, this becomes simple past : Correct Direct Speech Structure :
“she needed” Matches tense, subject, and demonstrative
Pronoun Change : accurately
“I” (spoken by She ) becomes “she” in reported
form. Solution 37: ✅ Correct Answer: They requested
Final Structure : me to bring the basket there.
Combine the reporting verb and the transformed 🔍 Step-by-Step Analysis:
clause: → She said that she needed some rest. Direct Speech Given :
This is a polite request using “please”
Solution 34: ✅ Correct Answer: Rima said, "If I Verb: bring
had wings, I would fly in the sky." Place reference: here → needs adjustment in
🔍 Step-by-Step Explanation: indirect speech
Reported Sentence : Indirect Speech Rules :
Tense Pattern : “Please” → often replaced by “requested”
This is a second conditional sentence. “Bring” stays in base form after “to”
Structure: If + past tense , would + base verb “Here” → becomes “there” (shift in perspective)
Pronoun Shift (Indirect to Direct) : Subject Insertion :
“She” becomes “I” (Rima speaking about herself). “me” is added as the person being requested
Verb Consistency :
“had” stays as past tense Solution 38: ✅ Correct Answer: The man
“would fly” remains unchanged in direct speech. ordered them to be quiet and listen carefully.
🔍 Step-by-Step Analysis:
Solution 35: ✅ Correct Answer: She said that Direct Speech Given :
she wished she were a bird. This is an imperative sentence
🔍 Step-by-Step Analysis: (command/request).
Direct Speech : The tone is firm—suggests command , not
This is an exclamatory wish using subjunctive advice or casual request.
mood ("were" instead of "was"). Indirect Speech Rules for Imperatives :
Reporting Verb : Use verbs like ordered , requested , or advised
“said” remains unchanged. depending on tone.
No need for “exclaimed” or “wished” as “said” is Remove quotation marks.
neutral and acceptable here. Use “to + base verb” structure.
Mood & Meaning : Adjust pronouns if needed (e.g., “you” → “them”).
The sentence expresses a wish contrary to Tone Justification :
reality , not a factual statement. “Be quiet and listen carefully” is directive and firm
Tense & Pronoun Shift : → ordered fits best.
“I” → “she”
“were” remains unchanged due to subjunctive Solution 39: No ✅ Correct Answer
mood Rukmini asked Sanjana why she had not
“O that…” is omitted in indirect speech. appeared in the UPSC Preliminary Examination
the previous year.
Solution 36: ✅ Correct Answer: He said, "This 🔍 Point-wise Explanation
was in the box." Reporting Verb Change
🔍 Step-by-Step Analysis: said to → becomes asked (because the
Indirect Sentence Given : sentence is interrogative)
Tense Used : Reporting Structure
Past perfect : had been Direct question → becomes embedded
In direct speech, this often becomes simple past statement (no question format in indirect speech)
: was Pronoun Shift
Pronoun & Demonstrative Consistency : you → becomes she (referring to Sanjana)
Telegram (Previous year papers PDFs [SSC,Railway,DSSSB,UP SI]): https://t.me/RBE_S
YouTube (Free lectures and job updates): https://www.youtube.com/c/RBERevolutionByEducation
Download RBE application for SSC Exams Complete Preparation
Tense Backshift rest of the sentence is grammatically correct.
did not appear → becomes had not appeared
Rule: Simple past → past perfect in indirect Solution 44: The correct answer is "They said
speech that they enjoyed watching cricket." When
Time Expression Change changing from direct to indirect speech, the
last year → becomes the previous year present simple "enjoy" changes to the past simple
Rule: Time expressions shift to maintain temporal "enjoyed" due to the rules of reported speech.
clarity The other options either do not change the tense
or use the incorrect perfect tense.
Solution 40: ✅ Correct Answer
He told Renuka that they had to do it then and go Solution 45: The correct answer is " The farmer
there as soon as possible. said to his wife, 'Why are you upset when I
🔍 Point-wise Explanation have just returned from the city ?'" This
Reporting Verb Change sentence correctly uses present perfect tense to
said to → becomes told (since the sentence is indicate an action that has just now been
being reported to someone) completed, matching the reported speech's time
Pronoun Shift frame. The other options use incorrect or less
We → becomes they (referring to “He and natural wording and tenses.
Renuka” from the speaker’s perspective)
Tense Backshift Solution 46: The correct answer is "They said to
must → becomes had to the officer, 'Please permit us to go on leave
Rule: In indirect speech, modal verbs like must for three days.'" Direct speech should match
often change to had to in past reporting the natural request format using "please," and
now → becomes then "us" is the correct pronoun . Other options are
Time expressions shift in indirect speech either ungrammatical or do not represent the
Place Reference polite request form.
there → remains there (already correct for
indirect speech) Solution 47: The correct answer is "She said she
dared not oppose him." When changing to
Solution 41: The correct answer is "Ravi said indirect speech, the present tense "dare not" is
that he would join the new office soon." When shifted to simple past "dared not" to maintain
changing direct to indirect speech, the future correct tense sequence. "Dared not oppose him"
tense "will" changes to "would". While “would be is the grammatically appropriate and standard
joining” could be correct, the sentence in context indirect form of the original sentence.
most naturally reduces to “would join” for clarity
and style. The other options either use incorrect Solution 48: The correct answer is " The captain
tense or are not typical constructions in reported ordered them to stand at ease ." Imperative
speech. sentences in direct speech are generally reported
using verbs like "ordered," especially in a military
Solution 42: The correct answer is "They said, context, and the structure is "ordered + object + to
'You needn’t bring anything.'" The sentence + base verb." This matches both the context and
uses "needn’t," which is the contraction of "need grammar of the sentence.
not," and matches the meaning and structure of
the reported speech. The other options either Solution 49: The correct answer is " He told me
change the tense, use incorrect forms, or alter the to go there ." In indirect speech, "come here"
intended meaning. changes to "go there," and the reporting verb
"told" is followed by the object ("me") and the
Solution 43: The correct answer is " in the prize infinitive form of the verb ("to go"). The other
distribution function ." The proper preposition options have either incorrect grammar or do not
should be "at" instead of "in "—the idiomatic change the words appropriately for indirect
usage is "at the prize distribution function." The speech.

Telegram (Previous year papers PDFs [SSC,Railway,DSSSB,UP SI]): https://t.me/RBE_S


YouTube (Free lectures and job updates): https://www.youtube.com/c/RBERevolutionByEducation
Download RBE application for SSC Exams Complete Preparation
Solution 50: The correct answer is " He told me meaning.
to go there ." In indirect speech, "come here"
changes to "go there," and the verb "told" is Solution 57: The direct speech uses "had eaten"
followed by the object "me" and the correct (past perfect), so indirect speech retains the past
infinitive "to go." The other options have incorrect perfect tense. The correct transformation is "She
verb forms, misuse "to came ," or do not shift said that she had eaten breakfast." The other
"here" to "there" as required. options either use the wrong tense or change the
intended meaning.
Solution 51: The correct answer is " Madhubala
told Dilip that she had had a very sumptuous Solution 58: The reported speech "had been
meal the previous day." In indirect speech: "said writing" converts to direct speech in the present
to" becomes "told," "I" becomes "she," the past perfect continuous tense ("have been writing") to
perfect tense "had" becomes "had had" to match standard tense conversion rules. The
maintain the sequence of tenses, and "yesterday" correct direct statement is "I have been writing a
changes to "the previous day." Options 3 and 4 letter to the principal." Other options are
incorrectly use future time references instead of ungrammatical or incorrect tense forms.
past.
Solution 59: For indirect speech, questions with
Solution 52: The correct direct speech for "She "would you like" are converted to "S he asked me
told us that she had not seen the movie." is "She if I would like some coffee. " This accurately
said, 'I had not seen the movie.'" represents the original meaning, maintains
correct tense and structure, and complies with
Solution 53: To change the sentence to indirect indirect question formation rules. The other
speech, the reporting verb "said" (which denotes options are either grammatically incorrect or
a question/request) changes to "asked" and the change the meaning.
verb tense shifts from "can" to "could" in
accordance with sequence of tenses. Therefore, Solution 60: The reported speech "was learning"
the correct indirect speech is "She asked whether becomes "am learning" in direct speech when the
she could ask a question." reporting verb ("said") is in the past and the
sentence refers to a current ongoing action at the
Solution 54: When changing direct speech time of speaking. "I am learning French" is the
(present perfect - "have completed") to indirect direct statement that transforms correctly from the
speech, it becomes past perfect ("had given indirect form.
completed") to reflect reported speech rules. The
correct conversion is "He said that he had Solution 61: For direct speech, the original verb
completed the task." The other forms do not forms and modality (“could have been
accurately represent the reported statement. controlled...had been revised earlier”) are
retained as indicated in the indirect statement.
Solution 55: The indirect speech uses "kept The other options either use incorrect tense or
rising" and "would threaten." The correct direct change the meaning of the original sentence.
speech mirrors this structure using the past tense
("kept rising") and the modal "would threaten." Solution 62: The correct direct speech form is
This matches the original form and correctly "Were you aware of the policies of the company?"
retains the conditional aspect of the statement. This maintains the simple past tense and uses
the correct auxiliary ("were") for the subject "you."
Solution 56: In transforming direct speech ("I The other options have errors in tense, structure,
don’t like spicy food.") into indirect speech, the or word form.
tense changes from present simple to past simple
("didn't like"). The correct indirect form is "He said Solution 63: The direct speech form that matches
that he didn’t like spicy food." Other options either the reported speech "she had never encountered"
do not reflect the correct tense shift or change the is "I had never encountered such behavior." The
reporting verb in past tense and the perfect tense
Telegram (Previous year papers PDFs [SSC,Railway,DSSSB,UP SI]): https://t.me/RBE_S
YouTube (Free lectures and job updates): https://www.youtube.com/c/RBERevolutionByEducation
Download RBE application for SSC Exams Complete Preparation
structure of the action are retained for accurate form.
transformation. Other options use incorrect tense
or aspect. Solution 70: When converting direct speech to
indirect speech, the present perfect "have"
Solution 64: The direct question "Will you finish changes to "had", "now" becomes "then", and
your task on time?" changes to indirect speech as "said to" becomes "told." Therefore, the correct
"He asked me if I should finish my task on time." indirect form is: "Rohan told Manasi that he had
Question marks are not used in indirect speech no rose to give her then." The other options either
unless the reporting verb is "asked," and the use incorrect tense or fail to change "now" to
sentence does not become a question. The other "then."
options either change the meaning or incorrect
pronoun use. Solution 71: When converting a direct question
("Has he left already?") to indirect speech, the
Solution 65: While changing from direct to word order changes to "if he had left already."
indirect speech with exclamation, "exclaimed" The reporting verb "asked" is used for questions,
should be used as the reporting verb. The and the auxiliary verb "had" follows the subject
sentence should be converted to past perfect "he." The other options either use incorrect word
tense to reflect reported speech, hence "had been order or wrong reporting verbs.
very foolish." Other options either do not use the
correct reporting verb or do not have the correct Solution 72: When converting “must” in reported
backshift of tense. speech, especially when expressing supposition
or deduction, “must” often changes to “might” in
Solution 66: The phrase "lonely as a cloud" indirect speech. Thus, the correct indirect form is
compares the speaker's loneliness to a cloud "He said it might be raining outside." The other
using "as," which is the defining feature of a options either change the meaning or incorrectly
simile—a figure of speech that shows similarity convert the modal verb.
between two different things using "like" or "as."
Other options do not fit this usage in the line. Solution 73: ✅ Correct Answer: She said that
she wished she could meet him once.
Solution 67: The correct direct speech conversion This is an exclamatory sentence expressing a
of an exclamatory sentence uses "How" for strong desire or wish , not a question or
positive exclamations. "He exclaimed that the command.
flower was looking very beautiful..." changes to The reporting verb “said” remains unchanged.
"He said, 'How beautiful the flower is looking, The exclamatory tone is softened in indirect
which is placed in the vase!'" The other options speech.
are grammatically incorrect or do not use the The phrase “How I wish…” becomes “wished”
standard exclamatory structure. in reported form.
“ could meet” remains intact as it’s already in
Solution 68: The imperative sentence "Rina told the past modal form.
Suman to leave the city immediately" is best
expressed in direct speech as "Rina said to Solution 74: ✅ Correct Answer: Mohan told
Suman, 'Leave the city immediately.'" This Radha that he had seen her many times in the
structure correctly conveys a direct command. market since the previous year.
The other options either change the wording or do 🧠 Explanation:
not use the standard imperative format.
Direct Speech: ➡ Mohan told Radha, "I have
seen you many times in the market since last
Solution 69: The direct question "Where are you
year."
going?" in indirect speech is "He asked me where
Step-by-step transformation:
I was going." The present continuous tense
Reporting verb “told” → Past tense, so the
changes to past continuous, and the question
tense in the reported speech must shift.
word order changes to statement word order. The
“I have seen” → becomes “he had seen”
other options either use incorrect tense or verb
(present perfect → past perfect).
Telegram (Previous year papers PDFs [SSC,Railway,DSSSB,UP SI]): https://t.me/RBE_S
YouTube (Free lectures and job updates): https://www.youtube.com/c/RBERevolutionByEducation
Download RBE application for SSC Exams Complete Preparation
“you” → changes to “her” (Radha is the Solution 78: ✅ Correct Answer: Ravi said to
listener). his friend, "Please turn the music down."
“ last year” → becomes “the previous year” 🔄 Explanation:
(time expression shift in indirect speech). Sentence given: Ravi requested his friend to turn
✅ Final Indirect Speech: ➡ Mohan told Radha the music down. This is an example of reported
that he had seen her many times in the market speech in the form of a polite request .
since the previous year. To convert it into direct speech , we need to:
Use imperative form (request tone)
Solution 75: ✅ Correct Answer: He asked me Avoid modal verbs like will or are you going to
where I was going at that time of the day. which imply questions
🧠 Step-by-Step Explanation: Keep the tone polite and direct
Direct Speech: ➡ He said, "Where are you going ✅ Correct direct speech: ➡ Ravi said to his
at this time of the day?" friend, "Please turn the music down."
Let’s convert it into indirect speech :
Reporting verb “said” + question → becomes Solution 79: ✅ Correct Answer: She asked
“asked” who knew the answer.
Question word “Where” → retained 🔄 Direct to Indirect Speech Transformation:
“ are you going” → changes to “I was going” Direct Speech: She said, "Who knows the
(present continuous → past continuous) answer?" 🔸 This is a reporting of a wh -
“ this time of the day” → changes to “that time question in simple present tense .
of the day” (time reference shift) ✅ Rules Applied:
Reporting verb “said” becomes “asked”
Solution 76: ✅ Correct Answer: They said, (because it's a question).
"We were supposed to arrive earlier." “Who knows” changes to “who knew”
🧠 Explanation: (backshift of tense from present to past).
Indirect Speech: ➡ They said that they were No use of quotation marks in indirect speech.
supposed to arrive earlier. No need for “that” before a wh-word.
Let’s reverse it into Direct Speech : ✅ Final Indirect Sentence: ➡ She asked who
Reporting verb “said” → remains unchanged. knew the answer.
“ they were supposed to” → becomes “we
were supposed to” (pronoun shift from third Solution 80: ✅ Correct Answer: Sobhit said, "I
person to first person). can speak more fluently than Rahul."
Tense remains past → “were supposed to” stays 🧠 Explanation:
unchanged.
Indirect Speech: Sobhit said that he could
speak more fluently than Rahul.
Solution 77: ✅ Correct Answer: They said that To convert this into Direct Speech , we reverse
they were watching a movie. the tense shift and adjust the pronoun:
🧠 Explanation: “ he could speak” → becomes → “I can
Direct Speech: ➡ They said, "We are watching a speak”
movie." “ more fluently than Rahul” stays the same
Step-by-step transformation to Indirect Reporting verb “said” remains unchanged
Speech: ✅ Final Direct Speech: ➡ Sobhit said, "I can
Reporting verb “said” → in past tense, so the speak more fluently than Rahul."
tense in the reported clause must shift.
“We are watching” → becomes “they were Solution 81: ✅ Correct Answer: He said that he
watching” (present continuous → past had to reach early.
continuous). 🧠 Why This Is Correct:
Pronoun “we” → changes to “they” to match the
“Must” often changes to “had to” in indirect
subject.
speech when the reporting verb is in the past
tense (e.g., said ).
This maintains both the tense consistency and
Telegram (Previous year papers PDFs [SSC,Railway,DSSSB,UP SI]): https://t.me/RBE_S
YouTube (Free lectures and job updates): https://www.youtube.com/c/RBERevolutionByEducation
Download RBE application for SSC Exams Complete Preparation
the necessity conveyed in the original sentence. The reporting verb “said” remains unchanged.
The pronoun “they” becomes “we” (first person) to
Solution 82: Answer: The correct indirect speech reflect the speaker’s original words.
transformation of the sentence “The teacher
said, ‘Let him try on his own.’” is: Solution 88: ✅ Correct Answer: Option 2 – She
Option 2 – The teacher suggested that he try said that she was tired.
on his own. 🔍 Explanation:
Direct Speech: She said, “I was tired.”
Solution 83: ✅ Correct Direct Speech: The reporting verb “said” is in the past tense .
She said, "I like chocolate." The original statement is also in past tense (“was
tired”), so no tense shift is needed.
Solution 84: 🔄 Voice Transformation: Indirect The pronoun “I” changes to “she” to match the
→ Direct Speech speaker.
Given Sentence (Indirect Speech): She said
that she was going to the market. Solution 89: ✅ Correct Answer: Option 3 –
✅ Correct Direct Speech: “Rarely had I seen such a breathtaking view in
She said, "I am going to the market." all my travels.”
🔍 Detailed Explanation:
Solution 85: ✅ Correct Answer: Option 2 – She Indirect Speech:
said that she had to go then. The explorer remarked that rarely had he seen
🔍 Explanation: such a breathtaking view in all his travels.
Direct Speech: She said, "I must go now." We need to convert this into Direct Speech ,
Indirect Speech Rules Applied: preserving:
Reporting Verb “said” → No change (no object The reporting verb : remarked (past tense, so no
given). change in tense needed)
Modal “must” → Changes to “had to” in indirect The inversion used for emphasis: Rarely had he
speech. seen... → this is a classic case of negative
Time word “now” → Changes to “then” . adverbial inversion , which must be retained in
Direct Speech.
Solution 86: ✅ Correct Answer: Option 4 – He
prayed that God might bless me. Solution 90: ✅ Correct Answer: Option 4
🔍 Explanation: 🔍 Explanation:
Direct Speech: He said, "May God bless you." Indirect Speech:
This is a blessing or prayer , not a factual The astronomer noted that Jupiter’s rotation was
statement or command. So, when converting to the fastest among all planets in the solar system.
Indirect Speech , we use a verb that reflects the
intention behind the statement—like “prayed” , Solution 91: ✅ Correct Answer: Option 3 –
“wished” , or “hoped” . “They said that their teacher would explain
“May” changes to “might” in indirect speech. the topic the next day.”
“Said” becomes “prayed” to match the emotional 🧠 Grammar Rules Applied:
tone of the original. Reporting Verb “said” → Past tense → triggers
backshifting .
Solution 87: ✅ Correct Answer: Option 3 – “will” → changes to “would” .
They said, "We are enjoying the trip." “tomorrow” → changes to “the next day” .
🔍 Explanation: Pronoun “our” → changes to “their” (depends
Indirect Speech: They said that they were on speaker context).
enjoying the trip. No change in sentence structure needed
To convert this into Direct Speech , we reverse beyond tense and pronoun shifts.
the tense shift:
“ were enjoying” in indirect speech becomes Solution 92: 🧠 Step-by-Step Logic:
“are enjoying” in direct speech. Reporting Verb:
Telegram (Previous year papers PDFs [SSC,Railway,DSSSB,UP SI]): https://t.me/RBE_S
YouTube (Free lectures and job updates): https://www.youtube.com/c/RBERevolutionByEducation
Download RBE application for SSC Exams Complete Preparation
told him → implies the speaker is directly
addressing him .
Subject Consistency:
I remains I (speaker is the same).
him is the listener.
Tense Correction:
was busy is the backshifted form of am busy .
In Direct Speech, we restore the original tense:
am busy .
Final Direct Form: 👉 I told him, “I am busy.”

For SSC/Railway/DSSB Exams Complete Preparation (Download RBE Application)


(Learn from those who have cleared the exam themselves)
https://play.google.com/store/apps/details?id=com.revolution.education

Telegram (Previous year papers PDFs [SSC,Railway,DSSSB,UP SI]): https://t.me/RBE_S


YouTube (Free lectures and job updates): https://www.youtube.com/c/RBERevolutionByEducation
Download RBE application for SSC Exams Complete Preparation

SSC Steno 2025 T-1 Jumbled Sentences All 99 Questions with Detailed Solution and Answer key

Q1. Which of the given options is the most 3. His ideas continue to influence modern
logical sequencing of the following sentences technology and design.
to form a meaningful paragraph? 4. Apple Inc. introduced products like iPhone and
His observations eventually contributed to his iPad.
groundbreaking book, On the Origin of Species. (a) 2143 (b) 1243
The Galápagos Islands are famous for their (c) 4312 (d) 3124
diverse wildlife and their role in Charles Darwin’s
theory of evolution. Q5. Which of the given options is the most
This led him to propose the idea of natural logical sequencing of the following sentences
selection, where species evolve over time to to form a meaningful paragraph?
adapt to their environment. 1. The primary factor in its formation was the
Darwin observed that finches on different islands Colorado River, which gradually carved through
had distinct beak shapes suited to their specific layers of rock, deepening and widening the
diets. canyon.
(a) 2-4-3-1 (b) 1-3-2-4 2. The Grand Canyon was formed over millions of
(c) 2-4-1-3 (d) 4-2-3-1 years through the combined forces of geological
activity and erosion.
Q2. Rearrange the following parts (P, Q, R, S) 3. weathering from wind, rain, and temperature
to form the most meaningful and changes further shaped the canyon's intricate
grammatically correct sentence: rock formations.
P. though he lacked international experience 4. Tectonic uplift of the Colorado Plateau
Q. was selected for the Olympic squad increased the river's flow and erosive power,
R. the young shooter allowing it to cut deeper into the landscape.
S. due to his exceptional performance at the (a) 3214 (b) 1342
national trials (c) 2143 (d) 4321
(a) R Q P S (b) P S R Q
(c) R Q S P (d) P R Q S Q6. Rearrange the parts of the sentence In the
correct order.
Q3. Rearrange the parts of the sentence in the
correct order. Inflation
The economic slowdown P. making it difficult for households
P. reducing consumer spending Q. to manage daily expenses
Q. in multiple industries R. affects purchasing power
R. straining household incomes and S. and leads to increased cost of living,
S. has led to job losses (a) SRQP (b) QSPR
(a) QSPR (b) SQRP (c) RSPQ (d) PQRS
(c) PRSQ (d) RSPQ
Q7.
Q4. Which of the given options is the most Arrange the parts to form a correct sentence:
logical sequencing of the following sentences
to form a meaningful paragraph? (1) The athlete trained relentlessly.
1. He co-founded Apple inc. along with Wozniak. (2) He overcame several injuries.
2. Steve Jobs was a visionary entrepreneur and (3) He qualified for the finals.
innovator. (4) He inspired young players.
Telegram (Previous year papers PDFs [SSC,Railway,DSSSB,UP SI]): https://t.me/RBE_S
YouTube (Free lectures and job updates): https://www.youtube.com/c/RBERevolutionByEducation
Download RBE application for SSC Exams Complete Preparation
(a) 2134 (b) 1324 by lightning, and the meta l key conducted the
(c) 1243 (d) 1234 electrical charge.
(a) 1243 (b) 1234
Q8. Rearrange the parts of the sentence in the (c) 1324 (d) 1432
correct order.
Q12. Arrange the sentence parts:
The update (A) should follow ( B ) traffic rules (C) strictly ( D )
P. ensuring security everyone
Q. enhanced encryption (a) DABC (b) DACB
R. and data autonomy (c) DCAB (d) DBCA
S across user devices
(a) SRPQ (b) SPQR Q13. Which of the given options is the most
(c) QRPS (d) PSRQ logical sequencing of the following sentences
to form a meaningful paragraph?
Q9. Arrange the parts to form a correct
sentence: 1. The advancements in science led by figures
like Copernicus, Galileo and Descartes also
(1) The exhibition opened to the public. questioned its established views of the
(2) visitors admired the artworks. universe.
(3) Interactive sessions were conducted.
(4) Feedback was overwhelmingly positive. 2. It Encouraged the growth of secularism and
(a) 1234 (b) 1243 individualism, drawing inspiration from the
(c) 2314 (d) 2314 classical works of ancient Greece and
Rome.
Q10. Which of the given options is the most
logical sequencing of the following sentences 3. The invention of the printing press further
to form a meaningful paragraph? accelerated this shift, making books more
accessible and promoting literacy and
1. He has inspired millions through his speeches individual thinking.
and writings.
2. He came to be known as the "Missile Man of 4. The Renaissance ushered in the new era that
India" for his contributions. questioned the monopoly of the Roman Catholic
3. In the year 2002, he became the President of Church on knowledge and spiritual
India. guidance.
4. Dr. A.P.J. Abdul Kalam worked as a scientist at (a) 4213 (b) 1342
ISRO for forty years. (c) 2143 (d) 3214
(a) 2431 (b) 1324
(c) 4231 (d) 2341 Q14. Which of the given options is the most
logical sequencing of the following sentences
Q11. Which of the given options is the most to form a meaningful paragraph?
logical sequencing of the following sentences
to form a meaningful paragraph? 1. Feminism began as a fight for women’s
suffrage and then expanded to include social
1. Franklin hypothesized that lightning was a form discrimination, workplace inequalities and
of electrical discharge. reproductive rights.
2 . Franklin flew a kite made of a silk string and
attached a metal key to it during a storm. 2. Nevertheless, the movement received
3. He wanted to prove this by conducting an significant support from progressive thinkers who
experiment to capture the electrical charge from a believed in gender equality and the dismantling of
thunderstorm. patriarchal structures.
4. As the storm approached, the kite was struck
3. Conservatives viewed the feminist movement
Telegram (Previous year papers PDFs [SSC,Railway,DSSSB,UP SI]): https://t.me/RBE_S
YouTube (Free lectures and job updates): https://www.youtube.com/c/RBERevolutionByEducation
Download RBE application for SSC Exams Complete Preparation
as threatening to family values, with fears that 3. When the eruption occurred, a thick layer of hot
women would abandon traditional roles as wives ash and pumice rapidly buried the town.
and mothers. 4. Unlike other ruins, Pompeii remained frozen in
time due to Mount Vesuvius' sudden eruption.
4. The evolution of feminism has been a (a) 1243 (b) 4231
transformative journey, reflecting changing (c) 3142 (d) 2431
societal attitudes and the struggles for gender
equality. Q19. In the following question, the first and the
(a) 3421 (b) 1342 last parts of the sentence/passage are numbered
(c) 2314 (d) 4132 1 and 6. The rest of the sentence/passage is split
into four parts — P, Q, R, and S. These four parts
Q15. Choose the correct sequence to form a are not given in their proper order. Read the
meaningful sentence: sentence/passage and find the most logical order
of the parts between 1 and 6.
(A) is very interesting ( B ) reading ( C ) books ( The invention of the wheel changed the world. P:
D) fiction Transport became easier. Q: Machines could now
(a) B DCA (b) BDCA be built. R: Travel became faster. S: Roads
(c) BACD (d) BADC improved quickly. 6. It was a great invention.
(a) PQRS (b) PRSQ
Q16. Arrange the sentence parts: (c) PSRQ (d) PRQS

(A) to submit ( B ) asked him (C) the report ( D ) Q20. In the following question, the first and the
the manager last parts of the sentence/passage are numbered
(a) DCAB (b) DBAC 1 and 6. The rest of the sentence/passage is split
(c) DBCA (d) BACD into four parts — P, Q, R, and S. These four parts
are not given in their proper order. Read the
Q17. Rearrange the following steps of the sentence/passage and find the most logical order
Union Budget preparation in the correct order: of the parts between 1 and 6.
Exercise keeps the body fit and active. P: It also
P. The finance minister presents the budget in boosts mental health. Q: Regular workouts
Parliament reduce stress levels. R: Walking, cycling, and
yoga are easy options. S: Physical activity
Q. The Ministry of Finance collects estimates strengthens the heart and muscles. 6. So,
from various ministries exercise should be part of our routine.
(a) SPRQ (b) QRSP
R. The budget is passed by both houses of (c) SQPR (d) PRQS
Parliament
Q21. In the following question, the first and the
S. The Cabinet approves the final draft of the last parts of the sentence/passage are numbered
budget 1 and 6. The rest of the sentence/passage is split
(a) QPRS (b) QSPR into four parts — P, Q, R, and S. These four parts
(c) PQSR (d) SQPR are not given in their proper order. Read the
sentence/passage and find the most logical order
Q18. Which of the given options is the most of the parts between 1 and 6.
logical sequencing of the following sentences Artificial Intelligence is reshaping industries. P: It
to form a meaningful paragraph? also improves customer service through
automation. Q: In healthcare, it helps in quick
1. I t cut off oxygen and prevented the usual diagnosis. R: AI systems reduce human error in
decay caused by air, moisture, and bacteria. complex calculations. S: From manufacturing to
2 . The excavation of Pompeii provided a medicine, its impact is visible. 6. Its future role will
remarkably well-preserved glimpse intg life in only expand.
ancient Rome. (a) S Q R P (b) Q R P S
Telegram (Previous year papers PDFs [SSC,Railway,DSSSB,UP SI]): https://t.me/RBE_S
YouTube (Free lectures and job updates): https://www.youtube.com/c/RBERevolutionByEducation
Download RBE application for SSC Exams Complete Preparation
(c) RSQP (d) Q P R S (c) RQSP (d) QPRS

Q22. Arrange the sentence parts: Q27. Rearrange the parts of this sentence:
(A) after the meeting (b) the team members (C) (A) despite poor rainfall (b) the farmers (C)
went (D) to a restaurant managed to harvest (D) a good crop
(a) ABCD (b) BACD (a) BADC (b) CBAD
(c) BADC (d) BCAD (c) BDAC (d) BACD

Q23. In the following question, the first and the Q28. Choose the correct sequence to form a
last parts of the sentence/passage are numbered meaningful sentence:
1 and 6. The rest of the sentence/passage is split (A) the guests (b) arrived (C) at the party (D)
into four parts — P, Q, R, and S. These four parts early
are not given in their proper order. Read the (a) CBAD (b) BADC
sentence/passage and find the most logical order (c) ABDC (d) BACD
of the parts between 1 and 6.
The invention of the telescope changed Q29. Arrange the sentence parts:
astronomy. P: Scientists could observe distant (A) opened (b) suddenly (C) the door (D) he
galaxies. Q: They discovered stars not visible to (a) CBAD (b) DACB
the naked eye. R: Earlier, knowledge was limited (c) DCAB (d) DABC
to nearby celestial bodies. S: The new tool
opened a whole new universe. 6. Astronomy Q30. Rearrange the parts of this sentence:
became a precise science. (A) their demands (b) the workers (C) went on
(a) RPSQ (b) SRPQ strike (D) after the management rejected
(c) PQRS (d) RSPQ (a) DACB (b) BACD
(c) B C DA (d) DBAC
Q24. Rearrange the parts of this sentence:
(A) to protect endangered species (b) wildlife Q31. In the following question, the first and the
sanctuaries (C) have been established (D) across last parts of the sentence/passage are numbered
the country 1 and 6. The rest of the sentence/passage is split
(a) CBAD (b) BADC into four parts — P, Q, R, and S. These four parts
(c) BC DA (d) BACD are not given in their proper order. Read the
sentence/passage and find the most logical order
Q25. Arrange the sentence parts: of the parts between 1 and 6.
(A) invited (b) to the event (C) many guests (D) Recycling helps in environmental protection. P: It
were reduces the amount of waste in landfills. Q:
(a) CDAB (b) DCAB Recycling paper saves trees and water. R: It also
(c) DABC (d) DACB cuts down pollution caused by burning waste. S:
But awareness among people is still low. 6. Public
Q26. In the following question, the first and the campaigns can improve participation.
last parts of the sentence/passage are numbered (a) QPRS (b) QRPS
1 and 6. The rest of the sentence/passage is split (c) PQRS (d) PRSQ
into four parts — P, Q, R, and S. These four parts
are not given in their proper order. Read the Q32. Arrange the sentence parts:
sentence/passage and find the most logical order (A) were injured (b) many passengers (C) in the
of the parts between 1 and 6. accident (D) yesterday
Riya enjoys learning different languages. P: She (a) BADC (b) BCDA
attends French classes. Q: Her Spanish is (c) BACD (d) BCAD
improving. R: She knows some German. S: She
uses an app to practice daily. 6. She wants to be Q33. In the following question, the first and the
a polyglot. last parts of the sentence/passage are numbered
(a) QRSP (b) RPQS 1 and 6. The rest of the sentence/passage is split
into four parts — P, Q, R, and S. These four parts
Telegram (Previous year papers PDFs [SSC,Railway,DSSSB,UP SI]): https://t.me/RBE_S
YouTube (Free lectures and job updates): https://www.youtube.com/c/RBERevolutionByEducation
Download RBE application for SSC Exams Complete Preparation
are not given in their proper order. Read the businesses. P: Businesses struggle to maintain
sentence/passage and find the most logical order profit margins. Q: Rising prices reduce the
of the parts between 1 and 6. purchasing power of individuals. R: Governments
The Earth's climate has changed over centuries. may intervene through monetary policy. S: It
P: But current warming is caused by human creates uncertainty in long-term investment
actions. Q: Historically, it changed due to natural planning. 6. Managing inflation is crucial for
causes. R: Burning fossil fuels increases economic stability.
greenhouse gases. S: Deforestation worsens the (a) QPRS (b) PQRS
problem. 6. This calls for urgent action. (c) QRSP (d) SQPR
(a) SQRP (b) PRSQ
(c) QPRS (d) QPSR Q37. Which of the given options is the most
logical sequence of the following sentences to
Q34. In the following question, the first and the form a meaningful paragraph?
last parts of the sentence/passage are numbered 1. Cybersecurity is a growing concern in the
1 and 6. The rest of the sentence/passage is split digital age.
into four parts — P, Q, R, and S. These four parts 2. Strong passwords, encryption, and awareness
are not given in their proper order. Read the can help prevent cybercrimes.
sentence/passage and find the most logical order 3. Hackers use advanced techniques to steal
of the parts between 1 and 6. personal and financial data.
Honeybees are vital for pollination. P: They 4. It means protecting computer systems from
transfer pollen between flowers. Q: This helps in attacks by hackers.
the growth of fruits and seeds. R: Without bees, (a) 2-3-1-4 (b) 1-4-3-2
many crops would fail. S: But bee populations are (c) 4-1-3-2 (d) 3-2-4-1
declining globally. 6. Protecting bees is essential
for food security. Q38. Rearrange the following steps of the national
(a) PSQR (b) PRQS census process in the correct order:
(c) RPQS (d) PQRS P. Enumerators visit households to collect data
Q. Data is compiled and analyzed at central
Q35. In the following question, the first and the offices
last parts of the sentence/passage are numbered R. The government announces census dates and
1 and 6. The rest of the sentence/passage is split guidelines
into four parts — P, Q, R, and S. These four parts S. A final census report is published and shared
are not given in their proper order. Read the with the public
sentence/passage and find the most logical order (a) P → Q → S → R (b) R → P → Q → S
of the parts between 1 and 6. (c) P → Q → S → R (d) S → Q → P → R
Art is a reflection of society. P: It communicates
emotions beyond words. Q: From cave paintings Q39. In the following question, the first and the
to modern murals, art evolves. R: Artists express last parts of the sentence/passage are numbered
views on culture, politics, and identity. S: It also 1 and 6...
raises awareness on social issues. 6. Hence, art The digital divide continues to deepen social
holds immense power in society. inequalities. P. Unequal access to the internet
(a) PRQS (b) QRPS limits educational opportunities. Q. Those in rural
(c) QPRS (d) RSPQ or underserved areas lag behind. R. It prevents
skill development and employment access. S.
Q36. In the following question, the first and the Technology's benefits are unequally distributed.
last parts of the sentence/passage are numbered 6. Bridging this gap is critical for inclusive growth.
1 and 6. The rest of the sentence/passage is split (a) SQPR (b) QPRS
into four parts — P, Q, R, and S. These four parts (c) PQRS (d) SPRQ
are not given in their proper order. Read the
sentence/passage and find the most logical order Q40. Arrange the sentence parts:
of the parts between 1 and 6. (A) by the end of the year (b) complete the project
Economic inflation affects both consumers and (C) the construction company (D) promised to
Telegram (Previous year papers PDFs [SSC,Railway,DSSSB,UP SI]): https://t.me/RBE_S
YouTube (Free lectures and job updates): https://www.youtube.com/c/RBERevolutionByEducation
Download RBE application for SSC Exams Complete Preparation
(a) CDBA (b) CADB Farmers work hard despite unpredictable
(c) ACBD (d) DACB weather. Q: They grow crops that feed the nation.
R: Yet, they face several challenges. S: Modern
Q41. Choose the correct sequence to form a technology can support better yield. 6. Ensuring
meaningful sentence: farmer welfare is national duty.
(A) the winner (b) declared (C) was (D) yesterday (a) PRQS (b) QPRS
(a) ACBD (b) CBAD (c) QRSP (d) PQRS
(c) CABD (d) BCAD
Q47. Rearrange the parts of the sentence in the
Q42. Arrange the sentence parts: correct order.
(A) his advice (b) the students (C) accepted (D)
happily Advances in satellite imaging
(a) ABCD (b) BCAD P. across diverse geographic regions
(c) BACD (d) BADC Q. enabling accurate climate modelling
R. and disaster prediction systems
Q43. Arrange the sentence parts: S. have transformed environmental monitoring,
(A) on time (b) the bus (C) arrived (D) at the stop (a) SQRP (b) QPSR
(a) BCDA (b) ABCD (c) RPSQ (d) PRQS
(c) CBAD (d) BCAD
Q48. Which of the given options is the most
Q44. Choose the correct sequence to form a logical sequencing of the following sentences to
meaningful sentence: form a meaningful paragraph?
(A) served (b) dinner (C) they (D) late
(a) BACD (b) BADC 1. They are formed by tiny animals called coral
(c) ACBD (d) CABD polyps.
2. Coral reefs are home to many marine species.
Q45. In the following question, the first and the 3. This is caused by rising ocean temperatures
last parts of the sentence/passage are numbered and pollution.
1 and 6. The rest of the sentence/passage is split 4. Corals are losing their wonderful colours due to
into four parts—P, Q, R, and S. These four parts coral bleaching.
are not given in their proper order. Read the (a) 2-3-1-4 (b) 2-1-4-3
sentence/passage and find the most logical order (c) 4-3-2-1 (d) 1-3-2-4
of the parts between 1 and 6.
1. The spacecraft-initiated descent towards the Q49. In the following question, the first and the
red planet. P: The data gathered during this last parts of the sentence/passage are numbered
process will aid future missions. Q: The heat 1 and 6. The rest of the sentence/passage is split
shield protected it from atmospheric entry. R: into four parts — P, Q, R, and S. These four parts
Soon after, the parachutes deployed successfully. are not given in their proper order. Read the
S: Engineers monitored every second of the sentence/passage and find the most logical order
landing sequence. 6. This landing marked a of the parts between 1 and 6.
historic step in interplanetary exploration.
(a) QRSP (b) RQSP Festivals play a big role in cultural unity. P: They
(c) QRSP (d) SQRP bring together people of different beliefs. Q: Each
region celebrates with its unique customs. R:
Q46. In the following question, the first and the Shared celebrations promote tolerance and joy.
last parts of the sentence/passage are numbered S: They also preserve traditional art forms. 6.
1 and 6. The rest of the sentence/passage is split Hence, festivals strengthen the social fabric.
into four parts — P, Q, R, and S. These four parts (a) QSPR (b) RPSQ
are not given in their proper order. Read the (c) PRQS (d) PQRS
sentence/passage and find the most logical order
of the parts between 1 and 6. Q50. In the following question, the first and the
1. Agriculture is the backbone of our economy. P: last parts of the sentence/passage are numbered
Telegram (Previous year papers PDFs [SSC,Railway,DSSSB,UP SI]): https://t.me/RBE_S
YouTube (Free lectures and job updates): https://www.youtube.com/c/RBERevolutionByEducation
Download RBE application for SSC Exams Complete Preparation
1 and 6. The rest of the sentence/passage is split of the parts between 1 and 6.
into four parts — P, Q, R, and S. These four parts
are not given in their proper order. Read the Computers help us in many ways.
sentence/passage and find the most logical order P: Writing reports is easier.
of the parts between 1 and 6. Q: Data is stored safely.
R: Meetings happen online.
1 . The United Nations was formed after the S: Calculations done fast.
Second World War. P: It was established to 6. Offices depend on them daily.
prevent future global conflicts. Q: Over the (a) QRPS (b) PQSR
decades, it has expanded its roles. R: Now it (c) PRSQ (d) PQRS
addresses issues like climate change and human
rights. S: Its peacekeeping missions have played Q55. Rearrange the parts of this sentence:
a vital role in conflict zones. 6. Despite criticisms, (A) rising inflation (b) causes difficulties (C) for
the UN remains a key international body. middle-class families (D) everywhere
(a) PRSQ (b) PSQR (a) BACD (b) BCAD
(c) QPRS (d) RPSQ (c) BACD (d) ABCD

Q51. Arrange the sentence parts: Q56. In the following question, the first and last
(A) announced (b) the winner (C) was (D) parts of the sentence/passage are numbered 1
yesterday evening and 6. The rest of the sentence/passage is split
(a) BACD (b) CBAD into four parts — P, Q, R, and S. These four parts
(c) BCAD (d) ACBD are not given in their proper order. Read the
sentence/passage and find the most logical order
Q52. Arrange the sentence parts: of the parts between 1 and 6.
(A) read (b) the instructions (C) before filling (D)
carefully Email has changed modern communication.
(a) BACD (b) DA BC P: It is faster than traditional letters.
(c) BCAD (d) BDCA Q: Instant messaging is also gaining popularity.
R: Emails allow file sharing and formal contact.
Q53. In the following question, the first and last S: However, they may lack personal touch.
parts of the sentence/passage are numbered 1 6. Choosing the right medium depends on
and 6. The rest of the sentence/passage is split context.
into four parts — P, Q, R, and S. These four parts (a) RPSQ (b) PRSQ
are not given in their proper order. Read the (c) QPRS (d) PQSR
sentence/passage and find the most logical order
of the parts between 1 and 6. Q57. In the following question, the first and last
parts of the sentence/passage are numbered 1
1. I visited a zoo last weekend. and 6. The rest of the sentence/passage is split
P: The lions were resting under trees. into four parts — P, Q, R, and S. These four parts
Q: I saw many wild animals there. are not given in their proper order. Read the
R: Monkeys were swinging branches. sentence/passage and find the most logical order
S: The elephants were eating sugarcane. of the parts between 1 and 6.
6. It was a fun and learning experience.
(a) QPSR (b) QSPR 1. Solar energy is gaining popularity worldwide. P:
(c) PQSR (d) PRSQ It is available in abundance. Q: This makes it a
sustainable option. R: Unlike fossil fuels, it
Q54. In the following question, the first and the doesn’t emit harmful gases. S: Also, it
last parts of the sentence/passage are numbered reduces dependency on non-renewable
1 and 6. The rest of the sentence/passage is split sources. 6. Hence, governments are investing in
into four parts — P, Q, R, and S. These four parts solar technology.
are not given in their proper order. Read the (a) RSPQ (b) PRSQ
sentence/passage and find the most logical order
Telegram (Previous year papers PDFs [SSC,Railway,DSSSB,UP SI]): https://t.me/RBE_S
YouTube (Free lectures and job updates): https://www.youtube.com/c/RBERevolutionByEducation
Download RBE application for SSC Exams Complete Preparation
(c) PQRS (d) QPSR S: Stories often explore future societies.
6: Thus, sci-fi has real-world influence.
Q58. In the following question, the first and the (a) RQPS (b) SRQP
last parts of the sentence/passage are numbered (c) PQRS (d) QPRS
1 and 6. The rest of the sentence/passage is split
into four parts — P, Q, R, and S. These four parts Q63. In the following question, the first and the
are not given in their proper order. Read the last parts of the sentence/passage are numbered
sentence/passage and find the most logical order 1 and 6. The rest of the sentence/passage is split
of the parts between 1 and 6. into four parts — P, Q, R, and S. These four parts
are not given in their proper order. Read the
Ravi opened his new geometry box. sentence/passage and find the most logical order
P: It had a ruler, compass, and protractor. of the parts between 1 and 6.
Q: He used it in his maths class. 1: A triangle has three sides.
R: He showed it to friends. P: The angles can be equal or different.
S: It was made of metal and looked shiny. Q: It is one of the basic shapes.
6. He liked his new box very much. R: Every triangle has three angles.
(a) SRPQ (b) PSRQ S: The sum of angles is always 180 degrees.
(c) PRQS (d) PRSQ 6: Triangles are important in geometry.
(a) RPQS (b) PRSQ
Q59. Rearrange the parts of this sentence: (c) QRPS (d) QPRS
(A) before the heavy rains (b) the villagers (C)
repaired the damaged bridge (D) with the help of Q64. Rearrange the parts of this sentence:
officers (A) for higher education (b) many students (C)
(a) DACB (b) ABCD prefer to go abroad (D) these days
(c) BACD (d) BCAD (a) ABCD (b) BDAC
(c) BADC (d) BACD
Q60. Arrange the sentence parts:
(A) the headmaster (b) for his punctuality (C) Q65. Arrange the sentence parts:
praised (D) the student (A) every week (b) we (C) play (D) cricket
(a) ADCB (b) BDCA (a) BDCA (b) BCDA
(c) ABCD (d) ACBD (c) CADB (d) DACB

Q61. Choose the correct sequence to form a Q66. Arrange the sentence parts:
meaningful sentence: (A) met (b) his friend (C) at the station (D) Rohit
(A) has a long (b) India (C) and rich (D) cultural (a) DACB (b) DBAC
history (c) DCBA (d) DABC
(a) BACD (b) BDCA
(c) BCAD (d) BADC Q67. Arrange the sentence parts:
(A) my mother (b) bought (C) a beautiful saree
Q62. In the following question, the first and the (D) yesterday
last parts of the sentence/passage are numbered (a) BACD (b) CBAD
1 and 6. The rest of the sentence/passage is split (c) ABCD (d) ACBD
into four parts — P, Q, R, and S. These four parts
are not given in their proper order. Read the Q68. Arrange the sentence parts:
sentence/passage and find the most logical order (A) to the office (b) reached (C) Mr. Sharma (D)
of the parts between 1 and 6. at 9 am
(a) CBAD (b) DCAB
Science fiction fuels technological imagination. (c) CDBA (d) CABD
P: It inspires young minds to innovate.
Q: Many past inventions were once fictitious Q69. Rearrange the parts of the sentence in the
ideas. correct order.
R: It encourages thinking beyond limits.
Telegram (Previous year papers PDFs [SSC,Railway,DSSSB,UP SI]): https://t.me/RBE_S
YouTube (Free lectures and job updates): https://www.youtube.com/c/RBERevolutionByEducation
Download RBE application for SSC Exams Complete Preparation
Postcolonial literature are not given in their proper order. Read the
P. interrogates historical narratives, sentence/passage and find the most logical order
Q. while reclaiming suppressed voices of the parts between 1 and 6.
R. and offers alternative perspectives
S. rooted in indigenous experience, Social media influences public opinion.
(a) QRSP (b) RSPQ P: Posts can go viral within minutes.
(c) SRPQ (d) PS QR Q: It helps in sharing views widely.
R: However, misinformation also spreads fast.
Q70. Choose the correct sequence to form a S: This can lead to confusion or panic.
meaningful sentence: 6. So, digital literacy is essential today.
(A) opened (b) slowly (C) the door (D) he (a) QPRS (b) PRQS
(a) DACB (b) BACD (c) QPSR (d) QSRP
(c) DABC (d) DCAB
Q74. In the following question, the first and the
Q71. In the following question, the first and last last parts of the sentence/passage are numbered
parts of the sentence/passage are numbered 1 1 and 6. The rest of the sentence/passage is split
and 6. The rest of the sentence/passage is split into four parts — P, Q, R, and S. These four parts
into four parts – P, Q, R, and S. These four parts are not given in their proper order. Read the
are not given in their proper order. Read the sentence/passage and find the most logical order
sentence/passage and find the most logical order of the parts between 1 and 6.
of the parts between 1 and 6.
The evolution of language is deeply linked to
1: A compass is an important tool in navigation. human history. P: Early humans used symbols to
P: It always points towards the magnetic north. convey meaning. Q: Writing systems emerged
Q: Even in cloudy weather, it shows direction. thousands of years later. R: Spoken language
R: It does not depend on GPS or electricity. likely developed alongside tool use. S: Over time,
S: Hikers and sailors rely on it during travel. languages diversified across regions. 6. Today,
6: Thus, it remains a reliable tool even today. language continues to evolve with technology.
(a) PRSQ (b) RQPS (a) RPSQ (b) PRSQ
(c) PRQS (d) SRPQ (c) QRSP (d) RPQS

Q72. In the following question, the first and last Q75. In the following question, the first and the
parts of the sentence/passage are numbered 1 last parts of the sentence/passage are numbered
and 6. The rest of the sentence/passage is split 1 and 6. The rest of the sentence/passage is split
into four parts — P, Q, R, and S. These four parts into four parts — P, Q, R, and S. These four parts
are not given in their proper order. Read the are not given in their proper order. Read the
sentence/passage and find the most logical order sentence/passage and find the most logical order
of the parts between 1 and 6. of the parts between 1 and 6.

Plants need sunlight to grow. Human memory is a complex neurological


P: Photosynthesis needs light. process. P: Short-term memory stores data
Q: No food is made without it. temporarily for immediate use. Q: Information can
R: Leaves turn yellow without it. be transferred to long-term memory with
S: Plant dies slowly. repetition. R: The brain filters and encodes
6: Sunlight is very important. information efficiently. S: Long-term memory
(a) PSQR (b) SPQR holds information for an extended period. 6.
(c) PQRS (d) SQPR Understanding memory helps improve learning
strategies.
Q73. In the following question, the first and last (a) PSRQ (b) PRSQ
parts of the sentence/passage are numbered 1 (c) RPSQ (d) RQPS
and 6. The rest of the sentence/passage is split
into four parts — P, Q, R, and S. These four parts
Telegram (Previous year papers PDFs [SSC,Railway,DSSSB,UP SI]): https://t.me/RBE_S
YouTube (Free lectures and job updates): https://www.youtube.com/c/RBERevolutionByEducation
Download RBE application for SSC Exams Complete Preparation
Q76. In the following question, the first and the are already affecting agriculture. Q: Melting
last parts of the sentence/passage are numbered glaciers are disrupting water supplies in many
1 and 6. The rest of the sentence/passage is split regions. R: Its impacts are visible across the
into four parts — P, Q, R, and S. These four parts globe. S: Rising sea levels threaten low-lying
are not given in their proper order. Read the coastal areas. 6. Urgent global collaboration is
sentence/passage and find the most logical order essential to tackle it.
of the parts between 1 and 6. (a) PRSQ (b) QSPR
(c) RPQS (d) RQPS
It started raining in the evening. P: People ran to
shelters. Q: Water collected on roads. R: Traffic Q81. In the following question, the first and the
moved slowly. S: Many opened their umbrellas. 6. last parts of the sentence/passage are numbered
It was a heavy shower. 1 and 6. The rest of the sentence/passage is split
(a) PSRQ (b) SRPQ into four parts — P, Q, R, and S. These four parts
(c) PQRS (d) QPRS are not given in their proper order. Read the
sentence/passage and find the most logical order
Q77. In the following question, the first and the of the parts between 1 and 6.
last parts of the sentence/passage are numbered Artificial Intelligence is transforming global
1 and 6. The rest of the sentence/passage is split industries. P: However, ethical concerns about its
into four parts — P, Q, R, and S. These four parts use persist. Q: It improves efficiency in sectors
are not given in their proper order. Read the like healthcare and finance. R: There is fear of job
sentence/passage and find the most logical order loss due to automation. S: Policymakers are
of the parts between 1 and 6. debating how to regulate its growth. 6. A
Time management helps improve productivity. P: balanced approach is needed to harness AI’s
Procrastination leads to wasted hours. Q: potential.
Creating a schedule prevents delays. R: It gives (a) QSRP (b) QRSP
direction and reduces stress. S: Staying (c) RPQS (d) QPRS
disciplined is the key. 6. Success often depends
on how we manage time. Q82. Arrange the sentence parts:
(a) SRPQ (b) QPRS (A) the painter (b) has completed (C) the mural
(c) PQSR (d) R PQ S (D) beautifully
(a) ABDC (b) BACD
Q78. Choose the correct sequence to form a (c) ABCD (d) ACBD
meaningful sentence:
(A) playing (b) children (C) were (D) in the park Q83. Arrange the sentence parts:
(a) BCAD (b) BACD (A) in his bag (b) kept (C) the documents (D) he
(c) BDCA (d) CBAD (a) DCBA (b) DBCA
(c) BACD (d) DABC
Q79. Rearrange the parts of this sentence:
(A) several measures (b) have been introduced Q84. In the following question, the first and the
(C) by the government (D) to control inflation last parts of the sentence/passage are numbered
(a) CBAD (b) BACD 1 and 6. The rest of the sentence/passage is split
(c) A BC D (d) BCAD into four parts — P, Q, R, and S. These four parts
are not given in their proper order. Read the
Q80. In the following question, the first and the sentence/passage and find the most logical order
last parts of the sentence/passage are numbered of the parts between 1 and 6.
1 and 6. The rest of the sentence/passage is split Mathematics is essential in daily life. P:
into four parts – P, Q, R, and S. These four parts Calculating discounts helps in shopping. Q:
are not given in their proper order. Read the Managing time also involves math skills. R: From
sentence/passage and find the most logical order money to measurements, it is everywhere. S:
of the parts between 1 and 6. Even cooking depends on ratios and quantities. 6.
Climate change remains one of the biggest So, learning math has practical value.
challenges of our time. P: Shifting rainfall patterns (a) PRSQ (b) RPQS
Telegram (Previous year papers PDFs [SSC,Railway,DSSSB,UP SI]): https://t.me/RBE_S
YouTube (Free lectures and job updates): https://www.youtube.com/c/RBERevolutionByEducation
Download RBE application for SSC Exams Complete Preparation
(c) QRSP (d) RPSQ Q90. In the following question, the first and the
last parts of the sentence/passage are numbered
Q85. Choose the correct sequence to form a 1 and 6. The rest of the sentence/passage is split
meaningful sentence: into four parts — P, Q, R, and S. These four parts
(A) the cat (b) sleeping (C) is (D) under the table are not given in their proper order. Read the
(a) ADCB (b) ACBD sentence/passage and find the most logical order
(c) ABDC (d) ADBC of the parts between 1 and 6.

Q86. In the following question, the first and the Books have always been a source of knowledge.
last parts of the sentence/passage are numbered P: They allow us to travel across time and space.
1 and 6. The rest of the sentence/passage is split Q: We learn about different cultures and
into four parts — P, Q, R, and S. These four parts civilizations. R: Stories in books fuel our
are not given in their proper order. Read the imagination. S: From science to art, books cover
sentence/passage and find the most logical order every field. 6. Therefore, reading must be
of the parts between 1 and 6. encouraged in youth.
Rainforests are essential for biodiversity. P: Trees (a) SRPQ (b) PRQS
store carbon and release oxygen. Q: They are (c) PSQR (d) SQPR
home to countless species. R: Yet, deforestation
threatens these ecosystems. S: Some plants Q91. Rearrange the parts of this sentence:
even hold potential medical cures. 6. So, (A) the country (b) was devastated (C) by a
preserving rainforests is a global concern. powerful cyclone (D) last month
(a) QRSP (b) PQSR (a) ACBD (b) ABCD
(c) QPSR (d) SRPQ (c) BACD (d) ABDC

Q87. Arrange the sentence parts: Q92. Arrange the sentence parts:
(A) I have never (b) seen such (C) a huge (A) a scholarship (b) for higher studies (C) was
gathering (D) before today awarded to (D) the topper
(a) ABCD (b) BACD (a) DABC (b) DCAB
(c) ABDC (d) ACBD (c) DACB (d) DBCA

Q88. Rearrange the parts of this sentence: Q93. Arrange the sentence parts:
(A) to improve road safety (b) stricter traffic rules (A) took care of (b) her younger brother (C) while
(C) should be implemented (D) immediately their parents (D) were away
(a) BCAD (b) ABCD (a) BACD (b) ACBD
(c) CBAD (d) BACD (c) CBAD (d) ABCD

Q89. In the following question, the first and the Q94. In the following question, the first and the
last parts of the sentence/passage are numbered last parts of the sentence/passage are numbered
1 and 6. The rest of the sentence/passage is split 1 and 6. The rest of the sentence/passage is split
into four parts — P, Q, R, and S. These four parts into four parts — P, Q, R, and S. These four parts
are not given in their proper order. Read the are not given in their proper order. Read the
sentence/passage and find the most logical order sentence/passage and find the most logical order
of the parts between 1 and 6. of the parts between 1 and 6.
A stray dog entered the school gate. P: The An astronaut must train before going to space.
children ran away in fear. Q: It started barking P: They use a simulator to practice.
loudly. R: A peon chased it away. S: Some Q: Their body must get used to zero gravity.
teachers came out of the staff room. 6. Order was R: They learn emergency procedures.
soon restored. S: They also learn how to wear the space suit.
(a) QSPR (b) PSRQ 6. All this helps them prepare well.
(c) QPSR (d) PQSR (a) PRSQ (b) SRQP
(c) QSPR (d) RPSQ

Telegram (Previous year papers PDFs [SSC,Railway,DSSSB,UP SI]): https://t.me/RBE_S


YouTube (Free lectures and job updates): https://www.youtube.com/c/RBERevolutionByEducation
Download RBE application for SSC Exams Complete Preparation
Q95. In the following question, the first and the (c) RQPS (d) RSPQ
last parts of the sentence/passage are numbered
1 and 6. The rest of the sentence/passage is split Q97. Arrange the sentence parts:
into four parts — P, Q, R, and S. These four parts (A) water the plants (b) every morning (C) before
are not given in their proper order. Read the breakfast (D) we
sentence/passage and find the most logical order (a) DACB (b) DCAB
of the parts between 1 and 6. (c) DABC (d) DBCA
The classroom was silent. P: The teacher entered
with a test sheet. Q: Everyone looked anxious. R: Q98. In the following question, the first and the
She distributed the papers. S: The exam was last parts of the sentence/passage are numbered
about to begin. 6. The students began writing. 1 and 6. The rest of the sentence/passage is split
(a) QSPR (b) PSQR into four parts — P, Q, R, and S. These four parts
(c) PQRS (d) QPSR are not given in their proper order. Read the
sentence/passage and find the most logical order
Q96. In the following question, the first and the of the parts between 1 and 6.
last parts of the sentence/passage are numbered The Earth is facing a serious environmental crisis.
1 and 6. The rest of the sentence/passage is split P: Forests are being cut down. Q: Oceans are
into four parts — P, Q, R, and S. These four parts filling with plastic. R: The air is becoming toxic. S:
are not given in their proper order. Read the Climate change is accelerating. 6. We must act
sentence/passage and find the most logical order before it's too late.
of the parts between 1 and 6. (a) PRQS (b) QPRS
Oceans cover more than 70% of Earth’s surface. (c) PQRS (d) RQPS
P: Marine species are crucial for food chains. Q:
But plastic waste threatens marine life. R: Oceans Q99. Rearrange the parts of this sentence:
regulate global temperature and climate. S: They (A) by adopting modern methods (b) farmers can
provide food and employment to millions. 6. increase (C) agricultural production (D) effectively
Ocean conservation must be prioritized. (a) BADC (b) BCAD
(a) PRSQ (b) SRPQ (c) BACD (d) ABCD

Answer Key:

Q1 a Q2 c Q3 b Q4 a Q5 c
Q6 c Q7 d Q8 c Q9 a Q10 c
Q11 c Q12 a Q13 a Q14 d Q15 a
Q16 b Q17 b Q18 d Q19 b Q20 a
Q21 a Q22 b Q23 a Q24 c Q25 a
Q26 c Q27 d Q28 c Q29 b Q30 c
Q31 a Q32 c Q33 c Q34 d Q35 b
Q36 a Q37 b Q38 b Q39 c Q40 a
Q41 a Q42 b Q43 a Q44 d Q45 c
Q46 d Q47 a Q48 b Q49 d Q50 b
Q51 c Q52 b Q53 a Q54 d Q55 d
Q56 b Q57 c Q58 c Q59 d Q60 d
Q61 a Q62 d Q63 c Q64 d Q65
Q66 d Q67 c Q68 a Q69 d Q70 a
Q71 c Q72 a Q73 a Q74 a Q75 c
Q76 c Q77 d Q78 a Q79 c Q80 c
Q81 d Q82 c Q83 b Q84 b Q85 d
Q86 a Q87 a Q88 b Q89 c Q90 b
Q91 c Q92 b Q93 d Q94 c Q95 c
Q96 d Q97 c Q98 c Q99 d

Telegram (Previous year papers PDFs [SSC,Railway,DSSSB,UP SI]): https://t.me/RBE_S


YouTube (Free lectures and job updates): https://www.youtube.com/c/RBERevolutionByEducation
Download RBE application for SSC Exams Complete Preparation
Solution 1: Explanation describing weathering forces that further shaped
First, sentence 2 introduces the Galápagos the canyon.
Islands and their role in Darwin’s theory.
Then it naturally leads into Darwin’s specific Solution 6: Explanation
observations about finches (the unnumbered "Inflation" is the subject (R).
sentence). "Affects purchasing power" (S) follows the subject
Sentence 3 follows because it explains the and verb sequence.
implication of these observations — natural "And leads to increased cost of living," (P)
selection concept. continues the effect of inflation.
Finally, sentence 1 concludes with how these "Making it difficult for households" (Q) is the result
findings contributed to Darwin’s book. or consequence.
"To manage daily expenses" completes the idea
Solution 2: Explanation smoothly.
"The young shooter" (R) is the subject.
"was selected for the Olympic squad" (Q) is the Solution 7: Explanation
main verb phrase. Sentence 1 introduces the athlete and his effort
"due to his exceptional performance at the (training).
national trials" (S) explains the reason for Sentence 2 follows with challenges he faced
selection. (injuries).
"though he lacked international experience" (P) is Sentence 3 shows the outcome (qualifying for
a concessive clause showing a contrast, placed finals).
at the end for emphasis. Sentence 4 concludes with the effect of his
achievements (inspiring others).
Solution 3: Explanation:
"The economic slowdown" is the subject. Solution 8: Explanation
"has led to job losses" is the main verb phrase "The update" is the subject (implied in the
showing the effect. question).
"in multiple industries" specifies where the job "Enhanced encryption" (Q) is the main verb
losses occurred. phrase.
"straining household incomes and reducing "And data autonomy" (R) connects another object
consumer spending" shows the consequences of to the verb.
the job losses. "Ensuring security" (P) explains the consequence
or result.
Solution 4: Explanation "Across user devices" (S) specifies where this
Sentence 2 introduces Steve Jobs and sets the occurs.
context.
Sentence 1 follows, mentioning his foundational Solution 9: Explanation
achievement (Apple). It starts with the opening of the exhibition (1).
Sentence 4 describes the groundbreaking Followed by visitors' reaction (2).
products Apple introduced. Then describes activities during the exhibition (3).
Sentence 3 sums up with the lasting impact of his Ends with the feedback (4).
ideas on modern technology.
Solution 10: Explanation :
Solution 5: Explanation: Sentence 4: "Dr. A.P.J. Abdul Kalam worked as a
Sentence 2 introduces the formation of the Grand scientist at ISRO for forty years." (Early career)
Canyon over millions of years. Sentence 2: "He came to be known as the
Sentence 1 specifies the primary factor in the 'Missile Man of India' for his contributions."
formation—the Colorado River carving through (Known for missile development)
rock layers. Sentence 3: "In the year 2002, he became the
Sentence 4 explains the tectonic uplift that President of India." (Later career/presidency)
increased the river’s power enhancing erosion. Sentence 1: "He has inspired millions through his
Sentence 3 completes the paragraph by
Telegram (Previous year papers PDFs [SSC,Railway,DSSSB,UP SI]): https://t.me/RBE_S
YouTube (Free lectures and job updates): https://www.youtube.com/c/RBERevolutionByEducation
Download RBE application for SSC Exams Complete Preparation
speeches and writings." (Legacy and inspiration) Sentence 2 concludes with the support from
progressive thinkers.
Solution 11: Explanation: Therefore, the correct answer is 4132.
Sentence 1: Franklin hypothesized that lightning
was a form of electrical discharge. (This is the Solution 15: Explanation:
initial idea.) B (reading) is the subject (gerund).
Sentence 3: He wanted to prove this by D (fiction) modifies "books."
conducting an experiment to capture the electrical C (books) is the object noun.
charge from a thunderstorm. (He plans to test his A (is very interesting) is the predicate describing
hypothesis.) the subject.
Sentence 2: Franklin flew a kite made of a silk Hence, the answer is BDCA.
string and attached a metal key to it during a
storm. (Describes the experiment setup.) Solution 16: Explanation :
Sentence 4: As the storm approached, the kite D = The manager (subject)
was struck by lightning, and the metal key B = asked him (verb + indirect object)
conducted the electrical charge. (The experiment A = to submit (infinitive verb phrase)
result.) C = the report (direct object of the infinitive)

Solution 12: Explanation: Solution 17: Explanation:


Subject: "Everyone" (D) Q: The Ministry of Finance collects estimates
Verb: "should follow" (A) from various ministries.
Object: "traffic rules" (b) S: The Cabinet approves the final draft of the
Adverb: "strictly" (C) budget.
This follows the standard English sentence P: The finance minister presents the budget in
structure: Subject + Verb + Object + Adverb. Parliament.
Hence, the correct sequence is DABC. R: The budget is passed by both houses of
Parliament.
Solution 13: Explanation: Therefore, the correct sequence is QSPR.
The Renaissance ushered in a new era that
questioned the monopoly of the Roman Catholic Solution 18: Explanation:
Church on knowledge and spiritual guidance. 2. The excavation of Pompeii provided a
It encouraged the growth of secularism and remarkably well-preserved glimpse into life in
individualism, drawing inspiration from the ancient Rome.
classical works of ancient Greece and Rome. 4. Unlike other ruins, Pompeii remained frozen in
The advancements in science led by figures like time due to Mount Vesuvius' sudden eruption.
Copernicus, Galileo and Descartes also 3. When the eruption occurred, a thick layer of hot
questioned its established views of the universe. ash and pumice rapidly buried the town.
The invention of the printing press further 1. It cut off oxygen and prevented the usual decay
accelerated this shift, making books more caused by air, moisture, and bacteria.
accessible and promoting literacy and individual Therefore, the correct answer is 2431.
thinking.
This order logically flows from the general Solution 19: ✅ Correct Answer: PRSQ
historical context to specific developments in 🧠 Logical Arrangement Breakdown:
thought an d technology. Let’s reconstruct the passage using PRSQ :
1. The invention of the wheel changed the world.
Solution 14: Explanation: ➡ Introduces the central idea.
Sentence 4 introduces the evolution and
P. Transport became easier. ➡ Immediate impact
transformation of feminism.
of the wheel.
Sentence 1 describes the beginnings with
women’s suffrage and expansion to other issues. R. Travel became faster. ➡ Builds on the idea of
Sentence 3 explains the conservative opposition easier transport.
and fears. S. Roads improved quickly. ➡ Infrastructure
evolved to support wheeled movement.
Telegram (Previous year papers PDFs [SSC,Railway,DSSSB,UP SI]): https://t.me/RBE_S
YouTube (Free lectures and job updates): https://www.youtube.com/c/RBERevolutionByEducation
Download RBE application for SSC Exams Complete Preparation
Q. Machines could now be built. ➡ Long-term The team members, after the meeting, went to a
technological impact. restaurant.
6. It was a great invention. ➡ Concludes with a This sequence maintains proper subject–time–
summarizing statement. action–place order.

Solution 20: ✅ Correct Sequence: SQPR Solution 23: ✅ Correct Sequence: RPSQ
🔍 Step-by-Step Logical Flow: Sentence 1: The invention of the telescope
changed astronomy. – This introduces the impact
Sentence 1: Exercise keeps the body fit and
of the telescope.
active. – This introduces the general idea of
R: Earlier, knowledge was limited to nearby
exercise and its benefits.
celestial bodies. – Provides the context before
S: Physical activity strengthens the heart and
the invention—limited scope.
muscles. – First, we elaborate on the physical
P: Scientists could observe distant galaxies. –
benefits of exercise.
Shows the new capability brought by the
Q: Regular workouts reduce stress levels. – Then
telescope.
we shift to the mental health benefits , starting
S: The new tool opened a whole new universe. –
with stress reduction.
Emphasizes the transformational impact of the
P: It also boosts mental health. – This reinforces
invention.
the previous point and broadens the mental
Q: They discovered stars not visible to the naked
health angle.
eye. – Adds a specific discovery enabled by the
R: Walking, cycling, and yoga are easy options. –
telescope.
Now we give practical examples of how to
Sentence 6: Astronomy became a precise
incorporate exercise.
science. – Concludes with the result of all these
Sentence 6: So, exercise should be part of our
advancements.
routine. – A logical conclusion based on all the
benefits and examples.
Solution 24: ✅ Correct Answer: BCDA
Solution 21: ✅ Correct Sequence: S Q R P Wildlife sanctuaries have been established across
the country to protect endangered species .
Sentence 1: Artificial Intelligence is reshaping
industries. – This introduces the broad impact of
AI. Solution 25: ✅ Correct Answer: CDAB
S: From manufacturing to medicine, its impact is 👉 Correct arrangement: Many guests were
visible. – Expands the scope by showing AI’s invited to the event , which forms a meaningful
presence across sectors. sentence.
Q: In healthcare, it helps in quick diagnosis. –
Focuses on a specific domain with a clear benefit. Solution 26: The most logical order of the parts
R: AI systems reduce human error in complex between 1 and 6 is RQSP .
calculations. – Adds another technical advantage Passage Arrangement
relevant to multiple fields. Riya enjoys learning different languages.
P: It also improves customer service through R: She knows some German.
automation. – Concludes the examples with a Q: Her Spanish is improving.
business-oriented benefit. S: She uses an app to practice daily.
Sentence 6: Its future role will only expand. – A P: She attends French classes.
fitting conclusion that looks ahead. She wants to be a polyglot.
Explanation
Solution 22: ✅ Correct Sequence: BACD R shows existing knowledge of German, intro
B: the team members – This is the subject of the ducing her language background.
sentence. Q logically follows, indicating improvement in
A: after the meeting – This gives the time Spanish, a nother language she's learning.
context. S explains her practice method, relevant after disc
C: went – This is the verb showing the action. ussing both German and Spanish.
D: to a restaurant – This tells us the destination. P adds French classes, rounding out her efforts in
Final sentence:
Telegram (Previous year papers PDFs [SSC,Railway,DSSSB,UP SI]): https://t.me/RBE_S
YouTube (Free lectures and job updates): https://www.youtube.com/c/RBERevolutionByEducation
Download RBE application for SSC Exams Complete Preparation
multiple languages before the conclusion. (b) many passengers – This is the subject of the
sentence.
Solution 27: Correct Answer : BACD (A) were injured – This is the verb phrase, directly
📌 Rearranged Sentence : following the subject.
The farmers, despite poor rainfall, managed to (C) in the accident – This prepositional phrase
harvest a good crop. tells us where the injury occurred.
Logical Flow : Subject → Condition → Action → (D) yesterday – This gives the time reference,
Object. i.e. → B (the farmers) → A (despite poor placed naturally at the end.
rainfall) → C (managed to harvest) → D (a good ✔ Final Sentence:
crop) Many passengers were injured in the accident
yesterday.
Solution 28: ✅ Correct Answer : ABDC It follows a clear and logical order: Subject →
📜 Reconstructed Sentence : Verb → Place → Time
The guests arrived early at the party.
Sequence Logic : Subject → Verb → Time → Solution 33: ✅ Correct Answer: QPRS
Place. i.e. → A (the guests) → B (arrived) → D 🔍 Logical Flow Analysis:
(early) → C (at the party) Opening (1) : The Earth's climate has changed
over centuries. → This sets up a general
Solution 29: ✅ Correct Answer : DACB historical context.
Logical Flow: Q: Historically, it changed due to natural causes.
Subject + Verb + Object + Adverb → He + → Smooth continuation from the opening —
opened + the door + suddenly → This follows explains past reasons.
natural English sentence structure. P: But current warming is caused by human
actions. → Contrast introduced with “But” — shifts
Solution 30: ✅ Correct Answer : BCDA from natural to human causes.
Sentence Structure Insight: R: Burning fossil fuels increases greenhouse
Subject (b) → Verb (C) → Reason Clause (D + A) gases. → Specific example of human action
→ the workers → went on strike → after the causing warming.
management rejected their demands S: Deforestation worsens the problem. → Another
example — complements R logically.
Ending (6): This calls for urgent action. →
Solution 31: ✅ Correct Answer : QPRS
Concludes with a call to respond to the human-
📜 Reconstructed Passage :
driven crisis.
1. Recycling helps in environmental protection. Q:
Recycling paper saves trees and water. P: It
Solution 34: ✅ Correct Answer: PQRS
reduces the amount of waste in landfills. R: It also
Logical Flow:
cuts down pollution caused by burning waste. S:
P explains how bees help in pollination.
But awareness among people is still low. 6. Public
Q shows the result of that action—growth of fruits
campaigns can improve participation.
and seeds.
Why QPRS Works Best:
R introduces the consequence of losing bees.
Q → P → R builds a layered case for recycling
S highlights the current threat —declining
with increasing environmental benefits.
populations.
S introduces the challenge, making 6 a natural
6 concludes with a call to action for food security.
solution.
Follows a benefit-benefit-impact-problem-solution
Solution 35: ✅ Correct Answer: QPRS
structure, ideal for informative writing.
🔍 Step-by-Step Analysis:
Solution 32: ✅ Correct Answer: BACD Opening Statement (1) :
Sets a general tone—now we need a logical
📜 Reconstructed Sentence :
build-up.
Many passengers were injured in the accident
Best Starting Link (Q) :
yesterday.
Chronological anchor—traces art’s journey
🔍 Explanation:
through time.
Telegram (Previous year papers PDFs [SSC,Railway,DSSSB,UP SI]): https://t.me/RBE_S
YouTube (Free lectures and job updates): https://www.youtube.com/c/RBERevolutionByEducation
Download RBE application for SSC Exams Complete Preparation
Next Logical Idea (P) : P: Explains the first consequence—unequal in
Explains art’s expressive power after its evolution. ternet access limits education.
Then (R) : Q: Specifies the affected groups, particularly rur
Deepens the idea—art as a medium for societal al and underserved populations.
commentary. R: Adds further impact on skill developmen t and
Followed by (S) : employment opportunities.
Extends the impact—art as a tool for activism. S: Summarizes the overarching imbalance in te
Closing Statement (6) : chnology benefits distribution.
Concludes with a summarizing insight. 6: Concludes with the necessity of bridging the
digital gap for inclusive growth.
Solution 36: ✅ Correct Answer: QPRS
🔍 Step-by-Step Analysis: This logical flow builds from problem
Opening Statement (1) : introduction to specific consequences, then
Sets the stage for dual impact—on individuals summary, and lastly a solution-oriented
and enterprises. conclusion, making PQRS the most coherent
First Logical Link (Q) : sequence .
Begins with consumer impact—direct
consequence of inflation. Solution 40: ✅ Correct Answer : CDBA
Next (P) : "The construction company promised to
Follows naturally—shows how inflation affects complete the project by the end of the year."
producers. ✅ Here's the breakdown:
Then (R) : (C) the construction company → subject
Introduces response mechanism—policy-level (D) promised to → verb phrase
reaction. (b) complete the project → action
Followed by (S) : (A) by the end of the year → time frame
Wraps up with broader economic implications.
Closing Statement (6) : Solution 41: ✅ Correct Sequence :
Summarizes the need for control and balance. ACBD → The winner was declared yesterday.

Solution 37: ✅ Correct Answer: 1-4-3-2 Solution 42: ✅ Correct Sequence :


🔍 Step-by-Step Analysis: BCAD → The students accepted his advice
Opening Statement (1) : happily.
Introduces the topic clearly and sets the context. 🔍 Point-wise Explanation
Definition (4) : Subject Identification
Explains what cybersecurity actually involves. (b) the students → Subject of the sentence
Threat Description (3) : Verb Placement
Illustrates the danger that cybersecurity aims to (C) accepted → Main verb follows the subject
prevent. Object Placement
Preventive Measures (2) : (A) his advice → Object of the verb “accepted”
Offers solutions and concludes the paragraph Adverb Placement
logically. (D) happily

Solution 38: ✅ Correct Answer: R → P → Q → S Solution 43: "The bus arrived at the stop on time .
🔍 Step-by-Step Logic: " This sequence logically and grammatically
connects the subject, verb, place, and time.

Solution 39: The correct order for the sentence Solution 44: To make it grammatically correct, it
segments is PQRS. should read "They served dinner late," which is C
AB D .
Chronological Logic Explanation
1: Introduces the main theme about the digital
divide deepening social inequalities.
Telegram (Previous year papers PDFs [SSC,Railway,DSSSB,UP SI]): https://t.me/RBE_S
YouTube (Free lectures and job updates): https://www.youtube.com/c/RBERevolutionByEducation
Download RBE application for SSC Exams Complete Preparation
Solution 45: ✅ Correct Sequence: Q R S P Solution 49: The correct answer is " PQRS ." The
🧠 Logical Arrangement Breakdown: most logical sequence is: P (They bring together
Let’s reconstruct the passage step-by-step: people of different beliefs), Q (Each region
1. The spacecraft-initiated descent towards the celebrates with its unique customs), R (Shared
red planet. ➡ This sets the scene for a technical celebrations promote tolerance and joy), S (They
also preserve traditional art forms). This order
landing sequence.
connects the idea of unity, diversity in customs,
Q. The heat shield protected it from atmospheric
the benefits of shared celebration, and the
entry. ➡ First technical step during descent.
preservation of culture, making the passage clear
R. Soon after, the parachutes deployed
and cohesive before the concluding sentence
successfully. ➡ Follows logically after heat shield
about strengthening the social fabric.
activation.
S. Engineers monitored every second of the
Solution 50: ✅ Correct Answer: PSQR
landing sequence. ➡ Human involvement during
🧠 Logical Arrangement Breakdown:
the critical phase.
Let’s reconstruct the passage using PSQR :
P. The data gathered during this process will aid
1. The United Nations was formed after the
future missions. ➡ Reflects on the outcome and
Second World War. ➡ Introduces the origin of the
future implications.
UN.
6. This landing marked a historic step in
P. It was established to prevent future global
interplanetary exploration. ➡ Concludes with a
conflicts. ➡ Explains the founding purpose.
broader significance.
S. Its peacekeeping missions have played a vital
role in conflict zones. ➡ Shows how it fulfilled that
Solution 46: The correct answer is " PQRS." The
purpose.
logical sequence is: P (Farmers work hard
despite unpredictable weather), Q (They grow Q. Over the decades, it has expanded its roles. ➡
crops that feed the nation), R (Yet, they face Transition to broader evolution.
several challenges), S (Modern technology can R. Now it addresses issues like climate change
support better yield). This order follows a logical and human rights. ➡ Specific examples of its
flow: describing the farmers’ hard work, their expanded scope.
contribution, the challenges they face, and the 6. Despite criticisms, the UN remains a key
role of modern technology in supporting them. It international body. ➡ Concludes with a balanced
leads coherently to the concluding sentence perspective.
about ensuring farmer welfare.
Solution 51: The correct answer is "BCAD ."
Solution 47: The correct answer is " SQRP." The When arranged in this order, the sentence reads:
sentence in correct order is: "Advances in satellite " The winner was announced yesterday
imaging have transformed environmental evening." This follows proper English sentence
monitoring, enabling accurate climate modelling structure with subject (the winner), verb (was
and disaster prediction systems across diverse announced), and time phrase (yesterday
geographic regions." This structure provides evening). The other arrangements do not form
logical flow and connects all the elements grammatically correct sentences.
meaningfully.
Solution 52: 1 ⃣ BACD → B → A → C →
Solution 48: The correct answer is "2-1-4-3." The D
logical sequence starts with the general "The instructions read before filling carefully"
introduction about coral reefs (2), describes how ❌ Incorrect word order. Sounds awkward and
they are formed (1), explains the problem they grammatically off.
face (4), and ends with the cause of the problem 2 ⃣ DABC → D → A → B → C
(3). This arrangement creates a clear and
"Carefully read the instructions before filling"
coherent paragraph, moving from description to
✅ Perfect! This is clear, grammatically correct,
problem and its cause.
and natural.
3 ⃣ BCAD → B → C → A → D
Telegram (Previous year papers PDFs [SSC,Railway,DSSSB,UP SI]): https://t.me/RBE_S
YouTube (Free lectures and job updates): https://www.youtube.com/c/RBERevolutionByEducation
Download RBE application for SSC Exams Complete Preparation
"The instructions before filling read carefully" Solution 59: The correct sequence is "the
❌ Disjointed and confusing. Doesn’t follow logical villagers repaired the damaged bridge with the
sentence flow. help of officers before the heavy rains." This order
4 ⃣ BDCA → B → D → C → A (B-C-A-D) first identifies the doer ("the villagers"),
"The instructions carefully before filling read" then their action ("repaired the damaged bridge"),
❌ Word order is jumbled. Meaning is lost. followed by "with the help of officers," and sets it
in time with "before the heavy rains." This
arrangement presents the sentence logically and
Solution 53: The logical sequence is QPSR: "I
meaningfully.
saw many wild animals there. The lions were
resting under trees. The elephants were eating
Solution 60: The correct order is "the headmaster
sugarcane. Monkeys were swinging branches."
praised the student for his punctuality." (A: the
This order guides the reader through the zoo
headmaster, C: praised, B: for his punctuality, D:
observations most naturally, culminating in the
the student). This arrangement gives logical
closing statement about the experience.
structure: subject, verb, object, reason. Options
with other orders do not convey the intended
Solution 54: The most logical order is PQRS:
meaning or proper sentence flow.
"Writing reports is easier. Data is stored safely.
Meetings happen online. Calculations done fast."
Solution 61: The sequence BACD gives the
This order flows naturally, listing computer
meaningful sentence: "India has a long and rich
benefits before concluding that offices depend on
cultural history." Placing "India" at the beginning
them daily.
aligns the subject first, followed by the correct
arrangement of adjectives and the object,
Solution 55: The correct arrangement is ABCD:
resulting in a grammatically complete statement.
"Rising inflation causes difficulties for middle-
class families everywhere."
Solution 62: The most logical arrangement is
QPRS: Q (past inventions were once fictitious
Solution 56: The most logical order is PRSQ: "It
ideas) connects sci-fi with reality, P (inspires
is faster than traditional letters. Emails allow file
innovation) follows as an effect, R (encourages
sharing and formal contact. However, they may
thinking beyond limits) logically extends
lack personal touch. Instant messaging is also
innovation, and S (stories explore societies) ties
gaining popularity." This flow introduces benefits
into future possibilities before the conclusion
and limitations before discussing alternatives.
(real-world influence). This sequence maintains
coherence and develops the core theme
Solution 57: The most logical order is PQRS: "It
stepwise.
is available in abundance. This makes it a
sustainable option. Unlike fossil fuels, it doesn’t
Solution 63: Q introduces the triangle as a basic
emit harmful gases. Also, it reduces dependency
shape — a natural continuation of sentence 1. R
on non-renewable sources." This sequence
adds a factual detail — every triangle has three
presents the advantages in a natural flow: starting
angles. P elaborates on the nature of those
with availability and sustainability, highlighting
angles — equal or different. S concludes the
environmental benefits, and ending with reduced
technical explanation — the sum of angles is
dependency before the final sentence about
always 180°. 6 wraps up with the importance of
government investment.
triangles in geometry.
Solution 58: The best sequence is PRQS: P
Solution 64: The correct sequence is BACD:
(describes the contents), R (shows to friends), Q
"Many students prefer to go abroad for higher
(uses in class), S (describes its appearance). This
education these days." This arrangement puts the
gives a logical flow: he opens the box and sees
subject ("many students") first, followed by the
what's inside, shows it to his friends, uses it for
verb ("prefer to go abroad"), the purpose ("for
class, and then mentions its material and shine
higher education"), and the time phrase ("these
before ending with his fondness for the box.
days"), forming a grammatically and contextually
Telegram (Previous year papers PDFs [SSC,Railway,DSSSB,UP SI]): https://t.me/RBE_S
YouTube (Free lectures and job updates): https://www.youtube.com/c/RBERevolutionByEducation
Download RBE application for SSC Exams Complete Preparation
meaningful sentence. slowly."

Solution 65: Let’s evaluate each option: Solution 71: The most logical order between 1
Option A (BDCA): "We cricket every week play" and 6 is P RQ S .
— Incorrect word order; sounds unnatural.
Option B (BCDA): "We play cricket every week" Solution 72: The logical order is PSQR. First, it is
— Perfect! Grammatically correct and natural. explained that photosynthesis requires light (P),
Option C (CADB): "Play every week cricket we" followed by the idea that no food is produced
— Incorrect; subject is misplaced. Option D without it (Q). Then, the effect on leaves (R) is
(DACB): "Cricket every week play we" — described, and finally, the outcome of a plant
Awkward and grammatically flawed. dying slowly (S) completes the flow, which
Correct Answer: Option B (BCDA) naturally leads to the concluding statement that
Final Sentence: We play cricket every week. sunlight is very important.

Solution 66: The correct order is DABC: "Rohit Solution 73: The logical order is QPRS. First, Q
met his friend at the station." The subject ("Rohit") establishes that social media helps in sharing
comes first, followed by the verb ("met"), the views widely, followed by P which explains how
object ("his friend"), and the adverbial phrase ("at quickly posts can spread. R then introduces the
the station"). This order forms a grammatically issue of speedy misinformation, and S gives the
correct and meaningful sentence. consequence. This sequence logically builds the
argument towards the need for digital literacy
Solution 67: The correct sequence is ABCD: "My stated at the end.
mother bought a beautiful saree yesterday." This
order places the subject ("my mother"), the verb Solution 74: The most logical sequence is RPSQ:
("bought"), the object ("a beautiful saree"), and Spoken language likely developed alongside tool
the time expression ("yesterday") in a clear, use (R), then early humans used symbols to
grammatically correct structure. The other convey meaning (P), then writing systems
arrangements do not form a logical or emerged (Q), and finally, languages diversified
grammatically accurate sentence. across regions (S). This order properly traces the
chronological development of language.
Solution 68: The correct sequence is CBAD: "Mr.
Sharma reached to the office at 9 am." This order Solution 75: The most logical sequence is RPSQ:
places the subject ("Mr. Sharma") first, followed First, the brain filters and encodes information
by the verb ("reached"), then the prepositional efficiently (R), then short-term memory stores
phrase ("to the office"), and finally the time ("at 9 data for immediate use (P), next, information can
am"), forming a clear and grammatically correct be transferred to long-term memory with
sentence. The other arrangements do not form a repetition (S), and finally, long-term memory holds
natural or coherent sentence. information for an extended period (Q). This
sequence follows how memory functions in
Solution 69: The correct order is PSRQ: stages from processing to storage.
“Postcolonial literature interrogates historical
narratives, rooted in indigenous experience, while Solution 76: ✅ Correct Answer: PQRS
reclaiming suppressed voices and offers 📘 Logical Sequence Breakdown:
alternative perspectives.” This arrangement 1. It started raining in the evening. ➡ This sets
logically introduces the main action (P),
the scene — a sudden rain.
elaborates with context (S), explains an ongoing
P: People ran to shelters. ➡ Immediate human
process (Q), and concludes with an outcome (R).
reaction to rain.
Other sequences disrupt the logical flow of the
sentence. Q: Water collected on roads. ➡ Natural
consequence of continued rainfall.
Solution 70: The sequence D ACB gives the R: Traffic moved slowly. ➡ Result of
meaningful sentence: "He opened the door waterlogging and poor visibility.
Telegram (Previous year papers PDFs [SSC,Railway,DSSSB,UP SI]): https://t.me/RBE_S
YouTube (Free lectures and job updates): https://www.youtube.com/c/RBERevolutionByEducation
Download RBE application for SSC Exams Complete Preparation
S: Many opened their umbrellas. ➡ Another Solution 81: ✅ Correct Answer: QPRS
human response, slightly delayed or 🔍 Logical Arrangement:
simultaneous. 1. Artificial Intelligence is transforming global
6. It was a heavy shower. ➡ Concluding remark industries. Let’s now arrange the middle parts
summarizing the intensity. logically:
Q: It improves efficiency in sectors like healthcare
Solution 77: ✅ The correct sequence is R PQ S and finance. 🔸 Introduces the positive impact of
Correct Option: 4) RPQS AI.
Order: P: However, ethical concerns about its use
Time management helps improve productivity. persist. 🔸 Introduces a contrasting concern .
R: It gives direction and reduces stress. R: There is fear of job loss due to automation. 🔸
P: Procrastination leads to wasted hours. Adds a specific negative consequence .
Q: Creating a schedule prevents delays. S: Policymakers are debating how to regulate its
S: Staying disciplined is the key. growth. 🔸 Leads toward solutions and
Success often depends on how we manage time.
governance .
Why: Start with the general benefit (R), point out
6. A balanced approach is needed to harness AI’s
the problem caused by poor time use (P), offer a
potential. 🔸 Concludes with a call for balance ,
practical solution (Q), then state the key principle
tying all points together.
(S) before the concluding sentence.

Solution 82: ✅ Correct Answer: ABCD


Solution 78: ✅ Correct Answer: BCAD
🧠 Sentence Arrangement:
🧠 Sentence Formation:
Let’s break down the parts:
Let’s arrange the parts logically:
(A) the painter → Subject
(b) children → Subject
(b) has completed → Verb (Present Perfect)
(C) were → Helping verb
(C) the mural → Object
(A) playing → Main verb (present participle)
(D) beautifully → Adverb (modifies the verb)
(D) in the park → Prepositional phrase (place)
🔁 Logical Order: ➡ Subject → Verb → Object
➡ Final sentence: Children were playing in the
→ Adverb ✅ Final sentence: The painter has
park.
completed the mural beautifully.
Solution 79: Final sentence:
A + B + C + D = several measures have been Solution 83: ✅ Correct Answer: D BCA
introduced by the government to control Final Sentence: He kept the documents in his
inflation ✅ bag.
Correct Option: ABCD ✔
Solution 84: ✅ Correct Answer: RPSQ
Solution 80: Step 1: After 1 (introductory 🧠 Logical Arrangement:
sentence) Let’s reconstruct the passage step-by-step:
Best link is R: Its impacts are visible across the 1. Mathematics is essential in daily life. ➡ This
globe , because it generalizes effects before introduces the general idea.
giving specific examples. R. From money to measurements, it is
Step 2: Examples of impacts everywhere. ➡ Broadens the scope and supports
P (rainfall & agriculture) the opening statement.
Q (melting glaciers & water supplies) P. Calculating discounts helps in shopping. ➡
S (rising sea levels & coastal areas) Gives a specific, relatable example.
These are concrete impacts, and they flow S. Even cooking depends on ratios and
smoothly after R. quantities. ➡ Another practical example,
Step 3: Logical order continuing the theme.
So, the correct order is:
Q. Managing time also involves math skills. ➡
R→P→Q→S
Adds a final example, rounding out the variety of
applications.
Telegram (Previous year papers PDFs [SSC,Railway,DSSSB,UP SI]): https://t.me/RBE_S
YouTube (Free lectures and job updates): https://www.youtube.com/c/RBERevolutionByEducation
Download RBE application for SSC Exams Complete Preparation
6. So, learning math has practical value. ➡ 1. A stray dog entered the school gate. Q. It
Concludes with a logical summary. started barking loudly. P. The children ran away in
fear. S. Some teachers came out of the staff
Solution 85: Correct Answer: (D) ADBC room. R. A peon chased it away. 6. Order was
Explanation: soon restored.
ADBC = The cat is sleeping under the table.
This sequence forms a grammatically correct and Solution 90: ✅ Correct Answer: Option 2 –
meaningful sentence. PRQS
🔍 Logical Sequence Breakdown:
Solution 86: Correct Answer: ✅ Option 1 – 1. Books have always been a source of
QRSP knowledge. This introduces the general idea of
🔍 Logical Sequence Breakdown: books as valuable tools.
1. Rainforests are essential for biodiversity. P: They allow us to travel across time and
This introduces the topic and sets the tone. space. → A broad imaginative benefit — sets the
Q: They are home to countless species. → tone.
Expands on biodiversity by highlighting species R: Stories in books fuel our imagination. →
richness. Deepens the idea of imagination sparked by
R: Yet, deforestation threatens these books.
ecosystems. → Introduces the problem or threat Q: We learn about different cultures and
to biodiversity. civilizations. → Shifts from imagination to
S: Some plants even hold potential medical educational value.
cures. → Adds value to conservation by S: From science to art, books cover every
mentioning medicinal potential. field. → Concludes the examples with a
P: Trees store carbon and release oxygen. → comprehensive scope.
Highlights environmental benefits, reinforcing the 6. Therefore, reading must be encouraged in
need for preservation. youth. → Logical conclusion based on all the
6. So, preserving rainforests is a global benefits listed.
concern. → Concludes with a call to action based
on all the points above. Solution 91: ✅ Correct Answer: Option 3 –
BACD
Solution 87: Correct Answer: ✅ Option 1 – 🧠 Explanation:
ABCD To form a grammatically correct and meaningful
🔍 Sentence Arrangement: sentence, we need to follow the logical structure:
Let’s piece it together step by step: Subject: the country
(A) I have never → Subject + auxiliary verb Verb: was devastated
(b) seen such → Verb + intensifier Cause: by a powerful cyclone
(C) a huge gathering → Object Time: last month
(D) before today → Time expression
Solution 92: ✅ Correct Arrangement: DCAB
Solution 88: ✅ Correct Answer: Option 2 – ➡ The topper was awarded a scholarship for
ABCD higher studies.
🔄 Rearranged Sentence: This structure maintains:
(A) to improve road safety (b) stricter traffic rules Subject → The topper
(C) should be implemented (D) immediately Verb phrase → was awarded
👉 Final sentence: To improve road safety, Object → a scholarship
stricter traffic rules should be implemented Purpose → for higher studies
immediately.
Solution 93: ✅ Correct Answer: Option 4 –
Solution 89: ✅ Correct Answer: Option 3 – ABCD
QPSR 🔍 Sentence Parts:
🔄 Logical Sequence Breakdown: (A) took care of
Telegram (Previous year papers PDFs [SSC,Railway,DSSSB,UP SI]): https://t.me/RBE_S
YouTube (Free lectures and job updates): https://www.youtube.com/c/RBERevolutionByEducation
Download RBE application for SSC Exams Complete Preparation
(b) her younger brother Q: But plastic waste threatens marine life. 🔹
(C) while their parents Introduces the problem → sets up the need for
(D) were away action.

Solution 94: ✅ Correct Answer: Option 3 – Solution 97: 🧠 Rearranged Sentence:


QSPR We water the plants every morning before
🔍 Explanation: breakfast.
This sequence follows a progressive and 🔍 Step-by-Step Logic:
realistic training flow : Subject (D): we → starts the sentence.
Q – Physical adaptation is the first step (zero Verb Phrase (A): water the plants → action
gravity). performed by the subject.
S – Then comes learning to handle the space Time (b): every morning → tells us when the
suit. action happens.
P – Followed by simulator-based practice. Sequence (C): before breakfast → adds a
R – Finally, mastering emergency procedures. specific time frame.
Each step logically builds toward the concluding
sentence: All this helps them prepare well. Solution 98: 1. The Earth is facing a serious
environmental crisis. This sets the stage for
Solution 95: ✅ Correct Answer: Option 3 – listing multiple environmental threats.
PQRS 🧩 Middle Parts:
🧠 Logical Arrangement: P: Forests are being cut down. 🔹 Begins with
1. The classroom was silent. P. The teacher deforestation — a major crisis.
entered with a test sheet. Q. Everyone looked Q: Oceans are filling with plastic. 🔹 Adds
anxious. R. She distributed the papers. S. The marine pollution — another global issue.
exam was about to begin. 6. The students began R: The air is becoming toxic. 🔹 Introduces air
writing.
pollution — escalating the concern.
S: Climate change is accelerating. 🔹
Solution 96: ✅ Correct Answer: Option 4 –
Culminates with the overarching threat — climate
RSPQ
change.
🔍 Logical Sequence Analysis:
6. We must act before it's too late. A natural
Let’s build the passage step-by-step from 1 to 6 : conclusion after listing the dangers.
1. Oceans cover more than 70% of Earth’s
surface. This introduces the vastness and Solution 99: 🧠 Explanation:
importance of oceans.
Let’s analyze the logical flow:
🧩 Middle Parts:
(A) sets the condition or method → By adopting
R: Oceans regulate global temperature and modern methods
climate. 🔹 Scientific significance → expands on (b) introduces the subject and action → farmers
their planetary role. can increase
S: They provide food and employment to (C) specifies what is being increased →
millions. 🔹 Socioeconomic value → connects agricultural production
oceans to human livelihood. (D) adds the manner of increase → effectively
P: Marine species are crucial for food chains. This order maintains clarity, coherence, and
🔹 Ecological importance → zooms in on grammatical correctness .
biodiversity.

Telegram (Previous year papers PDFs [SSC,Railway,DSSSB,UP SI]): https://t.me/RBE_S


YouTube (Free lectures and job updates): https://www.youtube.com/c/RBERevolutionByEducation
Download RBE application for SSC Exams Complete Preparation

SSC Steno 2025 T-1 Reading Comprehension All 80 Questions with Detailed Solution and Answer key

Q1-5. Read the passage carefully and answer following is NOT cited as a challenge for small
the following questions: retailers?
The rise of e-commerce has dramatically (a) Scr u t iny
reshaped retail landscapes around the world. (b) High utility bills
Consumers now enjoy unprecedented (c) Dependence on delivery services
convenience, i.e., with only a click, we can (d) Environmental scrutiny
compare prices, read reviews, and deliver goods
directly to their doors. Brick-and-mortar stores, in The tone of the passage can be best
contrast, must contend with higher overheads— described as:
rent, utilities, and staffing—that online retailers (a) Alarmist (b) Analytical
often avoid. Many physical shops have adopted (c) Enthusi astic (d) Sarcastic
“omnichannel” strategies to stay competitive,
integrating their online and in-store experiences. Which inference is best supported by the
For example, customers might order online and passage?
pick up in-store or use a mobile app to check real- (a) All retailers will eventually go fully online.
time stock levels before visiting. However, some (b) Personalized experiences are key to futur e
challenges still persist for small retailers because retail success.
they lack the technical expertise or capital to (c) Third-party delivery is always unprofitable.
develop user-friendly websites, and excessive (d) Packaging waste has minimal impact on
reliance on third-party delivery services can erode consumers
profit margins. Moreover, the environmental
impact of rapid shipping—packaging waste and Q6-10. Read the following passage and
increased carbon emissions—is increasingly answer the questions based on the passage:
under scrutiny. As e-commerce continues its
ascent, the most successful retailers will combine Wildlife conservation has increased in importance
digital innovation with sustainable practices and a since governments and NGOs have realized that
personalized customer experience. locals are vital guardians of the environment.
Instead of mandating top-down orders, successful
What is the primary purpose of this passage? initiatives recruit villagers to protect threatened
(a) To advocate for brick and mortal retail over e- species, serve as informants, and generate
commerce. income from ecotourism. In other parts of Africa
(b) To describe how omnichannel strategies are and Asia, community rangers are taught tracking
best and first aid, while women's organizations
(c) To examine both opportunities and challenges produce handicrafts out of sustainable forest
in modern retail products. These programs not only restrict
(d) To explain why delivery services are harmful poaching but also create ownership and pride in
the maintenance of biodiversity. There are still
In context, the word “overheads” most nearly obstacles, however: low funding levels, human-
means: wildlife conflict, and uncertain land-use rights can
(a) Profits (b) Fixed expenses undo success. Continued dialogue between
(c) Revenues (d) Discounts stakeholders and equitable benefit-sharing
mechanisms are essential in guaranteeing that
According to the passage, which of the conservation outcomes are directed to community
well-being.
Telegram (Previous year papers PDFs [SSC,Railway,DSSSB,UP SI]): https://t.me/RBE_S
YouTube (Free lectures and job updates): https://www.youtube.com/c/RBERevolutionByEducation
Download RBE application for SSC Exams Complete Preparation
when properly planned, public transportation
What is the primary focus of the passage? promotes
(a) Criticize eco-tourism tor harming wildlife economic efficiency, social mobility, and a cleaner
(b) Describe the role of communities in city.
conservation
(c) Explain how poachers operate What is the central idea of the passage?
(d) Advocate for stricter government enforcement (a) Public transport is out d ated in modern cities
(b) Investment i n roads to private vehicles is
In context, the phrase “top-down regulations” more practical
most nearly means: (c) A strong public transport system is essential
(a) Ru l es created by local villagers for sustainable ang inclusive cities
(b) P olicies imposed by higher authorities (d) Public transport IS mostly beneficial for
(c) Guidelines developed by scientists tourists
(d) Suggestions from tourists
According to the passage, integrated fare
According to the passage, which activity systems :
helps villagers earn income? (a) Create financial burden to low-income people
(a) Poaching endangered species (b) Make switching between travel modes easier
(b) Crafting handicrafts from sustainable products (c) Are only for long-distance travel
(c) Clearing forests for agriculture (d) Encourage use of private vehicles
(d) Mining minerals
As per the passage, what is needed to
Which challenge is NOT mentioned as maintain strong public transport?
affecting community conservation? (a) Minimal re g ulation
(a) Human-wildlife conflicts (b) Frequent tech upgrades
(b) Inadequate funding (c) Sufficient funding and long-term planning
(c) Climate change (d) Higher taxes on private vehicles
(d) Unclear land-use rights
How does public transport contribute to
The tone of the passage is best described as: equity?
(a) Pessimistic (b) Promotional (a) By keeping fares high
(c) Objective (d) Sarcastic (b) By focusing only on commercial zones
(c) By connecting low-income communities with
Q11-15. Read the following passage and wealthy areas,
answer the questions based on the passage: (d) By offering free rides on weekends

Good public transport networks are the pillars of What is the tone of the passage?
urban sustainability. Buses, trams, and metro (a) Skeptical (b) Analytical ang supportive
lines can transport big groups of passengers (c) Neutral (d) Aggressive
without relying so much on private cars-—and
therefore eliminating traffic snarls and emissions. Q16-20. Read the following passage and answer
integrated fares across modes of travel make it the questions based on the passage:
easy to move around: one ticket could pay for Universal immunization campaigns have
bus, train, and bike-share travel. Spending money eradicated or controlled fatal conditions such as
on exclusive smallpox and polio. Mass immunization relies on
lanes and priority signaling also makes them cold-chain availability: vaccines have to be stored
faster and more dependable. Nonetheless, at precise temperatures from manufacturing
constructing and retaining such infrastructure through inoculation. Solar-powered refrigerators
requires capital and long-term vision. Equity and insulated containers are lifelines for remote
issues are also important: lines need to serve locations. Door-to-door interactions by community
low-income communities along with commercial health workers educate households regarding
hubs so that all citizens have a benefit. Finally, dosage routines and myths. Vaccine hesitancy
Telegram (Previous year papers PDFs [SSC,Railway,DSSSB,UP SI]): https://t.me/RBE_S
YouTube (Free lectures and job updates): https://www.youtube.com/c/RBERevolutionByEducation
Download RBE application for SSC Exams Complete Preparation
fueled by misinformation is also a major obstacle scale up microenterprises. In most villages, these
to herd immunity. Campaigns of public loans support poultry farming, handloom weaving,
education—capitalizing on radio, social media, or tiny retailing outlets. MFIs go beyond credit by
and school initiatives—seek to revive public faith regularly offering financial education training,
in science. Preserving accessibility and teaching borrowers to cope with cash flows and
communication in an open nature is crucial in interest charges. Critics warn, however, that
order to secure public health. excessive interest rates and repayment pressure
can result in over-indebtedness. To avoid such
What does “cold-chain logistics” refer to in risks, some MFIs limit interest rates and spread
the passage? repayment terms. Research indicates that when
(a) Transporting vaccines under controlled complemented by market linkages—such as
temperatures guaranteed buyers for fruits and vegetables—
(b) Selling vaccines in cold climates microfinance can trigger sustainable economic
(c) Developing new vaccine formulas growth and poverty reduction.
(d) Distributing medicines door-to-door
The primary function of microfinance
institutions, according to the passage, is to:
According to the passage, which method (a) Charge high interest rates to rural borrowers
helps maintain vaccine potency in remote (b) Provide large corporate loans
areas? (c) Offer financial services to those excluded from
(a) Air-conditioned cars traditional banking systems
(b) Solar-powered refrigerators (d) Regulate national monetary policy
(c) Vaccine-heated containers
(d) Underground storage In the context of the above passage,
“microenterprises” most nearly means:
Vaccine hesitancy is primarily caused by: (a) Multination corporations
(a) Lack of refrigerators (b) Misinformation (b) Small-scale businesses
(c) Cold-chain failures (d) Overproduction (c) Government agencies
(d) Family-owned farms
Which strategy is NOT mentioned as part of
public-education initiatives? Which benefit MFIs is mentioned besides
(a) Radio broadcasts lending money?
(b) Social-media influencers (a) Granting land titles
(c) School programs (b) Financial literacy training
(d) Financial incentives (c) Tax exemptions
(d) Import-export facilitation
The passage suggests that to protect public
health, it is essential to: What is a key criticism of microfinance
(a) Eliminate door-to-door highlighted in the passage?
(b) Halt solar-powered technology (a) It fails to reach women borrowers.
(c) Ensure equitable access and clear (b) It offers only savings accounts.
communication (c) High interest and repayment pressure can
(d) Focus solely on urban areas cause debt.
(d) It mandates market linkages.
Q21-25. Read the following passage and answer
the questions based on the passage: According to the author, sustainable impact
Microfinance institutions (MFIs) have become occurs when microfinance is combined with:
crucial agents for advancing financial inclusion in (a) High interest rates
rural areas. Through extending small loans and (b) Market linkages
savings accounts to people who don’t have (c) Shorter loan tenures
access to conventional banks, MFIs empower (d) Government bailouts
entrepreneurs—most often women—to initiate or
Telegram (Previous year papers PDFs [SSC,Railway,DSSSB,UP SI]): https://t.me/RBE_S
YouTube (Free lectures and job updates): https://www.youtube.com/c/RBERevolutionByEducation
Download RBE application for SSC Exams Complete Preparation
Q26-30. Read the passage and answer the (c) Simple (d) Compound-Complex
questions:
As artificial intelligence (AI) technologies continue What does the word ‘Proliferate” most nearly
to proliferate across every sector of human mean as used in the passage?
activity, the ethical implications of their (a) Decline slowly (b) Spread rapidly
deployment have become a matter of urgent (c) Change unpredictably (d) Improve steadily
concern. Once relegated to the realm of
speculative fiction, AI now permeates decision- In the context of the passage, the phrase
making in fields as diverse as healthcare, finance, “algorithmic discrimination” most likely refers
education, and criminal justice. While proponents to:
argue that AI can increase efficiency, reduce (a) Computers correcting human mistakes.
human bias, and democratize access to services, (b) AI systems treating individuals unfairly based
critics warn of the potential for algorithmic on biased programming.
discrimination, erosion of privacy, and the (c) AI systems achieving perfect objectivity.
deepening of existing social inequalities. (d) The classification of data without errors.
At the heart of the debate lies a fundamental
tension between technological progress and According to the passage, what is one major
moral responsibility. Machines, no matter how obstacle to ensuring fairness and
sophisticated, lack consciousness and intrinsic accountability in AI systems?
values; they merely execute instructions based on (a) The high cost of building AI models.
data inputs and programmed algorithms. Thus, (b) The limited availability of data scientists.
when an AI system makes a life-altering (c) The inability to fully explain how AI systems
decision—such as denying a loan application or reach conclusions.
predicting a defendant’s likelihood of (d) The lack of government funding.
reoffending—it is not the machine itself that is
accountable, but the human architects behind its Q31-35. Read the passage and answer the
creation and training. questions:
Moreover, the opacity of many AI systems, often Changes that seem small and unimportant at first
referred to as the "black box" problem, poses a will compound into remarkable results if you’re
significant ethical challenge. When even willing to stick with them for years. We often
developers cannot fully explain how a machine convince ourselves that massive success
arrived at a given outcome, the prospect of requires massive action. Whether it’s losing
ensuring fairness, transparency, and weight, building a business, or learning a new
accountability becomes deeply problematic. skill, we put pressure on ourselves to make some
Addressing these concerns will require not only earth-shattering improvement that everyone will
technical innovations, such as more explainable notice.
AI, but also rigorous interdisciplinary collaboration However, improvement by just 1 percent each
between engineers, ethicists, policymakers, and day may not be noticeable in the moment, but
the public. over time it can be far more meaningful. The
same way that money compounds with interest,
Which of the following correctly identifies the the effects of your habits multiply as you repeat
tense used predominantly in the first them. The outcomes you see in your life today
paragraph? are often the result of small choices made over
(a) Past perfect time, rather than dramatic changes.
(b) Present continuous This is why habits are the compound interest of
(c) Present perfect continuous self-improvement. The longer you stick with
(d) Present simple them, the greater the returns. But just as positive
habits can compound in your favor, negative
“At the heart of the debate lies a fundamental habits can compound against you. A slight slip in
tension between technological progress and discipline today may not seem like a big deal, but
moral responsibility.” if repeated, it can steer you in a completely
(a) Compound (b) Complex different direction.
Telegram (Previous year papers PDFs [SSC,Railway,DSSSB,UP SI]): https://t.me/RBE_S
YouTube (Free lectures and job updates): https://www.youtube.com/c/RBERevolutionByEducation
Download RBE application for SSC Exams Complete Preparation
Success, then, is not a single event—it is the alternatives.
result of daily choices, made consistently over Freedom of speech is a cornerstone of
time. democratic societies, allowing individuals to
express ideas, question authority, and promote
Identify the sentence with the correct use of change. However, in the digital world, this right
subject-verb agreement: encounters complex challenges. Social media
(a) The effects of your habits multiplies as you platforms have become arenas where opinions
repeat them can be voiced instantly—but also where hate
(b) The outcomes you see in your life today is speech, misinformation, and cyberbullying thrive.
often the result of small choices made over time. Balancing the right to speak freely with the
(c) The effects of your habits multiply as you responsibility to prevent harm is now a pressing
repeat them. debate. Should platforms censor harmful content,
(d) The outcomes of your daily decisions or would that violate freedom? While some argue
determines your long-term success. that regulation is essential to protect vulnerable
groups, others fear it might lead to suppression of
"Changes that seem small and unimportant at dissent. The problem deepens when private
first will compound into remarkable results if companies—not elected bodies—decide what
you’re willing to stick with them for years." speech is allowed or banned. Striking the right
(a) To decrease gradually balance requires thoughtful policies, digital
(b) To mix different elements literacy, and legal frameworks rooted in both
(c) To accumulate and grow over time human rights and public safety. In a world where
(d) To remain constant words travel faster than thought, protecting free
speech without enabling harm may be one of the
Choose the best antonym for the word defining tests of modern democracy.
"discipline" in the sentence:
"A slight slip in discipline today may not seem What central conflict is explored in the
like a big deal, but if repeated, it can steer you passage?
in a completely different direction." (a) Law vs. religion
(a) Commitment (b) Carelessness (b) Profit vs. loss
(c) Determination (d) Focus (c) Freedom vs. online harm
(d) Privacy vs. piracy
What is the main idea of the passage?
(a) Success comes from making one major Why is regulation of speech on social media
change in life. controversial?
(b) Small daily improvements, when repeated (a) It is illegal in all democracies
consistently, lead to significant long-term results. (b) It gives too much power to users
(c) Negative habits have no real impact unless (c) It may suppress dissenting voices
they are extreme. (d) It slows down communication
(d) Success is mostly determined by luck rather
than effort. What is the concern about private companies
controlling speech?
What type of pronoun is the word “ This ” in (a) They are too small to influence politics
the sentence, “This is why habits are the (b) They might have vested interests
compound interest of self-improvement.”? (c) They lack legal authority or accountability
(a) Demonstrative pronoun (d) They promote academic debates
(b) Reflexive pronoun
(c) Abstract pronoun According to the author, what is required to
(d) Subjective pronoun resolve this conflict?
(a) Removing all harmful users
Q36-40. Read the following passage carefully and (b) Allowing complete freedom
answer the questions that follow by selecting the (c) Combining rights with safety through law and
most appropriate option from the given literacy
Telegram (Previous year papers PDFs [SSC,Railway,DSSSB,UP SI]): https://t.me/RBE_S
YouTube (Free lectures and job updates): https://www.youtube.com/c/RBERevolutionByEducation
Download RBE application for SSC Exams Complete Preparation
(d) Banning social media worldwide (c) More apps mean less productivity
(d) Less data brings more security
What tone does the author adopt in the
passage? According to the author, what may be the
(a) Dismissive (b) Alarmist solution?
(c) Balanced and reflective (d) Aggressive (a) Fatser internet plans
(b) Reducing social media apps
Q41-45. Read the following passage carefully and (c) Mindful engagement and limits
answer the questions that follow by selecting the (d) Following more influencers
most appropriate option from the given
alternatives. What is the tone of the passage?
In the age of endless options—from streaming (a) Sarcastic (b) Cautionary
services to social media platforms—modern (c) Indifferent (d) Aggresive
individuals face what psychologists call “decision
fatigue.” Unlike earlier generations who had Q46-50. Read the passage and answer the
limited choices, today’s youth are constantly questions:
required to decide: what to watch, whom to follow, Much of the discussion in this book is about
what to share, and how to respond. While biases of intuition. However, the focus on biases
freedom of choice is often celebrated, too many does not mean that intuition is always wrong—far
choices can lead to stress, dissatisfaction, and from it. Most of the time, the quick judgments of
even paralysis. Ironically, abundance can become our intuitive mind, what I call System 1, are
overwhelming. Algorithms, designed to predict accurate enough to help us navigate the world.
preferences, often narrow exposure rather than This system operates automatically and quickly,
broaden it, creating digital bubbles. People may with little or no effort.
feel in control, yet their choices are subtly shaped On the other hand, System 2 is the slower, more
by unseen patterns. Over time, the constant need deliberate mode of thinking. It allocates attention
to choose can drain mental energy, leaving to effortful mental activities, such as solving
individuals anxious or indifferent. True digital complex problems or evaluating logical
freedom may lie not in limitless options, but in arguments. System 2 is more methodical and
mindful engagement and setting personal rational, but it is also lazier—it prefers to endorse
boundaries. Recognising when to choose—and the intuitions of System 1 rather than question
when to step back—could be the new wisdom of them.
the connected age. This division of labour works well most of the
time, but it also creates predictable errors.
What is the main issue discussed in the Because System 1 is prone to biases and snap
passage? judgments, we often make decisions that are not
(a) Rise of social media trends as rational as we believe. Understanding these
(b) Overuse of mobile apps two systems can help us recognize when we are
(c) Stress caused by excessive digital choices relying too much on intuition and when we need
(d) Lack of internet facility to engage in more deliberate thinking.

What does the author imply about Identify the sentence with correct subject-verb
algorithms? agreement:
(a) They always give better choices (a) The quick judgments of our intuitive mind,
(b) They expand one’s interests what I call System 1, is accurate enough to help
(c) They limit exposure to diverse content us navigate the world.
(d) They reduce online ads (b) This division of labor work well most of the
time, but it also creates predictable errors.
What paradox is presented in the passage? (c) Understanding these two systems can helps
(a) Choice leads to control but also exhaustion us recognize when we are relying too much on
(b) Technology helps only the rich intuition.
(d) System 2 allocates attention to effortful mental
Telegram (Previous year papers PDFs [SSC,Railway,DSSSB,UP SI]): https://t.me/RBE_S
YouTube (Free lectures and job updates): https://www.youtube.com/c/RBERevolutionByEducation
Download RBE application for SSC Exams Complete Preparation
activities, such as solving complex problems or Students learn that solar power does not cause
evaluating logical arguments. pollution like coal or petrol. The school has also
inspired nearby houses to install solar panels.
Choose the sentence that correctly uses a This change made the village cleaner and more
relative pronoun? aware of the environment. Even on cloudy days,
(a) System 1 is the intuitive mode of thinking, who the solar battery stores enough power to keep the
operates automatically and quickly. school running. With small steps like this, more
(b) The biases, which influence our judgments, schools and homes can help protect nature and
often lead to predictable errors. build a better future.
(c) Understanding the two systems, whom have
different roles, is crucial for decision-making. What energy source does the school use?
(d) The effortful mental activities whose System 2 (a) Petrol (b) Wind
performs include solving complex problems. (c) Solar (d) Gas

What is the meaning of the word "deliberate" What does the school do with the saved
in the sentence: electricity money?
"System 2 is the slower, more deliberate mode (a) Build a playground
of thinking." (b) Buy food
(a) Accidental (b) Careful and intentional (c) Buy books and improve classrooms
(c) Quick and effortless (d) Biased (d) Hire more teachers

Choose the best synonym for the word Why is solar energy better than coal?
"endorse" in the sentence: (a) It is cheaper to burn
"System 2 is more methodical and rational, (b) It does not cause pollution
but it is also lazier—it prefers to endorse the (c) It is easier to store
intuitions of System 1 rather than question (d) It is available in bottles
them."
(a) Approve (b) Reject What did the school inspire villagers to do?
(c) Criticize (d) Delay (a) Use less water
(b) Buy new uniforms
According to the passage, what is one (c) Install solar panels
drawback of System 1 thinking? (d) Visit the school more
(a) It always leads to incorrect decisions.
(b) It requires too much mental effort. What helps the school on cloudy days?
(c) It can be prone to biases and snap (a) Electricity from the city
judgements. (b) Fans from other schools
(d) It completely replaces System 2 in decision- (c) Solar batteries
making. (d) Teachers' phones

Q51-55. Read the following passage carefully and Q56-60. Read the following passage carefully and
answer the questions that follow by selecting the answer the questions that follow by selecting the
most appropriate option from the given most appropriate option from the given
alternatives. alternatives.

In a small village, a government school has The rise of genetic engineering has redefined
become an example of clean energy use. The what is scientifically possible, but not necessarily
school uses solar panels to run fans, lights, and what is ethically acceptable. While gene editing
even computers. This helps the school save offers hope in treating inherited diseases and
money on electricity bills. The saved money is improving crop resilience, it also raises profound
used to buy books and improve classrooms. The moral questions. Should we edit genes to
teachers also explain to students how solar enhance intelligence or physical ability? Where
energy works and why it is good for the planet. do we draw the line between therapy and
Telegram (Previous year papers PDFs [SSC,Railway,DSSSB,UP SI]): https://t.me/RBE_S
YouTube (Free lectures and job updates): https://www.youtube.com/c/RBERevolutionByEducation
Download RBE application for SSC Exams Complete Preparation
enhancement? These questions remain alternatives.
unsettled. Some argue that altering the human Theme: The Dandi March
genome for non-medical reasons may lead to a Sector: Indian History / Freedom Movement
new form of inequality—where the genetically In 1930, Mahatma Gandhi started a peaceful
modified have social advantages over others. protest called the Dandi March. He and his
Others fear unintended consequences: once a followers walked 240 kilometers from Sabarmati
gene is changed, its effects may ripple through Ashram to Dandi, a village near the sea. The
generations. Science often moves faster than purpose of the march was to protest the British
legislation, leaving ethical frameworks lagging law that taxed salt. Salt is a basic need for
behind. True progress lies not in what we can do, everyone, and Gandhi believed it was unfair to
but in what we choose to do responsibly. As with make people pay for it. During the march, more
all powerful tools, genetic engineering demands and more people joined him. The British
caution, reflection, and a shared sense of human government tried to stop the movement, but
values. Gandhi remained calm and peaceful. He made
salt from the sea at Dandi, breaking the law in a
What is the central concern of the passage? non-violent way. The Dandi March became a
(a) Costs of gene therapy strong symbol of India’s fight for freedom. It
(b) Government control of science showed the world that peaceful protest could be
(c) Ethical use of genetic engineering powerful. Even today, it is remembered as a
(d) Advances in medical devices brave step in India’s struggle for independence.

What does the author suggest about using Who led the Dandi March?
genes to enhance traits? (a) Subhas Chandra Bose
(a) It improves global health (b) Jawaharlal Nehru
(b) It is encouraged by law (c) Bhagat Singh
(c) It risks creating inequality (d) Mahatma Gandhi
(d) It always benefits society
Why was the march started?
Why are ethical frameworks struggling to (a) To support education
keep up? (b) To protest salt tax
(a) Scientists disagree (c) To celebrate independence
(b) Public lacks interest (d) To fight a war
(c) Laws are faster than research
(d) Science progresses rapidly How did Gandhi break the law?
(a) By fighting soldiers
What does the author imply about gene (b) By leaving the country
editing’s long-term impact? (c) By making salt from the sea
(a) It is fully reversible (d) By closing schools
(b) It can affect future generations
(c) It has no proven results What was the distance covered in the march?
(d) It only benefits the rich (a) 100 km (b) 500 km
(c) 240 km (d) 50 km
Which phrase best captures the author’s
message? What does the Dandi March symbolize today?
(a) “Edit now, regret later” (a) Peaceful protest and courage
(b) “Act fast and innovate” (b) Use of violence in freedom
(c) “Caution must guide progress” (c) Marching for exercise
(d) “All science is safe” (d) Tax collection by British

Q61-65. Read the following passage carefully and Q66-70. Read the following passage carefully and
answer the questions that follow by selecting the answer the questions that follow by selecting the
most appropriate option from the given most appropriate option from the given
Telegram (Previous year papers PDFs [SSC,Railway,DSSSB,UP SI]): https://t.me/RBE_S
YouTube (Free lectures and job updates): https://www.youtube.com/c/RBERevolutionByEducation
Download RBE application for SSC Exams Complete Preparation
alternatives. (a) Casual and humorous
As algorithms increasingly curate our digital (b) Cautious and reflective
experiences—what we read, watch, buy, or even (c) Dismissive and skeptical
whom we date—they quietly shape our (d) Optimistic and enthusiastic
perception of the world. This phenomenon, known
as "algorithmic living," promises efficiency but Q71-75. Read the following passage carefully and
risks eroding autonomy. When choices are answer the questions that follow by selecting the
filtered through predictive systems, personal most appropriate option from the given
agency may weaken, and individuals might alternatives.
mistake convenience for control. Moreover, In today’s digital age, convenience often comes at
algorithmic personalization often traps users in the cost of privacy. Every time a user browses a
feedback loops, reinforcing existing preferences website, clicks a link, or uses an app, their data is
and limiting exposure to diverse perspectives. collected—sometimes without their knowledge.
Over time, this can lead to cognitive narrowing, Companies use this information to target ads,
where users become less open to alternative predict behavior, or even sell insights to third
views or new ideas. While algorithms are not parties. While this can lead to better services, it
inherently harmful, their unchecked influence also raises ethical concerns. Who controls this
raises ethical concerns about manipulation and data? How is it stored, and can users truly opt
passive consumption. True digital well-being out? Many people accept terms and conditions
demands critical awareness—not of the machine, without reading them, unknowingly giving
but of how we engage with it. In a world permission for data tracking. Some countries
increasingly optimized for us, the question is not have passed laws to protect user data, but
what the algorithm shows, but whether we are still implementation remains weak. As surveillance
choosing to look beyond it. tools grow smarter, the line between public and
What is the main concern of “algorithmic living”? private becomes blurred. Individuals must now
(a) Faster online shopping learn to balance the benefits of technology with
(b) Decline in physical activity the right to privacy—a task that requires
(c) Loss of autonomy due to filtered choices awareness, regulation, and responsible digital
(d) Increased cost of internet behavior.

What does the term “feedback loop” imply in What is the main concern raised in the
the passage? passage?
(a) Randomized search results (a) Lack of internet speed
(b) Repeated exposure to similar (b) High cost of mobile data
(c) Feedback from social media (c) Threats to digital privacy
(d) Technological malfunctions (d) Overuse of technology

How does algorithmic living affect thinking According to the passage, how is user data
over time? often collected?
(a) It enhances creative thinking (a) Only with government approval
(b) It encourages scientific research (b) through printed forms
(c) It narrows perspective and reduces openness (c) Without users being fully aware
(d) It develops emotional intelligence (d) After signing hardcopy documents

What is suggested as a way to maintain digital What does the passage say about laws
well-being? protecting data?
(a) Ignore all online content (a) They are strong and global.
(b) Follow only algorithmic suggestions (b) They are strict in all countries.
(c) Practice critical engagement (c) They exist but are poorly enforced.
(d) Increase screen time (d) They are unnecessary.

What is the tone of the passage?


Telegram (Previous year papers PDFs [SSC,Railway,DSSSB,UP SI]): https://t.me/RBE_S
YouTube (Free lectures and job updates): https://www.youtube.com/c/RBERevolutionByEducation
Download RBE application for SSC Exams Complete Preparation
What is one positive use of user data performance data is stored online. Despite these
mentioned in the passage? issues, many educators believe AI can be a
(a) Reducing screen time useful tool when used wisely. Like any
(b) Improving service experiences technology, the key lies in balancing human
(c) Blocking global access guidance with machine efficiency.
(d) Avoiding social media
What does AI mainly help with in education?
What does the author suggest is needed to (a) Giving marks (b) Personalizing learning
protect privacy? (c) Hiring teachers (d) Fixing school buildings
(a) Avoiding all apps
(b) Stronger surveillance tools What is a possible risk of overusing AI?
(c) Awareness and responsible behavior (a) Higher costs (b) Fewer classrooms
(d) Free mobile phones for all (c) Poor thinking skills (d) Better results

Q76-80. Read the following passage carefully and Why are AI chatbots used in schools?
answer the questions that follow by selecting the (a) To monitor cameras
most appropriate option from the given (b) To take exams
alternatives. (c) To answer students’ questions
Artificial Intelligence (AI) is gradually transforming (d) To conduct interviews
the way students learn. From virtual tutors to
personalized learning paths, AI helps identify What is a concern related to student data?
each learner’s strengths and weaknesses. With (a) Attendance drops (b) Internet speed
the help of data, AI systems can suggest specific (c) Data privacy (d) Exam cheating
topics where students need improvement.
Schools are also using AI chatbots to answer What is the author’s final advice?
basic queries, reducing the pressure on teachers. (a) Replace teachers
However, experts warn that over-dependence on (b) Use only AI
AI may limit critical thinking. If students rely too (c) Balance tech and human input
much on machines for answers, they may lose (d) Avoid all AI
problem-solving skills. There are also concerns
about data privacy, especially when student

Answer Key:

Q1 c Q2 b Q3 b Q4 b Q5 b
Q6 b Q7 b Q8 b Q9 c Q10 c
Q11 c Q12 b Q13 c Q14 c Q15 b
Q16 a Q17 b Q18 b Q19 d Q20 c
Q21 c Q22 b Q23 b Q24 c Q25 b
Q26 d Q27 c Q28 b Q29 b Q30 c
Q31 c Q32 c Q33 b Q34 b Q35 a
Q36 c Q37 c Q38 c Q39 c Q40 c
Q41 c Q42 c Q43 a Q44 c Q45 b
Q46 d Q47 b Q48 b Q49 a Q50 c
Q51 c Q52 c Q53 b Q54 c Q55 c
Q56 c Q57 c Q58 d Q59 b Q60 c
Q61 d Q62 b Q63 c Q64 c Q65 a
Q66 c Q67 b Q68 c Q69 c Q70 b
Q71 c Q72 c Q73 c Q74 b Q75 c
Q76 b Q77 c Q78 c Q79 c Q80 c

Telegram (Previous year papers PDFs [SSC,Railway,DSSSB,UP SI]): https://t.me/RBE_S


YouTube (Free lectures and job updates): https://www.youtube.com/c/RBERevolutionByEducation
Download RBE application for SSC Exams Complete Preparation
Solution 1: Explanation sarcastic; rather, it carefully analyzes the situation
The passage discusses the benefits of e- and provides thoughtful insights.
commerce and the challenges faced by brick-and-
mortar stores and small retailers. Solution 5: Explanation
It provides examples of omnichannel strategies The passage concludes by stating that the most
and highlights issues like technical difficulties for successful retailers will combine digital innovation
small businesses and environmental concerns, with sustainable practices and a personalized
rather than advocating for one model over customer experience, emphasizing the
another or focusing only on omnichannel importance of personalization.
strategies. It does not claim that all retailers will go fully
The passage analyzes how retail is changing, online; rather, it discusses omnichannel strategies
presenting a balanced view of both opportunities integrating online and in-store experiences.
and challenges in the modern retail landscape. The passage mentions that reliance on third-party
delivery can erode profit margins but does not say
Solution 2: Explanation it is always unprofitable.
The passage states that brick-and-mortar stores Packaging waste is described as an
“must contend with higher overheads—rent, environmental impact under increasing scrutiny,
utilities, and staffing,” directly providing examples indicating it is significant, not minimal.
of overheads as recurring business expenses
necessary for day-to-day operation. Solution 6: Explanation
Therefore, "overheads" refers to the regular, fixed The passage emphasizes how local communities,
costs or expenses a business must pay, especially villagers, play a vital role in wildlife
regardless of specific business activities. conservation efforts.
It highlights community participation through
Solution 3: Explanation activities like protecting species, serving as
The passage mentions that brick-and-mortar informants, earning from ecotourism, and crafting
stores face challenges like higher overheads handicrafts.
(which include rent, utilities, and staffing), but The passage also mentions challenges like
these are not specifically cited as challenges funding, human-wildlife conflict, and land rights
for small retailers. but stresses cooperative dialogue and benefit-
For small retailers, the passage lists challenges sharing.
such as lack of technical expertise , insufficient It does not criticize ecotourism, explain poachers’
capital , and dependence on third-party delivery actions, or advocate for stricter government
services . enforcement.
"Scrutiny" and "environmental scrutiny" relate to
e-commerce shipping impacts, not specific Solution 7: Explanation
challenges faced by small retailers in developing The passage contrasts "top-down regulations"
their business. with community-based conservation, where locals
"High utility bills" is not mentioned as a unique are actively involved.
challenge for small retailers in the context of the "Top-down" implies decisions or rules made by
passage—it applies broadly to physical stores, higher authorities such as governments or
not specifically to small retailers. organizations without local input.
This phrase therefore means policies or
Solution 4: Explanation mandates imposed from above, rather than
The passage presents a balanced examination of suggestions or rules emerging from local villagers
the opportunities and challenges in modern retail or other stakeholders.
without exaggeration or emotional bias. Options like "rules created by local villagers,"
It discusses positive aspects like convenience "guidelines developed by scientists," or
and innovation alongside challenges such as "suggestions from tourists" do not fit the meaning
technical difficulties and environmental concerns of top-down regulations as described in the
in a clear, reasoned manner. passage.
The tone is neither alarmist, enthusiastic, nor
Telegram (Previous year papers PDFs [SSC,Railway,DSSSB,UP SI]): https://t.me/RBE_S
YouTube (Free lectures and job updates): https://www.youtube.com/c/RBERevolutionByEducation
Download RBE application for SSC Exams Complete Preparation
Solution 8: Explanation Other options like public transport being outdated,
The passage mentions that women's favoring private vehicles, or mostly benefiting
organizations produce handicrafts out of tourists are not supported by the passage.
sustainable forest products as one of the ways
communities generate income. Solution 12: Explanation
Ecotourism is also mentioned as a source of The passage states that integrated fares across
income for villagers. different modes of travel allow passengers to use
Poaching endangered species, clearing forests one ticket for bus, train, and bike-share travel.
for agriculture, and mining minerals are not This convenience eliminates the need to buy
mentioned as sources of income but rather separate tickets for each mode, making
activities that harm conservation efforts. movement around the city easier.
Therefore, crafting handicrafts aligns with the Other options like creating financial burden, being
sustainable and community-friendly activities only for long-distance travel, or encouraging
highlighted in the passage. private vehicle use are not supported by the
passage.
Solution 9: Explanation
The passage mentions specific challenges such Solution 13: Explanation:
as low funding levels, human-wildlife conflict, and The passage states that constructing and
uncertain land-use rights. retaining public transport infrastructure requires
It does not mention or discuss climate change as capital and long-term vision.
one of the obstacles affecting community This indicates that adequate funding and proper
conservation. planning over time are essential to maintain and
Other potential challenges like climate change, develop effective public transport.
pollution, or habitat destruction are not listed in The passage does not mention minimal
the passage. regulation, frequent technology upgrades, or
higher taxes on private vehicles as the key factors
Solution 10: Explanation needed for maintenance.
The passage presents information about wildlife
conservation in a neutral, factual manner without Solution 14: Explanation
emotional language or persuasive intent. The passage mentions that equity issues are
It explains the role of communities, successful important and that transit lines need to serve low-
initiatives, challenges, and the importance of income communities along with commercial hubs.
continued dialogue. This ensures that all citizens have access and
The tone does not express pessimism, sarcasm, benefit from public transport.
or promotional bias, but rather provides balanced The passage does not mention keeping fares
and informative content. high, focusing only on commercial zones, or
This objective tone is typical of informative texts offering free rides on weekends as equity
aiming to educate readers on a topic with clarity measures.
and fairness.
Solution 15: Explanation
Solution 11: Explanation The passage presents facts, reasons, and logical
The passage highlights how public transport explanations about the benefits and challenges of
systems like buses, trams, and metros support public transport.
urban sustainability by reducing private car use, It supports the importance of good public
traffic congestion, and emissions. transport networks for urban sustainability.
It explains the benefits of integrated fares, The author examines various aspects, including
exclusive lanes, and priority signaling in making equity, infrastructure needs, and environmental
public transport efficient. benefits, with a balanced and reasoned approach.
It also addresses the need for long-term The tone is not skeptical , neutral, or aggressive,
investment and equitable access to ensure that but analytical as it provides justification and
low-income communities benefit along with evidence supporting public transport.
commercial areas. It is also supportive as it highlights the positive
Telegram (Previous year papers PDFs [SSC,Railway,DSSSB,UP SI]): https://t.me/RBE_S
YouTube (Free lectures and job updates): https://www.youtube.com/c/RBERevolutionByEducation
Download RBE application for SSC Exams Complete Preparation
role of public transport in cities. access to vaccines and transparent
communication are key strategies for protecting
Solution 16: ✅ Correct Answer: Transporting public health.
vaccines under controlled temperatures
The passage clearly states: Solution 21: ✅ Correct Answer: Offer financial
“Mass immunization relies on cold-chain services to those excluded from traditional
availability: vaccines have to be stored at precise banking systems
temperatures from manufacturing through The passage states:
inoculation.” “Through extending small loans and savings
This means cold-chain logistics refers to accounts to people who don’t have access to
maintaining specific temperature conditions conventional banks…” This clearly highlights that
throughout the vaccine's journey—from the core role of MFIs is to provide financial
production to delivery—to ensure effectiveness. services—like loans and savings—to individuals
who are excluded from traditional banking
Solution 17: ✅ Correct Answer: Solar-powered systems , especially in rural areas.
refrigerators
The passage clearly states: Solution 22: ✅ Correct Answer: Small-scale
“Solar-powered refrigerators and insulated businesses
containers are lifelines for remote locations.” This The passage describes microenterprises as
means that in areas lacking reliable electricity, ventures supported by small loans in rural
solar-powered refrigerators are essential to areas—such as poultry farming, handloom
maintain the cold-chain and preserve vaccine weaving, and tiny retail outlets . These are
potency. clearly small-scale businesses , often run by
individuals or families, especially women, with
Solution 18: ✅ Correct Answer: limited capital and local reach.
Misinformation “In most villages, these loans support poultry
The passage clearly states: farming, handloom weaving, or tiny retailing
“Vaccine hesitancy fueled by misinformation is outlets.” This line directly links microenterprises to
also a major obstacle to herd immunity.” This small, local businesses , not large corporations
means that misinformation —false beliefs, or government bodies.
myths, or rumors —plays the biggest role in
making people reluctant to get vaccinated. Solution 23: ✅ Correct Answer: Financial
literacy training
Solution 19: ✅ Correct Answer: Financial The passage clearly states:
incentives “MFIs go beyond credit by regularly offering
The passage outlines several strategies used in financial education training, teaching borrowers to
public education campaigns to combat vaccine cope with cash flows and interest charges.” This
hesitancy and promote immunization. means that besides lending money , MFIs also
Specifically, it mentions: help borrowers understand financial concepts,
“Campaigns of public education—capitalizing on manage their income, and handle repayment—
radio, social media, and school initiatives…” this is financial literacy training .
This confirms that radio broadcasts , social
media , and school programs are part of the Solution 24: ✅ Correct Answer: High interest
strategy. However, financial incentives are not and repayment pressure can cause debt
mentioned anywhere in the passage. The passage clearly states:
“Critics warn, however, that excessive interest
Solution 20: ✅ Correct Answer: Ensure rates and repayment pressure can result in over-
equitable access and clear communication indebtedness.” This highlights the main criticism
The final line of the passage states: of microfinance: while it empowers borrowers, it
“Preserving accessibility and communication in an can also lead to financial stress and debt if
open nature is crucial in order to secure public interest rates are too high or repayment terms are
health.” This clearly emphasizes that equitable
Telegram (Previous year papers PDFs [SSC,Railway,DSSSB,UP SI]): https://t.me/RBE_S
YouTube (Free lectures and job updates): https://www.youtube.com/c/RBERevolutionByEducation
Download RBE application for SSC Exams Complete Preparation
too rigid. The phrase “across every sector” implies wide
and fast expansion .
Solution 25: ✅ Correct Answer: Market The verb “continue to proliferate” suggests
linkages ongoing, rapid growth , not decline or
The passage concludes with: unpredictability.
“Research indicates that when complemented by
market linkages—such as guaranteed buyers for Solution 29: ✅ Correct Answer: AI systems
fruits and vegetables—microfinance can trigger treating individuals unfairly based on biased
sustainable economic growth and poverty programming
reduction.” 🔍 Explanation:
This clearly shows that market linkages , like Let’s decode the phrase “algorithmic
assured buyers for local produce, enhance the discrimination” in context:
long-term impact of microfinance by creating “…critics warn of the potential for algorithmic
income stability and growth opportunities. discrimination , erosion of privacy, and the
deepening of existing social inequalities.”
Solution 26: ✅ Correct Answer: Present simple 🔸 Key Clues:
🔍 Explanation: The phrase is listed alongside negative
Let’s analyze the first paragraph and match it with outcomes like erosion of privacy and social
tense logic: inequality.
🔸 Key Verbs in the First Paragraph: The passage discusses how AI systems, though
“AI technologies continue to proliferate…” → efficient, can amplify human biases if trained on
Present simple flawed or prejudiced data.
“AI now permeates decision-making…” →
Present simple Solution 30: ✅ Correct Answer: The inability to
“Proponents argue …” → Present simple fully explain how AI systems reach conclusions.
“Critics warn …” → Present simple 🔍 Explanation:
These verbs reflect ongoing, general truths or Let’s extract the key line from the passage:
current realities , which is the hallmark of present “When even developers cannot fully explain how
simple tense . a machine arrived at a given outcome, the
prospect of ensuring fairness, transparency, and
Solution 27: ✅ Correct Answer: Simple accountability becomes deeply problematic.”
🔍 Explanation: 🔸 This directly refers to:
Let’s dissect the sentence: The “black box” problem — where AI systems
“At the heart of the debate lies a fundamental operate in ways that are not fully interpretable ,
tension between technological progress and even by their creators.
moral responsibility.” This opacity makes it difficult to audit decisions ,
🔸 Key Features: detect bias , or assign responsibility , which
Single independent clause undermines fairness and accountability.
→ Subject: a fundamental tension
→ Verb: lies Solution 31: The correct answer is " The effects
→ Objects/Modifiers: between technological of your habits multiply as you repeat them."
progress and moral responsibility This is because when you perform a habit
repeatedly, the results compound, leading to
Solution 28: ✅ Correct Answer: Spread rapidly bigger changes over time. This principle is central
🔍 Explanation: to the concept of the compounding effect of
habits, as seen in both life and behavioral
Let’s analyze the usage of “proliferate” in the
science. The other options reference outcomes
passage:
and decisions, but do not specifically state the
“As artificial intelligence (AI) technologies
multiplying effect of repetition.
continue to proliferate across every sector of
human activity…”
Solution 32: "To accumulate and grow over
🔸 Contextual Clues:
time." This phrase refers to the idea of
Telegram (Previous year papers PDFs [SSC,Railway,DSSSB,UP SI]): https://t.me/RBE_S
YouTube (Free lectures and job updates): https://www.youtube.com/c/RBERevolutionByEducation
Download RBE application for SSC Exams Complete Preparation
compounding, where effects, benefits, or passage.
quantities increase incrementally as actions or
elements are repeated or added. In finance, Solution 37: The correct answer is " It may
science, and habit-building, this principle means suppress dissenting voices." When platforms
that growth is not simply linear but exponential, as regulate content to protect users or reduce harm,
results build upon previous results and keep a significant concern is that such regulations can
increasing. The other options do not capture this also limit freedom of expression, particularly by
accumulating and increasing nature. silencing those with unpopular or dissenting
opinions. This is seen as a major drawback
Solution 33: The correct answer is because it can undermine the very principle of
"Carelessness." Discipline refers to self-control, free speech in democratic societies. The other
order, and adherence to rules or standards. Its options do not address the nuanced challenge
opposite is carelessness, which means lack of highlighted in discussions of regulation and
attention, caution, or discipline. The other democracy
options—commitment, determination, and
focus—are qualities that support discipline, not Solution 38: The correct answer is " They lack
oppose it, making them unsuitable as antonyms legal authority or accountability." The passage
in this context. highlights that private companies—unlike elected
bodies—are not directly answerable to the public
Solution 34: The correct answer is "Small daily and may make decisions about what speech is
improvements, when repeated consistently, allowed or banned without sufficient checks,
lead to significant long-term results." The transparency, or accountability. This raises
passage states that success is not the result of a concerns about fairness and the proper limits of
single event, but of consistent daily choices. This their power over public discourse. Other options
highlights the cumulative effect of regular, positive do not address the central issue of power and
habits over time, rather than dramatic changes or accountability in content moderatio n.
luck. The other options do not accurately reflect
the core message of gradual, consistent Solution 39: The correct answer is "Combining
improvement. rights with safety through law and literacy ."
The author suggests that resolving the conflict
Solution 35: The correct answer is between freedom of speech and online harm
"Demonstrative pronoun ." In the sentence, the requires a balanced approach that includes legal
word “This” is used to point to a fact or idea that frameworks, digital literacy, and policies that
has just been mentioned or will be explained, uphold rights while ensuring safety. This solution
which is the function of a demonstrative pronoun. addresses both the protection of free speech and
Demonstrative pronouns (this, that, these, those) the need to mitigate online dangers, rather than
are used to indicate specific things. The other resorting to extremes like unchecked freedom or
options do not fit the grammatical role of “This” in outright bans.
the context of the sentence.
Solution 40: The correct answer is "Balanced
Solution 36: The correct answer is "Freedom vs. and reflective." The author considers different
online harm." The passage discusses the viewpoints regarding free speech and online
challenge of balancing the right to free speech harm, highlights the complexity of the issue, and
with the need to prevent online harms such as emphasizes the need for thoughtful, measured
hate speech, misinformation, and cyberbullying. It solutions. The tone is neither extreme nor
explores how protecting freedom of expression dismissive, but rather carefully weighs potential
must be weighed against the responsibility to consequences and solutions.
safeguard individuals and society from digital
risks, which is the central conflict presented in the Solution 41: The most appropriate option for the
text. The other options are mentioned neither main issue discussed in the passage is Stress
specifically nor as primary conflicts in the caused by excessive digital choices .
Reasoning
Telegram (Previous year papers PDFs [SSC,Railway,DSSSB,UP SI]): https://t.me/RBE_S
YouTube (Free lectures and job updates): https://www.youtube.com/c/RBERevolutionByEducation
Download RBE application for SSC Exams Complete Preparation
The passage describes how an abundance of lies not in having limitless options but in mindful
options and continuous decision-making in the engagement and setting personal boundaries.
digital age led to "decision fatigue," mental Recognizing when to choose and when to step
exhaustion, stress, dissatisfaction, and digital back is described as the new wisdom of the
bubbles. connected age, implying the importance of
It specifically focuses on the negative effects of conscious and limited decision-making.
having too many choices online, not on social Other options like faster internet plans, reducing
media trends, mobile app overuse, or internet social media apps, or following more influencers
facility issues. are not mentioned as solutions in the passage.
Thus, the correct answer is: Mindful
Solution 42: The author implies that algorithms engagement and limits .
limit exposure to diverse content .
Explanation Solution 45: The tone of the passage is
The passage states that algorithms, although Cautionary .
designed to predict preferences, often narrow Explanation
exposure rather than broaden it, creating digital The passage warns about the negative effects of
bubbles. having too many choices in the digital age, such
This means that instead of expanding one's as stress, dissatisfaction, paralysis, and mental
interests, algorithms tend to restrict content to exhaustion (decision fatigue).
what they predict the user will prefer, limiting It highlights the risks and downsides of seemingly
diversity. positive freedom of choice, advising mindful
The options "always give better choices," "expand engagement and setting boundaries.
one’s interests," and "reduce online ads" do not The tone is not sarcastic, indifferent, or
reflect the passage’s description of algorithms aggressive; rather, it carefully cautions readers
shaping choices subtly and narrowing exposure. about the potential pitfalls of excessive digital
Hence, the correct answer is: They limit options and encourages wise decision-making.
exposure to diverse content . Therefore, the correct answer is: Cautionary .

Solution 43: The paradox presented in the Solution 46: Explanation:


passage is: Choice leads to control but also 1- "The quick judgments of our intuitive mind,
exhaustion . what I call System 1, is accurate enough to help
Explanation us navigate the world."
The passage explains that while freedom of Incorrect: The subject "judgments" is plural, so
choice is often celebrated and individuals may the verb should be "are," not "is."
feel in control, having too many choices can 2- "This division of labor work well most of the
cause stress, dissatisfaction, paralysis, and time, but it also creates predictable errors."
mental exhaustion (decision fatigue). Incorrect: The subject "division" is singular, so the
This creates a paradox where abundance and verb should be "works," not "work."
freedom of options, which should empower 3- "Understanding these two systems can helps
people, instead overwhelm and drain their mental us recognize when we are relying too much on
energy. intuition."
Other options like technology helping only the Incorrect: After "can," the verb should be in base
rich, more apps reducing productivity, or less data form " help ," not "helps."
leading to more security are not the central 4- "System 2 allocates attention to effortful
contradictions described in the passage. mental activities, such as solving complex
Hence, the correct answer is: Choice leads to problems or evaluating logical arguments."
control but also exhaustion . Correct: The subject "System 2" is singular, and
the verb "allocates" agrees with it.
Solution 44: According to the author, the solution Thus, the correct sentence is the 4 th option.
may be Mindful engagement and limits .
Explanation Solution 47: Explanation:
The passage suggests that true digital freedom 2- "The biases, which influence our judgments,
Telegram (Previous year papers PDFs [SSC,Railway,DSSSB,UP SI]): https://t.me/RBE_S
YouTube (Free lectures and job updates): https://www.youtube.com/c/RBERevolutionByEducation
Download RBE application for SSC Exams Complete Preparation
often lead to predictable errors." It can be prone to biases and snap judgments.
Correct use of the relative pronoun "which" to add Explanation:
non-essential information about "biases." The passage explicitly mentions that System 1 is
Other sentences have incorrect pronoun usage: prone to biases and snap judgments.
1- "System 1 is the intuitive mode of thinking, Because of this, decisions made by System 1 are
who operates automatically and quickly." often not as rational as we believe.
Incorrect: "who" is used for people, not for "mode" System 1 operates automatically and quickly but
(an object or concept). Correct pronoun would be can lead to predictable errors due to these
"which." biases.
3- "Understanding the two systems, whom have Other options like "always leads to incorrect
different roles, is crucial for decision-making." decisions," "requires too much mental effort," or
Incorrect: "whom" is used for people as an object, "completely replaces System 2" are incorrect
but here it should be "which" or "that" referring to based on the passage information.
"two systems." Thus, the correct answer is: It can be prone to
4- "The effortful mental activities whose System 2 biases and snap judgments .
performs include solving complex problems."
Incorrect use of " whose " with "activities" — Solution 51: The passage clearly states that the
better phrasing would be "The effortful mental school uses solar panels to generate clean
activities that System 2 performs..." energy for running fans, lights, and computers.
Solar power is highlighted as a non-polluting
Solution 48: The meaning of the word option that has made the school and the village
"deliberate" in the sentence "System 2 is the cleaner and more environmentally aware. Other
slower, more deliberate mode of thinking." is: options are neither mentioned nor relevant in the
Careful and intentional passage.
Explanation:
System 2 thinking is described as slower and Solution 52: The passage clearly states that the
more effortful, involving careful attention to saved money from electricity bills is used by the
complex problems and logical arguments. school to buy books and improve classrooms.
"Deliberate" here refers to conscious, intentional, This reinvestment helps enhance educational
and thoughtful mental effort, contrasting with the resources and infrastructure. Other options are
quick and automatic nature of System 1. not mentioned in the passage.
It is not accidental, quick and effortless, or biased.
Therefore, the correct option is: Careful and Solution 53: The passage specifically mentions
intentional . that solar power does not cause pollution like coal
or petrol. This is the chief reason why solar
Solution 49: The best synonym for the word " energy is considered better than coal in the
endorse " in the sentence: context provided, focusing on environmental
"System 2 is more methodical and rational, but it protection and cleanliness of the village. Other
is also lazier—it prefers to endorse the intuitions options are not supported by the passage.
of System 1 rather than question them."
is Approve . Solution 54: The passage clearly states that the
Explanation: school inspired nearby villagers to install solar
To "endorse" means to give approval or support panels in their homes, leading to a cleaner village
to something. and increased environmental awareness. The
Synonyms like "approve," "support," "back," and other options are not mentioned as actions
"champion" align with this meaning. motivated by the school.
The other options, "reject," "criticize," and "delay,"
do not convey approval or support. Solution 55: The passage explains that on cloudy
Thus, the correct answer is: Approve . days, the solar battery stores enough power to
keep the school running. Solar batteries are
Solution 50: According to the passage, one therefore essential for providing stored energy
drawback of System 1 thinking is: when sunlight is not available. Other options are
Telegram (Previous year papers PDFs [SSC,Railway,DSSSB,UP SI]): https://t.me/RBE_S
YouTube (Free lectures and job updates): https://www.youtube.com/c/RBERevolutionByEducation
Download RBE application for SSC Exams Complete Preparation
not mentioned or relevant to the school's Direct reference from the passage .
functioning on cloudy days.
Solution 63: ✅ Correct Answer: By making salt
Solution 56: The passage focuses on the ethical from the sea .
and moral implications of genetic engineering, Direct reference from the passage .
discussing potential inequalities, unintended
consequences, and the importance of responsible Solution 64: ✅ Correct Answer: 240 km
choices. The central concern is navigating what is Direct reference from the passage .
ethically acceptable in the use of genetic
engineering, rather than just scientific capability, Solution 65: ✅ Correct Answer: Peaceful
regulation, or technological advancement.
protest and courage
Peaceful protest : Gandhi’s method was non-
Solution 57: The passage warns that altering
violent.
genes for non-medical reasons may create new
Courage : Despite British opposition, Gandhi
forms of inequality, where genetically modified
remained calm and led the march bravely.
individuals have social advantages over others.
The author expresses concern about fairness,
Solution 66: ✅ Correct Answer: Loss of
unintended consequences, and ethical
autonomy due to filtered choices .
considerations, rather than unconditional benefits
Central concern of the passage.
or legal encouragement.

Solution 67: ✅ Correct Answer: Repeated


Solution 58: The passage states that scientific
advancements often outpace the development of exposure to similar .
legislation and ethical frameworks. This rapid Direct reference from the passage
progress creates a gap, making it difficult for
ethical guidelines to keep up with what is Solution 68: ✅ Correct Answer: It narrows
technologically possible. Other options do not perspective and reduces openness
align with the main point discussed in the Meaning of Cognitive Narrowing:
passage. Refers to a reduction in mental flexibility
Users are repeatedly exposed to similar content,
Solution 59: The passage notes that gene editing which limits their openness to diverse
may have "unintended consequences" and that perspectives.
its effects "may ripple through generations," Effect on Thinking:
indicating concern about how changes could Instead of expanding creativity or emotional
persist and impact the future. Other options are depth, algorithmic living reinforces existing
either contradicted or not discussed by the preferences .
author. This leads to mental rigidity and confirmation
bias .
Solution 60: The author emphasizes the
importance of acting responsibly and thoughtfully Solution 69: ✅ Correct Answer: Practice
with powerful technologies like genetic critical engagement
engineering, advocating for caution, reflection, Meaning of “Critical Engagement”:
and ethical consideration. The other options Being mindful and analytical about how we
either exaggerate consequences, promote haste, interact with digital platforms
or wrongly imply all science is inherently safe. Not blindly accepting algorithmic suggestions
Actively seeking diverse perspectives and
Solution 61: Mahatma Gandhi was the central questioning what is shown
figure and leader of the Dandi March.
Direct reference from the passage . Solution 70: ✅ Correct Answer: Option 2 –
Cautious and reflective
Solution 62: ✅ Correct Answer: To protest the Tone Definition:
British law that taxed salt . Cautious : Expresses concern about potential
Telegram (Previous year papers PDFs [SSC,Railway,DSSSB,UP SI]): https://t.me/RBE_S
YouTube (Free lectures and job updates): https://www.youtube.com/c/RBERevolutionByEducation
Download RBE application for SSC Exams Complete Preparation
risks are irrelevant to the issue.
Reflective : Encourages thoughtful consideration
and self-awareness Solution 76: ✅ Correct Answer: Option B –
Personalizing learning
Solution 71: Correct Answer: (C) Threats to 🧠 Explanation: The passage clearly states that
digital privacy AI helps in identifying each learner’s strengths
Explanation: and weaknesses and suggests specific topics for
The passage emphasizes how user data is improvement—this is the essence of
collected, often without consent, and used for personalized learning . The other options are
commercial purposes. not supported by the passage.
It highlights ethical concerns, weak
implementation of data protection laws, and the Solution 77: ✅ Correct Answer: Option C –
blurring line between public and private life.
Poor thinking skills
The central theme revolves around digital
🧠 Explanation: The passage clearly states that
privacy risks in the modern tech-driven world.
over-dependence on AI may limit critical
thinking and cause students to lose their
Solution 72: Correct Answer: Without users
problem-solving abilities . This makes Option C
being fully aware
the most accurate and relevant choice. The other
Explanation :
options are either unrelated or contradict the
The passage clearly states that user data is
passage.
collected "sometimes without their knowledge"
, especially when users accept terms and
Solution 78: ✅ Correct Answer: Option C – To
conditions without reading them.
This highlights the lack of awareness and answer students’ questions
transparency in digital data collection practices. 🧠 Explanation: AI chatbots are designed to
assist with routine queries, making learning
Solution 73: Correct Answer: (C) They exist smoother and freeing up teachers’ time. The
but are poorly enforced other options are not mentioned or supported by
Explanation: the passage.
The passage mentions that “some countries
have passed laws to protect user data, but Solution 79: ✅ Correct Answer: Option C –
implementation remains weak.” Data privacy
This clearly indicates that while such laws do 🧠 Explanation: The passage explicitly mentions
exist, they are not effectively enforced or concerns about data privacy , making it the
uniformly applied. most appropriate choice. The other options are
not discussed or relevant to the context.
Solution 74: Correct Answer: (b) Improving
service experiences Solution 80: ✅ Correct Answer: Option C –
Explanation: Balance tech and human input
The passage states that data collection “can lead 🧠 Explanation: The author clearly advocates for
to better services,” which is a positive outcome a balanced approach , where AI supports but
of using user data. does not dominate the learning process. This
This refers to personalized experiences, targeted ensures that students benefit from both
recommendations, and improved functionality technological tools and human mentorship.
based on user behavior.

Solution 75: Explanation:


The author clearly states that protecting privacy in
the digital age requires awareness , regulation ,
and responsible digital behavior . Avoiding all
apps is impractical, stronger surveillance tools
worsen privacy concerns, and free mobile phones
Telegram (Previous year papers PDFs [SSC,Railway,DSSSB,UP SI]): https://t.me/RBE_S
YouTube (Free lectures and job updates): https://www.youtube.com/c/RBERevolutionByEducation
Download RBE application for SSC Exams Complete Preparation

SSC Steno 2025 T-1 Miscellaneous All 2 Questions with Detailed Solution and Answer key

Q1. Which of the following is an example of (a) The chef has arranged the cherries in a
onomatopoeia? pattern.
(a) Baloon (b) Horrible (b) The chef has arranged the cherries by a
(c) Silence (d) Murmur pattern.
(c) The chef has arranged the cherries of a
pattern.
Q2. Choose the sentence with correct collocation (d) The chef has arranged the cherries with a
for the word ' pattern ' pattern.

Answer Key:

Q1 d Q2 a

Solution 1: Explanation "Silence" is the absence of sound, not a word that


Onomatopoeia are words that imitate the natural imitates a sound.
sounds associated with things.
"Murmur" imitates the sound of soft, indistinct
talking or whispering, fitting the definition of Solution 2: The correct answer is "The chef has
onomatopoeia. arranged the cherries in a pattern." The usual
The other options: and correct collocation in English is "in a pattern
"Balloon" is not a sound word. ," which means the cherries are organized
"Horrible" is an adjective describing something following a particular design or order. The other
unpleasant. prepositions ('by,' 'of,' or 'with') are not standard or
grammatically correct in this context.

Telegram (Previous year papers PDFs [SSC,Railway,DSSSB,UP SI]): https://t.me/RBE_S


YouTube (Free lectures and job updates): https://www.youtube.com/c/RBERevolutionByEducation
RBE COMBO TEST SERIES
https://rbelearning.com/test-series

Maths+English+GK+Reasoning
FULL LENTH MOCKS
SECTIONAL MOCKS
ALL SUBJECTS
TYPE - WISE

TOPPERS
की पसंद
SSC CGL 2025
Air Air Air Air Air
8 16 13 22
5

SSC CGL 2024 Toppers - RBE paid Test Series Users

RBE WEBSITE - rbelearning.com


DOWNLOAD RBE APPLICATION NOW
https//play.google.com/store/apps/details?id=com.revolution.education

For SSC Exams Complete Preparation


FREE PLAYLISTS ON YOUTUBE

MATHS REVISION COURSE LATEST GOVT. JOBS

COMPUTER GK IMP. VIDEOS

GK CRASH COURSE MONTHLY TOP 100 CA

SSC BEST QUESTIONS SCIENCE THEORY + PYQS

You might also like